You are on page 1of 246

,

HU
,U IN
NGUYN

,
, ,
`
BANG

GIAI TOAN
PHUONG PHAP

, ,
BIN

AI
LU
O
NG
B
T
.
.
,
,
(T
ai ban l`n thu hai)

,
BAN GIAO
` XUT
DUC
NHA
.

517
21/727-05
GD 05

M
a s: 8I004T5-TTS

,
`
`
U
LOI NOI

,
D bt bin, ung van
. bin
,,
Tn tu (Binh ph
ap)

,
,
, ,,
,,
ta thu. c hin
o mt
Ta h
ay tuong tuo. ng c
. h. thng trn do
. nhung thao
,
,
,
t
ac kh
ac nhau. Ta c
o th phn tch trang
th
ai cua h. thng d x
ac dinh
. vi.
.
,
,
,
,
,
tr (muc
ac. Mt
. dch) c`n dat
. duo. c t`u nhung vi. tr kh
. trong nhung cng cu.
,
rt manh
cho vic
a tnh cht bt bin cua mt
ai
.
. phn tch h. thng l`
. s d
.
, ,,
,,
,
,
,
,

i nhung
luo. ng trong h. thng. Nhung d
ay khng thay di duo
ai
. luo. ng n`
,
, ,
,
`ng
thao t
ac kh
ac nhau trong h. thng. Su. bt bin c
o th d`
ung d ch ra ra
,
,
,
i mt
t`u mt
ac. Xuyn sut cun
. cu h`nh khng th dat
. to
. cu h`nh kh
,
, , ,,
s
ach n`
ay l`
a tu tuong bt bin thng qua c
ac chu d` trong s hoc,
. dai
. s,
,
h`nh hoc
a tr`
o choi to
an hoc,...
. v`
.
,
,,
,,
,
. t tn l`
Cun s
ach duo. c da
a Phuong ph
ap d
a mong
ai
y l`
. luo. ng bt bin, d
,
,
,,

mun cua t
ac gia mun bin soan
ac phuong ph
ap to
an hoc
. mt
. loat
. c
. trong
,
,
,,
,,

bin soan
hoc
a nghin cuu. Truoc dy t
ac gia da
ap
. tp
. v`
. mt
. s phuong ph
,
,, ,
giai to
an trong c
ac cun s
ach [6], [7], [8], [9], [10], [11]. Ngu`oi xua l`
am
,
,
,
,
p dung
ra Binh ph
ap d a
v`
a giai quyt nhung cuc
.
. chin tranh, din h`nh
,
,
,,
trong t`u,ng
nht l`
a b. Binh ph
ap cua Tn tu v`
a b. Binh ph
ap cua Tn Tn
,
,
,
,
trai nghim
p dung
th`oi k` m`
a hai soan
aa
v`
ao thu. c tin.
a
. gia trn d
. qua v`
.
,
,
,
,
,,
bit Binh ph
Nhu ta d
ap ch c`n c
o 36 muu k m`
a ho
a giai d
a
uo. c h`u ht
,
,,
,,
. t ra. a
. c bit
m duo. c Binh
c
ac t`nh th cua c
ac cuc
a
. chin tranh d
. ngu`oi na
,
,
,
p dung
ph
ap v`
aa
n
o v`
ao thu. c t nhu th n`
ao l`
a mt
ang tao
.
. vn d` s
. cua
,
,
,
,,
,
,,
,
,
t`ung ngu`oi v`
a t`ung th`oi d
o th n
oi c
ac phuong ph
ap giai to
an l`
a nhung
ai.
. C
3

,,
,,
Phuong ph
ap d
ai
. luo. ng bt bin

,
,
,
ai tp
an. T
ac gia cun s
ach mao
muu k trong khi giai b`
. to
. mui
. bin soan
.
,
,
,
,
nhung phuong ph
ap giai to
an v`
a hoc
an c
ung hi vong
th`
anh b. To
an
. to
.
,
,

`
ph
ap cho m`nh v`
a c
ac ban
tham
kh
ao.
R
t
nhi
u
v
n
d

trong
c
a
ch
gi
ai

.
, ,
,
,
,
suu t`m v`
to
an v`
a hoc
an c
o th tng kt lai,
ac gia d
a chon
a
. to
. t
. loc
. nhung
,,
,
,
,c kh
. c trung nht, nhung b`
phuong ph
ap da
ai tp
a c
o mu
ai qu
at cao
. hay v`
, ,

`
c. Ban
mang ni
an hoc
a su sa
ao c
ac
. dung to
. co ban v`
. doc
. t`m thy phn n`
,
,,

phuong ph
ap h
oa giai c
ac b`
ai tp
ho
a
c
m
t
c
a
ch
nh`
n
trong
h
oc
t
p
c
u
ng
.
.
.
. .
,
, ,
,
nhu thu. c h`
anh tu. m`nh giai b`
ai tp.
.
,
,
,
,
,,
,

duo. c xut ban, cun s


ach n`
ay
C`
ung voi nhung cun s
ach cua t
ac gia d
a
,,
,,
vn d` d
,i 5 vn d`, mi
quan tm to
oi gon
trong
m
t
chu
o
ng
v`
a
l`
a
uo. c g
.
.
,
,,
,,
,
,
,
,

p dung
mt
ap giai a
dai
i
. phuong ph
.
. luo. ng bt bin. Mi chuong tuong d
,
,
,
,
,

`
i nhau, sau mi tit nho trong chu d l`
. t ra
dc
a mt
. lp
. vo
. vn d duo. c da
,
,
, , ,,

v`
a c
o v du. minh hoa,
sau
d
o
l`
a
b`
a
i
t
p
a
p
d
ung
nh
u
ng
tu
tu
o
ng
c
ua
tit.

.
.
.
,
,
,
,
chuong c

. c trung kh
. c bit
Mi
o nhung tit da
ac nhau. a
. ph`n cui mi
,
,
,,
,,
,
,
,
,
chuong l`
a mt
phuong ph
ap cua chuong
` hay th hin
. chuyn d
. su dung
.
,
,
,
,,

`
, mi vn d o dy c
d
o th ph
at trin th`
anh ni
oi chuyn
o
. dung mt
. bui n
.
, ,
,
,,

ngoai
kh
o
a.
Nh
u
ng
b`
a
i
t
p
d
u
o
c
a
p
d
ung
c
a
ch
gi
ai
c
ua
c
a
c
b`
a
i
m
u
nn
.
. .
.
,
,
,
,,
,,
,
v`
khng c
o giai chi tit. Ch c
o hai chuong c
o huo
ng dn
a go. i y cua b`
ai tp
.
,
,
,
,
,

t nhu sau:
trong ca chuyn d`. Ni
om ta
` d
uo. c t
. dung cua mi chuyn d
,,
,
Chuong 1: Nguyn l bt bin. Nhi`u b`
ai to
an cho bit thu. c hin
. mt
. s
,
,
,
,
. c
nhu c
thao t
ac trn mt
ao do
ac s, qun b`
ai, qun c`o hoa
. h. di tuo. ng n`
,
,
,
,
,
,
c tap
a nhung dai
cho. Tuy b`
nhung bin d
ai to
an c
o phu
a
. nhung n chu
.
,
,
,
,,
,

n le hoa
. c tng, tch cua c
luo. ng bt bin nhu tnh cha
ac bin khng thay
,
,
,
,

i.
Nh
o
ph
a
t
hi
n
ra
ho
a
c
c
t`
nh
d
u
a
ra
nh
u
ng
bi
n
c
o tnh cht bt bin

.
.
,
,
,
,
,
. c don diu
hoa
ao nhung du kin
n kt
. bt bin, nh`o v`
. bt bin dua ta d
,
,
,
,
,
,
lun
ai to
an. Nhung b`
ai to
an cua chuong n`
ay l`
a du. a v`
ao c
ac tnh cht
. cua b`
s hoc,
ai to
an c
ung mang tnh c
ac b`
ai tp
a thit lp
. b`
. s hoc.
. Chuyn d` l`
.
,
,
ng dung
c
ac h`
am bt bin v`
au
c
ua
n
o
.
.
,
,,
,c d
,ng hai bin. Tnh bt bin th hin
Chuong 2: a thu
o vic
inh
i xu
.
. r
. d

,
L`oi n
oi d
`u

,
,c di xu
,ng: Mt
,c hai bin goi
,ng nu ta
ngha nhung da thu
ad
i xu
. da thu
. l`
,
,
,
,c khng
thay di vai tr`
o v`
a vi. tr giua hai bin cho nhau, gi
a tri. cua da thu
,
,
,,
,ng minh d
,c di xu
,ng hai
thay d
i. T`u d
inh
uo. c moi
a thu
. ngha trn ta chu
. d
,
,
,
,
,c cua c
,c d
,ng co, so,, m`
bin d`u biu din nhu d
ac d
a c
ac
a thu
a thu
i xu
,
,
,
,
,
,
,
,
1

c di xu
ng co so lin quan d
c nghim

da thu
n cng thu
. cua Vi`ete . T`u do
,
, ,
,

p dung
a
d
ang loat
ac vn d
a thuc cho h`
trong dai
.
. c
. s so cp nhu giai h.
,
,,
,,
,
v cua
,c di xu
,ng;
,i mi
phuong tr`nh vo
c
ac phuong tr`nh l`
a nhung da thu
,
,
`ng c
,c ra th`u,a s; giai phu,o,ng tr`nh ba
. t n s phu;
phn tch d
ach da
a thu
.
,
,,
,
,
,

`
ng, ... Chuyn d l`
i h. s di xu
p b`
giai phuong tr`nh vo
a mt
ai to
an d
a
. lo
,
,,
,c bc
,ng, u
,ng dung
thu
o c
ac h. s di xu
. cao c
. chuyn d` giai phuong tr`nh
`ng c
bc
ach ha. bc.
. cao ba
.
,
,
,,
, tu.,. Khi chu
,c cua c
,ng minh
ng thu
Chuong 3: Bt da
ac d
ay s d`ng thu
,
,
,c th` vai tr`
ng thu
. c c
ng
bt da
o c
ac bin hoa
ac bin s tham gia trong bt d
a
,
,
,
,c ngang nhau. Nhi`u khi su., chuyn d
thu
o v`
a vi. tr giua c
ac bin
i vai tr`
,
,
,
khng thay di, do do
`ng nhu,ng
c vn
ng thu
ta cho ra
cho nhau th` bt da
,,
,,
,
, tu., n`
v`
bin s n`
ay duo. c xp theo mt
u
ao do
a nh`o thm di`u kin
. th
. duo. c
,
,
,
,
,
,
,
p m`
ng minh d
c d d`
ng thu
sa
a chu
ang hon. Trong chu d` n`
ay
uo. c bt da
,
,,
,
,
, ,
,

ng thuc duo. c tao


p xp theo mt

o sa
ta xet bt d
a
. boi hai b. s c
. thu tu. da
,
,
,,
,c v` tng c
ng thu
. p s trong hai b. s duo. c sa
p xp
ac ca
bit, d
a bt d
y l`
a
,
,
,
,
, ,
,ng minh bt
ti uu nht. T`u d
l co ban cua chuyn d
o th chu
inh
`, ta c
.
,
,
,
c Cauchy2 , Chebyshev3 v`
ng dung
ng thu
da
a nhi`u u
kh
ac. Chuyn d
a
` l`
.
,
,
,
,
,
,
,
,
,

a
ng
th
u
c
n
i
mt
b
t
d
a
ng
th
u
c
r
t
t
ng
qu
a
t,
c
o
th
nh
n
d
u
o
c
c
a
c
b
t
d

.
. .
, ,
,
ting kh
ac. Kt qua cua chuyn d
a c
ac ban
o th s
ang tao
` l`
. c
. ra rt nhi`u
,
,
,
c mo
i v`
ng thu
bt da
a rt d
ep.
.
, ,
,,
,,
,,
,,
Chuong 4: Phuong tr`nh h`
am. giai mt
am, ngu`oi ta
. phuong tr`nh h`
,
,
,
,,
,,
,
,,
thu`ong thay d
a tri. cua di s d
i gi
nhn
uo. c nhung phuong tr`nh thch
. d
,
,
,
,
,c cua
,i nhu,ng gi
ng thu
ho. p r`i suy ra h`
am phai t`m. Vo
a tri. kh
ac nhau, d
a
,,
Vi`ete (1540-1603): Nh`
a to
an hoc
ap.
. ngu`oi Ph
,,
,
Louis Cauchy (1789-1857): Nh`
a to
an hoc
uc.
. ngu`oi
,`,
3 Pafnuty Lvovich Chebyshev (1821-1894): Nh`
a to
an hoc
. nguoi Nga.
1 Francois

2 Augustin

,,
,,
Phuong ph
ap d
ai
. luo. ng bt bin

,
,,
,
,,
. c bt bin, ta dua ra phuong ph
phuong tr`nh h`
am khng di, hoa
ap th
, ,
,,
,
,
,
,
,
hai d
c
ac gi
a tri.
ap thu
am l`
a phuong ph
ap
giai phuong tr`nh h`
. Phuong ph
,
,
,
,
`

dim bt d
ng.
Chuyn
d

l`
a
nh
u
ng
d
a
th
u
c
giao
ho
a
n,
c
a
ch
t`
m
nh
u
ng
d
.

a
,
c giao ho
thu
an.
,,
,
,
,
Chuong 5: Nh
ung tr`
o choi to
an hoc.
ai to
an tr`
o choi t`m kim
. s b`
. Mt
,
,
,,
,
,
ng v`
nhung chin thut
a ho`
a cua ngu`oi choi c
ung d`u du. a v`
ao tnh bt
. tha
,
,
,
,
cho. T`m kim mt
bin cua nhung di`u kin
ai to
an tr`
o
. da
. l`oi giai cho b`
,
,
,
,
,
,
,

`
`
choi l`
a mt
vi
c
r
t
kh
o
,
ngu
o
i
ta
thu
o
ng
ph
a
t
hi
n
ra
nh
u
ng
bi
n
d
o
n

.
.
.
,
,,

theo mt
diu
b
t
bi
n,
ngh
a
l`
a
c
o
t
a
ng
ho
a
c
gi
am
m
t
d
ai
lu
o. ng n`
ao do

.
.
. .
.
,
,,
,,

quy lut
Nhiu b`
ai thi hoc
ac nuo
c, v` n
o
.
. c dinh.
. sinh gioi rt hay ra o c
,
`
`
doi hoi hoc
ach logic v`
a thng minh. Chuyn d l`
a c
ac
. sinh suy lun
. mt
. c
, `,
,
,

chin thut
trong
tr`
o
cho
i
Nim,
m
t
tr`
o
cho
i
xu
t
x
u
t
u
Trung
Qu
c.
Tr`
o
.
.
,
,
,
,,
, ,,

ng.
choi d
a ch ra buoc d
uo. c phn tch k v`
i d
cho kt qua tha
,
,
,
,
,
duo. c lu. a chon
ac nhau, vic
Nhi`u b`
ai tp
. t`u nhi`u ngu`n kh
. da
. giai
,
,
`m m ta tt phu,o,ng ph
. t ra, c`
d
v du. v`
a b`
ai tp
ap da
on nhung c
ach
a
. nha
,
,
,
,
,
,
ng minh kh
bo qua (v du. nhu bt
chu
ac c
o th c
o v`
a hay hon t
ac gia d
a
,
,
c Cauchy).
ng thu
d
a
,
,
Cun s
ach d`
anh cho hoc
an, hoc
a gioi
. sinh ph thng yu to
. sinh kh
mn to
an, c
ac th`y c gi
ao, sinh vin d
anh to
an, ng`
anh tin hoc
ai
. hoc
. ng`
.
,
,
,
,`,

v`
a nhung nguoi yu thch to
an hoc
. ph thng. Trong bin soan
. khng th
,
,,
mong ban
tr
anh khoi sai s
ot v`
a nh`m ln
op y gui v`
oc
. d
. cho y kin. Moi
. g
,
,
,
d
a xut ban Gi
ao duc,
a Ni.
ia
. ch: Nh`
. 81 Tr`n Hung ao,
. H`
.
H`
a Ni,
ang 9 n
am 2004
. th
,
,
Nguyn Huu in

, ,
CHUONG

BIN

NGUYN LI BT

,,
,,
,
dai
i thiu
1.1. Gio
. phuong phap
. luo. ng bt bin . . . . . . . . . . . . . . . . . . . . . . . . . . . . .
hin
` toan
........................................
1.2. Phat
. bt bin trong bai
,
,,
ba
` ng dai
1.3. Giai toan
. luo. ng bt bin . . . . . . . . . . . . . . . . . . . . . . . . . . . . . . . . . . . . . .
,
1.4. Bt bin do
n di
u
. ........................................................
,
` toan
nng cao . . . . . . . . . . . . . . . . . . . . . . . . . . . . . . . . . . . . . . . . . . . . . . .
1.5. Nhung bai
` bt bin . . . . . . . . . . . . . . . . . . . . . . . . . . . . . . . . . . . . . . . . . . . .
1.6. Chuyn d
` v` ham

1.6.1. inh
am bt bin trn trang
ai . . . . . . . . . . . .
. ngha h`
. th
,

`
1.6.2. H. thng bt bin dy du . . . . . . . . . . . . . . . . . . . . . . . . . .

7
9
24
27
43
51

52
58

,,
,,
,
dai
i thiu
1.1. Gio
. phuong phap
. luo. ng bt bin
,
,
,
,
Cho a, b, c l`
a nhung s thu. c. Ta xet tng S = a + b + c. Nu ta d
a
i ch
,
,
,
cho b, b cho c v`
a c cho a, th` tng S lun lun ch l`
a mt.
ay khng
. Tng n`
,
,
,
,
,
,

tu. thu. c hin


, tu.,
i thu
thay di di vo
D`
u a, b, c c
o thay d
i thu
. phep cng.
.
,
,
,
khng thay d
,i
nhu th n`
ao ch
ang nua S vn
a S bt bin d
i, ngha l`
i vo
,
,
,
vic
ac bin kh
ac. Trong thu. c t c
ung nhu trong to
an hoc,
. thay di c
. rt
,,
,u lai
nhi`u vn d` lin quan d
n mt
. s di tuo. ng nghin cu
. bt bin di
,
,
,
,,
theo
,i su., thay di cua nhi`u d
vo
ac. V du,
i tuo. ng kh
. nu ta chuyn ch
,
,,
. t pha
ng hoa
. c trong khng gian,
trong mt
huo
ng mt
ao d
o
. h`nh khi n`
. ma

,,
Chuong 1. Nguyn l bt bin

,
,
,n c
th` d
ai c
ac canh,
d. lo
ac g
oc cua h`nh, khi khng thay di...
. d`
.
, ,
,,
,a
Ta c
o th hiu l`
a : Moi
a quan h. giu
ai
inh
. d
. luo. ng d
. tnh hay tnh cht v`
, ,
, ,
,
,ng ph`n tu, cua mt
nhu
. c mt
a khng thay d
oi mt
i v
. hoa
. s tp
. ho. p m`
. bin
,
,,
d
d
ao d
a bt bin.
i n`
o
uo. c goi
. l`
,
`ng su., bt bin c
Nhi`u v du. ch ra ra
o trong mn s hoc,
a h`nh
. dai
. s v`
,
hoc,...
Nhung b`
ai to
an c
o lin quan dn bt bin chia l`
am hai loai
.
. :
,
,
,
. c kt qua) phai t`m.
1. Nhung b`
ai to
an ly bt bin l`
am kt lun
. (hoa
,
,
,,
,,
Nhung b`
ai to
an loai
ay duo. c lit
. n`
. k rt nhi`u trong chuong 1 cua
,
, ,
,
cun s
ach [11]. Nhung bt bin cua b`
ai to
an l`
a kt qua cua qu
a tr`nh
. p trong ni
la
ai to
an.
. dung b`
,
,
,,
2. Nhung b`
ai to
an ly tnh bt bin l`
am phuong ph
ap giai. Trong cun
,
,,
s
ach [11] c
o mt
ay. Chuong n`
ay
. ph`n rt nho d` cp
. dn vn d` n`
,
ta d
ac kha canh
cua loai
ai to
an n`
ay, c
ung l`
a tr`nh b`
ay
` cp
n c
. d
.
. b`
,
,,
,
p b`
phuong ph
ap giai cho mt
ai to
an.
. lo
,
,
B`
ai to
an xut ph
at nhu mt
. chuyn
. c tch :
,
,`,
,
,
,
Nguoi Nng dn tr`ng d
o 99 qua chua chn m`
au
uo. c mt
. cy kh th`n c
,
,

chn m`
xanh v`
a 1000 qua d
au v`
ang. Mt
n mi ng`
ay hai qua
a
n a
. con Qua. d
, ,
,,
,
n mt
kh v`
a n
oi v
oi ngu`oi nng dn: "A
ang, may t
ui ba gang
. qua tra cuc
. v`
,
,
,

mang d
ad
n hai qua kh bt k` khng phn bit
i m`
u. ng". Qua. d
n a
. qua xanh
,
,
,
v`
a qua v`
ang. Nu Qua. a
n mt
ang v`
a mt
. qua v`
. qua xanh th` cy kh lai
. sinh
,
,
,

ra mt
n hai qua v`
ang, th` cy kh lai
. qua xanh. Nu Qua. a
. sinh ra mt
. qua
,
,
,
v`
ang. Nu Qua. a
n hai qua xanh th` cy kh lai
ung qua v`
ang. Hoi c
o
. sinh c
, ,
,
,`,
,

th xay ra truong ho. p qua kh cui c`


ung c`
on lai
a m`
au v`
ang khng?
. trn cy l`
,
,
,
,
thun
a X; qua kh
. tin
. cho vic
. giai ta k hiu
. : Qua kh xanh l`
,
,
n qua l`
b`
v`
ang l`
a V; qua. a
a + v`
a cy kh sinh ra qua l`
a =. Khi d
ai to
an c
o
o
,
n gon:
th vit lai
. nga
.
V + V = V,

X + X = V,

V + X = X.

1.2. Ph
at hin
ai to
an
. bt bin trong b`

,
,
,
`ng s lu,o.,ng qua xanh hoa
. c l`
T`u c
ach vit trn ta thy ra
a khng thay di
,
,
,
l`n a
l`n Qua. a
. c l`
n (mi
n hai qua). V` trn cy
hoa
a giam d
i 2 qua sau mi
,
,
,
,
,
n, nn
s nhung qua m`
au xanh l`
a le, c`
on s nhung qua m`
au v`
ang l`
a cha
,
,
n qua
qua cui c`
ung trn cy se l`
a m`
au xanh, khng phu. thuc
ao c
ach a
. v`
,
cua Qua.
.
,
,
Tnh bt bin trong b`
ai to
an trn l`
a g`?
o l`
a s nhung qua xanh d`
u
,
,
,
,

n qua nhu th n`
. c nu n
Qua. c
oa
ao di nua th` n
o khng thay di hoa
o thay
,
,
,
,

di th` thay di mt
ach c dinh
a giam di hai qua. Chnh d
iu bt bin
. l`
. c
,
,
,
,
,
,
,
i qua xanh v`
di vo
a gia thit b`
ai to
an dua ta dn l`oi giai. Nhu vy
. vic
.
,
,
,,

cho cua b`
t`m ra tnh bt bin trong nhung d
ai to
an l`
a rt quan
ai
. luo. ng da
trong.
.
,
,
,,
Nhung b`
ai to
an c
o dang
an thu`ong t`n
. nhu mt
. quy tr`nh hay thut
. to
,,
,
,,
,,
,
tai
th
ai khoi d`u v`
a mt
ay nhung buo
c di ho. p l. (buo
c bin
. mt
. trang
.
. d
,
,
,
,
,
,
,
,
,
di). Kt lun
ai to
an loai
ay thu`ong phai tra l`oi nhung cu
. cua nhung b`
. n`
,
hoi sau dy :
,
,i du,o.,c trang
cho khng ?
1. C
o th d
ai cui c`
ung d
at
a
. to
. th
,
,
,i ?
2. T`m tt ca trang
ai cui c`
ung c
o th d
at
. to
. th
,i han
,i mt
3. C
o t`n tai
ai cui c`
ung khng ?
. gio
. tin to
. trang
. th
,
,
4. T`m tt ca chu k` c
o th c
o trong d
ay trang
ai ?
. th
,
,c t`m tnh bt bin trong mt
Ph`n sau dy ta xet nhung c
ach thu
ai
. b`
,
to
an nhu th n`
ao thng qua c
ac v du.
.

hin
` toan

1.2. Phat
. bt bin trong bai
,
,,
,
Thng qua v du. sau d
y, ban
oc
uo. c nh`n t`u
. d
. se thy su. bt bin d
,
,,
,
nhung kha canh
kh
ac nhau d`u giai duo. c b`
ai to
an.
.
,
`, tai
V du. 1.1. Trn bang ta vit 10 du cng
a 15 du tru
ac vi. tr bt k`.
. v`
. c

10

,,
Chuong 1. Nguyn l bt bin

,
mt
v`
oa hai du bt k` trong d
a vit v`
ao d
oa
Ta thu. c hin
o
o
. x
. du cng
. nu x
,
,
,
` nu x
hai du ging nhau v`
a du tru
oa hai du kh
ac nhau. Hoi trn bang
,
c`
on lai
ac trn 24 l`n?
. du g` sau khi ta thu. c hin
. thao t
,
du cng
du tr`u, ba
`ng s 1, c`
`ng s
`,i giai. C
Lo
ach 1: Ta thay mi
on mi
. ba
,
,
1. Thao t
ac thu. c hin
oa hai s v`
a vit lai
a tch cua ch
ung.
. x
. mt
. s se l`
,
,
,
,

V` th tch cua tt ca c
ac s vit trn bang se khng thay di. V` vy
. ngay
,
,
,
`
`

ng 1, th` cui c`
t`u du gia thit cho tch c
ac s trn bang ba
ung c
ung c`
on
,
,

lai
a trn bang c`
on lai
. s 1, ngha l`
. du tr`u.
, `
du cng
`ng s 0, c`
ng s 1.
C
ach 2: Ta lai
ba
on du tr`u ba
. thay mi
.
,
,
,
n th` ta vit lai
Thao t
ac thu. c hin
a tng cua hai s x
oa di l`
a s cha
. l`
. s
,
,
,
,
. c l`
0. Nhu vy
ac s trn bang sau khi thu. c hin
ac hoa
a
. tng c
. mt
. thao t
,
,
,
,
`

. c l`
khng thay di hoa
a giam di 2. u tin tng c
ac s trn bang l`
a mt
. s
,
,
,
`ng 15), th` s cui c`
le (ba
ung trn bang c`
on lai
a s le, vy
a s 1. Ngha
. l`
. l`
,
,
l`
a trn bang c`
on du tr`u.
, ,,
,
l`n thao t
`ng c
C
ach 3: Ba
ach thay nhu o c
ach 1, by gi`o sau mi
ac, s
,
,
,
. c l`
. c l`
1 hoa
a khng thay di hoa
a giam di 2. V` th l
uc ban d
`u s chu s
,
,
,
1 l`
a le, th` cui c`
ung ch c`
on lai
a c`
on lai
. mt
. s 1, ngha l`
. mt
. du tr`u.

,
, ,
,
Phn tch ba c
ach giai ta thy c
ach 1 lo. i dung
tnh khng di cua tch
.
,
, ,
,
,,
n c
c
ac s vit trn bang; c
ach 2 su dung
tnh khng di cua tng cha
ac s
.
,
,
,
,
,
,

n c
ac du tr`u. Nhu vy
ach giai
v`
a c
ach 3 l`
a su. khng d
i cua s cha
. trong c
,
,
, ,
,,
,,
n le cua
. c s luo. ng cha
su dung
ta da
tnh cht bt bin cua tch, tng hoa
.
,
,
`ng khi ga
,p b`
. p nhung lo
c
ac s. Qua c
ach giai trn ta thy ra
ai to
an m`
a thao
,
,
,
,

. p di la
. p lai,
t
ac la
a t`m ra nhung tnh bt bin cua thao
. ta phai bin di v`
,
,
`
ng c
t
ac ta thu. c hin.
u y ra
ac thao t
ac ta thu. c hin
ao
. Ch
. khng phu. thuc
. v`
,
,
,
,
,

`
ng minh n
t d
thao t
ac trn hai s n`
ao ba
o tuong tu. nhu c
ach
u, vic
. chu
, ,
,,

l`
am trn, ban
am b`
ai tp
ay). Mt
. doc
. l`
. 1.18 (o cui phn n`
. bin th cua

1.2. Ph
at hin
ai to
an
. bt bin trong b`

11

,,
,,
,
b`
ai to
an trn duo. c cho duo
i dang
. nhu v du. sau:
,
,
,,
,,
on
V du. 1.2. Bn k tu. X v`
a n
am k tu. O d
uo. c vit xung quanh mt
u`ong tr`
. d
,
,
,
,

theo mt
a nhu nhau th` ta vit thm
. thu tu. bt k`. Nu hai k tu. canh
. nhau l`
,
,
,
,
,

a ch
,a ch

v`
ao giu
ung X m
oi, nguo. c lai
ao giu
ung O m
oi. Sau d
o
. ta vit thm v`
,
,
,
ng k tu. c
ta x
oa nhu
u X v`
aOd
ac trn la
. p lai
i. Ta thu. c hin
. thao t
. mt
. s
,
,
,
,
,
l`n. Hoi sau mt
a tr`nh trn, ta c
o nhn
uo. c chn k tu.
. s l`n thu. c hin
. qu
. d
,,
O quanh d
on hay khng?
u`ong tr`
,
,
,
,
`,i giai. Nu ta d
. t X = 1 v`
ch
Lo
a O = 1, khi do
u y l`
a giua nhung k tu.
a
,
,
,
,,
`ng tch cua ch
canh
nhau duo. c thay ba
ung. Nu ta xet tch P cua tt ca gi
a
.
,
,,
,
,
`ng P mo
`ng
i ba
tri. truoc v`
a sau khi thu. c hin
i, ta se thy ra
. mt
. l`n thay d
,
,,
,
`ng 1 sau khi thu. c hin
P lun lun ba
b`nh phuong cua P c
u. Do do
. thay
,
,
, ,
,
,
`i hoi P = 1 khng bao gi`o c
di. Nhung chn k hiu
o th xay ra.
. O do

,,

V du. 1.3. Mt
o canh
anh 16 vung, mi
uo. c chia th`
. h`nh vung c
. 4 cm d
,
vung d
` mt
nh
nh du cng
vung c
o canh
1cm. Trong mi
a
a
.
. (+), tru
. d
,
,
,,
,
,
,
` (). Nhu
ng du o c
du tru
ac vung c
o th thay di d
ang,
`ng th`oi theo h`
,
,
,`,
,
u han
ct
. c d
o kha n
ang sau hu
c
uong cheo. C
i du theo nguyn ta
. hoa
. l`n d
,
dn tt ca c
trn dn
ac vung d
o du cng
`u c
. (+) khng?.
,
, , ,
, `
`,i giai. Ta thay du cng,
ng c
Lo
tr`u ba
ac s tuong u
ng 1 v`
a 1. Trang
ai
.
. th
, ,,
,,
,,
,
,
ban d
a h`nh 1.1. ai
a tch c
ac s o c
ac
`u gia su l`
. luo. ng bt bin o dy l`
,
,
c
o gach
ac m ta trong b`
ai to
an
. cheo trong h`nh 1.2, v` sau nhung thao t
,,
,,
dai
ay lun lun c
o gi
a tri. 1. Ngha l`
a trong c
ac duo. c gach
. luo. ng n`
. cheo
,
,
,
,
lun lun t`n tai
o s 1, suy ra khng th nhn
uo. c bang khng
. mt
. c
. d
,
,
a mt
chu
ao.
. du tr`u n`

,
,
,
V du. 1.4. Trn bang ta vit tp
ac s 0, 1 v`
a 2. Ta thu. c hin
. ho. p s g`m c
.
,
,

s cua s c`
chu
x
oa d
ac nhau v`
ad
ao do
on lai
a 2 thay
i hai s kh
in v`
. (ngha l`
,
`
cho 0 v`
a 1; 1 thay cho 0 v`
a 2; 0 thay cho 2 v`
a 1). Chung minh ra
ng nu sau

,,
Chuong 1. Nguyn l bt bin

12

H`nh 1.1

H`nh 1.2

,
,
,
, s duy nht
mt
ac trn, trn bang ch c`
on lai
. s l`n thu. c hin
. thao t
. mt
. chu
, , ,
, s d
c
khng phu. thuc
o
ao thu tu. thu. c hin
ac thao t
ac c
ac s d
th` chu
a
o
. v`
. c
,
trn bang.
,
,,
loai
`,i giai. Ta thu.,c hin
Lo
ac th` s luo. ng mi
. mt
. l`n thao t
. trong ba loai
.
,
,
,
,

. c giam d
s trn t
ang ln hoa
ac loai
s
thay
d

i
t
nh
i 1, suy ra s luo. ng c

.
,
,
,
n le. Khi trn bang ch c`

on lai
m
t
s
,
ngh
a
l`
a
hai
trong
c
ac s 0, 1 v`
a2
cha
.
.
,
,
,
,
`ng khng, c`
`ng mt.
ba ba
a ngay t`u d`u s
on s thu
c
o s luo. ng ba
. Ngha l`
,
,
,
,,
n le v`
luo. ng hai s trong ba s trn bang phai c
o c`
ung tnh cha
a mt
. loai
. s
,
,
,
,
n le kh
tu. thu. c hin
c`
on lai
o tnh cha
ac. V` th khng phu. thuc
ao thu
. c
. v`
.
,

thao t
ac, cui c`
ung ch c`
on mt
trong
c
a
c
s
0,
1
v`
a
2
c`
o
n
l
ai,
s
n`
a
y
c
o
s
.
.
,
, ,
,
,,
,
,
n le cua hai s kia.
n le kh
i s luo. ng cha
luo. ng cha
ac vo
,
,
,,
,ng minh b`
Trong chu
ai to
an trn, nu s luo. ng ca ba loai
. s trn bang
,
,
,
n le th` d`
c
o c`
ung tnh cha
u c
o thu. c hin
ac thao t
ac trn th n`
ao d
i nua,
. c
,
,
,
,
cui c`
ung c
ung khng th c`
on mt
ach giai nhung
. s duy nht trn bang. C
,
, ,
,
,,
v du. trn l`
a rt din h`nh, d cung c phuong ph
ap giai ta xet mt
ai b`
ai
. v`

to
an sau :

1.2. Ph
at hin
ai to
an
. bt bin trong b`

13

,
ta x
oa d
a vit
V du. 1.5. Trn bang ta vit ba s nguyn. Sau d
o
i mt
. s v`
,
,
,
`d
tng hai s c`
v`
ao d
on lai
ac nhu vy
. p lai
a
o
i 1. Thao t
. tru
. la
. mt
. s l`n v`
,
,
,
,
ng s d
cui c`
ung ta nhn
ang nhu
`u tin
uo. c ba s 17, 1967, 1983. Phai ch
. d
,
,,
trn bang d
a 2, 2, 2 ?
uo. c vit l`
,
,,
`,i giai. B`
d
Lo
ai to
an n`
ay l`
a mt
ai to
an kh
o da
uo. c ra trong mt
. k` thi hoc
.
. b`
, ,,
,
,
,,
,
,
,
,
,
,
`

sinh gioi o Nga. Ta tuong ch`ung nhu phai t`m lai


c
a
c
bu
o
c
th
u
c
hi
n
t
u
ba
.
.
.
,
, ,
,
,i c
s kt qua d
ac s ban d`u. Nhung b`
ai to
an c
o cu tra l`oi l`
a: Vo
ac
n c
, ,
, ,
ra, ta khng th thu. c hin
thao t
ac da
`u dn ba s
. bin di t`u ba s ban d
,
,
,,
c
kt qua, khi do
ac c
ach thu l`
a v vong.
B`
ai to
an c
o cho thao t
ac bin di
.
,
t d`u t`u, s n`
, tu., ra sao? Th
ba s nhung khng cho bit g` v` ba
ao v`
a thu
,
th` c
ai g` bt bin trong b`
ai to
an n`
ay ? Ta xet cu. th :
,, `
,
,,
Sau buoc du tin t`u ba s 2, 2, 2 ta nhn
ay c
o hai
. duo. c 2, 2, 3, ba s n`
,
,
,,
,
,
,
,

hai tro di th` kt qua lun lun c


c thu
n v`
s cha
a mt
o hai
. s le. T`u buo
,
,
, ,
,

`
`

n v`
t d
s cha
a mt
u ta thu. c hin
ao (v` nhung s
u tu bt cu s n`
. s le d`
. ba
,
,
, ,
,
,
,
n ba
`ng tng cua mt
n v`
cha
a mt
a tng cua hai
. s cha
. s le tr`u di 1; s le l`
,
, ,
n tr`u, di 1). Nhu,ng trong kt qua da
,i
cho d
s cha
a ba s le ca, nn vo
`u l`
,
,
,
cho v`
thao t
ac da
a xut ph
at t`u 2, 2, 2 khng th cho kt qua.
,
, ,
,,
,
n le cua ba s khng
B`
ai to
an trn duo. c giai nh`o ph
at hin
. ra tnh cha
,
,
,
,,
thay di, nn t`u trang
th
ai xut ph
at khng th nhn
th
ai kt
uo. c trang
.
. d
.
,
qua.
,
,,
,ng s 1, 2, 3, ..., 1974 d
V du. 1.6 (Kiev 1974). Nhu
uo. c vit trn mt
. bang.
,
,
,,
Ngu`oi ta thay hai s bt k` ba
`ng mt
. c l`
a tng hoa
. c l`
a hiu
. s hoa
. cua hai s
,
,
,
. Chung minh ra
d
`ng sau 1973 l`n thu. c hin
ac trn, ch c`
on mt
o
. thao t
. s
,
,
c`
on lai
a s 0.
. trn bang khng th l`

,
,
,
,,
n le
`,i giai. Vo
,i kinh nghim
Lo
ai to
an truo
c, ta quan tm dn tnh cha
. giai b`
,
,
,,
l`n thao t
n le nhu, th n`
cho v`
cua c
ac s d
a sau mi
ac duo. c s cha
ao. Khi
a
,
,
,
,
,
,

t du trn bang c
ba
o 987 s le. Mi ln ta thu. c hin
. thay di, s cua nhung

14

,,
Chuong 1. Nguyn l bt bin

,
,
n le kh
. c l`
. c
s le hoa
a c`
on nguyn (khi ta ly hai s c
o tnh cha
ac nhau hoa
,

n) hoa
. c l`
hai s c`
ung tnh cha
a giam d
ung tnh
i hai s (khi ta ly hai s c`
,
,
,
,
,
,
,

le). Nhu vy
on lai
. s cua nhung s le c`
. sau mi l`n thu. c hin
. thay di lun
,
,
,
lun l`
a mt
on lai
ung trn bang th` n
o phai
. s le. Vy
. khi c`
. mt
. s cui c`
,
,
n
l`
a s le, do d
o khng th l`
a 0.
o

,
,,
,ng s 1, 0, 1, 0,
V du. 1.7. Mt
on d
anh s
au re quat.
uo. c chia th`
. h`nh tr`
. Nhu
,
,,
, ,
,,
0, 0 d
ao trong c
ac re quat
ay thu tu. theo nguo. c chi`u kim d
uo. c vit v`
`ng h`.
. n`
,
,
,
,
Ta thu. c hin
ac la
. p: T
ang s cua hai re quat
nhau ln 1 d
on vi.
. thao t
. canh
.
.
,
,
,
,

Khi thu. c hin


c
a
c
thao
t
a
c
trn
c
o
d
u
a
d

n
k
t
qu
a
c
a
c
s
trong
c
a
c
r
e
qu
at

.
.
d
`ng nhau khng ?
`u ba
,
,
,
,
`,i giai. Ky hiu
Lo
a nhung s trong c
ac re quat
. a1 , a2 , ..., a6 l`
. hin
. th`oi. Khi
,
,,
,,
s S = a1 a2 + a3 a4 + a5 a6 l`
do
a mt
. dai
. luo. ng khng di. Khoi
,
d`u ta c
o S = 2. ch cui c`
ung ta mun l`
a S = 0 se khng bao gi`o d
at
.
dn.

,
,
V du. 1.8. Ngo`
ai bin dng, trn mt
on d
`n la
`n c
o
ao sinh sng ging tha
. h`
,
ba loai
au: m`
au x
am c
o 133 con; m`
au nu 155 con v`
a m`
au d
o 177 con.
o c
. m`
,
,
Nu hai con tha
`n la
`n kh
ac m`
au ga
. p nhau, th` ch
ung d
au sang
`ng th`oi d
i m`
,
,
m`
au thu ba. (V du. nu tha
`n la
`n m`
au x
am ga
. p tha
`n la
`n m`
au nu, th` ca hai
,
,
,ng tru,`o,ng ho.,p hai con tha
con d
au d
`n la
`n c`
ung
`u d
i sang m`
o.) Trong nhu
,
,
,
nguyn khng d
m`
au ga
. p nhau th` ch
ung giu
au. C
o xay ra t`nh trang
a
i m`
. l`
,
,
,,
,
,
trn d
`n la
`n tro th`
anh c`
ung mt
au d
ao tt ca tha
uo. c khng ?
. m`
,
,
,
,
,,
,
n le o,, c
`,i giai. Tnh cha
ac b`
ai truo
c l`
a nhung bt bin rt tt d
Lo
ta giai
,
,,
,,
,,
,
n le c
to
an, tnh cha
ung duo. c x
ac d
ac s cho 2. Tuong tu.
inh
. boi phep chia c
,
,,
,
nhu vy
a 0.
. ba s nguyn 133, 155, 177 chia cho 3 ta duo. c b. s du 1, 2 v`
,,
`n la
`n x
`n la
`n nu, th`
. p mt
Ta thu xet nu mt
am ga
. con tha
. con tha
,
,
,
ta c
ch
ung d`ng th`oi d
anh m`
au do. Khi do
o 132 con x
am, 154 con nu
i th`

1.2. Ph
at hin
ai to
an
. bt bin trong b`

15

,
,
, ,
, , ,
v`
a 179 con d
ng l`
a 0, 1 v`
a
a 179 cho 3 tuong u
o. Nhung s du cua 132, 154 v`
,
,
. p lai
c
2, ngha l`
a lai
ac s du da
o.
`y d
u c
. ga
. d
,
`n la
`n x
`n la
`n m`
. p con tha
Nu mt
am ga
au do, th` ch
ung d`ng
. con tha
,
,
`n la
`n x
`n la
`n nu
ta c
th`oi d
au th`
anh nu. Khi d
o 132 tha
am, 157 tha
i m`
o
,
,
,
,
,
`n la
`n do. Ly nhung s trn chia cho 3 cho s du tuong u
,ng l`
v`
a 176 tha
a
,
,
,
,

. p ca ba kha n
0, 1 v`
a 2, ngha l`
a lai
ang cua s du.
. ga
,
,
`n la
`n nu v`
`n la
`n do ga
. p nhau, th` ch
Nu con tha
a tha
ung c`
ung di m`
au
`n la
`n x
`n la
`n nu v`
`n la
`n
c
th`
anh x
am. Khi do
o 135 tha
am, 154 tha
a 176 tha
,
,
,
,
,
,
,

`n la
`n trn chia cho 3 tuong u
ng l`
do. S du cua nhung s tha
a 0, 1 v`
a 2, vn
,
,
c
o d`y du c
ac s du khi chia cho 3.
,
,,
,
, ,
Bt bin o d
a d`
u thay di m`
au nhu th n`
ao th` s du cua c
ac s
y l`
,
,,
`
`
n la
n chia cho 3 d`u c
luo. ng tha
o d`y du ba s 0, 1, 2.
,
,
,
,
`n la
`n trn dao l`
S luo. ng tt ca tha
a 133 + 155 + 177 = 465 l`
a mt
. s
,
,
,
`n la
`n d
chia ht cho 3. Nu tt ca tha
ung mt
au th` s du cua s
`u c`
. m`
,
,,
,
,
`n la
`n m`
,ng l`
luo. ng tha
au x
am, nu v`
ad
a 0, 0, 0. i`u
o chia cho 3 tuong u
,
,
,
,
,
n`
ay v l v` c
ac s du phai c
o d`y d
ac s du n`
ay khi chia cho 3. Nhu vy
u c
.
, ,
, ,,
cu tra l`oi l`
a khng th d
uo. c.
,
,,
,,
,
Vic
ai to
an
. t`m ra dai
. luo. ng bt bin cua mt
. di tuo. ng trong du kin
. b`
,
,
,
,
,
tht
ai to
an loai
ay. Su. da dang
cua c
ac b`
ai to
an
. lo. i hai
. khi giai nhung b`
. n`
.
,,
,,
n`
ay duo. c lit
. k duoi dy:
,
,,
vung trong bang 8 8 d
o
V du. 1.9. Tai
uo. c vit mt
. s nguyn. Ta c
. mi
,
,
,
,
,
,

th chon
. c 4 4 v`
a t
ang tt ca s trong bang nho
. bt k` bang nho 3 3 hoa
,
,
,,
,
,ng s chia ht cho 3
ln 1. V
oi c
ach l`
am nhu vy
o th nhn
uo. c nhu
. liu
. c
. d
,
,
,
,
,u han
trong tt ca vung cua bang 8 8 sau mt
. s hu
. l`n thu. c hin
. khng

,
,
,
,
`,i giai. Khng, khng bao gi`o, c
Lo
o kt qua nhu vy.
. Tht
. vy,
. ta tnh tng
,,
,,
h`nh vung
c
ac s trong c
ac duo. c gach
. cheo trong h`nh 1.3. Boi v` mi

,,
Chuong 1. Nguyn l bt bin

16

,
h`nh vung co, 3 3 chu
,a 12 gach
,a 6 hoa
. c 9
co 4 4 chu
on mi
. cheo, c`

,
th` sau mt
ac tng c
ac s
. thao t
trong c
ac gach
o s
. cheo chia cho 3 c
,
,
,
du khng d
i. V` th nu ngay t`u d`u
,, ,
tnh duo. c tng khng chia chia ht cho
,
3, th` trong nhung gach
cheo lun
.
,
,

,a
c
lun chua nhung m`
a trong do
o chu
,
,
c
ac s khng phai bi
. cua 3.
,
,
C`
ung l lun
o th
oc
. nhu vy
. ban
. d
. c
,
giai b`
ai tp
. 1.19.
H`nh 1.3
,
V du. 1.10. Trn mt
o 8 8 vung bao g`m 32 tra
ng
. bang vung c
,
,
,
,
,
o th thay tt ca tra
ng th`
anh d
a
v`
a 32 d
en v`
en. Nu mt
. ngu`oi choi c
,

d
anh tra
ng c`
ung mt
uc trong mt
ang hoa
. c mt
o th
en th`
. l
. h`
. ct
. bt k`, th` c
,
,
,
,
,
,,
,
,u han
ng mt
thu. c hin
on d
i nhu vy
trn bang ch c`
u
. hu
. buoc thay d
. d
.

d
en hay khng?
,
,,
`,i giai. Khng. Nu c
. c mt
ng k den trong mt
Lo
od
ang hoa
c
u
. h`
. ct
. truo
,
,
,
,
`

khi thu. c hin


. thay di, th` sau khi thu. c hin
. mt
. ln thay di, s den
,
,
. c ct
hoa
se l`
trong h`
ang do
a 8 k, su. thay di s den l`
a (8 k) k =
. do
, ,
,
,

n le cua s nhung
n, tnh cha
8 2k den trn bang. V` 8 2k l`
a mt
. s cha
,
,
,
giu, nguyn tru,o
t d
,c c
ung nhu sau thu. c hin
den vn
`u
. thay di. Do ba
, ,
,
,,
c
o 32 d
on lai
c
en, nn khng th ch c`
. mt
. den trn bang tai
. mt
. buo
,

.
bin d
ao d
i n`
o

V du. 1.11.
,
,,
Cho mt
o canh
10 cm, d
anh mt
uo. c chia ra th`
. bang h`nh vung c
.
. 100
, ,
,
,
nht
25 h`nh chu
vung nho v
oi canh
1cm. Ngo`
ai ra ta da
. t ln d
o
.
. nhu nhau
,,
h`nh chu
, nht
c
o chi`u cao 4cm v`
a chi`u rng
anh
uo. c chia ra th`
. 1cm, mi
. d

1.2. Ph
at hin
ai to
an
. bt bin trong b`

17

,
,
,ng h`nh chu
, nht
p d
. t nhu
4 vung c
o canh
l`
a 1cm. C
o th sa
a
. trn bang
.
,
,
h`nh vung sao cho ch
ung phu to`
an b. bang vung hay khng ? (Khng chp
,
,
,
, nht
nhn
o h`nh chu
ao l`i ra khoi canh
cua bang).
. c
. n`
.
,
,
`ng m`
`,i giai. Ta t bang vung ba
Lo
au
,

ng sao cho nhu h`nh 1.4. Ta nhn


den tra
.
,,
ng. Ta ch
a 75 tra
duo. c 25 den v`
u y
,
,
,
. t nhung h`nh chu nht
l`
ad
a
. trn bang
,
vung cua
vung sao cho mi
h`nh
,
,
i mt
chu nht
ung vo
ao
. tr`
. vung n`
,
,
,
cua bang vung. H`nh chu nht
do
ay
. n`
,
. c l`
. c l`
se phu ln hoa
a 2 hoa
a 0 vung
,
,
. t tt ca 25 h`nh
suy ra khi d
den. T`u do
a
H`nh 1.4
,
,
chu nht
ung se
. trn bang vung, ch
,
,
n nhu,ng vung den. Bo,,i v` s lu,o.,ng cua vung
phu kn mt
. s cha
,
, ,
n. Nhu, vy
t l`
den da
a 25, n
o khng phai l`
a mt
. s cha
. khng th phu
`ng 25 h`nh chu, nht
cho.
ba
. trn h`nh vung da
,
,,
,,
V du. 1.12. Cho n l`
a mt
ac s 1, 2, ..., 2n
. s nguyn duong le. Ngu`oi ta vit c
,
,`,
nguoi ta ly hai s bt k` a, b thuc
ln bang. Sau d
ay trn, x
oa ch
ung d
o
i
. d
,
,
,
|a b|. Chung minh ra
v`
a vit v`
ao d
`ng sau mt
o
. s l`n thu. c hin
. nhu vy,
.
,

mt
s
l
e
s
e
c`
o
n
l
ai
cu
i
c`
u
ng.
.
.
,
,
,
,
,
,,
,
,
`,i giai. K hiu
Lo
a tng cua tt ca nhung s trn bang (chua x
oa). Khoi
. S l`
,
,
bu,o
,c S bi. giam
d`u S = 1 + 2 + + 2n = n(2n + 1) l`
a mt
. s le. Sau mi
,
n. Nhu, vy
n le
di 2 min(a, b) = |a + b| |a b|, dy l`
a mt
. s cha
. tnh cha
,
,
,
,,
cua S khng d
a tr`nh giam d`n ta c
o S 1 (mod 2). Khoi d`u
i. Trong qu
,
,
,
S l`
a mt
uc se c
ung l`
a mt
. s le. Nhu vy
. kt th
. s le.
,
,
V du. 1.13. Tai
ac dnh cua mt
ac l`i, ta ghi c
ac s: 8, 3, 12,
. c
. h`nh luc
. gi
,
,
,,
,
l`n thu.,c hin
1, 10 v`
a 6. Mi
o th thm hoa
. c b
ot v`
ao hai
i ngu`oi ta c
. thay d

,,
Chuong 1. Nguyn l bt bin

18

,
,
,
,
d
uy y c`
ung mt
o th
nh lin tip t`
. s. Sau mt
. s l`n thu. c hin
. nhu vy,
. ta c
,,
,,
,,
, ,
d
au s: 5, 2, 14, 6, 13 v`
a4d
ao s
au s trn
at
uo. c s
uo. c thay l`n luo. t thu tu. v`
. d
khng?
,
, ,
,
,
,,
`,i giai. Gia su, tai
, tai
Lo
ao do
ac dnh cua luc
ac c
o c
ac s:
i n`
. mt
. luo. t d
. c
. gi
,
a1 , a2 , a3 , a4 , a5 , a6 . Xet tng S = a1 a2 + a3 a4 + a5 a6 . Khi thm
,
giu, nguyn gi
v`
ao hai s canh
nhau c`
ung mt
o r`
ang l`
a tng S vn
a
.
. s th` r
,
,
,`,
,

tri.
ai to
an, tng S = 20 trong d
ay s kt qua v`
a d
ay
. Trong truong ho. p b`
, ,
,,

`
`
s khoi d
ung nhau. Vy
ach chuyn tu 8, 3, 12, 1, 10, 6
u tr`
. ta di t`m mt
. c
I1,II1

, nht v`
, hai tr`u, d
th`
anh 5, 2, 14, 6, 13, 4. K hiu
l`
a s thu
a s thu
i 1:
.
8, 3, 12, 1, 10, 6
III+2,IV+2

7, 2, 14, 3, 10, 6

5, 2, 14, 6, 13, 4.

IV+3,V+3

I1,II1

7, 2, 12, 1, 10, 6

5, 2, 14, 3, 10, 4

I2,VI2

,
m
V du. 1.14. Ta sa
p d
. t ba m
ay tu. d
ay nhn
a
ng
. trn mt
. dy chuy`n, mi
.
,
, ,
,

d
oc
m
t
t
m
th
e
c
o
ghi
hai
s
nguyn
v`
a
d
u
a
ra
m
t
t
m
th
e
m
o
i
theo
nguyn
.

.
.
,
,
,

ta
c sau: Sau khi d
oc
th
e
c
o
ghi
c
a

p
s
(a,
b),
m
a
y thu nht (I) in ra the c
o
.
.
,
,
,
ca
. p s (a b, b), m
ay thu hai (II) in ra the c
o ca
. p s (a + b, b) v`
a m
ay thu
,
,
,,
ba (III) in ra the c
o ca
. p s (b, a). Khoi d`u ta cho the c
o ca
. p s (19, 87). C
o
,
,
, ,
,
,
,
,
,
th d`
ung ba m
ay tu. d
oi
ng
nhn
uo. c the v
. trn theo mt
. thu tu. bt k` d
. d
ca
. p s:
a) (41, 14)?

b) (18, 81)?

,
`ng c
`,i giai. T`m bt bin trong b`
Lo
ai to
an n`
ay ba
ach quan s
at h. thng in
,
,
,
,
`ng ca ba m
. p s da
cho v`
the v`
a nhung ca
a kt qua. Ta thy ra
ay d`u thay
,
,
,
,
,,
,n nht cua n
,
di s in ra nhung uo
c s chung lo
o khng thay di do ta nho
,
`ng u,o
,c
dn thut
an Euclid v`
a c
ach tnh s in ra cua ba m
ay. Ta thy ra
. to
,n nht: (19, 87) = 1 v`
s chung lo
a (18, 81) = 9 l`
a hai s kh
ac nhau, d`
u kt
,
,
,
,
,,
,
ho. p ba m
ay nhu th n`
ao th` kt qua b) c
ung khng th xay ra. C`
on tru`ong

1.2. Ph
at hin
ai to
an
. bt bin trong b`

19

,
, ,
,
i c
ho. p a) tnh bt bin thoa m
ac s (14, 41)
an v` (14, 41) = 1. Vy
. the vo
,
,
,, `
,
,
,
ng c
c
o th nhn
ach kt ho. p c
ac m
ay tu. d
trn. Nhung ta phai
ng
. duo. c ba
.
,
,
,,
,
,i l`
ch c
ac buo
c thu. c hin
a c
ach giai tron
ai to
an. Tt nhin c
o rt
. mo
. ven
. b`
,
,
,
,
,
,
`

`
nhiu c
ach thu. c hin
ay d in ra kt qua, ta ch cn ch ra mt
. kt ho. p ba m
.
,
,,
,

n l`
phuong a
ad
on c
o ti uu hay khng d`
anh cho ban
K hiu
u (c`
. doc).
.
. s
,
m
ay trn m
ui tn l`
a thu. c hin
ay n`
ay:
. in theo s m
I4

III

III

III

(19, 87) (87, 19) (11, 19) (19, 11) (8, 11) (11, 8)
III

III

III

11

II

III

(3, 8) (8, 3) (2, 3) (3, 2) (1, 2) (2, 1) (13, 1)


II

III

II

(1, 13) (14, 13) (13, 14) (41, 14).


,
,
Nhung b`
ai to
an v` thay di vi. tr c
ac s trong mt
ung c
o c
ach
. b. s c
,
,
,
,

giai t`m bt bin tuong tu. .

,,
, ,
,ng s 1, 2, 3, ..., n d
. Mt
V du. 1.15. Nhu
p xp theo mt
ao do
uo. c sa
. thu tu. n`
.
,
,
,

phep bin d
ad
nhau
trong
b

s
c
o
s
a

n.
Ch
u
ng
i l`
i ch bt k` hai s canh
.
.
,
,
,
,

minh ra
`ng nu ta thu. c hin
s
l
e
l
n
ph
e
p
bi
n
d

i
nhu
vy,

.
. th` lun lun
,
,,
,
nhn
ac v
oi b. s ban d
ac vi. tr cua c
ac s 1, 2, ..., n.
uo. c mt
`u v` c
. d
. s kh
,
,
`,i giai. Ta k hiu
Lo
a mt
an vi. cua b. s 1, 2, ..., n. Ta n
oi
. a1 , a2 , ..., an l`
. ho
,
`ng hai s ai v`
a aj trong ho
an vi. n`
ay l`
a nghich
ra
ao nu i < j th` ai > aj .
. d
,
Khi ta thay d
nhau trong ho
an vi,
a ch
ung ta t
ang
i hai s canh
. ngha l`
.
,
,
,
,
,
,
,
. c giam s luo. ng nghich
hoa
ac nhu
ao di 1. Ta thu. c hin
. d
. s le l`n thao t
,
, ,
,
n le cua nhu,ng s nghich
bin d
ngha
vy,
a
i tnh cha
o
. dao, di`u d
. th` ta d
, ,,
,
,
,
,
,
`ng c
ng minh tuong tu. ta c
thay di ho
ach chu
o th mo
l`
a ta da
an vi.
. Ba
,
,
rng
b`
ai to
an n`
ay nhu trong b`
ai tp
ay).
.
. 1.20 (trong ph`n cui cua tit n`

,,
,ng bn t kh
V du. 1.16. Tai
ac nhau trong mt
ung
. nhu
. tuyn du`ong t c`
,`,

xut ph
at mt
uc 25 t theo c`
ung chiu (mt
a mt
. l
. tuyn duong t l`
. con
, ,,
,
,`,

ng t n`
duong duy nht v`
a khep kn). Theo nguyn ta
c, nhu
ay c
o th vuo. t
,, ,,
nhau, nhu,ng khng d
,ng t n`
qua ln
a hai xe. Nhu
ay c`
ung kt
uo. c vuo. t qu

,,
Chuong 1. Nguyn l bt bin

20

,
,
xut ph
at. Chung minh ra
`ng
th
uc mt
ong d`ng th`oi tai
. bn t m`nh da
. v`
,,
,
trong to`
an b. th`oi gian s l`n xe vuo. t nhau l`
a mt
n.
. s cha
,
,
,
`ng m`
`,i giai. Ta k hiu
Lo
au v`
ang, nhung
. mt
. trong nhung chic t ba
,,
,
, tu., xut ph
nh s 1, 2, 3, ..., 24 theo thu
chic c`
on lai
at t`u vi. tr xe
. duo. c da
,,
l`n c
a n
o l`
a xe ghi s v`
a di sau xe m`
au
m`
au v`
ang. Khi mi
o xe vuo. t m`
,
,
,
,
,
,
v`
ang, th` d
ay s thay d
ay
i hai s li`n nhau trong tru`ong ho. p l`n xe vuo. t n`
khng tham gia xe m`
au v`
ang.
,,
,,
,
vuo. t chic t m`
Ta xet tru`ong ho. p mt
ao do
au v`
ang. Nu
. chic xe n`
,
,
,
,
,,
,
,
truoc th`oi dim vuo. t nhung s trong bang l`
a mt
an vi. a1 , a2 , ..., a24 , th`
. ho
,
,,
`ng
sau khi vuo. t ch
ung tao
anh ho
an vi. a2 , a3 , ..., a24 , a1 . Ta gia thit ra
. th`
,
, ,,
an to`
an lin tip 23 l`n thay d
ay s: a1 , a2 , ..., a24
su. vuo. t nhau ho`
i d
a2 , a3 , ..., a24 , a1 a3 , a4 , ..., a24 , a1 , a2 ... a2 , a3 , ..., a24 , a1 .
,
,,
,
Nu xe m`
au v`
ang thu. c hin
an vi. a1 , a2 , ..., a24
. mt
. l`n vuo. t th` t`u ho
,
,,
,
ta nhn
an vi. a24 , a1 , a2 , ..., a23 , t`u ho
an vi. n`
ay c
ung bin di 23
. duo. c ho
,
l`n thay di d
ay s.
,
,
,,
,
,
l`n vu,o.,t d
Nhu vy
u trong tru`ong ho. p n`
ao th` mi
ua dn mt
. d`
. s le
,
,
,
,
,,
,,
,
l`n chuyn d
ay s. Nu tru`ong ho. p tng qu
at s l`n vuo. t l`
a le, th` c
o
i d
,
,
,
,
,
,
i b. s
s le l`n chuyn di. Do b`
ai to
an 1.15 nhn
ay s kh
ac vo
. duo. c d
,,
,
,
i vi. tr c
ban d`u (kh
ac vo
ac xe l
uc ban d`u), nhung cui c`
ung c
ac xe lai
. tro
,,
n l`n c
v` vi. tr ban d`u, nn ta c
o s cha
ac xe vuo. t nhau.
,,
Rt nhi`u dang
ai to
an t`m d
ai
. b`
. luo. ng bt bin, v du. sau dy tnh bt
,
,
,,
bin duo. c cho ngay trong gia thit cua d
ai:
` b`
,
,
,
,
V du. 1.17. Cho d
. t pha
ng v
oi 0 < b < a, ta du. ng
im S = (a, b) trn ma
,
d
ay c
ac d
c sau dy:
im (xn , yn ) theo quy ta

x0 = a,

y0 = b,

xn+1 =

x n + yn
,
2

,
,
,,
,
T`m gi
oi han
ac d
im sinh ra o trn.
. cua c

yn+1 =

2xn yn
.
x n + yn

1.2. Ph
at hin
ai to
an
. bt bin trong b`

21

,
,,
,
`,i giai. B`
,c
ai to
an n`
ay d t`m thy dai
Lo
ac cng thu
. luo. ng bt bin. T`u c
,
,
,i moi
trn suy ra xn+1 yn+1 = xn yn vo
anh giam ch s dn
. n. Ta tin h`
,,
,
,
,
i moi
xn yn = ab vo
ad
ai
. n. y chnh l`
. luo. ng bt bin. Khoi d`u ta d`u
,i moi
c
o y0 < x0 . Mi quan h. n`
ay c
ung bt bin vo
. n: yn < xn . Tht
. vy,
.
, ,,
,
,
,
,
`
ng minh ba
i mt
ng phuong ph
ta chu
ap quy nap,
ao
. gia su yn < xn vo
. n n`
,
,
,
,
`
`
ng minh ra
ng yn+1 < xn+1 . V` trung b`nh diu h`
, ta phai chu
do
oa thu. c su.
, ,
nho hon trung b`nh cng,
nn
.
xn yn xn yn
xn yn
0 < xn+1 yn+1 =
<

xn + yn
2
2
,
,
,
,
,
i moi
vo
ay giam thu. c su. , nn ta c
o lim xn = lim yn = x. Nhu
. n. Do d
n+
n+

2
vy,
a x = ab.
. x = ab, ngha l`
,
,,
ai
ai to
an trn gi
up ta rt nhi`u, ta phai d`
ung
. luo. ng bt bin trong b`
,
, ,
,
,
,
,
dn hai dai
ai to
an kiu th n`
ay c
o th t`m
. luo. ng bt bin d giai. Nhung b`
,
,
thy rt nhi`u trong chuong 2 cun s
ach [7].

` tp
Bai
.
,
,
,
`ng nhu,ng c
p dung
Ba
ach l`
am cua c
ac v du. trn ta c
o th a
v`
a giai c
ac
.
,
,
,

b`
ai tp
ai c
o dua ra go. i y, c`
on nu khng c
o go. i y, c
ac
. sau dy. Mt
. s b`
,
,,
,
,
,
,

`
c v`
ban
ai mu o phn truo
a thu. c hin
ach giai ho`
an to`
an
. xem lai
. nhung b`
. c
,,
,
tuong tu. .
,
,
. 1.18. Trn bang d
(+) v`
a mt
en vit mt
. s du cng
.
. s du tr`u ().
Cho phep x
oa d
a nu ch
ung c`
ung du ta vit lai
i hai du bt k` v`
. du cng
.
,
,
`
u

a
ng
d
(+), c`
on nu ch
ung kh
ac du ta vit lai
d
u
tr
u
().
Ch
u
ng
minh
r
.
,
,
,
c lin tip thu. c hin
cui c`
ung khng phu. thuc
ao c
ac buo
ac trn
. v`
. thao t
,
,
,
c).
(Go. i y: Xem lai
ai h
ai kh ph`n truo
. b`
,
,
, ,
,
. 1.19. T`u bang trong gia thit cua v du. 1.9 v`
a c
ac thao t
ac nhu vy
o
. c
,
,,
,
,

a mt
n n`
nhn
ao khng? (Go. i y: Xem lai
ach
. duo. c bang khng chu
. s cha
. c
,
giai v du. 1.9).

22

,,
Chuong 1. Nguyn l bt bin

,
`ng kt lun
,ng minh ra
ng, nu trong
. 1.20. Chu
on d
u
. cua v du. 1.15 c`
,
,
,
,

cho. (Giai nhu v


phep bin d
an vi. da
i ta di ch hai s bt k` trong ho
du. 1.15).
,
,
,
. 1.21. Ta xe mt
anh 10 manh, trong mt
o
. ming giy th`
. s manh giy d
,
,
,
,
,
,

ta lai
anh 10 manh nho, v`
a tip tuc
o th
. xe mi manh th`
. nhu vy.
. Hoi ta c
,
,,
nhn
ach l`
am trn 1975 manh giy khng?
. duo. c theo c
,
. 1.22. Trn bang ta vit c
ac s 1, 2, ..., 1975. Cho phep x
oa hai s bt k`
,
,
,
s du cua phep chia tng hai s n`
v`
a vit v`
ao do
ay cho 13. Sau mt
. s l`n
,
,
,
,
,
,
c
l`
am nhu vy
on lai
o th l`
a nhung s
o
. trn bang ch c`
. mt
. s, hoi s d
n`
ao?
,
,
,
trong mang
. c du tr`u. Ta
. 1.23. Trong mi
co 4 4 ta vit du cng
. hoa
,
,
,
,i du ngu,o.,c lai.
. c theo ct
thu. c hin
ang hoa
i d`ng th`oi theo h`
. bin d
. vo
. S
,
,
,
,
,,
,
,
,
,
luo. ng du tr`u nho nht ta c
o th tnh duo. c t`u mang ban d
a c
ac mang
`u v`
,
,
,
,
,
,
. c trung n`
goi
bin di sau do
a s d
. c trung cua bang n`
ay. S d
ay c
o th
a
a
. l`
,,
,
nhn
a tri. nhu th n`
ao?
. duo. c gi
,,
ng v`
a 20 kin den.
. 1.24. Trn mt
on c
o 30 con kin: 10 kin tra
u`ong tr`
. d
,
,
Cho phep di ch hai con kin m`
a giua ch
ung c
o ba con kin kh
ac. Hai vi.
,
, ,
,
,
,
,
,
,
tr cua hai con kin bt k` duo. c goi
a tuong d
ung c
o th d
uong nu ch
i
. l`
,
,
,
,
cho nhau sau mt
nhu trn. Hoi t`n tai
ch
. s l`n thu. c hin
. phep di ch
.
,
,,
,,
,
bao nhiu vi. tr khng tuong duong cua nhung con kin?
,
,,
, tu., n`
. 1.25. Nhung s 1, 2, ..., 2003 duo. c vit theo thu
ay. Mt
. phep bin
,
. t lai
chim
di l`
a chon
ung v`
ad
ac vi. tr ch
ung d
a
a
. bn s bt k` trong ch
. c
,
,
, ,
,
,
,
,
,
`
tu. c
ng phep bin d
nhung theo thu
ac s nguo. c lai.
o th ba
ay d thu. c
i n`
. C
,,
p xp th`
hin
anh 2003, 2002, ..., 2, 1 khng ?
uo. c vic
. d
. sa
,
,,
l`n thu.,c hin
. 1.26. Cho bn s 4, 5, 6, 7. Mi
i ngu`oi ta vit 4 s
. thay d
,
s mo
`ng trung b`nh cng
,i thay v`
,i ba
c
mo
ao bn s c
u: Mi
cua ba s d
o
a
.

1.2. Ph
at hin
ai to
an
. bt bin trong b`

23

,
,,
,,
,
`ng sau mt
,ng minh ra
truo
c. Chu
i ta khng bao gi`o dat
uo. c
. s l`n thay d
. d
,
,
,
`ng tng cua bn s a, b, c, d v`
nh
om 4 s: 5, 6, 7, 3 (Go. i y: Ch
u y ra
a bn s
a+b+c b+c+d c+d+a a+b+d
`ng nhau).
,
,
,
lun lun ba
3
3
3
3
,
,
,
,,
,
ng phn b mt
. 1.27. Trn mt
im. Giua moi
im
. du`ong tha
. s d
. hai d
,
,
,
nh du thm mt
giua moi
lin tip ta da
o
im lin tip
. dim. Sau d
. hai d
,
,
,,
nh du thm mt
trong c
ac d
a vn vn. S
im nhn
uo. c ta lai
. d
. da
. dim v`
,
,
,
,
,
,,
,

n hay le sau khi ta thu. c hin


ach thm
luo. ng nhung dim nhn
. duo. c cha
. c
,
, ,
,
dim trn 1000 l`n (Tra l`oi: S le).

,
,
,,
. t hai m
. 1.28. Ta da
ac thao t
ac sau d
ay
ay tu. d
o th l`
am d
y: M
ng
uo. c c
. c
,
,
. c A + 15 theo
I cho the v`
ao c
o s nguyn A in ra trn n
o tng A + 10 hoa
, ,
,
, `
su. d
ay II cho the v`
ao c
o s nguyn A in ra trn n
o hiu
iu khin cua ta. M
.
, ,
,
, `
`ng
. c A 15 theo su. diu khin cua ta. Cho the v`
A 10 hoa
ao c
o s 0, ba
,
,
,
,,
. c b) 123 d
hai m
ay tu. d
trn ta c
o th in ra the c
o s: a) 125 hoa
ng
uo. c
.
, ,
,
,
khng? (Tra l`oi: a) C
o th; b) Khng th).

,
,
. 1.29. Chia mt
anh nhung vung c
o canh
1cm. N
o
. h`nh chu nht
. th`
.
,
,
,,
,
duo. c phu boi to`
an b,
a khng th`
oi ra mep ngo`
ai,
. khng ch`ng ln nhau v`
,
,
,
,
,
,
`
ng s luo. ng da
bit nhung manh c
ba
o hai dang:
Nhung manh h`nh vung
.
,
,
c
o canh
2cm v`
a manh h`nh chu nht
o chi`u d`
ai 4cm v`
a rng
1cm. Nu
.
. c
.
,
,
,
`
`ng c
ng mt
ach
ta thay mt
a ba
. manh h`nh chu nht
. v`
. manh h`nh vung ba
,
,
,
,
,
,
,

chuyn ch nhung manh c`


on lai,
o duo. c phu
. th` h`nh chu nht
. ban d`u c
,
,
,
,
kn khng? (Go. i y: T h`nh chu nht
ai 1.11. Ch
u y mt
. nhu b`
. manh h`nh
,
,
,
n vung tra
ng v`
n vung d
chu nht
a cha
on manh h`nh
en, c`
. phu s cha
,
,
ng. Tra l`o,i: Khng du,o.,c).
vung phu 1 vung den v`
a ba vung tra

24

,,
Chuong 1. Nguyn l bt bin

,
,,
ba
` ng dai
1.3. Giai toan
. luo. ng bt bin
,
,,
`ng c
Ba
ach ph
at hin
ai to
an ta
. ra nhung dai
. luo. ng bt bin trong b`
, ,
c
o th giai nhi`u b`
ai to
an. Tuy nhin nu khng luyn
o v`
a
. tp
. th` rt kh
,
, ,
,
,,
khng c
o phuong ph
ap s
ang sua d giai. Tit n`
ay ta tip tuc
. t`m hiu thm
,
,
,
nhung c
ach t`m dai
ai to
an.
. luo. ng bt bin trong b`
,
,
,
,,
,,
,
,ng vin soi nho c
V du. 1.30. C
o ba d
o s luo. ng tuong ung
ng soi g`m nhu
,
,
,
,,
l`
a 19, 8 v`
a 9 (vin soi). Ta d
a chuyn mt
uo. c phep chon
ng soi v`
. hai d
. vin
, ,
,
,
,
,

`
ng d
chon
soi cua nhu
am
ng soi d
a
ng soi thu ba. Sau mt
. sang d
. s ln l`
,
,
,
,

`
nhu vy
o kha n
ang tao
o 12 vin soi khng ?
ng soi d
u c
. th` c
. ra moi
. d
,
,
, ,,
`,i giai. Khng. a
,ng l`
. t s vin soi trong ba d
Lo
a a, b v`
a c.
ng soi tuong u
,
,
,
,
Ta xet s du chia cho 3 cua nhung s n`
ay. Khi xut ph
at, nhung s d`ng
,
,
,
,
du n`
ay l`
a 1, 2, 0. Sau mt
ay l`
a 0, 1, 2 v`
. l`n chon
. thay di, nhung s du n`
,
,
,
,
,
,
,
ba. Nhu vy
hai dng soi c
o su. chuyn mt
. vin soi dn dng thu
. nhung
,
,,
, tu., kh
,i nhu,ng thu
s du lun lun l`
a 0, 1, 2 vo
ac nhau (d
ai
. luo. ng bt bin).
,
,
,
, ,,
tt ca c

Do d
ac dng soi d
o 12 vin soi l`
a khng th d
o
`u c
uo. c (v` khi do
,
,
,
s du cua ba dng soi l`
a 0, 0, 0, v l).

,,
,
,
, ,
V du. 1.31. Hai ngu`oi choi mt
o choi v
oi hai d
ng keo.
. tr`
. ng keo
. thu
,
,,
ngu,`o,i cho,i d
nht c
o 12 c
ai v`
ad
o 13 c
ai. Mi
ng keo
uo. c ly hai
. thu hai c
,
,
`, mt
`, d
c
ai keo
. c chuyn mt
ai keo
ng keo
ng thu nht
. tu
. trong hai d
. hoa
. c
. tu
,
,,
,
,,
,
,ng thao t
ac trn
sang d
ao khng th l`
am d
ng thu hai. Ngu`oi choi n`
uo. c nhu
,
,
,
,`,
, , ,
,
`

coi nhu l`
a thua. H
ay chung minh ra
ng nguoi choi di luo. t thu hai khng th
,
,,
c
thua. Ngu`oi d
o th tha
ng khng?
o
,
,
`,i giai. Ta k hiu
, hai tr`u,
Lo
a gi
a tri. tuyt
i cua s keo
. S l`
. d
. trong dng thu
,
,
l`n cho,i S se giam
, nht. Kho,i d
di dng thu
`u, S = |13 12| = 1. Sau mi
,
, ,

. c t
hoa
ang ln 2. Nhu vy,
o dang:
1, 3, 1, 3, .... Mi
. s du cua S chia cho 4 c
.
,
, , ,
,
,
l`n sau khi ngu`oi thu
nht choi, s du cua S chia cho 4 lun lun l`
a 3. Ta

,
,,
`ng dai
1.3. Giai to
an ba
. luo. ng bt bin

25

,
,,
`ng ngu,`o,i cho,i bi. thua khi v`
thy ra
a ch khi khng c`
on c
ai keo
ao o dng
. n`
,
,,
,
, nht v`
, hai, khi do
S = |1 0| = 1. Nhu
thu
a ch c`
on mt
ai keo
. c
. o dng thu
,
,,
,
,,
, ,
,

vy
hai lun lun c
o th thu. c hin
ach choi, do d
o
. ngu`oi choi thu
. duo. c c
,`,
khng thua.
nguoi do
,
,
,,
,,
`ng, hoa
. c l`
. c l`
Ta thy ra
a tng s keo
a s keo
. o hai dng giam di hoa
. o
,
,
,
,,
,
,
, nht giam d
ngu`oi choi
dng thu
o choi phai c
o kt th
uc, do d
i, nhu vy
o
. tr`
,
ng.
, hai phai tha
thu

,
,
th`
V du. 1.32. Mi
anh vin cua mt
o nhi`u nht l`
a ba d
i thu
. cu lac
. b. c
, ,,
,,
nhau). Chu,ng minh ra
trong cu lac
a tuong t
ac ln
`ng
y l`
. b. (di thu o d
,
,
,

ng th`
nhu
anh vin cua cu lac
o th chia th`
anh hai nh
om sao cho mi
. b. c
,
nh
th`
anh vin trong mi
om c
o nhi`u nht mt
ung nh
om.
i thu trong c`
. d
,
nhin ta chia nhu,ng th`
t d
`,i giai. Khi ba
Lo
anh vin trong cu lac
`u, ngu
.
,

. p di thu trong c`
b. th`
anh hai nh
om. K hiu
a s c
ac ca
ung nh
om. Nu
. S l`
,

mt
anh vin c
o t nht hai di thu trong c`
ung nh
om, th` th`
anh vin
. th`
,
,,
`

n`
ay c
o nhiu nht mt
om kh
ac. Th`
anh vin n`
ay d
uo. c di
. di thu trong nh
,
,
chuyn sang nh
om kh
ac, ta se giam S di t nht 1. V` S l`
a mt
. s nguyn
, ,
,,
,
,
khng m, n
o khng th giam m
ai duo. c. Nhu vy
. sau mt
. s huu han
. l`n
,
,
,
,
th`
chuyn di, mi
anh vin c
o th c
o nhi`u nht mt
ung
i thu trong c`
. d

mt
om.
. nh
,,
,,
,ng minh b`
: Phuong ph
Ch
uy
ap chu
ai to
an trn goi
a phuong ph
ap xung
. l`
, ,
,
,
,
,
`ng ta khng th giam m
dc v han.
o ch ra ra
ai s luo. ng khi n
o ch c
o
. N
,
,
,
,
huu han
a tri. (ban
o th xem k phuong ph
ap n`
ay trong cun s
ach
. gi
. doc
. c
[9]).
,,
,
,ng s a, b, c, d. Ta thay ch
V du. 1.33. Ta ba
t d
oi nhu
ung boi c
ac s
`u v
,
a 0 = a b, b 0 = b c, c 0 = c d, d 0 = d a. Ta thu. c hin
a tr`nh n`
ay
. qu
,
`

1996 ln. C
o kha n
ang d
ung A, B, C, D sao cho |BC AD|, |AC
n s cui c`
,

ng s nguyn t?
BD|, |AB CD| l`
a nhu

,,
Chuong 1. Nguyn l bt bin

26
,
,
`,i giai. Tra l`o,i: Khng.
Lo
,
,,
. p d
Bn buo
c la
`u tin cho kt qua
a-b
a-2b+c
a-3b+3c-d
2a-4b+6c-4d

b-c
c-d
d-a
b-2c+d
c-2d+a
d-2a+b
b-3c+3d-a
c-3d+3a-b
d-3a+3b-c
2b-4c+6d-4a 2c-4d+6a-4b 2d-4a+6b-4c
,
,,
n. Nhu, vy
. p tt ca c
sau 4 buo
Do d
c la
ac s d`u l`
a s cha
o
. sau 1996
,
,

. p tt ca c
phep la
ac s l`
a bi
. cua 2. V` th |BC AD|, |AC BD|, |AB CD|
,
,

tt ca l`
a bi
o s n`
ao l`
a s nguyn t.
. cua 4, suy ra khng c

,
,
,
V du. 1.34. Trong mt
o n n v
oi n l`
a mt
. bang vung c
. s le. Trong
,
,
ta vit +1 hoa
mi
. c 1. Goi
a tch tt ca c
ac s thuc
ang thu i v`
a bj
. ai l`
. h`
,
,
,
`

l`
a tch tt ca c
ac s thuc
ng
. ct
. thu j (i, j = 1, 2, ..., n). Chung minh ra
a1 + a2 + + an + b1 + b2 + + bn 6= 0.
,
,
,
`m o,, h`
`,i giai. Nu ta di du cua s na
, p v`
, q, nhu,ng s
Lo
ang thu
a ct
. thu
,
,
giu, nguyn. Ta xem su., thay
ap v`
a bq c
ung di du, c`
on nhung s kh
ac vn
, ,
,
,,
di cua ap + bq truo
c v`
a sau khi di du:
,,
truo
c

sau

,,
truo
c

sau

,,
truo
c

sau

,,
truo
c

sau

ap

-1

+1

-1

+1

+1

-1

+1

-1

bq

-1

+1

+1

-1

-1

+1

+1

-1

a p + bq

-2

+2

+2

-2

,
,
`ng ap + bq bin di ba
`ng c
Ta thy ra
ach thm v`
ao mt
. s bi
. cua 4. Ta xet
,
,
,
,i bang n`
bang ch c
o s +1, vo
ay a1 + a2 + + an + b1 + b2 + + bn = 2n
,
,
,
`
v`
a theo d
a mt
ay khng chia ht cho 4.
iu kin
. n l`
. s le, nhu vy
. tng n`
,
,
,
,,
,, ,
bang kh
`ng c
Boi v` mi
ac nhn
an s +1 ba
ach bin di
. duo. c t`u bang to`
,
,,
`ng tng a1 + a2 + + an + b1 + b2 + + bn
mt
a ta thy ra
. s ph`n tu v`
,
,
d`u khng chia ht cho 4 v`
a nhu vy
ay lun kh
ac 0.
. tng n`

,
1.4. Bt bin don diu
.

27

` tp
Bai
.
,
,
,,
`ng tng c
. 1.35. Moi
ac s t`u 1 d
ac s
n 1 000 000 duo. c thay ba
. s trong c
,
,
,
,
,
,
i d
am nhu vy
a tip tuc.
chu s cua n
o. Vo
ay s ta nhn
. l`
. v`
. Cui
. duo. c, lai
,,
,
,
,
c`
ung nhn
o mt
. duo. c mt
. triu
. s c
. chu s. Nhung s 1 nhi`u hon hay
,
,
,
,
,
,
,
nhung s 2 nhi`u hon? (Go. i y: H
ay su dung
s du cua mt
.
. s chia cho 9
,
,
,
,
,
,
,
,
`
ng s du cua tng c
ba
ac chu s chia cho 9. Tra l`oi: Nhung s 1 nhi`u hon.)
,
,
dnh
. t mt
. 1.36. (Hungari 1989) Tai
cua mt
a
. mi
. h`nh vung, ta d
. s
,
,
,
,,
que dim. Khoi d`u ch c
o mt
a ba d
nh v`
nh kia
. que dim trn mt
. d
,,
,`,
. t dim: Nguoi ta cho phep
khng c
o mt
ao. Buoc tin h`
anh da
. que dim n`
,
,
,
. t v`
ly mt
ad
ao hai dnh bn canh
nh v`
a
. s bt k` nhung que tai
. mt
. d
.
,
,
,
`

ng hai ln s que d
ly d
s que m`
a tng cua ch
ung ba
a
i. Sau mt
. s huu
,
,
. t dim, s dim tai
han
o th l`
a 1, 9, 8, 9 tnh
a
nh h`nh vung c
. l`n d
. bn d
,,
,,
`
`
`
`
. c l`
theo chiu kim d
a tnh theo nguo. c chiu kim d
ng h hoa
`ng h` duo. c
khng?
,
,,
, tu., n`
p xp theo mt
trn mt
. 1.37. Nhung s 1, 2, ..., n duo. c sa
ao d
o
. thu
.
,
,
,
,
,
,`,

ng minh
ng. Ta c
duong tha
o th chuyn di hai s bt k` canh
nhau. Chu
.
,
,
,
,
`

,i d
ng mt
ra
ay s lun lun kh
ac vo
ay
. s le nhung chuyn di tao
. ra d
,,
,
,

`
tu. ban du.
p xp theo thu
duo. c sa

,
1.4. Bt bin do
n di
u
.
,,
,,
,,
Theo d
c th` dai
a mt
inh
. ngha bt bin o ph`n truo
. luo. ng bt bin l`
.
,
,
, ,
,

tnh cht cua b`


ai to
an khng thay d
ac dng
bin di cua h.
i qua su. t
.
,
,
,
,,

thng. Nhung khi ta thay di h. thng m`


a c
o dai
i theo mt
. luo. ng bin d
.
,
,

th` sao ? V du. nhu khi h. thng bin di c


quy lut
ao do
o mt
ai
. n`
. d
.
,
,,
,,
,

. c lun lun giam mt


luo. ng lun lun t
ang hoa
V du. nhu
inh.
.
. luo. ng c d
,
,,
. c cp s nhn th` nhung s hang
cp s cng
hoa
sau kh
ac s hang
truo
c
.
.
.

28

,,
Chuong 1. Nguyn l bt bin

,,
,,
,
,,
. c cng bi.
ngu`oi ta tnh duo. c
mt
a cng sai hoa
o
. l`
. dai
. luo. ng c dinh
. T`u d
,
,
,
,c tng qu
. c nhung tng c
cng thu
at cho c
ac s hang
hoa
ac s hang
d
`u
.
.
,
,,
. c lun lun giam khi h. thng bi.
tin. Mt
ang hoa
. dai
. luo. ng lun lun t
,
,,
,,
,

t
ac dng
d
ad
ay ta giai
ai
iu.
uo. c goi
. luo. ng bt bin don d
. Trong ph`n n`
.
. l`
,
,,
,
,,
,
nhung b`
ai to
an c
o dai
ung nhu ph`n truo
c ta
on diu.
. luo. ng bt bin d
. C
,
,
,
,, ,
,,
,

phai t`m ra dai


ac bin cua b`
ai
iu
. luo. ng bt bin don d
. duoi su. thay di c
, ,
,
,
,
,
,

to
an. Chnh nhung dai
ai
on d
iu
n l`oi giai cua b`
. luo. ng bt bin d
. dn ta d
to
an.
,,
,,
`ng mt
,i mt
Ngu`oi ta thu`ong thy ra
at
. h. thng c`n d
. to
. cu h`nh cu.
,
,
,
,,
,
,
m`
th n`
ao d
a bt bin d
ai m
ai. Tru`ong ho. p
o
on diu
i m
. khng th thay d
,
,
,
,
,,
. c lun t
din h`nh l`
a mt
ai to
an phai giam (hoa
ang) v`
a
. dai
. luo. ng cua b`
,
,
,
,

`
khng bao gi`o d`ung lai.
Tr
a
i
l
ai,
quy
tr`
nh
bi
n
d

i
trong
d
i
u
ki
n
c
ua
b`
a
i

.
.
.
,
,
,
,
,
,
. c
to
an phai d`ung lai,
a huu han
a tri. hoa
. v` bt bin don diu
. ch l`
. nhung gi
,
,
,
,
,
,
,
i han
suy ra su. v l v`
l`
a mt
ac kha n
ang thay d
a
i. T`u do
. gio
. cua tt ca c
,
,,
b`
ai to
an duo. c giai. Ta xet v du:
.
,
,,
,
trong bang h`nh chu
, nht
V du. 1.38. Mi
. m n duo. c vit mt
. s thu. c.
,
,
Ta thu. c hin
ac s trn mt
ang hoa
. c mt
i du mt
. bin d
. l`n c
. h`
. ct
. bt k`.
, ,
,
,
`
`
Chung minh ra
ng ba
ng thao t
ac nhu trn, ta c
o th d
ang
u a d
n trong moi
. h`
,
,
v`
a trong moi
ac s cua n
o l`
a mt
. ct
. tng c
. s khng m.
,
,
,
,
`,i giai. thun
. c ct
Lo
ang hoa
a mt
. vt.
. Ta xet tng tt ca
. tin
. ta goi
. h`
. l`
,
,
,
,
,i su., thu.,c hin
c
ac s trong bang vo
ay se t
ang nu tng
i. Tng n`
. bin d
,
,
,
,
nhung s trong mt
ac l`
a mt
ay se giam khi tng
. vt
. thao t
. s m; tng n`
,
,
,
,,
c
ac s trong vt
ac l`
a mt
a tng khng thay di nu tng
. thao t
. s duong v`
,
,
,
`ng khng. Ngha l`
,i tng
cua vt
ac ba
a nu trong bang c
o mt
. thao t
. vt
. vo
,
,
`ng m, th` ba
`ng thao t
c
ac s ba
ac cho phep trn ta t
ang tng tt ca c
ac s
,
trong bang.
,
,
,
,
,
`ng thao t
Liu
ac s trong bang ba
ac trn c
o th t
ang
. tng cua tt ca c

,
1.4. Bt bin don diu
.

29

,
,
,,
,
ln v han
ac l`
am t
ang
. khng? Khng th nhu vy
. duo. c v` nhung thao t
,
,
,
,
,
,
,
tng c
ac s trong bang ch c
o huu han
a ch c
o huu han
. l`n, ngha l`
. nhung
,
s trong mt
,i s ban
. c l`
bang s kh
ac nhau. Tht
a tr`
ung vo
. vy,
. mi
. hoa
,
,,
,
`ng mt
,i s ban d`u ba
. c l`
d`u hoa
a kh
ac vo
. du tr`u. V` th s luo. ng nhung
,
, ,,
,
,
,
bang kh
ac nhau khng th vuo. t qu
a 2mn , ngha l`
a tng cua tt ca c
ac s
,
,
,
,
,
trong bang ch c
o th nhn
a tri. kh
ac nhau.
. huu han
. nhung gi
,
,
,
Ta xet bang s ban d`u. Ta chon
o nhung vt
o tng c
ac s l`
a
. trong n
. c
,
,
,
,
,

duo. c giai). Ta thu. c hin


m (nu khng c
o vt
ao nhu vy
ai to
an d
a
. n`
. th` b`
.
,
,
,
,,

`
phep bin di trn vt
ay. Trong bang vua nhn
o
. n`
. duo. c lai
. t`m thy vt
. c
,
,
,,
,

tng c
ac s l`
a m th` ta lai
o v`
a nhn
. thu. c hin
. phep bin di trn n
. duo. c
,
,
,
,
,
,

bang moi v`
a tip tuc
i
. nhu vy.
. Nhu vy
. mi ln thu. c hin
. phep bin d
,
,
,
,
,
,
th` tng c
ac s trong bang t
ang ln. Tng n`
ay ch c
o th nhn
. huu han
.
,,
,
,, ,
`n

gi
a tri,
nn
ho
a
c
l`
a
t
ai
m
t
bu
o
c
th
u
c
hi
n
n`
a
o
d
o
ta
nh
n
d
u
o
c
b
ang
c

.
.
.
.
.
.
. .
,
,
,
,
,
,
,
,
m hay mun
i kha n
. c l`
t`m, hoa
a so
ung nhn
ang tng cu. c
uo. c bang vo
. ta c
. d
,
,
,
,i tnh cht nhu, vy
dai.
on mt
o tng m
. Bang vo
. ta phai t`m v` nu c`
. vt
. c
,
,
,

n`
ao t`n tai
o th` ta lai
. trong n
. thu. c hin
. mt
. l`n nua phep bin di, khi do
,
,
,
,
,
,
,
,
i n`
n hon tng cu. c dai
tng c
ac s trong bang mo
ay t
ang ln v`
a lo
ac
. cua c
,
s trong bang, d
ay v l.
i`u n`

,
,
,
V du. 1.39. Trn ma
. t pha
ng cho n d
o ba d
ao na
`m trn
im, khng c
im n`
,
,
,
,`,
,`,
,`,
mt
ng v`
and
ng, khng c
o hai duong tha
ng n`
ao song
uong tha
uong tha
. d
,
,
,
,
,
,
,
,ng
` c
song v
oi nhau. Chung minh ra
`ng tu
ac d
ay c
o th ha. d
im n`
uo. c nhu
,
,,
,,
,ng d
cho
d
oc d
ng d
u`ong vung g
i mt
u`ong tha
a
. khng giao nhau ln nhu
,
,
,
,
,
,
,
,
d
oc.
sao cho trn mi
ng duo. c ha. ch mt
u`ong vung g
u`ong tha
. d
,
,
,
,,
,
,,
`,i giai. Ta ha. c
ng
Lo
ac du`ong vung g
oc t`u c
ac dim xung c
ac du`ong tha
,
,,
d
cho mt
da
ach bt k` (mi
oc xung mt
im ha. vung g
u`ong ring).
. c
. d
,,
,,
duo. c
Nu khng c
o hai du`ong vung g
oc n`
ao giao nhau th` b`
ai to
an d
a
,
giai.

30

,,
Chuong 1. Nguyn l bt bin

,,
,,
,,
,
Trong tru`ong ho. p nguo. c lai,
oc giao nhau AA1
u`ong vung g
. ta xet hai d
,
,
,
,
,,
, , ,
,
ng tuong u
oc t`u c
ac dim A v`
a B ln d
v`
a BB1 , d
ng
uo. c ha. vung g
u`ong tha
,
,
,
,`,
ng vung g
v`
a (h`nh 1.5). Cho P l`
a giao d
oc n`
ay.
im cua hai duong tha
,
,
,
`
ng AA2 v`
By gi`o ta thay hai d
oc AA1 v`
a BB1 ba
a BB2 l`
a
u`ong vung g
,
,
,
,
,
,
,

ng. Ta se chu
ng minh
nhung d
oc ha. xung v`
a tuong u
u`ong vung g
,
,
,
,
,
,
,
`
i su. thay d
ng vo
ra
ai c
ac d
oc giam di.
i trn th` tng d. d`
u`ong vung g
Tht
o AA2 < AB1 < AP + PB1 v`
a BB2 < BA1 < BP + PA1 , cng
. vy,
. ta c
.
,
,
,

c trn cho kt qua AA2 + BB2 < AA1 +


ng thu
theo v hai bt d
a
B BB1 .
,
,
A
By gi`o ta tin h`
anh l lun
. nhu
,,
,
c tranh g`m n
b`
ai truoc, ta xet bu
,
,`,
,,
P
ng v`
duong tha
a n du`ong vung g
oc.
B2
,
,
A2
Ta ly hai du`ong vung g
oc giao
A1
B1
,
,
nhau v`
a thu. c hin
i: Thay
. bin d
,,
`ng hai
hai du`ong vung g
oc n`
ay ba
,

,`,

duong vung g
oc c
o tng d. d`
ai
,
, ,
,c tranh
cua ch
ung nho hon. Trong bu
,,
,,
nhn
. duo. c, ta lai
. t`m hai du`ong vung
H`nh 1.5
,
g
oc giao nhau v`
a lai
th
u
c
hi
n
thao
.
.
.
,
,
,,
. c l`
hoa
t
ac bin di trn v`
a tip tuc
a dn mt
c n`
ao
o
. nhu vy.
. Khi d
. buo
,
,
, , ,
,
,
i tt ca c
ta nhn
do
ac du`ong vung g
oc khng
. duo. c buc tranh c`n t`m vo
,
,
,
,
,
c tranh c
. c l`
giao nhau; hoa
a cui c`
ung ta nhn
o tng d
ai tt ca
uo. c bu
. d`
. d
,
,
,
,
,,
,
,,
c
ac du`ong vung g
oc nho nht (boi v` tng n`
ay ch c
o th nhn
. huu han
.
,
,`,
`

c
ac gi
a tri!).
y
l`
a
b
u
c
tranh
m`
a
ta
c
n
t`
m.
Th
t
v
y
n
u
c`
o
n
hai
d
u
o
ng

.
. .
,
t nhau th` ta lai
ca

a
p
d
ung
thao
t
ac thay d
vung g
oc n`
ao do
i mt
.
.
. l`n
,
, ,
,
,
,
,
,
,
,
c tranh vo
i tng d
nua v`
a nhn
ai c
ac du`ong vung g
oc nho hon
uo. c bu
. d`
. d
,
nua, v l!.
,
,
,,
`ng vic
Phn tch c
ach giai hai b`
ai to
an trn ta thy ra
uo. c tin
. giai d
,
,
,,
,,
, `
(b`
h`
anh theo mt
ao mt
ao d
ai truo
c l`
a tng
: Ta dua v`
o
. so d
. dai
. luo. ng n`

,
1.4. Bt bin don diu
.

31

,
,
,,
c
ac s trong bang, b`
oc) v`
a mt
ai n`
ay l`
a tng d. d`
ai c
ac d
u`ong vung g
.
,
, ,
,
,
,
,
,
ao
thao t
ac bin d
a kt qua cua su. bin d
am dai
i m`
i l`
ua v`
. luo. ng ta d
,
,
,,
,
,
c l`
duo. c bin d
ac dinh
ai truo
a tng t
ang,
i theo mt
. (trong b`
. quy lut
. x
,
,
,
,
,,
,
,
,
,
,
,
trong b`
ai sau l`
a tng giam). L`oi giai du. a trn co so dai
ao ch
. luo. ng dua v`
,
, ,
,
,
nhn
a tri. kh
ac nhau. Suy ra thao t
ac ch c
o th thu. c hin
. huu han
. gi
. huu
,
,
,,
ng d
han
a ta kha
ai to
an v` trang
ai c`n t`m.
inh
uo. c b`
. dua d
. l`n v`
. th
,
, ,
,
`i hoi su. s
Nhu vy,
ai to
an n`
ay do
ang tao
a nh`n b`
ai to
an
. giai loai
. b`
. cao v`
, ,
,,
,
,,
,

duoi g
oc d
. bin di voi vic
on d
iu
.
. t`m ra nhung dai
. luo. ng bt bin d
,
,
,
,
`

. c t
. c giam huu han
(hoa
ang, hoa
a nhung b`
ai
. ln). Nhung v du. sau dy l`
,
,
,,
, `
`
ng so d ta d
vach
to
an kh
o nhung duo. c giai ba
a
. ra:
,
,
,
V du. 1.40. Chung minh ra
`ng 2n d
. t pha
ng l`
ad
ut
im bt k` trn ma
`u m
,
,
cua n d
ng khng giao nhau.
oan
. tha
,
,
,
`,i giai. Ta ke n d
,i c
ng vo
Lo
ac
oan
. tha
,
,
,
d
D
cho (mi
dim m
ut l`
a c
ac dim da
im
A
,
, ,,
ng).
ch d
ad
ut mt
uo. c l`
`u m
oan
. d
. tha
,
,
ng
Nu tt ca c
ac d
tha
oan
.
, ,
,
t nhau ta c
khng ca
o l`oi giai cua b`
ai
,
,,
ng
. p doan
to
an. Nguo. c lai,
. ta xet ca
. tha
t nhau, cho do
l`
a AB v`
a CD (h`nh
ca
C
B
, ,,

1.6). Mt
ph
e
p
bi
n
d

i
d
u
o
c
x
a
c
d
inh

.
.
.
,
,
,

H`nh 1.6

a
ng
c
a
t
nhau
nhu sau: Ta d
th
i hai d
oan
.
,
ng khng
AB v`
a CD th`
anh hai d
oan
. tha
t nhau AC v`
ca
a BD.
,
,
,,
,
,
Ta d
ai c
ac
i t`m dai
on diu
. luo. ng bt bin d
. giam nhu sau: V` tng d. d`
,
,
,`,
,
,
,
`
gi
n hon tng d. d`
d
a CD cua tu
ac li ABCD lo
ai hai canh
uong cheo AB v`
.
,
,
,

ng l`
d
a BD, nhu vy
ai n doan
am bt bin
i din
. d`
. AC v`
. ta ly tng d
. tha
,
,
,
,
,
,
,
d
ay ch nhn
a tri. (v` ta ch
on diu.
. Hin nhin tng n`
. huu han
. nhung gi

32

,,
Chuong 1. Nguyn l bt bin

,
,
,,
,
,
ng). L lun
c
o huu han
ach tao
an to`
an tuong tu. nhu c
ac
oan
. c
. nd
. tha
. ho`
,
,
,
,,
,
,
i tng d
ng vo
b`
ai truoc dua d
ai nho nht,
n mt
oan
. d`
. h. thng n d
. tha
,
,
ng giao nhau.
trong h. thng n`
ay khng th c
o hai doan
. tha

, ,
,,
on ta d
. t n s. Nu thu tu. c
ac s a, b, c, d
V du. 1.41. Trn mt
a
u`ong tr`
. d
,
,
,
thoa m
an (a d)(b c) > 0, th` hai s b v`
acd
i ch cho nhau. Chung
,
,,
,,
minh ra
`ng sau mt
on khng c`
on b. tu n`
ao sa
p xp
u`ong tr`
. s buoc th` trn d
,
nhu vy.
.
,
,
,
,,
,
`,i giai. Phep bin d
d
Lo
ai to
an da
a di ch
i trong b`
uo. c cho. Thu. c cht l`
,
,,
giu, nguyn. Ta phai
hai s trn du`ong tr`
on, c`
on c
ac s kh
ac vn
t`m d
ai
.
,
,
,
,,
,
luo. ng bt bin c
o lin quan d
ay. Nhu vy,
n su. thay d
i hai s n`
. cho thu
,
tu. a, b, c, d sao cho (a d)(b c) > 0, ngha l`
a ab + cd > ac + bd. Nu
,
,
,
,
,

a, b, c, d chuyn th`
,
cho th` t`u b. tu
thu. c hin
anh b. tu
i da
. phep bin d
,
,
,
, ,
`ng tng cua tch c
a, c, b, d. Ta thy ra
ac s canh
nhau bi. giam d
i thu. c su. :
.
ab + bc + cd > ac + cb + bd.
,
,
,
,,
,
Ho`
an to`
an tu. nhin, dai
a tng cua tch c
ac s
on diu
. luo. ng bt bin d
. l`
,
,
,,
,
,
i phep bin d
cho th` dai
k` nhau lin tip trn du`ong tr`
on. Vo
i da
. luo. ng
,
,
, ,
,
,
bt bin don diu
a v` n
o ch c
o huu han
a tri. (tai
. giam thu. c su. , v`
. gi
. sao?),
, ,
,
, ,
,

nn phep bin di cua ta ch c


o th thu. c hin
ach
. huu han
. ln. Tip tuc
. c
,
,
,,
giai nhu c
ac b`
ai truoc d`
anh cho ban
oc.
. d
.
,
, ,
,
Nhung b`
ai to
an v` h`nh hoc
o dang
ung c
o th giai
i cu h`nh c
. c
. thay d
,,
`ng phu,o,ng ph
ap dai
ba
. luo. ng bt bin.

,
`, mt
V du. 1.42. Tu
ac l
om ta tin h`
anh thao t
ac nhu sau: Nu d
ac
a gi
a gi
. d
,
,
,
, ,`,
,
`
`

na
m v mt
oi d
ng AB, o d
a B hai d
i v
uong tha
y A v`
nh khng lin
. pha d
,
,
,
,,
,,

`
tip cua d
ac, th` mt
ac canh
ac duo. c chia ra boi hai d
a gi
a gi
im
. phn c
. cua d
,
, ,
,
,,
,
A v`
a B, d
a trung d
ng AB. Chung
uo. c ly d
i xung qua tm l`
im cua d
oan
. tha
,
,
,u han
cho
minh ra
`ng sau mt
ac nhu vy
ac d
a gi
a
. s hu
. l`n thu. c hin
. thao t
. d
`
se th`
anh d
ac li.
a gi

,
1.4. Bt bin don diu
.

33

,
,
`,i giai. Nhu, vy
Lo
. phep bin di
,
cho do
l`
da
a vic
i da
. bin d
X
Y
= YB
,,
,
AY
gi
ac. ai
on diu
.
. luo. ng bt bin d
A
B
,

Y X = XY
,i phep bin di n`
vo
ay l`
a din
.

,
X B = AX
,
tch cua d
ac t
ang don diu.
a gi
.
,
Y
X
`ng din
D thy ra
. tch cua da
, ,
gi
ac trong qu
a tr`nh bin d
i cua
,
,
,
da gi
ac ch c
o th nhn
. huu han
.
gi
a tri.
.
H`nh 1.7
,
Tht
. vy,
. ta xet b. vecto doc
. theo
,
,
,
,
,i mt
c
ac canh
ac. Vo
ac b. vecto khng thay di, m`
a ch
a gi
. cua d
. l`n thao t
,
,
,
, tu., lin tip cua c
thay d
ac vecto canh
nhau (h`nh 1.7 minh hoa
i thu
.
. mt
.
,
,
,
,`,
,,
,
,

ac nhn
truong ho. p cu. th). Ngha l`
a s luo. ng nhung d
a gi
. duo. c thng qua
,
,
,
,
,
,,
suy ra dai
thu. c hin
ac thao trn ch c
o th l`
a huu han.
o
. c
. T`u d
. luo. ng bt
,
,
,,
,
,
bin d
a tri.
on d
iu
.
. cua ta ch nhn
. duo. c huu han
. gi

,
,,
,,
ao c
ac ph`
ong cua mt
oa nh`
a 115
V du. 1.43. 2000 ngu`oi d
uo. c chia v`
. t`
,
,
,
,,
,
,
ph
bu`ng. Mi
ut khng phai moi
ong, m`
a mt
`u o trong ph`
. ngu`oi d
. s ngu`oi
,, ,
,
,, ,
,
`, ph`
d
ong t ngu`oi t
oi ph`
ong c
o nhi`u ngu`oi hon. Chung minh ra
`ng t
oi mt
i t u
.
,
,`, `

l
uc n`
ao d
o
t
t
c
a
m
oi
ngu
o
i
d

u
t
p
trung
t
ai
m
t
ph`
o
ng.

.
.
.
.
,
,
, ,,
,
,
,,
`,i giai. Kt lun
Lo
ai to
an tuong ch`ung nhu khng tin duo. c. B`
ai to
an
. cua b`
,
,,
,
,
,
,
,

i mi ph`
d
ong ta xet b`nh phuong
uo. c giai nh`o t`m ra bt bin don diu.
. Vo
,
,
,
,`,
,,

cua s nguoi trong ph`


ong. K hiu
ac s b`nh phuong l`
a S. Ta ch ra
. tng c
,
, ,,
,,
,,
,
ngu,`o,i d
`ng S t
,i mi
ra
ang vo
uo. c chuyn d
i. Tht
. vy,
. gia su mt
. ngu`oi di t`u
,,
,
, , ,
nhung s b`nh
ph`
ong c
o n ngu`oi to
i ph`
ong c
o m ngu`oi, m`
a m > n. Khi d
o
,
,
,,
,,
,
phuong cua s ngu`oi trong c
ac ph`
ong bin di t`u n2 v`
a m2 th`
anh (n 1)2
,
, , ,
,
v`
a (m + 1)2 tuong u
ng v`
a c
ac ph`
ong kh
ac th` khng di. Nhu vy
. S thay

,,
Chuong 1. Nguyn l bt bin

34
,
,
di nhu sau:
((n1)2 + (m + 1)2 ) (n2 + m2 ) =

= (n2 + m2 2n + 2m + 2) (n2 + m2 ) = 2(m n) + 2 > 2 > 0.


Vy
ang.
. S lun lun t
,
`ng s ngu,`o,i l`
Ta bit ra
a 2000 (mt
. bt bin). T`n tai
. mt
. s huu han
.
,`,
,
,
`
c
ach chia 2000 nguoi v`
ao c
ac ph`
ong kh
ac nhau, nhu vy
. tn tai
. huu han
.
,
,
,
,

kha n
ang gi
a tri. cua S. Ngha l`
a S khng th t
ang m
ai m
ai. Nhung dn mt
.
,
,`,
,,
, quy tr`nh b`
a
l
uc hai ph`
ong c`
on nguoi trong d
ai to
an duo. c thu. c hin
o
. v`
,
,
,
,

`
tt ca moi
S se t
ang v`
a qu
a tr`nh n`
ay d
uc do
n khi d`ung lai,
. l
. ngu`oi du
trong mt
ong.
. ph`

,
,
,,
V du. 1.44. Cho bn s a, b, c, d khng phai tt ca ba
`ng nhau. Khoi d
`u
ba
`ng b. (a, b, c, d) v`
a la
. p lai
`ng
. vic
. thay th (a, b, c, d) ba
,
(a b, b c, c d, d a). Chung minh ra
`ng khi la
. p lai
. nhi`u l`n vic
.
,,
,

thay th trn th` t nht mt


trong
b
n
s
s
e
tr
o
th`
a
nh
v
c`
u
ng
l
o
n.
.
,
`,i giai. a
. t Pn = (an , bn , cn , dn ) l`
. p. Khi d

Lo
a b. bn s sau n phep la
o
,,
,

ta c
o an + bn + cn + dn = 0 voi n 2. Ta khng bit su dung
bt bin
.
,
,
,
, ,,
,

n`
ay nhu th n`
ao. Nhung ta lin tuong toi trong h`nh hoc,
am khoang
. h`
,
,
, ,
,,
,c
,i biu thu
c
ach t`u d
im Pn dn d
im gc toa
. (0, 0, 0, 0) duo. c lin h. vo
. d
,
,,
,ng minh d
,c trn khng bi. cha
. n
a2n + b2n + c2n + d2n . Nu ta chu
uo. c biu thu
,
,,
trn th` b`
ai to
an d
uo. c giai.
,
,,
Ta di t`m mi lin h. giua hai buo
c lin tip Pn+1 v`
a Pn :
a2n+1 + b2n+1 + c2n+1 + d2n+1 =
= (an bn )2 + (bn cn )2 + (cn dn )2 + (dn an )2 =
= 2(a2n + b2n + c2n + d2n ) 2an bn 2bn cn 2cn dn 2dn an .

(1.1)

,
1.4. Bt bin don diu
.

35

,
,
By gi`o ta c
o th d`
ung an + bn + cn + dn = 0:
0 = (an + bn + cn + dn )2 = (an + cn )2 + (bn + dn )2 +
+ 2an bn + 2an dn + 2bn cn + 2cn dn .

(1.2)

,i (1.2):
Cng
. (1.1) vo
a2n+1 + b2n+1 + c2n+1 + d2n+1 =
= 2(a2n + b2n + c2n + d2n ) + (an + cn )2 + (bn + dn )2
2(a2n + b2n + c2n + d2n ).

,
,
,i n 2,
T`u mi quan h. bt bin n`
ay ta dua ra kt lun
. vo
a2n + b2n + c2n + d2n 2n1 (a21 + b21 + c21 + d21 ).
,
,
,
,
Khoang c
ach t`u dim Pn dn dim d
ang v han,
a t nht c
o
`u t
. ngha l`
,
,,
,
n bt k`.
mt
anh ph`n phai tro ln lo
. th`
,
,
,
,
H`
am khoang c
ach t`u mt
im d
`u toa
. rt quan trong,
. dim dn d
. d
.
,
,
,

khi n`
ao c
o mt
d
a
y
d
i
m
ta
ph
ai
ngh

ngay
t
o
i
h`
a
m
n`
a
y.

,
,,
ac d
V du. 1.45. Mt
an d
inh
uo. c x
. nhu sau:
. thut
. to
,,
,
Buoc xut ph
at: (x0 , y0 ) v
oi 0 < x0 < y0 ,
x n + yn

,,
Buoc tip theo: xn+1 =
, yn+1 = xn+1 yn .
2
,
,
T`m gi
oi han
ay (xn , yn ).
. cua d
,
,
,
`,i giai. T`u, nhu,ng gia thit d
cho v`
Lo
a tnh cht cua trung b`nh cng
v`
a
a
.
,
trung b`nh nhn cua hai s ta c
o:
yn x n
xn < yn = xn+1 < yn+1 , yn+1 xn+1 <
4
,
,
,
,,
,
,
i moi
i han,
vo
ac d
ay c
o gio
ay s duong v`
a giam
. n. Nhu vy
. c
. v` nhung d
,
,
,
,i han
,i han
gio
thu. c su. . Khi d
ac d
ay n`
ay tr`
ung nhau v`
a ta t`m gio
o
. cua c
.
chung lim xn = x = lim yn = y.
n

,,
Chuong 1. Nguyn l bt bin

36

,
,
,
Tnh bt bin c
ung gi
up ch cho vic
o hoi kh
ac
ai n`
ay. Nhung c
. giai b`
,
,,
,
,
ng giua c
l`
a khng c
o phuong ph
ap di xu
ac bin d t`m bt bin. T`n tai
.
,
, , xn
,
,
,
c bin di cua
. c yn xn khi
mt
ap x
ac d
hoa
inh
. biu thu
. s phuong ph
yn
, , ,,
c n d
bin d
i t`u buo
n n + 1.
,,
,,
,
a) Tru`ong ho. p xet thuong:
s
r
1 + yxnn
xn+1
xn+1
xn+1
.
(1.3)
=
=
=
yn+1
xn+1 yn
yn
2
r
,,
1 + cos

,
,
,
,
,
c n`
c cos =
i cng thu
. V` ta
T`u cng thu
ay ta lin tuong to
2
2
,
,,
xn
xn
. t
(1.3) tro th`
lun c
o0<
< 1, ta c
o th d
= cos n . Khi d
anh
a
o
yn
yn
n
0
cos n+1 = cos
n = n 2n n = 0 .
2
2
,,
,,
,
`
i
iu n`
ay tuong d
uong vo
xn
x0
2n arccos
= arccos .
(1.4)
yn
y0
,,
dy l`
a dai
. luo. ng bt bin.
,
,,
,
2
2
b) Tru`ong ho. p xet hiu:
anh c
an bc
. tr
. hai, ta xet yn xn thay v`
,,
yn xn v`
a ta nhn
. duo. c
q
q
y2 x2n
2 y2n+1 x2n+1 = y2n x2n
y2n+1 x2n+1 = n
4
ngha l`
a
q
q
2n y2n x2n =
,,
, hai.
dy l`
a dai
. luo. ng bt bin thu
,
,,
T`u (1.4) v`
a (1.5), ta nhn
. duo. c

y20 x20

(1.5)

y20 x20

x0
xn
n
n
n
arccos
= 2 arccos
= 2 arcsin
= 2 arcsin
.
y0
yn
yn
2n yn
q
y20 x20
,
, ,
,c trn tin to
,i
ng thu
V phai cua d
khi n . Cui c`
ung ta
a
y
p

y2n

x2n

,
1.4. Bt bin don diu
.

37

,,
nhn
. duo. c

y20 x20
x0 .
arccos
y0
,
`

y l`
a mt
b`
a
i
to
a
n
kh
o
n
u
ta
gi
ai
b
a
ng
c
ach kh
ac.
.
x=y=

s trong nhu
,ng s a1 , a2 , ..., an l`
V du. 1.46. Mi
a +1 hoa
. c 1 v`
a ta c
o
S = a1 a2 a3 a4 + a2 a3 a4 a5 + + an a1 a2 a3 = 0.

,
Chung minh ra
`ng n chia ht cho 4.

,
, ,
,
`ng
`,i giai. y l`
Lo
a b`
ai to
an v` l thuyt s, nhung ta c
ung c
o th giai n
o ba
,
,
`ng ai , th` S khng thay d
bt bin. Nu ta thay ai ba
i d`ng du theo 4
,
,
v` bn s hang
canh
nhau theo v`
ong tr`
on thay di du cua ch
ung (v du.
.
.
,

ta thay du cua a1 th` bn s hang


canh
nhau l`
a a1 a2 a3 a4 , an a1 a2 a3 ,
.
.
,

an1 an a1 a2 , an2 an1 an a1 ). Tht


. vy,
. nu hai trong s nhung s hang
.
,
,
,,

l`
a duong v`
a hai l`
a m th` tng S khng thay di. Nu ba s hang
. trong bn
, ,,
,,
s hang
o c`
ung du th` S se thay di boi 4 (nu c
o ba s hang
a
. c
. duong v`
,,
,

mt
m th` khi thay du d
o duong, m`
a lai
on tru 2
a mt 2 khi n
. s hang
.
. c`
,
,,
,

khi n
o m, vy
mt di 4. Tuong tu. , nu ba s hang
m v`
a mt
. tng cng
.
.
.
,
,
,,
,

`
s hang
du
o
ng
th`

t
ng
t
a
ng
ln
4).
Cu
i
c`
u
ng
t
t
c
a
b
n
th
u
a
s
trong
s
.
,
l`n thay du nu S da
mt 4 lai
hang
c`
ung du, th` S thay d
i 8 (v` mi
.
.
,
,
,
,
,
,
,
`

`
`
mt thm 4 khi ca bn s hang
d

u
du
o
ng,
tru
o
ng
h
o
p
c
a
b
n
s
h
ang
d

u
.
.
.
,,
m th` nguo. c lai).
.
,,
,
Khoi d`u ta c
o S = 0 di`u n`
ay keo theo S 0 (mod 4). By gi`o ta tin
,
,,
,,
du m thay ba
`ng du du,o,ng, c
ac buo
c n`
ay khng
h`
anh t`ung buo
c mi
,
,
c`
thay d
ac du tr`u th` vn
on S 0 (mod 4),
i S (mod 4). Khi ta thay ht c
,
S = n, ngha l`
nhung khi do
a n chia ht cho 4.

,,
,
,
vi. kh
V du. 1.47. Tai
o m`oi 2n quan kh
ach. Mi
ach duo. c m`oi
. mt
. hi
. nghi. c
,
,
,
c
o nhi`u nht l`
a n 1 ke th`
u. Chung minh ra
`ng c
ac vi. kh
ach c
o th ng`i

,,
Chuong 1. Nguyn l bt bin

38

,
,
u cua m`nh.
quanh mt
an tr`
on sao cho khng ai ng`i canh
ke th`
. b`
.

A
B

B
A

B
A

H`nh 1.8

H`nh 1.9

,
`,i giai. Tru,o
,c tin ta xp c
. t H l`
Lo
ac vi. kh
ach ng`i v`
ao vi. tr bt k`. a
a s
,
,
,
,
nhung d
ud
nhau. Ta phai t`m duo. c thut
an m`
a n
o
i th`
ich
. ng`i canh
.
. to
,
,
,
i B ng`i bn phai
. p th`
l`
am giam s H khi H > 0. Cho (A, B) l`
a ca
ud
ich
. vo
,
,,
,
,,
. p n`
A (h`nh 1.8). Ta phai t
ach duo. c ca
ay ra. i`u n`
ay thu. c hin
. duo. c nu
,
,
B, A 0 (h`nh 1.9) v`
. p kh
c
o mt
ac (A 0 , B 0 ), khi ta d
a H se giam nu
i ch
. ca
,
,
,
`ng
,ng minh ra
. p ke th`
(A, A 0 ) v`
a (B, B 0 ) khng l`
a nhung ca
u. Ta ch c`
on chu
,
,
,
0
0
0
0
i B ng`i bn phai cua A m`
. p (A , B ) lun lun t`n tai
mt
a A 0 l`
a
. ca
. vo
,
,
,
,
,
, ` `,
0
ban
a B l`
a ban
ad
i theo chi`u nguo. c
. cua A v`
. cua B. Ta khoi du tu A v`
,
,,
t ga
. p t nht n ngu`oi
kim d`ng h` (d
an. Ta se ba
i v` pha phai) quanh b`
,
, ,
,
, ,
ngu,`o,i ban
o
ban
u) cua A. V` pha phai cua mi
. (khng phai ke th`
. cua A c
,
,
,
,,
,

t nht n ch. Nhung ch n`


ay khng th bi. chim ht boi c
ac ke th`
u cua B
,
,
,
,
,
,
0
v` B ch c
o nhi`u nht n 1 ke th`
u. Nhu vy
. t`n tai
. ngu`oi ban
. A cua A
,
,
,,
,,
m`
a v` pha phai ngu`oi n`
ay l`
a B 0 , m`
a B 0 l`
a ngu`oi ban
. cua B.
,
,
Mt
ai to
an ta phai d`
ung dn hai bt bin don diu
a hai bt bin
. s b`
. v`
,i nhau.
n`
ay c
o mi lin h. vo
,
,,
,ng vung, mt
V du. 1.48. Trn mt
. t pha
ng chia luoi v han
. ma
. nhu
. s
,,
,, ,,
vung d
au theo nguyn ta
c sau
uo. c t den. Mt
uo. c t m`
. vung trong luoi d

,
1.4. Bt bin don diu
.

39

,
,,
vung d
a ch khi t nht c
o ba vung bn canh
d
en khi v`
y: Mi
uo. c t d
.
,`,
,,
,,
,

v`
ac th` t tra
ng. Quy tr`nh
a truong ho. p kh
n
o l`
ad
en trong buoc t truoc d
o
,,
,
`
n`
ay duo. c la
. p lai.
ng cui c`
ung khng c`
on mt
ao trn
en n`
. Chung minh ra
. d
,,
luoi vung.
,
,
,
,
`,i giai. Ta xet tt ca h`
,a s den. Ta xem xet co, h`
Lo
ang c
o chu
ang cua nhung
,
,
h`
ang n`
ay l`
a khoang c
ach chi`u doc
ang cao nht v`
a h`
ang thp nht.
. giua h`
,
,
,
`ng co, h`
,ng
Ta ch ra ra
ang n`
ay khng bao gi`o t
ang. Tht
o th chu
. vy,
. ta c
,
,
minh mnh
d` manh
hon: H`
ang bt k` khng chua vung d
en se khng
.
.
,
,,
,,

vung

cho. Boi v` tai


thay d
c t m`
au d
i theo nguyn ta
a
. mi buoc, mi
,,
c
trong h`
ang d
o nhi`u nht hai d
a duo
i (v` bn canh
bn
o
en trn v`
.
,
,
,

`
ng), nhu vy
ng cho
phai v`
a bn tr
ai n
od
a tra
o se giu nguyn l`
a tra
u l`
. n
,,
buoc sau.
,
,
,
,
,,
,
ng khng th t
Nhu vy
ang cua tt ca c
ac h`
ang khng tra
ang d
uo. c.
. co h`
,
,
,,
,
,
Tru`ong ho. p g` se xay ra khi co h`
ang cua c
ac h`
ang ta dang xet l`
a mt
. s
,
,
,
,
,
,
,
n hon 0 ? Trong tru`ong ho. p nhu vy
lo
ang pha trn
. ta xet co den trong h`
,
,
,
,
i vung den phai
nht (ngha l`
a khoang c
ach t`u vung d
ai nht to
en tr
,
,
,
nht). Khoang c
ach n`
ay khng th t
ang m`
a n
o lun lun giam: vung
ng (hu,o
,ng pha trn
den bn tr
ai nht se c
o t nht hai vung ln cn
. tra
,
,,
,
,,
,
ng. L lun
v`
a bn tr
ai n`
ay) v`
a nhu vy
o phai tro th`
anh tra
. tuong tu.
. n
,
,
,
,
,
ng cho den bn pha phai nht. Nhu vy
du
ang n`
ay giam
en cua h`
. co d
,
,,
,
ng,
h`
dn khi n
o tro th`
anh khng, tai
ang n`
ay to`
an b. l`
a tra
. th`oi dim do
,
,
,
,
,
ng c
ung giam.
nhu vy
ang cua h`
ang tt ca khng tra
. co h`
,
`ng dn mt
,i mt
uc t`n tai
ang vo
ai
i`u trn ch ra ra
. l
. t nht mt
. h`
. v`
,
,
,
,
,
,
,
, co den cua nhung h`
den trong do
ang nhu vy
ang
. se giam. Giam co h`
,
,
,
i trang
n`
ay khng phai m
ai m
ai, nhu vy
ung ta dat
ai khng c
o
. cui c`
. to
. th
,
,
,
ng.
a vung d
i tra
mt
ang n`
ao chu
a to`
an b. luo
en, ngha l`
. h`
,
,
,,
,,
o tr`nh b`
ay phuong ph
ap giai phuong
Trong cun s
ach [9] t
ac gia c

40

,,
Chuong 1. Nguyn l bt bin

,
,
,,
,,
,,
`ng phu,o,ng ph
tr`nh ba
ap giam d
ung. Ngu`oi ta thu`ong ch ra phuong
n v c`
,,
,,
tr`nh v d
o nghim
ap sau: Ngu`oi ta
inh
. khng c
. nguyn theo phuong ph
,
,
,
,
,
xy du. ng mt
am cua nghim
ac bin) m`
a gi
a tri. cua n
o
. h`
. (v du. nhu tng c
,,
,`,
,
,,

cho mt
l`
a s nguyn duong; khi d
nguoi ta lai
o
uo. c
. nghim,
.
. xy du. ng d
,
,
,

i, vo
i nghim
xy du. ng nhn
nghim
ay h`
am s da
a tri. nguyn
. mo
. n`
. gi
, ,
,
,,
,`,
,,
. p lai
duong nho hon. Nguoi ta la
a tr`nh n`
ay, nhn
ay giam v
uo. c d
. qu
. d
, ,,
,,
,ng
chu
han
ac gi
a tri. nguyn duong; d
ay khng th duo. c, ngha l`
ad
i`u n`
a
. c
,,
,
xy du. ng khng t`n tai,
minh d
ay s nguyn duong d
ai to
an v nghim.
a
. b`
.
,
,,
V du. 1.49. Chung minh ra
`ng phuong tr`nh v d
o nghim
inh
. sau khng c
.
nguyn kh
ac khng
8x4 + 4y4 + 2z4 = u4 .
,
, ,
`,i giai. Gia su, phu,o,ng tr`nh c
Lo
o nghim
. nguyn (x0 , y0 , z0 , u0 ) 6= (0, 0, 0, 0).
.
..
. t u0 = 2u1 .
Ta c
o 8x40 + 4y40 + 2z40 = u40 , suy ra u40 .. 2, vy
. u0 . 2. a
.
..
,,
. t z0 = 2z1 .
Ta duo. c 4x40 + 2y40 + z40 = 8u41 , suy ra z40 .. 2, vy
. z0 . 2. a
.
.
,,
.
. t y0 = 2y1 .
Ta duo. c 2x40 + y40 + 8z41 = 4u41 , suy ra y40 .. 2, vy
. y0 . 2. a
.
..
,,
. t x0 = 2x1 .
Ta duo. c x40 + 8y41 + 4z41 = 2u41 , suy ra x40 .. 2, vy
. x0 . 2. a
,
,,
Ta duo. c 8x41 + 4y41 + 2z41 = u41 . Vy
ung l`
a nghim
. (x1 , y1 , z1 , u1 ) c
. cua
,
x0 y0 z0 u0
,,
. p
cho v`
phuong tr`nh da
a c
o dang
ay c
o th la
. ( 2 , 2 , 2 , 2 ), quy tr`nh n`
x0 y0 z0 u0
,
,
c k: ( k , k , k , k ).
lai
ai d
n buo
. m
2 2 2 2
, ,
x0 y0 z0 u0
,i moi
C
ac s k , k , k , k l`
a c
ac s nguyn vo
k N. i`u n`
ay ch xay
.
2, 2 2 2
,,
`ng 0. Vy
cho khng c
o nghim
ra khi tt ca c
ac bin ba
. phuong tr`nh da
.
nguyn kh
ac 0.
,
,
,
,
t c
Ta c
o th t
om ta
ach giai theo bt bin don diu
. nhu sau:
,
,
, ,,
,,
,
i t
1. T`m dai
a ch ra n
o phai thay di duo
ac
on d
iu
. luo. ng bt bin d
. v`
,
.
d
ac n`
ao do
ng
. cua mt
. thao t

,
1.4. Bt bin don diu
.

41

,
,
,
, ,
`ng n
,ng
chu
2. H
ay ch ra ra
o ch c
o th thay d
i huu han
. l`n; khi do
, , ,
,
,
,
,
`ng n
.
minh ra
o c
o th ch d`ung su. thay di tai
ao do
. mt
. th`oi dim n`
,
,
,
,
,
,
,
,
,
c, th` ta phai xy du. ng n
o mt
ach
Nu su. chuyn d
i khng d
uo. c cho truo
. c
,
,
,
,
,

thch ho. p. C
o rt nhiu bt bin d
ac nhau c
o th xy du. ng nhu:
on diu
. kh
,
,
,
,
,,
c
ac tng, c
ac tch, gi
a tri. cu. c dai,
a tri. cu. c tiu v`
a nhi`u dai
. gi
. luo. ng thch
,
,
o,,
,i hu,o
,ng dn
ho. p kh
ac. Ban
oi trong muc
ai tp
ay vo
oc
. d
. t`m t`
. b`
. cua tit n`
trn.
,
,,
, ,
Truo
c khi chuyn sang b`
ai tp,
nua cua bt bin
. ta xet mt
. kha canh
.
,
,
,
don diu
a d`
ung bt bin d
o cu h`nh
on d
iu
. l`
. ch ra h. thng ta dang xet c
,
,
,
,
,
nhu mong mun. Trong tru`ong ho. p n`
ay bt bin don d
iu
. khng c`n thit
, ,
,
. t
di`u kin
ung vy
. thay di huu han
. l`n; c
. bt bin don diu
. khng c`n cha
,
,
,
,
,
,
,
m, n
`ng s trong mt
c chuyn di. Ta ch c`n ch
che la
o c
o th l`
a ha
. s buo
, ,
,
,
,,
,
ra bt bin d
o th ch thay di theo mt
ng nu n
o thay di
on d
iu
. c
. huo
,
, ,
, ,
,,
,i mt
tt ca v`
a n
o dat
ng kh
ac sinh ra.
i cua huo
. to
. cu h`nh t`u su. thay d
,
,
,,
d
V du. 1.50. (IMO 1986). Tai
u gi
ac ngu`oi ta g
an mt
nh cua mt
. mi
. ng
.
,
,
,
,,
,,

s nguyn sao cho tng cua n


am s l`
a duong. Nu ba d
an
nh lin tip d
uo. c g
,,
,
,
,

ng s x, y, z
c
ac s x, y, z v
oi y < 0, th` thao t
ac sau d
y d
uo. c thu. c hin:
. Nhu
,,
, , ,
,
,,
,,
,
ac nhu vy
. p lai
oi
d
uo. c la
uo. c thay boi x + y, y, z + y tuong ung. Thao t
. t
. d
khi t nht mt
am s l`
a s m. H
ay x
ac d
o mt
inh
. trong n
. c
. quy tr`nh c`n
,
,
,
,
u han
thit d
uc trong hu
kt th
. buoc khng ?
,
,
`,i giai. Thao t
. p lai
Lo
ac la
ai to
an lun lun kt th
uc. Ch`a kh
oa cua
. trong b`
,
,,
,ng minh l`
chu
a t`m ra mt
am gi
a tri. nguyn duong f(x1 , x2 , ..., x5 ) cua
. h`
,
,
,
,
nhung nh
an cua b
at gi
ac, m`
a gi
a tri. h`
am n`
ay giam khi ta thu. c hin
ac
. c
,
,
,
`
ng c
cho ta c
thao t
ac. Ba
ac du kin
ai to
an d
o th ly h`
am
a
. cua b`
f(x1 , x2 , x3 , x4 , x5 ) =

5
X
i=1

,,
o dy x6 = x1 ,

(xi xi+2 )2 ,

, ,,
,,
x7 = x2 . Gia su dn buo
c y = x4 < 0, f(x1 , x2 , x3 , x4 , x5 )

42

,,
Chuong 1. Nguyn l bt bin

,,
c
o gi
a tri. fc v`
a sau khi thao t
ac tai
c n`
ay f(x1 , x2 , x3 , x4 , x5 ) c
o gi
a tri.
. buo
,
,
fm fc = 2(x1 + x2 + x3 + x4 + x5 )x4 < 0, v` tng cua c
fm . Khi do
ac
,
,
,
s x1 , x2 , x3 , x4 , x5 lun lun l`
a s duong. Nu thao t
ac trn khng d`ung,
,
,,
,
,,
ta t`m d
ay v han
uo. c d
. giam f0 > f1 > f2 > ... nhung s duong khng
,
,
,
,
m. D
ay s nhu vy
a tr`nh phai d`ung lai
. khng t`n tai,
. nn qu
. tai
. mt
. th`oi
,
dim.

` tp
Bai
.
,
,,
p
. 1.51. (My 1997). Cho p1 , p2 , p3 , ... l`
a nhung s nguyn t, d
uo. c sa
,
,
,
,
tu. t
theo thu
ang d`n v`
a cho x0 l`
a mt
a 1. Cho s nguyn
. s thu. c giua 0 v`
,,
duong k, dinh
. ngha

0

nu xk1 = 0,
xk =
pk

nu xk1 6= 0,
xk1
,
,,
`ng vo
,ng minh ra
,i
od
a ph`n thp
ay t`m v`
a chu
y {x} k hiu
. l`
. phn cua x. H
,
,
,i 0.
tt ca x0 thoa m
an 0 < x0 < 1 th` d
ay s x0 , x1 , x2 , ... tin to
,
,
,,
. 1.52. (My 1993). Cho a, b l`
a nhung s duong le. inh
ay (fn )
. ngha d
, ,
,
,
,
,
`

i n 3 l`
c s le lo
n nht cua
ng c
. t f1 = a, f2 = b v`
ba
ach da
a ly fn vo
a uo
, ,
`ng s vo
`ng fn l`
,ng minh ra
,i n du lo
n v`
fn1 + fn2 . Chu
a ha
a x
ac d
a
inh
. gi
,
,
tri. n`
ay nhu h`
am cua a v`
a b.
,
,
,,
,,
2
2
. 1.53. T`m tt ca nghim
. nguyn duong cua phuong tr`nh x + y =
3(z2 + w2 ).
,
`ng c
,ng minh ra
. 1.54. (Paul Zeitz). Cho mt
ac
` thi. huu han
. d
. bt k`. Chu
,
,
,
,,
,

ng sao cho, vo
i mi d
dnh c
o th t m`u d
a tra
en v`
nh, t nht mt
. nua
,
,
,
,,
,,
,i m`u n
nhung d
od
o c
o.
nh ln cn
uo. c t m`u nguo. c lai
. cua n
. vo
,
,,
. 1.55. Mt
on duo. c chia ra l`
am 2000 re quat.
o 2001 con ch
. h`nh tr`
. C
,
,
trong c
ac re quat
ay. Lun lun t`n tai
ung mt
. n`
. hai con ch trong c`
. re

,ng b`
1.5. Nhu
ai to
an nng cao

43

,
,
, hai con ch nhu, vy
,i hai re quat
quat;
(theo chi`u
. cu
. nhay to
. bn canh
.
,
,
,
,,
,
`
ng minh ra
ng tai
t nht 1001 re
nguo. c nhau). Chu
ao d
o
. mt
. th`oi dim n`
quat
o ch.
. d`u c

,
` toan
nng cao
1.5. Nhung bai
,
,
,,
,
V du. 1.56. Mt
anh n
am manh; mt
uo. c xe th`
. t`o giy d
. s trong s 5 manh
,
,
, ,
,
,
,,
a, v`
nho n`
ay lai
anh 5 manh nho nu
a mt
ac manh nho
uo. c xe th`
. d
. s trong c
,
,
,
,
n`
ay lai
anh 5 manh, ... Vy,
o
uo. c xe tip th`
. d
. nu cu tip tuc
. xe nhu vy
. th` c
,
,
,,
,,

khi n`
ao ta duo. c 2002 manh giy hay khng? uo. c 2005 manh giy khng?
,
,
,
`,i giai. Khi ta chia t`o, giy l`
Lo
am 5 manh v`
a sau n`
ay chia c
ac manh giy ra
,
,
,
,
l`n s manh
, mi
l`
am 5 manh nho th` cu
giy t
ang thm 4. Vy
. s manh
,
l`n xe th` c
,c n`
giy, sau mi
o dang
4k + 1(k N ), biu thu
ay l`
a bt bin
.
trong qu
a tr`nh xe giy.

,
,
,,
V` 2002 6= 4k + 1, nn khng th xe duo. c 2002 manh.
,
,
, 501.
2005 = 501.4 + 1, vy
o th xe th`
anh 2005 manh sau l`n thu
. c
,
,,
d
V du. 1.57. Trong mt
o 8 8 . Trong mi
uo. c vit mt
.
. bang vung c
,
,
,`,
,

s tu. nhin. Nguoi ta cho phep thu. c hin


ac: T
ang mt
on vi. d
i
. mt
. thao t
. d
,
,
, ,
ng bang vung c
v
oi tt ca c
ac s na
`m trong nhu
o chi`u 3 3 hoa
. c 4 4
,
,
,
,

cho. Hoi sau khi thu. c hin


thuc
ac nhu trn
a
. bang vung d
. mt
. s thao t
,
,ng s trong bang c
th` to`
an b. nhu
o chia ht cho 10 khng?

,
,
,
, ,
,
,
`,i giai. Hin nhin t`n tai
a c
o th thu. c hin
Lo
. nhung bang m`
. nhung thao
,
,
,
,
,
,
t
ac trn. Nhung ta ch ra c
o nhung bang khi thu. c hin
ac trn
. nhung thao t
,
,
,

khng th cho kt qua. V` tnh cht chia ht cho 10, ta xet nhung s theo
,
,
,,
,
,
modulo 10. Hai bang s ta coi nhu nhau khi v`
a ch khi nhung ph`n tu
,
, , ,
,
,i d`ng nht nhu, vy
tuong u
ng cua n
od
o
`ng du theo modulo 10. Vo
. ta c
,
,
,
,
64
10 bang kh
ac nhau. Ta se ch ra thao t
ac cua ch
ung ta l`
a thun
. lo. i nu

44

,,
Chuong 1. Nguyn l bt bin

,
,
,
,, , ,
,,
, hai c
mt
ung c
o th nhn
ac, th` bang thu
uo. c t`u bang kh
uo. c
. bang nhn
. d
. d
,
,
,
p dung
l`
t`u bang ban d`u. i`u kin
aa
thm 9 l`n thao t
ac ta d
am.
u l`
a
. d
.
, , ,
,
,,
,
ng minh t`n tai
Theo phuong ph
ap nhu vy,
a n
o khng
u d chu
. d
. bang, m`
,
,
, , `, ,
,,
,
,

th nhn
an s khng. S luo. ng cua nhung h`nh vung co
. duo. c tu bang to`
,
h`nh vung nhu,
3 3 l`
a 36, c`
on h`nh vung co 4 4 l`
a 25. V` trong mi
,
, 9 a
p dung
p dung
vy
o th a
thao t
ac nhi`u nht 9 l`n (sau l`n thu
se
. ta c
.
.
,
,
,
,
,,
,
,,

at), s luo. ng chung cua nhung bang m`


a ta c
o th
nhn
. duo. c bang xut ph
,
,, , ,
,,
,
ng d

nhn
a 1061 . Nhu vy
inh
. da
. duo. c t`u bang khng, khng vuo. t qu
. kha
,,
,
ng minh.
duo. c chu

,
,,
V du. 1.58. Trong mt
o 100 100 d
uo. c d
i`n du cng
. bang vung c
.
,
,,
,,
,
,
`
ng du o mt
(+). Mt
ng c
ach d
an b. nhu
ang hoa
. c
i to`
. buoc thu. c hin
. ba
. h`
,
,
,
,
,
,
,
u han
sang du nguo. c lai.
mt
ao d
o kha n
ang sau hu
o
. ct
. n`
. C
. buoc nhu trn,
,
,,
,
` () ?
ng 1970 du tru
bang vung nhn
od
uo. c se c
u
. d
,
,
, ,
,
,,
,, ,
`,i giai. Gia su, c
Lo
o kha n
ang sau mt
c nhn
o
. s huu han
. buo
. duo. c bang c
,
,
,
,

`
i ta d
j ta
d
1970 du tr`u. Cho tai
ang thu
on tai
a
i du xi ln, c`
. h`
. ct
. thu
,
,

`
di du yj ln. Khi do
du tai
thay di xi + yj ln. Suy ra tai
da
. (i, j) da
.
,
,
,
,
,

n`
ay c
o du tr`u () khi v`
a ch khi xi + yj l`
a s le. Cho p l`
a s luo. ng s
,
,
,
,
,
,
,
s
le giua c
ac s xi , c`
on q l`
a s luo. ng nhung s le giua c
ac s yj . Khi d
o
,
,,
,
,
luo. ng chung nhung du tr`u trong bang se l`
a
p(100 q) + (100 p).q = 100p + 100q 2pq,
,,
,, ,
,c
ng thu
od
y ta nhn
a
. duo. c d
100p + 100q 2pq = 1970
hay l`
a
(p 50)(q 50) = 1515 = 15.101.
,,
,

Boi v` 101 l`
a mt
. s nguyn t, t nht mt
. trong nhung s p50, q50 chia
,
l`
ht cho 101. Ta cho d
a p50 chia ht cho 101. Nhung 50 p50 50,
o

,ng b`
1.5. Nhu
ai to
an nng cao

45

, ,
,
,i
nn su. chia ht p 50 cho 101 ch xay ra khi p 50 = 0, di`u n`
ay tr
ai vo
,
,c (p 50)(q 50) 6= 0.
ng thu
bt da

,
,
,
`, (),
nh du tru
V du. 1.59. Tai
ac d
nh A1 cua mt
a gi
`u 12 dnh d
a
. d
. d
,
,
,
,
,,
nh du cng
ad
c`
on tt ca c
ac d
on lai
(+). Mt
i
nh c`
a
. l`
. d
.
. buoc thu. c hin
,
,,
,
,
d
anh du nguo. c lai.
`ng
`ng th`oi ba du tai
nh lin tip th`
. ba d
. Chung minh ra
,
,
,
,
,
,
u han
khng c
o kha n
ang sau mt
. s hu
. buoc thu. c hin
. nhu trn se nhn
.
,
,
,,
,

`
d
u
o
c
A
c
o
d
u
tr
u
(),
c`
o
n
t
t
c
a
c
a
c
d
nh
c`
o
n
l
ai
mang
d
u
c
ng
(+).
.

2
.
.
,
,
,
`,i giai. Ta chia c
Lo
ac dnh cua d
ac d`u 12 canh
ra l`
am 3 nh
om:
a gi
.
{A1 , A4 , A7 , A10 }, {A2 , A5 , A8 , A11 } v`
a {A3 , A6 , A9 , A12 }.
,
,
dnh ro,i v`
D thy khi chon
ao mt
om. Suy ra
nh lin tip th` mi
. ba d
. nh
,
,
,
,
,
,
,
l`n di du o ba d
nh
sau mi
om
nh lin tip, s luo. ng du tr`u trong mi
,
,
,
,
,
. c giam d
t
ang ln hoa
om hai
i 1. T`u d
y suy ra s luo. ng du tr`u trong nh
,
,
n le. Tht
t d`u choi th` s
v`
a nh
om ba lun lun c`
ung tnh cha
. vy,
. khi ba
,
,
,
,,
,
,
`ng 0. Sau l`n d
nht c
,
luo. ng du tr`u ba
o 1 du tr`u, sau l`n di thu
i thu
,
,
nh
, ba l`
,
. c 2, sau l`n di thu
. c 3, sau l`n di thu
hai mi
om c
o 0 hoa
a 1 hoa
,
. c 4 v`
tu l`
a 0, 2 hoa
a vn vn.
,
,
,,
,
,
Tru`ong ho. p ring khng th c
o su. phn b trong nh
om hai c
o mt
. du
,
tr`u m`
a c
ac nh
om kh
ac khng c
o.

,
,
,
,ng ph`n
V du. 1.60. Cho bang s (h`nh 1.10) c
o tnh cht sau: Tng cua nhu
,,
,,
h`
ct
tu trong mi
ang, mi
. c d
ac cho
u`ong cheo chia ht cho 2. Mt
. hoa
. thao t
,
,
,,
,
phep chuyn mt
d
o
n
v
i
o
m
t

sang

bn
c
anh
(
bn
c
anh
c
ua
m
t
a
.
.
.
.
.
. l`
,
, ,
,
,
` h`nh 1.10 nhn
c
o chung canh).
C
o th tu
uo. c h`nh 1.11 sao cho tt ca
.
. d
,, ,,
ph`n tu o c
ac xung quanh l`
a s cha
n khng?
,
,
`,i giai. T`u, gia thit b`
. p s a2 , a7 v`
Lo
ai to
an suy ra ca
a a4 , a5 l`
a c`
ung tnh
, ,,

n le. Boi v` a1 + a4 + a6 v`
cha
a a6 + a7 + a8 chia ht cho 2 suy ra a1 + a4 +
2a6 + a7 + a8 chia ht cho 2, ngha l`
a a1 + a4 + a7 + a8 chia ht cho 2.

,,
Chuong 1. Nguyn l bt bin

46
a1

a2

a3

b1

b2

b3

a4

a5

b4

b5

a6

a7

a8

b6

b7

b8

H`nh 1.10

H`nh 1.11

,
cho ta c
C
ung t`u di`u kin
o a1 + 2 + a8 chia ht cho 2, ngha l`
a a1 + a8
. da

a4 + a7 chia ht cho 2 v`
chia ht cho 2. Khi d
a suy ra a4 v`
a a7 c`
ung tnh
o
,
,
,

n le.

n le. Nhu vy
cha
a a2 , a4 , a5 v`
a a7 c`
ung tnh cha
i d
n kt lun
. ta d
. l`
,
,
n, th` a1 , a3 , a6 v`
. c tt ca
Nu a2 , a4 , a5 v`
a a7 l`
a nhung s cha
a a8 hoa
,
,
,
,
,,
n hoa
n l`
. c tt ca d`u le. Khi d
. c
s luo. ng cua nhung s cha
d`u cha
a 5 hoa
o
l`
a 9.
,
,
,
. c tt ca d`u
Nu a2 , a4 , a5 v`
a a7 l`
a nhung s le, th` a1 , a3 , a6 v`
a a8 hoa
,
,
,
,,
,
n hoa
n l`
. c tt ca d
. c l`
s luo. ng cua nhung s cha
cha
a 1 hoa
a 5.
`u le. Khi d
o
,
,
,
,,
,
n trong bang l`
. c 9).
Nhu vy,
a mt
. s luo. ng nhung s cha
. s le (1, 5, hoa
,
,
,
,
Ta xet su. bin di trn nhung bn canh
mt
a y l`
a nhung
.
. . Nu x v`
,,
s o hai bn canh
nhau, th` ta c
o thao t
ac x, y x 1, y + 1. Ta xet tt
.
,
, ,
,
,
,`,
,
n, cha
n), (cha
n, le), (le, le). Sau
n le cho x v`
ca c
ac truong ho. p cha
a y: (cha
, ,
,
,
, , , , ,
n), (cha
n,
khi thu. c hin
ac ta nhn
ng (le, le), (le, cha
uo. c tuong u
. thao t
. d
n).
cha
,
,
,
,
n cua s lu,o.,ng s
Nhu vy,
ac bin d
i khng thay di tnh cha
. thao t
,
,
,
n (v` thay di s lu,o.,ng 0 hoa
. c 2). Trong h`nh 1.10 c
o s le nhung s
cha
,
n, c`
n, ngha l`
n. Suy ra bang nhu,
a s cha
on trong h`nh 1.11 c
o 8 s cha
cha
,,
,
vy
ac thao t
ac trn.
. khng nhn
. duo. c khi thu. c hin
. c

,
,
ta ghi
V du. 1.61. Cho mt
a trong mi
. bang h`nh vung ke 10 10 v`
, ,
,
,
`, s 1 d
`, 1
theo thu tu. mt
ang thu nht ghi tu
n s 100: H`
. s tu. nhin g`m tu
,
,
,
,
`, 11 d
d
ang thu hai ghi tu
`ng tng S cua 10 s
n 10; h`
n 20; ... Chung minh ra

,ng b`
1.5. Nhu
ai to
an nng cao

47

,
,
khng c
bt k` cua bang, trong d
o bt k` hai s n`
ao thuc
ung mt
ang
o
. c`
. h`
,
v`
a khng c
o bt k` hai s n`
ao thuc
ung mt
a mt
i. T`m
. c`
. ct,
. l`
. s khng d
s S.
,
,
,
`,i giai. Ta k hiu
Lo
. s hang
. cua tng S:
- Thuc
ang 1 l`
a a1 ;
. h`
- Thuc
ang 2 l`
a 10 + a2 ;
. h`
- ...
- Thuc
ang 10 l`
a 90 + a10 .
. h`
,
,
,
, c
Trong do
ac s tu. nhin a1 , a2 , ..., a10 bao g`m giua 1 v`
a 10, v`
a nhung
s n`
ay di mt
ac nhau, v` nu ta c
o a1 = a2 th` hai s a1 v`
a 10 + a2
. kh
,
,
,
`m trong c`
phai na
ung mt
o:
. ct
. cua bang. Ta c
S = a1 + (10 + a2 ) + (20 + a3 ) + + (90 + a10 )
= (10 + 20 + + 90) + (a1 + a2 + + a10 )
= 450 + (a1 + a2 + + a10 ).

,,
,
Boi v` c
ac s a1 , a2 , ..., a10 di mt
ac nhau v`
a nhn
a tri. nguyn t`u
. kh
. gi
,
,
,
mt
. t trong tng
1d
ac s tu. nhin t`u 1 dn 10 c
o ma
n 10, mi
. s trong c
,
,
,
i tu c

a1 + a2 + + a10 vo
ach l`
a mt
c
ung ch c
o mt
o
. s hang,
.
. l`n, do d
a1 + a2 + + a10 = 1 + 2 + 3 + 4 + 5 + 6 + 7 + 8 + 9 + 10 = 55.

,
,,
,i moi
Nhu vy
ad
ach chon
ai
i vo
. S = 450 + 55 = 505 l`
. luo. ng bt bin d
. c
.
,
,
tng c
ac s trong bang.

,
,,
,,
,,
`ng nhau.
V du. 1.62. Cho khi lp
. phuong tao
. boi 27 khi lp
. phuong nho ba
,
,
,,
,
khi lp
Trong mi
o chua mt
. c 1. (Hai khi nho
. phuong nho c
. s +1 hoa
,
goi
a canh
nhau nu ch
ung c
o chung mt
. t). Ta goi
. t ca
t" cua khi
. l`
.
. ma
. "ma
,
,
,,
,,
,ng khi lp
lp
a tt ca nhu
nhau v`
a na
`m trong
. phuong l`
. phuong nho canh
.
,
,
,
,
mt
. t pha
ng". C
o th thay d
. t ca
t c
o chung
i du d
`ng th`oi trong hai ma
. "ma

48

,,
Chuong 1. Nguyn l bt bin

,
,
,
,,
,
,,
,ng khi lp
khi lp
i du nhu
. phuong nho
. phuong nho, nhung khng thay d
chung.
,
,
,
,
,,
Ban d
ac khi nho c
o chua s 1. C
o th sau mt
`u trong tt ca c
. s buoc
,
, ,, ,
,
,
,,
thay d
ac khi nho o d
i du nhu trn th` c
nh cua khi lp
. phuong mang 1,
,
,,
,ng khi nho c`
c`
on nhu
on lai
`u mang +1 duo. c khng?
. d
,
,
,
,,
,
,,
`,i giai. Ta di x
Lo
ac dinh
i s luo. ng cua 1 trong khi lp
. su. thay d
. phuong
,
,
,,
,,
t
,i mt
. t ca
nho trong khi lp
c bin d
i bt k`. Trong hai ma
. phuong vo
. buo
,
,
,
,
,
,
nhung du d
o s luo. ng 12 s. Nu n trong ch
ung l`
a 1, c`
on
uo. c thay di c
,

sau khi bin di ta c


12 n l`
a +1, khi d
o n s +1 v`
a (12 n) s 1, ngha
o
,
,
,,
,,

l`
a s luo. ng cua 1 d
a 12 n n = 2(6 n), d
a mt
uo. c thay di l`
y l`
. s
,
,
,
,
,
,
,
,
buo
n. Suy ra tai
n le cua s luo. ng cua 1 giu,
c bin di tnh cha
cha
. mi
,
,,
nguyn. Ban d
a 27, c`
on ta mun c`
on 8 (trn c
ac d
`u s luo. ng s 1 l`
nh
,
,
, ,
,,
cua khi lp
ay khng th xay ra.
. phuong). Hin nhin di`u n`

,
,,
,,
V du. 1.63. Cho khi lp
`ng
. phuong bao g`m 27 khi lp
. phuong nho ba
, ,
,
,
,
,
,
,
,
ng khi lp
ng s
nhau. Trong nhu
. phuong nho cua khi lp
. phuong chua nhu
,
,
,,
,,
+1. Mt
o th thm c`
ung mt
ao hai khi lp
i ta c
. buoc bin d
. s v`
. phuong
,
,
,,
`, khi lp
cho
nho bn canh
o chung ma
. t). Tu
a
. nhau (hai khi nho c
. phuong d
,
,,
,,

c
o th nhn
uo. c khi lp
. d
. phuong sao cho
,
,
, ,,
,,
a) Trong tt ca khi lp
o 0, c`
on khi nho o trong
o 1;
. phuong nho c
. tm c
,
,
,
,
,,
,
b) Trong tt ca khi lp
o 0, c`
on khi nho o trong
. phuong nho c
. tm +1 ?
,
,
,,
,
,,
`,i giai. a) C
n.
Lo
o th nhn
uo. c khi nhu vy,
. d
. ban
. doc
. t`m mt
. phuong a
,
,
ng, nhu,ng khi nho
b) Ta c dinh
a goi
o l`
a khi tra
. mt
. khi nho v`
. n
,
,
,
canh
n
o l`
a nhung khi den, bn canh
nhung khi d
a nhung khi
en se l`
.
.
,
,
khi nho se hoa
ng v`
ng hoa
. c l`
. c l`
mi
nho tra
a vn vn. Khi d
a tra
a den,
o
,
,
,,

ng c
mi khi tra
o khi nho bn canh
l`
a den v`
a nguo. c lai.
a tng
.
. Cho Pt l`
,
,
,
,
ng, Pd l`
nhung s trong c
ac khi nho tra
a tng c
ac s trong c
ac khi nho

,ng b`
1.5. Nhu
ai to
an nng cao

49

,
, ,
,
,c P = Pt Pd khng thay di vo
i thao t
cho,
den. Biu thu
ac thay di da
,
v` ta cng
thm c`
ung mt
ao hai khi nho canh
nhau (ngha l`
a Pt v`
a
.
. s v`
.
,
,
,
,

Pd d`u gia t
ang nhu nhau). D tnh d
a tri. ban d
a 1, nu
uo. c gi
`u cua P l`
, ,,
,
,,
,

`i hoi b) lai
khi nho o tm l`
a den (P = 14 13). Khi lp
o
. phuong nhu do
. c
,
,,
`

P = 0 1 = 1. y l`
a diu v l, ngha l`
a khng th c
o khi lp
. phuong
, `
nhu diu kin
. b).
,
,
,
,,
,
Nhung b`
ai to
an t`m s buo
c thu. c hin
a rt kh
o v`
a
at
. d d
. kt qua l`
,
,,
,
khng c
o phuong ph
ap chung n`
ao. Ta xet mt
on gian sau d
y:
. v du. d

,
, ,
s
V du. 1.64. Trn bang vit mt
ay g`m N s theo mt
. d
. thu tu. bt k`, mi
,
,
hoa
d
. c l`
a 1 hoa
. c l`
a +1. Mt
ac s
o
i cho phep ta thay d
i du c
. phep bin d
, ,
,
,
,
,
,
o th d chuyn
trong mt
ay. T`m s buoc thu. c hin
. t nht c
. doan
. con cua d
,
`, d
tu
ay ban d
ay ch to`
an s +1.
`u v` d
,
,
,,
,,
,
`,i giai. ai
. p s
Lo
ai n`
ay l`
a s luo. ng c
ac ca
. luo. ng bt bin don diu
. cua b`
,
,
trong d
ay canh
nhau kh
ac du. Khi ta thu. c hin
.
. phep bin di thay du
,
,
,
,,
,n ho,n 2.
mt
o th thay di khng lo
. doan
. th` dai
. luo. ng bt bin ch c
,
,,
`ng t`u, b. s 1, +1, 1, +1, ... d nhn
,ng minh ra
Ta se chu
. duo. c b.
,
,, ,,
,
,i [ N+1
a ph`n
s +1, +1, +1, ... phai thu. c hin
y [x] k hiu
. l`
. vo
2 ] buoc (o d
,
,
,`,
,
n v`
nguyn cua x). Ta xet hai truong ho. p s N cha
a s N le.
, ,,
,,
,
d
Gia su N = 2k, ngha l`
a [ N+1
o
ai
iu
. luo. ng bt bin don d
.
2 ] = k. Khi d
,
,
,
,
`ng khng. Nhung vo
i k 1 buo
c thu.,c
ban d
a 2k 1, c`
on cui c`
ung ba
`u l`
,
hin
. se khng dat
. kt qua.
, ,,
,`,
,
Gia su N = 2k + 1, ngha l`
a [ N+1
ay d
ai
. p n`
.
2 ] = k + 1. Trong truong ho
,,
,,
,
,

luo. ng bt bin don d


a 2k, v` th sau k 1 buoc thu. c hin
iu
. ban du l`
.
,
,
,
`

ng minh
th` t`u trang
th
ai ban d
o d
ay to`
an s 1. Ta se chu
u khng th c
.
,
,,
,
,,
`

ng sau k buoc thu. c hin


ra
ai mong mun. Tht
uo. c trang
. phai dat
. d
. th
. vy,
.
,
,
,,
,
`
`

ng sau mt
ta ch
u y ra
i d
ai
on
. ln thu. c hin
. phep bin d
. luo. ng bt bin d
,
,
diu
a mt
.
. thay di khng qu
. don vi.

50

,,
Chuong 1. Nguyn l bt bin

,
,,
,
,
`ng vo
,ng minh ra
,i [ N+1
Ch c`
on chu
. c hin
. th` t`u b. s ban
2 ] buoc thu
,
,
,
,,
o 1. Ta dua v`
ao trong b. s ban d
ac
d`u se nhn
`u tt ca c
. duo. c b. s ch c
,
,
,
,
,
,
N+1
a 1 s luo. ng khng lo
n hon [ 2 ]. Tip tuc
nh
om c
o chu
. ta thay di du
,,
,

o c
ac s trong nhung nh
om n`
ay.

` tp
Bai
.
,
,
,
,
,i nhau th`
. t pha
ng cho N dim, t`u ch
. 1.65. Trn ma
ung c
o th ni vo
anh
,
,
,
,
`ng t`u mt
ng. Bit ra
nhung doan
at qu
a 11
. tha
. dim bt k` khng xut ph
,
,
,
,
,
`ng nhung d
`ng 4 m`
ng minh ra
ng. Chu
doan
ay c
o th t ba
au sao
im n`
. tha
,
,
,
,
n hon N.
ng c
cho nhung doan
o hai d
ut c`
ung m`
au khng lo
`u m
. tha
,
,
,
. 1.66. Cho mt
au d
a mt
au xanh. Mt
o v`
. s dim m`
. s dim m`
. s trong
,
,
,
`
i nhau th`
ng. Ta n
ng mt
ch
ung ni vo
anh d
oi ra
a da
. c bit,
oan
im l`
.
. tha
. d
,
,
,,
,
,
,
i n
i m`
nu hon mt
ac dim c`
on lai
o c
o m`
au kh
ac vo
au cua n
o.
. nua c
. ni vo
,
,
. c bit
. c bit
Nu t`n tai
ay v`
a t ln n
o m`
au
im da
. d
. th` ta chon
. dim da
. n`
,
,
,
,
,
`
ng minh ra
c khng c`
ng sau mt
. c
kh
ac. Chu
on mt
. s huu han
. buo
. dim da
bit
ao.
. n`
,
,
,,
. 1.67. Trn mt
on ta vit n s tu. nhin. Giua hai s canh
nhau
. du`ong tr`
.
,,
,
,
,

ta x
ta vit uoc s chung lon nht. Sau do
oa nhung s c
u di, nhung s c`
on
,
,
,
`

ng sau mt
lai
ac trn. Chung minh ra
. ta lai
. thu. c hin
. thao t
. s huu han
. l`n
,
,
,
,
`ng nhau.
thu. c hin
ac th` tt ca c
ac s trn d
on d`u ba
u`ong tr`
. thao t
,
,
. 1.68. Trn bang ta vit 10 s: Mt
a chn s kh
ac 0. Ta thu. c hin
. s 1 v`
.
,
`

chon
hai
s
b
t
k`

v`
a
thay
v`
a
o
v
i
tr

c
ua
ch
u
ng
b
a
ng
trung
b`
nh
c
ng
hai
s
.
.
.
,
,
,
,
,
n`
ay. S n`
ao l`
a s nho nht c
o th dat
ay
. duo. c tai
. vi. tr cua s 1 sau mt
. d
,
phep thu. c hin
. trn.
,ng th`
. t v`
. 1.69. Mt
anh mt
ong tr`
on v`
a quay ma
ao
. s em thiu nhi du
. v`
,
,

`
trong, mi em d
o mt
dng th`oi, mi em dua
u c
. s keo.
. Theo hiu
. lnh
.
,
,
,
,,
,,
l`
nua s keo
ao do
a
. cua m`nh cho ban
. o bn phai (nu s keo
. cua mt
. em n`

1.6. Chuyn d
am bt bin
` v` h`

51

,
,,
,
,
mt
le, th` ngu`oi phu. tr
ach dua thm cho em d
ai keo).
Hiu
choi
o
. c
.
. lnh
.
,
,
`ng dn mt
,ng minh ra
. p d
. p lai
nhu vy
uc, tt ca c
ac
i la
. la
. nhi`u l`n. Chu
. l
,
`
ng nhau.
em trong cuc
o s keo
. choi c
. ba
,
,u ting Anh.
. 1.70. Trong mt
a s
ach c
o N tp
ach tra cu
. thu vin
. trn gi
. s
,,
,
mt
Mt
ay mi
ut l`
am nhung cng vic
. ngu`oi m
. ph
. sau: Chon
. mt
. tp
. bt
,,
. t n
ng vi. tr v`
ng vi. tr (ngha l`
k`, m`
a n
o khng o d
a da
o v`
ao du
a nu s
u
,`,
,
ng minh
. t tp
tp
a k, th` nguoi m
ay da
ay v`
ao vi. tr k trn gi
a s
ach). Chu
. l`
. n`
,
,
,
,
`ng sau mt
p du
ng vi. tr.
ra
ac tp
ach se duo. c sa
. th`oi gian tt ca c
. s
,
,
,,
ma
. t cua mt
v`
. 1.71. Mi
o vit s trn do
a tt ca
. khi lp
. phuong c
,
,,
s d
`ng trung b`nh cng
c
ac s d`u khng ging nhau. Mi
uo. c thay ba
. cua
,
,
,,
. t bn canh.
nhung s trong bn ma
C
o kha n
ang nhn
ac s ban d
`u
.
. duo. c c
,
,
. t di din
trn ma
. sau t nht mt
. l`n thay di nhu trn khng ?
,
,
,
cua
. 1.72. Trn mi
bang ke vung 8 8 c
o ghi mt
ay l`
a tch
. s, s n`
,
,
,
,
cua ch s h`
ang v`
a ch s ct
a trong c
ac y
. cua y. Ly ra 8 bt k` v`
`
,ng
m trong c`
ung mt
ang hay c`
ung mt
khng c
o hai n`
ao na
. h`
. ct.
. Chu
,
,
`m trong c
`ng tch cua c

ac n`
ay l`
a khng d
minh ra
ac s na
i. Tnh tch d
o
2
(
ap s: (8!) = 1625702400).
,,
,
k
ta nhn
. 1.73. Cho mt
ac s +1 v`
a 1. T`u d
o
. b. s luo. ng 2 c
. b. s
s nhn vo
`ng c
,i s tip theo, s cui c`
,i s d
,i ba
ach: Mi
ung nhn vo
mo
`u
,
,
,
`ng
ng minh ra
i b. s mo
i lai
. p lai
tin. Vo
ac trn v`
a tip tuc.
. la
. thao t
. Chu
,
,
,
cui c`
ung ta nhn
o s +1.
uo. c ch c
. d

` bt bin
1.6. Chuyn d
` v` ham
,
,
,
,
`ng c
Ta tng qu
at h
oa nhung c
ach giai loai
ai to
an n`
ay ba
ach dua ra
. b`
,
,
,,
,,
,
nhung kh
ai nim
ai tuong duong v`
a nhung h`
am bt bin.
. nhung trang
. th

52

,,
Chuong 1. Nguyn l bt bin

` bt bin trn trang

1.6.1. .inh ngha ham


thai
.
,
,
,,
,
,,
`ng c
xet nhung b`
Nhung ph`n truo
c ta da
ai to
an giai ba
ac dai
. luo. ng bt
,
,
,
,
,
,
bin c
o dang:
Cho mt
o l`
a nhung trang
.
. tp
. ho. p M (nhung ph`n tu cua n
.
,
, ,

th
ai) v`
a mt
quy
t
a
c
bi
n
d

i
cho
ph
e
p
ta
chuy
n
t
u
tr
ang
th
a
i
n`
a
y
sang

.
.
,
,
,`,
`ng

trang
th
a
i
kh
a
c;
cu
h
oi
thu
o
ng
d
a
t
ra
l`
a
cho
m
t
tr
ang
th
ai c
o th ba

.
.
.
.
,
,
,
,
c bin di d
c thu. c hin
cho khng
mt
ai da
n mt
. s buo
. quy ta
. trang
. th
,
, ,
? N
oi c
ach kh
ac, cho hai trang
ai v`
a , c
o th bin di t`u trang
ai
. th
. th
,
,
c d
cho khng?
dn trang
ai sau mt
a
. th
. s huu han
. l`n thu. c hin
. quy ta
,
,
Hin nhin, ta c
o tnh cht sau dy: Nu mt
th
ai c
o th tin
. trang
.
,
,
,
,i trang
,i trang
to
th
ai v`
a t`u trang
th
ai c
o th tin to
th
ai , th` t`u c
o
.
.
.
,
,

`
i , tnh cht n`
th tin to
ay goi
a tnh ba
c cu.
. l`
,
,,
,
,
,i trang
Nu t`u mt
th
ai c
o th tin to
th
ai v`
a nguo. c lai
. trang
.
.
. t`u
,
,i trang
trang
th
ai c
o th tin to
th
ai , th` tnh cht n`
ay goi
a tnh d
i
.
.
. l`
,
,
,
ca
ng, v` vy
. p hai trang
xung. Khng phai mi
ai n`
ao c
ung c
o tnh d
i xu
. th
.
,
,

ta han
ch
ch

x
e
t
c
a
c
t
p
tr
ang
th
a
i
c
o
t
nh
d

i
x
u
ng.

.
.
.
,
,
`ng t`u trang
,i chnh m`nh, tnh cht
Ta cng nhn
ai c
o th tin to
. ra
. th
,
n`
ay goi
a tnh phan xa.
.
. l`
,,
,,
`ng hai trang
inh
ngha 1.1. Ta n
oi ra
th
ai v`
a l`
a tuong d
uong, nu
.
.
,
,
,,
c da
,i v`
cho t`u trang
theo quy ta
ai c
o th tin to
a nguo. c lai.
. th
. Hai trang
.
,,
,,
th
ai tuong d
a , k hiu
a : .
uong v`
. l`
,
,
`ng su., tu,o,ng du,o,ng cua nhu,ng trang
ai c
o tnh phan xa,
D thy ra
i
. th
. d
,
,
,
,
,
,
,
,

`
ng v`
p trang
c cu. Nhung lo
th
ai tuong d
xu
a ba
uong se phn chia tp
.
. ho. p
tp
trang
ai M th`
anh c
ac tp
a trong mi
. th
. con M = M1 M2 ..., m`
. con
,
,,
,,
,,
,
`
`

i nhau: Nu Mi v`
moi
ai cua n
o du tuong duong vo
a
. phn tu trang
. th
,,

Mi , th` . Nu Mi v`
a Mj (i 6= j) th` v`
a khng tuong
,,
,

duong. Mi ho. p con Mi ta goi


a mt
th
ai
ao.
. quy d
. Mt
. l`
. khng gian trang
.
,
,
,
cua b`
ai to
an l`
a ho. p nhung quy d
ac nhau.
ao
. kh

1.6. Chuyn d
am bt bin
` v` h`

53

inh
ngha 1.2. Mt
am s f x
ac d
ai M goi
a h`
am
inh
. trn tp
. h`
. trang
. th
. l`
.
,
,
,
,
,
,
,
. c d
bt bin hoa
th
ai tuong d
uong
ai
.
. luo. ng bt bin, nu trn nhung trang
n
o nhn
ung mt
a tri,
a
. ngha l`
. c`
. gi
nu th` f() = f().

(1.6)

,
,
,
, ,,
,,
T`u f() = f() khng th n
oi g` v` su. tuong duong cua hai trang
th
ai
.
,
,

i:
v`
a . Ta d
ao mt
am bt bin mo
ua v`
. loai
. h`
,,
inh
ngha 1.3. Mt
am bt bin f duo. c goi
a bt bin van
ang, nu
. n
. h`
. l`
.
,,
,,
,
hai trang
th
ai khng tuong d
o nhn
a tri. kh
ac
uong nhau th` n
.
. nhung gi
nhau:
nu 6 th` f() 6= f().
,
,i bt
Bt bin van
ang trn mt
a tri.
. V` th vo
. n
. quy dao
. nhn
. ch mt
. gi
bin van
ang
. n
f() = f().
,
,

. p trang
Nhu vy,
ang, cho mt
th
ai bt k` lun cho
. voi bt bin van
. n
. ca
.
,
,
,
,
phep ta kt lun
ung c
o tuong duong hay l`
a khng.
. ch
, ,
,
Nhung l`
am th n`
ao d kim tra mt
am bt bin l`
a van
ang? Khng
. h`
. n
,
,
,,
,
,
c
o phuong ph
ap chung nhung c
o mt
. tiu chun don gian sau dy:
inh
l 1.1. Nu
.
,,
a) t`n tai
ai 1 , 2 , ..., ` sao cho moi
ai M tuong
. ` trang
. th
. trang
. th
,,
,
duong v
oi mt
ung v`
a
. trong ch
,u han
b) h`
am bt bin f nhn
a tri. kh
ac nhau,
. hu
. ` gi
,
ng trang
th` f l`
a h`
am bt bin van
ang v`
a nhu
th
ai i , j (i 6= j) d
i mt
. n
.
.
,,
,,
,
khng tuong d
oi nhau.
uong v
, `
, `
T`u d
a ` quy d
iu kin
ao.
iu kin
. a) cho ta t`n tai
. khng qu
. T`u d
. b) cho
, ,
, ,,
`
`
ta tn tai
. khng nho hon ` quy dao.
. Suy ra tn tai
. ` quy dao.
. Trn co so

,,
Chuong 1. Nguyn l bt bin

54

,
,
cua b) lai
a tri. kh
ac nhau, ngha l`
a f l`
a bt
. cho ta bt bin f ch nhn
. ` gi
,
,
`m o,,
bin van
ang. Cui c`
ung t`u a) cho ta nhung trang
ai 1 , 2 , ..., ` na
. n
. th
,,
,,
,i nhau.
di mt
c
ac quy d
ac nhau, do d
ao
o
uong vo
. kh
. khng tuong d
,
Ta ly v du. cu. th:
,,
on. L
uc d
V du. 1.74. Ngu`oi ta tr`ng n cy xung quanh mt
`u
. h` h`nh tr`
,
,
,
,`,

ba
ng cu nhu ai d
t nhip:
mi cy d
o, nhung nguoi ta thy ra
o
u
.
. mt
. con c`

c`
Mi l`n c
o hai ch
u c`
o trn hai cy n`
ao d
ung bay ln v`
a du
o
. sang cy bn
,
,
canh,
mt
on con kia lai
`ng h` cua b`o h`, c`
.
. con bay theo chi`u kim d
. bay
,,
,,
,

` trang
nguo. c chi`u kim d
th
ai ban d
o o mt
`ng h`. Tu
`u : Mi con c`
.
. cy,
,
,
,

theo nhip
ac ch
u c`
o n
oi trn, d
uc n`
ao d
i chuyn ch cua c
n mt
o
. thay d
. l
,
,
,
,
d
t
oi trang
ai : Tt ca c
ac ch
u c`
od
ao d
u
o
uo. c khng?
. th
. d`n trn mt
. cy n`
,
, ,
`ng c
`,i giai. Ta c
Lo
o th giai b`
ai to
an ba
ach t`m c
ac h`
am bt bin theo c
ach
kh
ac nhau:

(a)

(b)

(c)

H`nh 1.12
ng. Khi
C
ach 1: Nu n = 2m. Ta t m`
au mt
ach mt
. cy xanh c
. cy tra
,
,
,
l`n di ch
s c`
ng hoa
. c l`
mi
do
o trong cy tra
a khng thay di nhu h`nh

1.6. Chuyn d
am bt bin
` v` h`

55

,
,
,
. c l`
. c l`
1.12(a), hoa
a t
ang ln 2 nhu h`nh 1.12(b), hoa
a giam di 2 nhu h`nh
,
,
,i mt
1.12(c). Vo
ai bt k` cua nhung con c`
o ta k hiu
a s
. trang
. th
. () l`
,,
,
,

ng. Ta xet h`
con c`
od
au tra
am p() nhu sau:
u
. o nhung cy m`

n ,
0 nu () l`
a s cha
,
p() =
1 nu () l`
a s le .
,,
,
n nhu,ng con c`
Theo nhung nhn
am s p c
o s cha
o trn
. xet o trn th` h`
,

ng, l`
cy tra
a khng di v`
a p() = 0.
,
n
,
,i trang
Nu m = 2k + 1, th`
l`
a mt
a, vo
th
ai ngay t`u
. s le. Ngha l`
.
2
,
d`u ta c
o p() = 1. T`u p() 6= p() suy ra hai trang
th
ai v`
a khng
.
,,
,,
,
,`,
,
,
tuong d
uong. Nhu vy
. trong truong ho. p (n = 2m, m = 2k + 1) t`u trang
.
,
,i trang
th
ai khng th tin to
th
a
i
.
.
n
,
n v`
l`
Nhung c`
on nu m = 2k th` sao? Khi d
a s cha
a p() = p() =
o
2
,`,
,
,,
,
0. Trong truong ho. p n`
ay h`
am bt bin p khng d
a c
o
ua ra kt lun
. duo. c l`
,,
,
,,
tuong duong giua hai trang
ai v`
a hay khng.
. th
,
,
,i h`
Nhu vy
am n`
ay khng dua dn kt lun.
u y l`
a nu f l`
a
. vo
. Mt
. ch
,
,

`
v`
h`
am bt bin th` t`u f() = f() khng suy ra diu g` ca: Khi do
a c
o
, ,,
,
,,
,,
,,

th tuong duong v`
a c
ung c
o th khng tuong d
uong. Nu f() 6= f() th`
,,
,,
,
`
trang
th
ai v`
a khng tuong duong (diu n`
ay suy ra t`u d
inh
. ngha). Ta
.
,
,

c
o th thit lp
am bt bin theo kiu kh
ac.
. h`

i+1

j1
j

H`nh 1.13

i1

H`nh 1.14

,
trn
,i trang
nh s c
C
ach 2: Ta d
ac cy t`u 1 d
ai c
ac con c`
o d
a
n n. Vo
. th

,,
Chuong 1. Nguyn l bt bin

56

,
, k vo
,i trang
cy bt k`, ta k hiu
a s c`
o tai
ai n`
ay. By gi`o
. ak () l`
. cy thu
. th
ta xet h`
am
q() = 1.a1 () + 2.a2 () + 3.a3 () + + n.an ().

(1.7)

,
H`
am q() c
o phai l`
a h`
am bt bin?
,
,
,
,,
Mt
ai thay di bt k` duo. c m ta trn h`nh 1.13 v`
a tng (1.7)
. trang
. th
,,
, ,,
,
ch anh huong dn 4 th`ua s sau:
+ i.ai () + (i + 1).ai+1 () + (j 1).aj1 () + j.aj () +

(1.8)

,
,
,,
,i vic
Vo
ai trong h`nh 1.13 th` tng (1.8) tro th`
anh
. thay di trang
. th
+i.[ai ()1]+(i+1).[ai+1 ()+1]+(j1).[aj1 ()+1]+j.[aj ()1]+

,
,
,
,
, ,,
,,
`ng tng v`u,a thay d
D kim tra thy ra
a tng truo
c khi thay d
i v`
i o trn
,
ng, v`
l`
a tr`
ung nhau. Phai ch
ang q() l`
a h`
am bt bin? i`u n`
ay khng du
,
,
,
,
`ng c
c`
on ba kha n
ang nua nhu c
ac h`nh 1.14, 1.15, 1.16. Ba
ach xet tng
,
,
nhu trn th` h`nh 1.14, q() giam di n, c`
on h`nh 1.15, q() t
ang ln n.
,
,
,`,
,
,
Truong ho. p cui c`
ung h`nh 1.16, q() khng thay d
a tri. cua
i. Nhu vy,
. gi
,
,
,
q() c
o thay d
a s n. Suy ra, d
am r() gi
a
i nhung ch l`
inh
. ngha mt
. h`
,
,
,
,
tri. cua n
o l`
a c
ac s du cua phep chia q() cho n, l`
a mt
am bt bin. T`u
. h`
,
trang
ai tt ca con c`
od
u
. th
. trn mt
. cy ` th`

a1 () = a2 () = ... = a`1 () = a`+1 () = ... = an () = 0,
a` () = n.
,i bt k` ` v`
. t kh
Ngha l`
a q() = `.n v`
a r() = 0 (vo
a n). Ma
ac, a1 () =
a2 () = ... = an () = 1. Vy
.
q() = 1 + 2 + + n =

n(n + 1)
.
2

n
,i n l`
Nu n = 2m, th` q() = n.m + m v`
a r() = m 6= 0. Suy ra vo
a s cha
,,
,,
,,
v`
th` ta nhn
a khng tuong d
uong.
. duo. c r() 6= r(), do do

1.6. Chuyn d
am bt bin
` v` h`

i+1

57

i
1

H`nh 1.15

H`nh 1.16

,
Nu n = 2m + 1, th` q() = n(m + 1) v`
a r() = 0. Nhu vy
. trong
,
,
,`,
,
,
,
truong ho. p n l`
a s le ta lai
o r() = r(). Tru`ong ho. p n`
ay ta lai
o
. c
. khng c
,
,
,
,
,
,
kt lun
ai v`
a .
. g` v` su. tuong duong cua hai trang
. th
, ,
`ng r() l`
ng minh ra
Ta d`
ung dinh
a bt bin van
ang. Tht
. l 1.1 d chu
. n
.
,,
,
,

vy,
k

hi
u

l`
a
tr
ang
th
a
i
sau:
M
t
con
c`
o
o
trn
cy
th
u
i,
c`
o
n
nh
u
ng
i
.
.
.
.
,
,
,,
,
,,

con c`
on lai
d
u
o
cy
th
u
n.
V
y

d
a trang
ai tt ca c
ac ch
u

uo. c hiu l`
n
. .
.
. th
,
n.
c`
o d`u trn cy thu
,
,,
,,
`ng mt
,i mt
Ta se ch ra ra
ai bt k` d`u tuong duong vo
. trang
. th
. trong
,
,

nhung trang
ai 1 , 2 , ..., n . Tht
a trang
ai bt k` cua c
ac
. th
. vy,
. cho l`
. th
,
,

`
n. Theo phep
ng dn tt ca c
ac ch
u c`
o v cy thu
ch
u c`
o trn cy. Ta c ga
,
,,
,
,

ra ta tin h`
bin d
ai da
anh nhu sau: Con c`
o o cy s 1 bay theo
i nhu b`
,,
,,
,
con s 2 bay theo chi`u nguo. c lai.
huong v` cy s n, d
uc do
`ng th`oi l
.
,
`

lai
Sau d
a con d
o
u
. dn con s hai bay v cy thu n v`
. trn cy s 3 bay
,,
,,
,
,
`

,
theo chiu nguo. c lai,
od
u
. ... cu tip tuc
. nhu vy
. cho dn con c`
. o cy thu
, (n 1) v` cy thu
, n th` con c`
, n
(n 1). Khi d`n ch
u c`
o thu
o trn cy thu
,,
, i n`
(i = 1, 2, ..., n 1). i`u n`
se bay nguo. c lai
ao do
ay c
o
n mt
. d
. cy thu
,

ngha l`
a i . Ta tnh r(i ). Voi i 6= n:

a1 (i ) = a2 (i ) = ... = ai1 (i ) = ai+1 (i ) = ... = an1 (i ) = 0,


ai (i ) = 1,

an (i ) = n 1.
Suy ra, q(i ) = i.1 + n.(n 1) v`
a r(i ) = i. Ngo`
ai ra q(n ) = n.n v`
a

,,
Chuong 1. Nguyn l bt bin

58

,
,
r(n ) = 0. Nhu vy
a tri. kh
ac nhau.
. r nhn
. n huu han
. gi
Theo dinh
a van
ang v`
a c
ac trang
th
ai 1 , 2 , ..., n
. l 1.1 bt bin r l`
. n
.
,,
,,
di mt
a van
ang nn
. khng tuong duong. V` r l`
. n
r() = r().
,,
,,
,ng minh d
chu
Ph`n truo
c ta d
a
uo. c
,
r() = r() n l`
a s le.
,
,
,
,
,,
an
Suy ra khi v`
a ch khi n l`
a s le. Nhu vy
ai to
an d
uo. c giai ho`
. b`
to`
an.
, ,
,
,
V` nhung h`
am bt bin, ban
o th giai nhung b`
ai tp
. doc
. c
. sau:
`ng nu f l`
,ng minh ra
1. Chu
a h`
am bt bin v`
a g l`
a mt
am s bt k`,
. h`
th` h`
am h() = g(f()) c
ung l`
a h`
am bt bin.
,
`
ng minh ra
ng nu h l`
2. Chu
a bt bin, f l`
a h`
am van
ang, th` t`n tai
. n
.
,
,
c sau du
ng thu
ng h() = g(f()).
mt
am s g sao cho d
a
. h`
,
,
,
,
3. T`u h`
am van
ang r cua l`oi giai trong v du. 1.74 ta dinh
. ngha hai
. n
,
2
2
,ng minh
h`
am bt bin nua: f() = [r()] v`
a g() = [r() 2] . Chu

`ng h`
am bt bin f l`
a van
ang, c`
on h`
am g khng van
ang.
ra
. n
. n
,
,
,

`
i d
4. Cho h`
am f bt bin van
ang. Vo
am s g d cho
iu kin
. n
. g` cua h`
,,
,c h() = g(f()) l`
h`
am h d
ac dinh
a mt
am bt
uo. c x
. theo cng thu
. h`
bin van
ang ?
. n
,
1.6.2. H. thng bt bin d
`y du
,,
,,
ng le di t`m v`
Nhi`u khi da
a su dung
h`
am bt bin van
ang, ngu`oi ta
.
. n
, ,,
,
di t`m h. bt bin d`y du duo. c d
inh
. ngha nhu sau:
,
,
,,
inh
ngha 1.4. Mt
ad
`y d
u,
uo. c goi
. h. nhung bt bin {f1 , f2 , ..., fk } d
. l`
.

1.6. Chuyn d
am bt bin
` v` h`
,
,
,c
ng thu
nu nhung da

f1 ()

f ()
2

fk ()

59

= f1 (),
= f2 (),
...
= fk ().

(1.9)

,
,,
,,
,
ng khi v`
d`ng th`oi du
a ch khi trang
ai v`
a tuong duong.
. th
,
,
,
,
T`u dinh
a h. nhung bt bin d
a tng qu
at
`y d
u l`
. ngha ta thy ngay l`
h
oa kh
ai nim
ang: Nu f l`
a bt bin van
ang, th` h. {f}
. bt bin van
. n
. n
,
g`m mt
a h. d`y du.
. bt bin l`
,
,
,
,ng s nguyn. u,o.,c phep thay
V du. 1.75. Trn bang ke co 2 2 ta vit nhu
,
d
i:
,,
,ng s c`
1. Trn mt
od
on
uo. c cng
. ct
. bt k`, mt
. s trong n
. thm 2, nhu
,
` di 2;
trong ct
. tru
,,
,ng s c`
2. Trn mt
ang bt k`, mt
od
on
uo. c cng
. h`
. s trong n
. thm 3, nhu
,
` di 3;
trong h`
ang tru
,
,
,
,,
,
ng bang tu,o,ng d
Nhu
o l`
a nhu th n`
ao?
uong cua n
,
,
`,i giai. Ta xet ba h`
,i mt
Lo
am: vo
. bang bt k` =


a
c


b
, k hiu
.
d

p() = a + b + c + d,
, ,
q() = s du cua s a + b chia cho 2,
, ,
r() = s du cua s a + c chia cho 3.
,
,
,
Nhung h`
am p, q, r trn l`
a h`
am bt bin. Khng kh
o kim tra mt
. bang
,
,,
,,
,i bang sau
bt k` tuong duong vo


0
q()
=
.
r() p() q() r()

,,
Chuong 1. Nguyn l bt bin

60
,
,
,c
ng thu
Tht
ac d
a
. vy,
. t`u c

p() = p(),
q() = q(),

r() = r(),

(1.10)

,
,
,
,
,,
,,
,i mt
ng dinh
kha
a c`
ung tuong duong vo
a
. nhung bang v`
. bang, ngha l`
,,
,,
,
i nhau.
ch
ung tuong duong vo
,
,
,
,,
, ,,
,,
,c
ng thu
Nguo. c lai,
a keo theo c
ac da
. su. tuong duong cua hai bang v`
,
trong (1.10), v` th p, q, r l`
a bt bin. Nhu vy
a h. bt bin d
`y
. {p, q, r} l`
,
du.
,
,
,
Ta c
o th d d`
ang giai nhung b`
ai tp
. sau:
,
`ng nu f1 , ..., fk l`
,ng minh ra
1. Chu
a nhung h`
am bt bin v`
a g l`
a mt
.

h`
am s k d
am
i s, th` h`
h() = g(f1 (), f2 (), ..., fk ())

(1.11)

l`
a mt
am bt bin.
. h`
,
`ng nu h l`
ng minh ra
2. Chu
a mt
am bt bin v`
a {f1 , ..., fk } l`
a h. bt
. h`
,
,
bin d
am g c
okd
an
`y du, th` t`n tai
i s sao cho thoa m
. mt
. h`
(1.11).

,
,
,
,
,
. p s thu. c (x, y). Xet phep chuyn d
3. Cho M l`
a tp
i duy
. ho. p nhung ca
. t
nht (x, y) (y, x). a

f1 (x, y) = xy,
f2 (x, y) = x + y.
,
`ng h. {f1 , f2 } l`
,ng minh ra
Chu
a mt
`y d
u.
. h. bt bin d
,
,
,
,
,
. p s thu. c (x, y, z). Xet phep chuyn di
4. Cho M l`
a tp
. ho. p nhung ca
. t
(x, y, z) (y, x, z) v`
a (x, y, z) (x, z, y). a

f1 (x, y, z) = xyz,
f2 (x, y, z) = xy + yz + zx,

f2 (x, y, z) = x + y + z.

1.6. Chuyn d
am bt bin
` v` h`

61

,
`ng h. {f1 , f2 , f3 } l`
,ng minh ra
Chu
a mt
. h. bt bin d`y du.
, ,
,
Ban
o th tng qu
at h
oa b`
ai tp
ung n`
ay. Nhung b`
ai tp
oc
. d
. c
. cui c`
.
,
,
,
,

c. Nhung h`
c trong d
n`
ay l`
a nhung vn d
am n`
ay
su sa
ai
. s nhung da thu
,
,
,

khng nhung c
o nhung tnh cht bt bin m`
a c`
on nhiu ch lo. i kh
ac trong
,,
,
,
,, `
,

nhung lop d
ay duo. c d
a thuc, vn d n`
cp
. trong chuong tip theo dy.
,
,, ,
,
,
,
,
kt th
uc chuong n`
ay c
ac ban
ai tp
ac
y nh`o c
. thu giai nhung b`
. sau d
,
,
,
c ta d
kin thu
a thu nhn
. duo. c trong ph`n chuyn d`.

` tp
Bai
.
,
c m
. 1.76. Mt
ay vi tnh g`m hai ph`n nhu h`nh 1.17, mt
. cng ta
. bn
,
,
,

m v`
m. C
nh s t`u 1 dn n nhung
c
o n l ca
a mt
o th da
. bn g`m n d`u ca
,
ca
l`n ni cng ta
m sao cho vo
c mt
m ro,i v`
,i mi
l
ao
. trong nhung d`u ca
,
,
,

ng s cua m`nh duo. c khng?


l c
o du

2 3 4
1
n n1

H`nh 1.17

10 11 12

10 11 12

13 14 15
(a)

13 15 14
(b)
H`nh 1.18

,
,
,
,i mt
. 1.77. Tr`
o choi d
ac s trong bang 16 vo
a 15 c
o
y c
. trng v`
,
,
,
,,

c
ac s nhu h`nh ve 1.18. uo. c phep mt
y s cua mt
. ln d
. sang bn
, ,
, ,

canh
c`
on trng. C
o th thu. c hin
ac phep dy t`u trang
th
ai
.
. lin tip c
.

,,
Chuong 1. Nguyn l bt bin

62

,,
h`nh 1.18(a) sang trang
th
ai 1.18(b) duo. c khng? H
ay t`m h`
am bt bin
.
,
,
van
ang cua tr`
o choi n`
ay.
. n
,
,, ,
,

. 1.78. Mt
o nhi`u co 11 11 duo. c bo d
i 22 sao cho trn mi
. bang c
,
,
,
,,
,,
h`
ng 2 . Hai vi. tr nhung duo. c bo di
ct
a trn mi
ang duo. c bo di d
u
. v`
,
,
,,
,,
,
tuong d
ac ct
ung v`
a
uong nhau nu ta chuyn di mt
. s bt k` c
. giua ch
,
,
,
,
,
nhung h`
ang giua ch
ung, th` t`u vi. tr n`
ay nhn
o bao nhiu
. duo. c vi. tr kia. C
,
,
,,
,,
,
,
,
vi. tr khng tuong d
uong cua nhung duo. c bo di?
,
,
,,
. 1.79. Tt ca c
ac s nguyn t`u 1 dn 2n duo. c vit th`
anh mt
ang. Ta
. h`
,
,
,
,
,

`ng
ng minh ra
i mi s cng
i s l`
. Chu
thu. c hin
v`
ao vo
a vi. tr cua s d
o
. vo
.
,
,
,, `
ng c
trong nhung tng nhn
ach trn c
o hai s chia 2n cho c`
ung
. duo. c ba
,
mt
. s du.
,,
,
,i nhu,ng d
. 1.80. Vo
u c`
o c`
ung bay ln
i`u kin
. o v du. 1.74, nhung hai ch
,
,
. c c`
d chuyn sang cy kh
ac theo hai chi`u kh
ac nhau hoa
ung mt
. chi`u
,
,
,
d`u duo. c. H
ay tnh h`
am bt bin van
ang cua b`
ai to
an n`
ay.
. n

H`nh 1.19
,
,,
,
. t v`
. c 1.
. 1.81. Trong bang co 3 3 , ngu`oi ta da
ao c
ac c
ac s +1 hoa
,
,,
. c mt
Mt
a thay c
ac du o c
ac theo mt
ang hoa
i l`
. phep bin d
. h`
. ct.
.
,
`ng
ng minh ra
Chu
a) S quy d
a 16;
ao
. l`

1.6. Chuyn d
am bt bin
` v` h`

63

,
quy d
,a 32 ph`n tu,;
b) Mi
ao
. d`u chu
,
,
,
,
c) Tch cua tt ca c
ac s trong h`nh vung bt k` co 2 2 trong bang l`
a

bt bin;
,
,
,,
,
nh du trong bn h`nh vung nhu
d) Tch cua tt ca c
ac s duo. c d
a
,
h`nh 1.19, tao
anh mt
. th`
. h. bt bin d`y du.
,
, tu., bt k`; d
H
ay giai b`
ai to
an n`
ay trong c
ac ph`n trn theo mt
. thu
,
nhau.
thy c
ac ph`n d
up ln
`u tro. gi
,
,,
,
,,
,
. 1.82. Vecto (a, b), o d
a nhung s nguyn, duo. c bin di th`
anh
y a, b l`
,
,
mt
ay t`m h`
am bt bin
. trong nhung vecto (a + b, b), (a b, b), (b, a). H
,
van
ang cua b`
ai to
an n`
ay.
. n
,,
,
,,
,
. p vecto (a, b), (c, d), o d
. 1.83. Ca
a nhung s nguyn, duo. c
y a, b, c, d l`
,
,
. p vecto (a+b, b), (c+d, d); (ab, b), (cd, d);
phep bin d
anh c
ac ca
i th`
, ,
(b, a), (d, c). H
ay tnh h. bt bin d
ai to
an n`
ay.
`y d
u cua b`

, ,
CHUONG

,
,
c d
ng hai bin
a thu
i xu

` tnh cht . . . . . . . . . . . . . . . . . . . . . . . . . . . . . . . . . . . . . . . . . . . . . . . . . . 65
2.1. .inh ngha va
,
, ,
2.2. .inh l co ban cho da
uc hai bin . . . . . . . . . . . . . . . . . . . . . . . . . . . . . . . . . . . . 67
th
,
,,
,
2.3. Giai h. phuong tr`nh d
ung . . . . . . . . . . . . . . . . . . . . . . . . . . . . . . . . . . . . . . . . . . 71
i x
,
,
,
,
2.4. ua v` h. phuong tr`nh dang
d
ung . . . . . . . . . . . . . . . . . . . . . . . . . . . . . . . . 77
i x
.
,
,
,
,
ng th
2.5. Ch
ung minh bt da
uc d
ung . . . . . . . . . . . . . . . . . . . . . . . . . . . . . . . . . . . 82
i x
,
` toan
v` tam th
2.6. Bai
uc bc
. hai . . . . . . . . . . . . . . . . . . . . . . . . . . . . . . . . . . . . . . . . . . . 87
,
,
,
2.7. Phn tch da
uc d
ung ra th`ua s . . . . . . . . . . . . . . . . . . . . . . . . . . . . . . . . . . 91
th
i x
,
` toan
khac
. . . . . . . . . . . . . . . . . . . . . . . . . . . . . . . . . . . . . . . . . . . . . . . . . . . . 95
2.8. Nhung bai
,,
,
`
`
2.9. Chuyn d
ung . . . . . . . . . . . . . . . . . . . . . . . . . . . 101
v phuong tr`nh h. s d
i x

,
, ,
,c h. s di xu
,ng . . . . . . . . .
2.9.1. inh
. l co ban cua da thu
,
,
,,
2.9.2. Nhung v du. giai phuong tr`nh bc
. cao . . . . . . . . . . . .

, ,
,,
,
` tra l`oi bai
` tp
2.10. Go. i y va
. chuong 2 . . . . . . . . . . . . . . . . . . . . . . . . . . . . . . . . . . . . .

101
105
110

,,
,
,c hai bin, xet mt
,c
Chuong n`
ay dua ra kh
ai nim
a thu
. v` d
. loai
. da thu
,
,
. c bit

da
a c
o gi
a tri. khng di khi ta thay di c
ac bin cho nhau. Sau d
o
. l`
,
,
,
,
,
,
,
`

c d
ng. inh
ng minh chi
l`
a xet mt
inh
i xu
. l duo. c chu
. l co ban v da thu
. d
,
,
,

ng dung
tit v`
a du. a trn dinh
ay ta u
giai h`
ang loat
ai tp
. l n`
.
. b`
. trong dai
. s
,
,
,,
,c ra th`u,a s, chu
,ng minh bt
nhu: Giai h. phuong tr`nh, phn tch d
a thu
,
,c, ...
ng thu
da

2.1. inh
a tnh cht
. ngha v`

65

` tnh cht
2.1. .inh ngha va
,
,
,c). Mt
. c s phu
Cho b l`
a mt
o th l`
a s thu. c hoa
am s
. s (c
. h`
,
,,
,
,
,
` k
i `, k l`
a mt
a nhung s
(x, y) = ax y d
on thuc hai bin x, y vo
uo. c goi
. d
. l`
,
,,
,
,c v`
nguyn khng m. S a duo. c goi
a h. s cua don thu
a s ` + k goi
a
. l`
. l`
,
,
,
,
,
`
k
bc
a duo. c k hiu
a deg(ax y ) = ` + k.
on thuc v`
. cua d
. l`
,
,
,c du,o.,c goi
Hai d
ad
nu ch
ung ch kh
ac nhau v` h.
`ng dang,
on thu
.
. l`
,
,c = ax`1 yk1 v`
a d`ng dang,
nu ch
ung phu.
a = bx`2 yk2 l`
s. on thu
.

thuc
ao c`
ung s bin v`
a `1 = `2 , k1 = k2 .
. v`
,
,
,
`ng tng cua hai do,n thu
,c d`ng dang
,c, v` khi
D thy ra
a mt
. l`
. don thu
,
,
,c d`ng dang
ax` yk + bx` yk = (a + b)x` yk . Tch cua hai d
do
c
ung
on thu
.
,
,
, ` k ` k
,
,
,
2` 2k

tao
on
. ra mt
. don thuc nhu ax y .bx y = (a.b)x y . Nhung tng hai d
,
,
,
,c khng d
,c.
thu
o th khng phai l`
a mt
`ng dang
on thu
. c
. d
,
inh
ngha 2.1. Mt
am hai bin P(x, y) goi
ad
o
a thuc hai bin, nu n
. h`
. l`
.
, ,
,
,
,
,
,
,
c hai bin.
c
o th biu din nhu mt
. tng huu han
. nhung don thu
,
,
p xp bc
,c hai bin c

Nhu vy
o th sa
ao do
a thu
. mt
. d
. theo mt
. bin n`
,
,
,
4
3
2

n dn nho goi
t`u lo
a sa
p xp theo bc
. bin. V du. P(x, y) = x + x y +
. l`
,
,,
2 4
2
p xp theo bc
x y + 2xy duo. c sa
. cua bin x.
,
,
, ,
,
,c hai bin tng qu
i
Ta se khng xet tt ca nhung d
at m`
a ch gio
a thu
,
,
,
,

c c
ng theo bc
p da thu
han
o tnh d
ac bin.
i xu
. cho mt
. lo
. cua c
,
,c hai bin P(x, y) goi
inh
ngha 2.2. Mt
a d
i xung, nu da
. da thu
. l`
.
,
,
,
`ng y v`
`ng x, ngha l`
,c n`
a
thu
ay khng thay d
a y ba
i khi chuyn di x ba
P(x, y) = P(y, x).
, ,
,c di xu
,ng hai bin nhu, :
Ta c
o th ch ra h`
ang loat
ac v du. v` d
a thu
. c
`ng y v`
,c di xu
,ng v` khi ta thay x ba
1) P(x, y) = x2 y + xy2 l`
a da thu
ay
,
,

`
c vn nhu c
ng x da thu
ba
u.

66

,,
,c d
,ng hai bin
Chuong 2. a thu
i xu

,c d
,ng.
2) P(x, y) = (x + y)3 + x5 + y5 l`
a da thu
i xu
,
`ng y
,c di xu
,ng, v` thay x ba
3) P(x, y) = x3 5y2 khng phai l`
a da thu
`ng x, P(y, x) = y3 5x2 6= P(x, y) = x3 5y2 .
v`
a y ba

,
,c d
,ng hai bin ta suy ra phep cng,
T`u d
ngha d
phep
inh
a thu
i xu
.
.
,
,
,
,
,
c di xu
ng c
c di xu
ng. Phep ly
tr`u, tch hai da thu
ung l`
a mt
. da thu
,
,
,
,
,
c d
ng c
c di xu
,ng. Ngha
l
uy th`ua cua mt
ung l`
a mt
a thu
i xu
. d
. da thu
,c di xu
,ng, th` P(x, y) Q(x, y),
l`
a nu P(x, y) v`
a Q(x, y) l`
a hai da thu
,
,c d
,ng, o, dy n l`
P(x, y).Q(x, y) v`
a (P(x, y))n c
ung l`
ad
a mt
a thu
i xu
. s
,
,,
tu. nhin duong.
,
,
, ,
,
,ng d
Mnh
d` 2.1. Mt
a y l`
a tng cua nhu
a thuc d
i xung hai bin x v`
on
. d
.
,
, ,
,
thuc dang
axk yk v`
ad
b(xn ym + xm yn ) v
oi a, b l`
a
a thuc d
i xung dang
.
.
,
,
,

ng h. s v`
ng s tu. nhin 0 m < n, 0 k.
nhu
a nhu
,ng minh. Cho P(x, y) l`
,c di xu
,ng, theo d
Chu
a da thu
inh
. ngha P(x, y) =
,
,
,
,
`
c. Nhu,ng do,n thu
,c n`
P(y, x) v`
a P(x, y) bao gm huu han
ay
. nhung don thu
,`,
,
c
o bc
ac nhau. Ta xet hai truong ho. p:
. kh
,
,
,c cua d
,c c
,i k
1) on thu
o c
ac bin d`ng bc
a axk yk vo
a thu
. ngha l`
,
,
,,
,
,c n`
ta chuyn di bin s th` don thu
l`
a mt
ay
. s nguyn duong. Khi do
,
khng di.
,,
,
n m ,
,c c
2) on thu
o dang
i n, m l`
a s nguyn duong v`
an>m
. bx y vo
,
,
,
,c trn
0, b l`
a mt
i hai bin s cho nhau th` d
on thu
. h. s. Khi ta chuyn d
,
,
,
,,
,
,c n`
tro th`
anh bxm yn , hai don thu
ay ho`
an to`
an kh
ac nhau. d
am bao
,
,
,
,
,c n`
,c,
. t trong tng cua d
P(x, y) = P(y, x) th` hai d
ay phai c
o ma
on thu
a thu
,
,c (bxn ym + bxm yn ) l`
,c. Nu tr
ngha l`
ad
a mt
cua da thu
ai
a thu
. s hang
.
,
,
,
c d
ng, v l.
cho khng phai l`
lai
a di xu
a
. th` da thu
,
,
,c d
,ng bao g`m tng nhu,ng d
,c vo
,i c
T
om lai,
ac
a thu
i xu
on thu
. mt
. d
,
,
,
,
,
,
,

c c`
i c
bin c`
ung bc
a tng cua hai don thu
ung h. s vo
ac bc
. v`
. o bin
,
,
chuyn di cho nhau.

, ,
,c hai bin
2.2. inh
. l co ban cho da thu

67

,
5
3 2
3 3
2 3
5
,c
Ta c
o th ly v du:
a da thu
. P(x, y) = x + 3x y x y + 3x y + y l`
,
,
,ng hai bin x, y, o, dy ta ch
,c x5 = x5 y0 v`
di xu
u y hai don thu
a y5 = x0 y5
,
v`
a tng 3(x3 y2 + x2 y3 ).
,
,
,c d
,ng hai bin d
Ta xet hai d
a
a thu
i xu
on gian nht P(x, y) = x + y v`
,
,
,,
,
, ,
, ,,

Q(x, y) = xy. Nhung da thuc n`


ay duo. c goi
a nhung d
a thuc d
i xung co so
. l`
,,

. c bit
hai bin v`
a duo. c k hiu
a
. d
.

1 = x + y,

2 = xy.

,
, ,
`ng da thu
,c di xu
,ng hai bin d`u c
Ph`n sau d
o th biu
y ta se ch ra ra
,
,
,
,c hai bin vo
,c co, so, trn.
,i c
din nhu mt
ac bin l`
a nhung da thu
. da thu

,
, ,
2.2. .inh l co ban cho da
uc hai bin
th
,
,
,
,
,c di xu
,ng d
bit tng, hiu,
Nhu ta d
uy th`ua nhung da thu
a
`u
. tch, l
,,
,
,
,
,
,
,
,
c d
ng. Nhu vy
c di xu
ng co so ta c
cho ta mt
o
a thu
i xu
. d
. t`u hai da thu
,
,
,
,
c di xu
ng t`
th tao
uy y. V du:
. ra nhung da thu
.
1) Q(1 , 2 ) = 31 1 2 = (x+y)3 (x+y)xy = x3 +2x2 y+2xy2 +y3 =
,c di xu
,ng hai bin theo x v`
P(x, y) l`
a mt
a y.
. da thu
,c d
a mt
2) Q(1 , 2 ) = 222 21 2 = x3 y xy3 = P(x, y) l`
a thu
i
. d
,
ng hai bin theo x v`
xu
a y.
,
,
`ng trong mt
,c hai bin 1 v`
a 2 , khi
Nhu vy
o th n
oi ra
. da thu
. ta c
,
,
,
c n`
,c d
thay 1 = x + y v`
a 2 = xy v`
ao d
ay se nhn
a thu
uo. c mt
a thu
i
. d
. d
,
ng hai bin x v`
xu
a y.
,
,
,,
,c di xu
,ng hai bin x v`
Cu hoi nguo. c lai
a: Mt
a y c
o biu din
. l`
. da thu
,
,,
,
,,
,c cua hai bin 1 = x + y v`
duo. c nhu mt
a 2 = xy duo. c khng ? V
a thu
. d

,,
,c d
,ng hai bin
Chuong 2. a thu
i xu

68

,
,c d
,ng bt k` nhu, x4 y + xy4 , ta bin d
du:
i xu
i
. Xet da thu
x4 y + xy4 = xy(x3 + y3 ) = xy(x + y)(x2 xy + y2 ) =
= xy(x + y)((x + y)2 3xy) = 2 1 (21 32 ).
,
, ,
,
,c l`
,i da thu
V du. kh
ac: Vo
a tng l
uy th`ua cua t`ung bin Sn = xn + yn ,
n = 1, 2, ... D thy
S1 = x + y = 1 ;
S2 = x2 + y2 = (x + y)2 2xy = 21 22 ;
S3 = x3 + y3 = (x + y)(x2 xy + y2 ) =
= (x + y)((x + y)2 3xy) = 1 (21 32 );
S4 = x4 + y4 = (x2 + y2 )2 2x2 y2 = (21 22 )2 222 .
,
,
, ,
,c ta xet mnh
dn
dn d
d`.
inh
. l co ban cua da thu
.
,
`,a hai bin Sn = xn + yn , n = 1, 2, 3, ... c
Mnh
d` 2.2. Moi
uy thu
o
. tng l
.
,
cng thuc truy h`i
Sn = 1 Sn1 2 Sn2 , n = 3, 4, ...
, ,
, ,
du,o,i dang
v`
a Sn c
o th biu din
a 2 .
a thuc cua hai bin 1 v`
. d

(2.1)

,
,,
,ng minh. 1) Ta c
,c (2.1) nhu, sau: Ta xet hai
Chu
o th t`m d
uo. c cng thu
,
,
,c theo
,c Sn1 = xn1 + yn1 v`
ng thu
ng thu
d
a 1 = x + y. Ta nhn hai da
a
,,
v v`
a nhn
uo. c
. d
1 Sn1 = (x + y)(xn1 + yn1 ) = xn + xyn1 + xn1 y + yn =
= xn + yn + xy(xn2 + yn2 ) = Sn + 2 Sn2 .
,
,
,
`ng quy nap
,ng minh ba
,c cua 1 v`
2) Ta chu
a 2 .
a thu
. Sn biu din nhu d
,
,,
,
,,
2
`
`
Tht
an o phn truoc th` S1 = 1 v`
a S2 = 1 22 du biu
. vy,
. nhu tnh to
, ,,
,
,
,c qua hai bin 1 v`
din nhu da thu
a 2 . Gia su Sn1 v`
a Sn2 c
ung biu
,
,
,
`ng Sn c
,c cua hai bin 1 v`
,ng minh ra
din nhu da thu
a 2 . Ta phai chu
ung

, ,
,c hai bin
2.2. inh
. l co ban cho da thu

69

,
,
,
,c hai bin trn. i`u n`
biu din nhu d
ay l`
a hin nhin v` theo cng
a thu
,
,c (2.1) v`
,ng minh. Nhu, vy
thu
a gia thit quy nap
i`u c`n chu
i`u
. suy ra d
. d
,
,
,
,
,
i moi
ng d
ng vo
kha
inh
u
. d
. s tu. nhin duong.
,
c (2.1) khng nhu,ng rt quan trong
Cng thu
trong mnh
d
a
` trn m`
.
.
,
,
,
,

c`
on l`
a mt
a 2 . Tu cng thuc (2.1)
. cng thuc tnh Sn biu din theo 1 v`
,
,,
`
ta c
o th tnh ln luo. t c
ac Sn :

S3 = 1 S2 2 S1 = 1 (21 22 ) 2 1 = 31 31 2 ;
S4 = 1 S3 2 S2 = 1 (31 31 2 ) 2 (21 22 ) = 41 421 2 + 222 ;
S5 = 1 S4 2 S3 = 1 (41 421 2 + 222 ) 2 (31 31 2 ) =
= 51 531 2 + 51 22 .
,
,
,
,
,,
uy th`ua tip theo. Ban
o
v`
a ta c
o th tip tuc
oc
. d
. c
. t`m duo. c nhung tng l
, ,
,
,
,
,
c tip theo. V` tnh quan trong
ng dung
th tu. tnh ly nhung biu thu
. cho u
.
,
,
n
n
`
`

sau n`
ay, ch
ung ti lit
k
10
t
ng
l
u
y
th
u
a
d

u
tin:
Bi
t
r
a
ng
S
=
x
+y

n
.
v`
a 1 = x + y, 2 = xy.
S1 = 1 ;
S2 = 21 22 ;
S3 = 31 31 2 ;
S4 = 41 421 2 + 222 ;
S5 = 51 531 2 + 51 22 ;
S6 = 61 641 2 + 921 22 232 ;

(2.2)

S7 = 71 751 2 + 1431 22 71 32 ;
S8 = 81 861 2 + 2041 22 1621 32 + 242 ;
S9 = 91 971 2 + 2751 22 3031 32 + 91 42 ;
8
6 2
4 3
2 4
5
S10 = 10
1 101 2 + 351 2 501 2 + 251 2 22 ;

,
, ,
, ,

inh
ac bin x v`
a y c
o th biu din
a thuc d
i xung bt k` cua c
. d
. l 2.1. Mt

70

,,
,c d
,ng hai bin
Chuong 2. a thu
i xu

,,
, ,
a 2 = xy.
duoi dang
a thuc cua 1 = x + y v`
. mt
. d
,
,
,ng minh. Mt
,c di xu
,ng hai bin bt k` d
Chu
a tng cua hai
a thu
`u l`
. d
,
,c nhu, mnh
dang
a da thu
d` 2.1.
. don v`
.
,
,,
,
,
,
,c l`
,c d`ng bc
,i c
1) Tru`ong ho. p nhung s hang
ad
ac
a thu
on thu
. cua d
. vo
,
,
k
k
c c
bin: on thu
o dang
. ax y . D thy
axk yk = a(xy)k = ak2 .
,
,
Nhu vy,
ay biu din theo 1 v`
a 2 .
. s hang
. n`
,
,,
,
,
,c c
2) Tru`ong ho. p nhung s hang
cua da thu
o dang
b(xn ym + xm yn ).
.
.
,
`ng n > m, khi do

Ta gia thit ra
b(xn ym + xm yn ) = bxm ym (ynm + xnm ) = bm
2 Snm .
, ,
,
,
,c cua 1 v`
theo mnh
d
o th biu din nhu mt
a 2 .
` 2.2, Snm c
a thu
.
. d
,
,
,
,

c cua 1 v`
Suy ra s hang
ung biu din nhu mt
a 2 .
. trn c
. da thu
,
,
,
,
,
c d
,c cua 1 v`
T
om lai,
a
a thu
`u biu din nhu d
a thu
. moi
. s hang
. cua d
,
,
,
,
,
,

c d
ng biu din nhu da thu
c cua 1 v`
2 , suy ra d
a 2 .
a thu
i xu
,
,
,
,
ng minh trn ta c
,c d
Thng qua c
ach chu
o th chuyn di moi
a thu
i
. d
,
,
,ng th`
,c cua 1 v`
,c
xu
anh d
a 2 nhu v du. sau: Xet da thu
a thu

P(x, y) = x6 + 3x3 y2 x5 y5 2xy4 + y6 + 3x2 y3 2x4 y.


,
,
,
,
p xp lai
,c v`
,c nhu, trong
a hai don thu
Ta sa
. theo tng nhung mt
. don thu
,ng minh:
chu
P(x, y) = x5 y5 + (x6 + y6 ) + 3(x3 y2 + x2 y3 ) 2(xy4 + x4 y)
hay
P(x, y) = x5 y5 + (x6 + y6 ) + 3x2 y2 (x + y) 2xy(y3 + x3 )
= 52 + S6 + 322 1 22 S3 ,

,
,,
,ng
2.3. Giai h. phuong tr`nh d
i xu

71

,c (2.2) ta c
v`
a theo cng thu
o
P(x, y) = 52 + S6 + 322 1 22 S3
= 52 + (61 641 2 + 921 22 232 ) + 322 1 22 (31 31 2 )
= 61 641 2 231 2 + 921 22 + 91 22 232 52 .

,
,,
,
2.3. Giai h. phuong tr`nh d
ung
i x
,
,
,,
,,
,
,,
. p h. phuong tr`nh hai n m`
Ta thu`ong ga
a nhung v tr
ai cua phuong
,
,
,,
,
,c di xu
,ng hai n x v`
tr`nh l`
a nhung d
a y. Trong tru`ong ho. p n`
ay ta
a thu
,
,,
,
,
,
chuyn h. phuong tr`nh th`
anh h. nhung phuong tr`nh phu. thuc
ao 1
. v`
,
,,
,
,
,
,
,
,
i n`
v`
a 2 v`
a giai h. phuong tr`nh mo
ay, thu`ong l`
a nhung h. phuong tr`nh
,
,
,
,
,
,
,
nh`o nhung gi
don gian hon rt nhi`u. Sau do
a tri. cua 1 v`
a 2 ta di t`m n
s x v`
a y.
,
,,
,
Truo
c khi xet nhung v du. cu. th, ta xet d
inh
. l:
,,
inh
l 2.2. Cho 1 v`
a 2 l`
a hai s bt k`. Phuong tr`nh bc
. hai
.
z2 1 z + 2 = 0
,,
v`
a h. phuong tr`nh

(2.3)

x + y = 1 ,
(2.4)
xy = 2
,,
,,
c
o mi lin h. tuong d
uong theo ngha sau:
,
,,
,,
Nu z1 , z2 l`
a nghim
. cua phuong tr`nh bc
. hai (2.3), th` h. phuong tr`nh
(2.4) c
o hai nghim
.


x1 = z1 ,
x2 = z2 ,
y1 = z2 ,
y2 = z1
,
,,
v`
a khng c
o nghim
ac; nguo. c lai,
a nghim
. kh
. nu x = a, y = b l`
. cua h.
,
,,
,,
,ng s a v`
phuong tr`nh (2.4), th` nhu
a b c
ung l`
a nghim
. cua phuong tr`nh
bc
. hai (2.3).

72

,,
,c d
,ng hai bin
Chuong 2. a thu
i xu

,
,,
,ng minh. Nu z1 v`
Chu
a z2 l`
a nghim
. hai (2.3), th`
. cua phuong tr`nh bc
,
c Viet
theo cng thu



z1 + z2 = 1 ,
x1 = z1 ,
x2 = z2 ,
ngha l`
a
z1 z2 = 2 ,
y1 = z2 ,
y2 = z1
,
,,
, ,
l`
a nghim
on khng c
o nghim
ac nua cua
. cua h. phuong tr`nh (2.4). C`
. kh
,,
,,
, ,
ng d
h. duo. c suy ra t`u kha
inh
. nguo. c lai.
.
,
,,
Nu x = a, y = b l`
a nghim
a
. cua h. phuong tr`nh (2.4), ngha l`

a + b = 1 ,
ab
= 2 .
ta c
Khi do
o
z2 1 z + 2 = z2 (a + b)z + ab = (z a)(z b).
,
,
,,
i`u n`
ay c
o ngha l`
a nhung s a, b l`
a nghim
. cua phuong tr`nh (2.3).
,
,,
V du. 2.1. Giai h. phuong tr`nh

x3 + y3
= 2,
xy(x + y) = 2.

,
,
,
,,
,
`,i giai. V tr
,c di xu
,ng
cho l`
Lo
ai cua nhung phuong tr`nh da
a nhung d
a thu
,
,,
,
,c
,i x v`
ng thu
. t 1 = x + y v`
di vo
a y. Ta d
a 2 = xy. Ta su dung
nhung da
a
.
,,
,,
,,
trong (2.2), t`m d
a h. phuong tr`nh tro th`
anh
uo. c x3 + y3 = 31 31 2 v`




31 31 2 = 2,
31 6 = 2,
31
= 8,
1 = 2,

1 2
= 2.
1 2
= 2.
1 2 = 2.
2 = 1.
,
,,
,
,
,
,
,
i
Nhu vy
. h. phuong tr`nh ban d`u tuong duong vo

x + y = 2,
xy
= 1.
,
,
,,
,,
H. phuong tr`nh d d`
ang giai duo. c, v` theo d
ay
inh
. l 2.2, vic
. giai h. n`
, ` ,
,,
,,
2
dua v giai phuong tr`nh bc
o
. hai z 2z + 1 = 0. Phuong tr`nh bc
. hai c
,
nghim
o nghim
a y = 1.
. bi
. z = 1 nn h. ch c
. x = 1 v`

,
,,
,ng
2.3. Giai h. phuong tr`nh d
i xu

73

,
,,
V du. 2.2. Giai h. phuong tr`nh

x5 + y5
x+y

= 33,
= 3.

,
,
,
,
,,
`,i giai. Ta su, dung
,c (2.2) d
Lo
chuyn h. phuong tr`nh trn v` h.
. cng thu
theo c
ac bin 1 v`
a 2 :

51 531 2 + 51 22 = 33,
1
= 3.
,
,
,
t`m 2 trong phuong tr`nh bc
T`u do
. hai
1522 1352 + 210 = 0, hay l`
a 22 92 + 14 = 0.
,
,
,
,,
,,
T`u phuong tr`nh n`
ay ta t`m d
a tri. cua 2 : 2 = 2 v`
a 2 = 7.
uo. c hai gi
,,
,,
t`m nghim
. cho h. phuong tr`nh ban d`u ta xet hai h. phuong tr`nh


x + y = 3,
x + y = 3,
v`
a
xy
= 2.
xy
= 7.
,
,
,
,
,c)
a s phu
Giai nhung h. n`
ay ta nhn
. l`
. duo. c (khng tnh nghim


x1 = 2, x2 = 1,
y1 = 1; y2 = 2.
,
,,
V du. 2.3. Giai h. phuong tr`nh

x3 y3 = 19(x y),
x3 + y3 = 7(x + y).

,
,
,,
`,i giai. Ta thy dy l`
,ng v`
a h. phuong tr`nh hai bin di xu
a ta c
ung c
o th
Lo
, ,,
,
,
,,
c du,o.,c nhu,ng s hang
gian uo
chung, nhung d
ac nghim
t`m d
uo. c ht c
.
.
ta xet:
,
,,
,,
1) Nu x = y, th` h. phuong tr`nh ch c`
on 2x3 = 14x (v` phuong tr`nh
,
,,
,,
, nht lun l`
thu
a d`ng nht), t`u phuong tr`nh n`
ay ta t`m duo. c ba nghim
.
sau


x1
y1

= 0,
= 0;


x2
y2

= 7,

= 7;

x3
y3

= 7,

= 7.

,,
,c d
,ng hai bin
Chuong 2. a thu
i xu

74

,,
, hai trong h. d`ng nht l`
2) Nu x = y, th` phuong tr`nh thu
a 0, c`
on
,,
,
,
,
,
3
nht l`
ta t`m thm duo. c hai nghim
phuong tr`nh thu
a 2x = 38x. T`u d
o
.
,
nua



x4 = 19,
x5 = 19,

y4 = 19;
y5 = 19.
, , ,,
,
,
c du,o.,c x y o, phu,o,ng tr`nh thu
3) Nu x 6= y, th` ta c
o th gian uo
,,
,,
,,
, hai v`
nht, x + y o phuong tr`nh thu
a nhn
. duo. c h.

x2 + xy + y2 = 19,
x2 xy + y2 = 7.
,
,c (2.2)
Ta bin d
ao bin 1 v`
a 2 theo cng thu
i v` h. phu. thuc
. v`

21 2
= 19,
2
1 32 = 7.
,
,,
,
T`u dy ta t`m d
o hai nghim
uo. c 21 = 25, 2 = 6, nhu vy
. ta c
.


1 = 5,
1 = 5,
2 = 6;
2 = 6.
,
,
,,
2
T`u hai h. trn d
ac phuong tr`nh bc
a
ua v` c
. hai z + 5z + 6 = 0 v`
,
,,
,,
2
z 5z + 6 = 0. Giai c
ac phuong tr`nh bc
ay ta nhn
ac
. hai n`
. duo. c c
nghim
.


x6
y6

= 2,
= 3;


x7
y7

= 3,
= 2;


x8
y8

= 2,
= 3;

x9
y9

= 3,
= 2.

,
,,
,
V du. 2.4. Giai h. phuong tr`nh v
oi x 6= 0, y 6= 0:
2
2

x + y
= 12,

y
x

1
1
1

+
= .
x y
3
,
s
`,i giai. H. phu,o,ng tr`nh da
,ng vo
,i x, y. Ta quy d
cho l`
Lo
ad
i xu
`ng mu

,
,,
,ng
2.3. Giai h. phuong tr`nh d
i xu

75

,
,,
hai phuong tr`nh v`
a chuyn sang h. c
o hai bin 1 v`
a 2 :


x3 + y3 = 12xy,
31 31 2 = 122 ,

3(x + y) = xy.
31
= 2 .
,
,,
,,
, hai thay v`
, nht, ta
Ly 2 cua phuong tr`nh thu
ao phuong tr`nh thu
,
,,
,,
nhn
a 1 (21 91 36) = 0. Giai phuong
uo. c 31 921 361 = 0, ngha l`
. d
,
,,
,i 1 cho kt qua 1 = 0, 1 = 3, 1 = 12. Ta nhn
tr`nh n`
ay di vo
uo. c
. d
ba h.



= 0,
1 = 12,
1 = 3,
= 0;
2 = 36;
2 = 9.
,
,
,
,
,

nht suy ra x = y = 0 v`
T`u h. phuong tr`nh thu
a dy khng phai l`
a
,
,
,
,
,
,
,
,
`
hai tuong u
ng vo
i phuong tr`nh bc
nghim
u. H. thu
. cua h. ban d
. hai
,,
2
cho c
z 12z + 36 = 0, phuong tr`nh n`
ay c
o nghim
o
a
. kep nn h. d
1
2

nghim
.


x1 = 6,
y1 = 6.
, ba tu,o,ng tu
,ng vo
,i z2 + 3z 9 = 0 c

C`
on h. thu
o hai nghim
a khi d
o
. v`

cho c
h. da
o thm hai nghim
.

3 3 5
3 + 3 5

, x3 =
,
2
2

y = 3 + 3 5 .
y = 3 3 5 ;
2
3
2
2
,
,,
V du. 2.5. Giai h. phuong tr`nh

= 7,
x + y z
x2 + y2 z2 = 37,

3
x + y3 z3 = 1.

x2

,
,
,,
`,i giai. Ta coi bin thu
, ba z nhu, l`
Lo
ad
c
. lai),
. lp
. (c dinh
. th` nhung phuong
,
,c d
,ng theo bin x, y.
cho c
tr`nh trong h. da
o v tr
ai l`
a nhung d
a thu
i xu

,,
,c d
,ng hai bin
Chuong 2. a thu
i xu

76

,
,,
,
,i c
Theo phuong ph
ap giai c
ac bin 1 v`
a 2 :
ac b`
ai trn ta dua v` h. vo

= 7,
1 z
2
2
1 22 z
= 37,

3
1 31 2 z3 = 1.
,,
,,
,,
,,
`ng c
, nht v`
, hai
Ba
ach ly z o phuong tr`nh thu
a 2 o phuong tr`nh thu
,
,,
,,
, ba. Sau khi tnh to
thay v`
ao phuong tr`nh thu
an ta duo. c 181 = 342. T`u
,
,,
d
y ta t`m duo. c nghim
. cua h. trn 1 = 19, 2 = 90, z = 12.
,
,
,
,,
,,
T`u d
a t`m duo. c c
ac nghim:
y d
ua v` giai phuong tr`nh bc
. hai v`
.

x2 = 10,
x1 = 9,
y2 = 9,
y1 = 10,

z2 = 12.
z1 = 12;

,
,
` tp
Bai
(Mt
a tra l`oi c
ac b`
ai tp
. s go. i y v`
. sau tai
. trang 110)
.
,
,
,,
H
ay giai nhung h. phuong tr`nh sau

. 2.6.

. 2.7.

. 2.8.


x + y = 5,
x2 xy + y2 = 7;
x + y = 5,
x3 + y3 = 65;
4(x + y) = 3xy,
x + y + x2 + y2 = 26;

x2 + y2 + x + y = 32,
12(x + y) = 7xy;

xy = 15,
. 2.10.
x + y + x2 + y2 = 42;
2
y2
x
+
= 18,
. 2.11.
y
x

x + y = 12;
. 2.9.

x2 y + y2 x = 30,
. 2.12.
1
1
5
+ = ;
y x
6

x + y = a,
. 2.13.
x3 + y3 = b(x2 + y2 );

x2 + y2 + 2(x + y) = 23,
. 2.14.
x2 + y2 + xy = 19;

xy(x + y) = 20,
. 2.15.
1
1
5
+ = ;
x y
4

x4 x2 y2 + y4 = 1153,
. 2.16.
x2 xy + y2 = 33.

,
,,
,ng
2.4. ua v` h. phuong tr`nh dang
. di xu

77

,
,,
,
2.4. ua v` h. phuong tr`nh dang
d
ung
i x
.
,,
,
,,
,,
,ng rt thun
O ph`n truo
c, ta thy vic
i xu
. giai mt
. h. phuong tr`nh d
.
, ,
,
,
,,

tin
v`
a
c
o
quy
t
a
c
chung
d

gi
ai.
Nhu
ng
khng
ph
ai
h

phu
o
ng
tr`
nh
d
a
cho

.
.
,,
,
,,
,ng. Nhu,ng c
n`
ao c
ung l`
a h. phuong tr`nh di xu
o mt
. s nhung h. phuong
,
,,
,
,,
,ng hoa
. c phuong tr`nh ta c
tr`nh khng di xu
o th dua v` h. phuong tr`nh
,
,
,
,
,
,ng d giai. T`
. t c
cho, ta se d
di xu
uy v`
ao nhung b`
ai to
an cu. th da
ac n
a
, ,
,
,,
,ng di vo
,i n s mo
,i. Ta
s phu. d
ai to
an v` h. phuong tr`nh d
d
ua b`
i xu
xet mt
. s v du.
,
,,
V du. 2.17. Giai h. phuong tr`nh

2
2

x + y

x y

5
xy,
2

1
xy.
4

,
,
`,i giai. Ta thy phu,o,ng tr`nh thu
, hai vi pham
,ng cua h. khi
Lo
. tnh di xu
,
,,
,,
. t z = y, h. phuong tr`nh tro th`
thay d
anh
i x, y cho nhau. Ta da

2
2

x + z = 2 xz,

x + z
= xz.
4
,,
,

,i x v`
. t 1 = x + z
d
y l`
a mt
h

phu
o
ng
tr`
nh
d

i
x
u
ng
d

i
v
o
a z. Khi do

a
. .
,,
,,
,
,
c (2.2)):
v`
a 2 = xz, ta nhn
. duo. c h. phuong tr`nh (t`u cng thu

2



1 22 = 2 2 ,
1 = 0,
1 = 2,
H. n`
ay c
o hai nghim
.

2 = 0;
2 = 8.
1

1
= 2 .
4
h. phu,o,ng tr`nh trn cho ta nghim
,i x v`
Mi
a z:
i vo
. d



x1 = x2 = 0,
x3 = 4,
x4 = 2,
z1 = z2 = 0;
z3 = 2;
z4 = 4.

78

,,
,c d
,ng hai bin
Chuong 2. a thu
i xu

,
,,
,,
Cui c`
ung ta nhn
uo. cnghim
`u:
. d
. cua h. phuong tr`nh ban d



x1 = x2 = 0,
x3 = 4,
x4 = 2,
y1 = y2 = 0;
y3 = 2;
y4 = 4.

,
,,
V du. 2.18. Giai h. phuong tr`nh
p

4
y3 1 + x = 3,
x2 + y3 = 82.
,
,
,,
`,i giai. H. phu,o,ng tr`nh da
,ng
cho khng phai l`
Lo
a h. phuong tr`nh d
i xu
p

,,
,
,i x v`
. t u = x v`
di vo
a y. Ta da
a v = 4 y3 1. H. phuong tr`nh dua v`


u + v = 3,
u + v = 3,

u4 + (v4 + 1) = 82;
u4 + v4 = 81.
,,
,ng di vo
,i hai bin u, v. Vo
,i 1 = u + v
H. phuong tr`nh trn l`
a h. di xu
,,
,,
,c (2.2) ta nhn
,i cng thu
v`
a 2 = uv, c`
ung vo
uo. c h. phuong tr`nh
. d

1 = 3,
41 421 2 + 222 = 81.
,,
,, 2
,,
Thay 1 = 3 v`
ao phuong tr`nh hai, ta nhn
. duo. c 2 182 = 0. Phuong
,,
tr`nh n`
ay c
o hai nghim
a 2 = 18. Ta nhn
. 2 = 0 v`
. duo. c hai nghim
.


1 = 3,
1 = 3,
2 = 0;
2 = 18.
,
,,
,,
2
, nht du,a v` vic
H. phuong tr`nh thu
. giai phuong tr`nh bc
. hai z 3z = 0
,
i u v`
v`
a theo dinh
o nghim
a v:
. l 2.2 ta c
. di vo




u1 = 3,
u2 = 0,
x1 = 9,
x2 = 0,

Khi d
o
v1 = 0;
v2 = 3.
y1 = 1;
y2 = 3 82.
,
,,
,
,,
2
, hai d
H. phuong tr`nh thu
ua v` giai phuong tr`nh bc
. hai z 3z + 18 = 0.
,,
,
, ,
nghim
Phuong tr`nh n`
ay khng c
o nghim
. thu. c. Do do
. thu. c cua h. ban
,
,,
d`u ch c
o hai b. s o trn.

,
,,
,ng
2.4. ua v` h. phuong tr`nh dang
. di xu

79

,
,,
V du. 2.19. Giai h. phuong tr`nh
r
r
x + y = 7 + 1,
y
x
xy
p
p 3
x y + y3 x = 78.
,
,
`ng x, y phai kh
`,i giai. T`u, h. phu,o,ng tr`nh ta thy ra
Lo
ac khng v`
a c
o c`
ung

,
,
. t u = x, v = y. Nu x, y l`
du. Nu x, y duong th` ta da
a s m th` ta

,`,
,
,
. t u = x, v = y. Trong moi
cho dua v`:
da
a
. truong ho. p h. d

v
7
u
+ =
+ 1,
v
u
uv
u3 v + v3 u = 78.
`ng c
,c (2.2) ta nhn
. t 1 = u + v v`
Ba
ach d
a 2 = uv v`
a theo cng thu
a
.
,,
duo. c

21 32 = 7
2 (21 22 ) = 78.
,,
,,
2 `,
, nht v`
, hai, nhn
Ta thay 1 tu phuong tr`nh thu
ao phuong tr`nh thu
.
,,
,,
duo. c 22 + 72 78 = 0. Ta nhn
d
u
o
c
hai
nghi
m

=
6
v`
a

=
13.
Do

2
2
.
.
.
,,
h. phuong tr`nh hai bin 1 v`
do
a 2 c
o nghim:
.


1 = 5
1 = 5
2 = 6;
2 = 6.
,,
,
,,
,
Boi v` u, v l`
a nhung s duong, nn 1 > 0 v`
a 2 > 0, ngha l`
a trong nhung
,
,
,
,,
, nht l`
ta
h. nghim
a 2 o trn ch c
o th. thu
a thoa m
an. Do d
o
. cua 1 v`
,,
nhn
. duo. c


u1 = 2,
u2 = 3,
v1 = 3;
v2 = 2.

,,
,
,
,
,i c
. t u = x,
cho vo
T`u d
ach da
y ta t`m x, y o h. phuong tr`nh da

,,
. c u = x, v = y, ta nhn
v = y hoa
uo. c bn nghim
. d
.




x1 = 4,
x2 = 9,
x3 = 4,
x4 = 9,
y1 = 9;
y2 = 4;
y3 = 9;
y4 = 4.

,,
,c d
,ng hai bin
Chuong 2. a thu
i xu

80

,
,,
V du. 2.20. Giai phuong tr`nh

4
97 x + 4 x = 5.

,,
,
`,i giai. Ta d
. t u = 4 x v`
phuong tr`nh dua v` dang
Lo
a v = 4 97 x. Khi d
a
o
.
. t kh
u + v = 5. Ma
ac
u4 + v4 = x + (97 x) = 97.
,,
,
,,
Nhu vy
uo. c h. phuong tr`nh
. ta nhn
. d

u + v = 5,
u4 + v4 = 97.
,
,,
`ng c
. t 1 = u + v v`
Ba
ach d
a 2 = uv, ta d
a
ua h. phuong tr`nh trn v` dang
.

1 = 5,
41 421 2 + 222 = 97.
,
,,
T`u dy ta t`m 2 trong phuong tr`nh bc
. hai
22 502 + 264 = 0.
,
,,
Giai phuong tr`nh trn c
o nghim
a 2 = 44. Ta c
o hai h.
. 2 = 6 v`


1 = 5
1 = 5
2 = 6;
2 = 44.
,
,,
, nht ta nhn
T`u h. thu
. duo. c


u1 = 2
u2 = 3
v1 = 3;
v2 = 2.
,

,,
V` u = 4 x nn nghim
a x1 = 16 v`
a x2 = 81.
. cua phuong tr`nh ban d`u l`
,,
,
, hai khng c
,i u v`
H. phuong tr`nh thu
o nghim
a v.
. thu. c di vo

,
,,
V du. 2.21. Giai phuong tr`nh
x+

x
x2

35
.
12

,
,,
,ng
2.4. ua v` h. phuong tr`nh dang
. di xu

81

,
,
,,
`ng x = 0 khng l`
`,i giai. Ta thy ra
cho. Ta
a nghim
Lo
. cua phuong tr`nh da

x2 1
1
,,
,ng
. t u = , v =
ta c
da
. Khi do
o h. phuong tr`nh di xu
x
x

u2 + v2 = 1
1
1
35
+ =
.
u v
12
,
,
,,
,,
`ng c
Ba
u v`
a v :
ach giai h. nhu muc
c ta t`m duo. c nghim
.
. truo
,
4
3
,
,
,
a
u1 = , u2 = . T`u d
y suy ra nghim
. cua phuong tr`nh ban d`u l`
5
5
5
5
x1 = v`
a x2 = .
4
3
, ,
,
` tp
Bai
(Mt
a tra l`oi c
ac b`
ai tp
. s go. i y v`
. sau tai
. trang 114)
.
,
,
,,
H
ay giai nhung h. phuong tr`nh sau


. 2.22.

. 2.23.

. 2.24.

x y = 2,
x3 y3 = 8;

a b

+ = 4;
x y

,
,
,,
Giai nhung phuong tr`nh sau:
r
r
1
5 1
+ x + 5 x = 1;
. 2.27.
2
2
. 2.28. sin3 x + cos3 x = 1;
. 2.29.

629 x +

. 2.30.

8+x+

x y = 2 xy,
x + y = 20;

x 43 + y 35 = 35,
. 2.26.
x 14 + y 15 = 5.

. 2.25.

x2 + y = 5,
x6 + y3 = 65;
x y

a + b = 1,

77 + x = 8;

8 x = 1;

,,
,c d
,ng hai bin
Chuong 2. a thu
i xu

82

1
35
1
+
=
;
2
x
12
1x

. 2.32. 3 10 x 3 3 x = 1;


19 x
19 x
. 2.33. x
x+
= 84;
x+1
x+1

. 2.34. x + 17 x2 + x 17 x2 = 9;
. 2.31.

. 2.35. x 3 35 x3 (x + 3 35 x3 ) = 30.

,
,
,
,
ng th
2.5. Ch
ung minh bt da
uc d
ung
i x
, , ,
,,
,c di xu
,ng rt hiu
ng minh nhu,ng
Phuong ph
ap d`
ung da thu
. qua d chu
,
,
,c. Nhu,ng bt d
,c c`n chu
,ng minh thu,`o,ng c
ng thu
ng thu
bt d
o dang
a
a
.
,
,
,
,
,

i P(x, y) l`

P(x, y) 0, vo
a mt
d
a
th
u
c
d

i
x
u
ng.
M
t
b
t
d
a
ng
th
u
c
khng

.
.
,
,
,c th` n
ng thu
thay di khi ta thay vai tr`
o c
ac bin cho nhau trong bt da
o
,
,
, ,
,
,,
,
,
ng minh bt da
c
ng thu
duo. c goi
a bt da
ng thuc d
i xung. Cng cu. d chu
. l`
,
,
ng l`
c (2.2) v`
di xu
a cng thu
ad
inh
y:
. l sau d
,
,ng s x, y x
nhu
inh
l 2.3. Cho hai s thu. c 1 v`
a 2 . Khi d
ac d
`ng
o
inh
. ba
.
,,
h. phuong tr`nh

x + y = 1 ,
xy = 2
,
,
,
,
,
,

ng s thu. c khi v`
l`
a nhu
a ch khi 1 v`
a 2 thoa m
an bt da
ng thuc
,
,
,
,,
21 42 0. a
ng thuc 21 = 42 ch dat
. duo. c khi x = y.
,
,,
,,
,
,ng s x, y x
`, h.
Tru`ong ho. p ring hay su dung:
cho nhu
ac d
tu
inh
.
.
,
,,
,ng s thu.,c, khng m, d
,ng
phuong tr`nh trn l`
a nhu
ad
i`u kin
u nhu
. c`n v`
,
,
,
,ng bt da
s 1 v`
a 2 thoa m
an nhu
ng thuc sau
21 42 0,

1 0,

2 0.

,
,ng minh bt da
,c di xu
,ng
ng thu
2.5. Chu

83

,
,
,,
,ng minh. Nhu,ng s x, y thoa m
Chu
an d
a l`
a nghim
inh
. l 2.2 v`
. cua phuong
tr`nh bc
. hai
,i c
ngha l`
a tr`
ung vo
ac s

z2 1 z + 2 = 0,
1

q
21 42

.
z1,2 =
2
,
,
,
,
,
,c du,o
,i du
V` th d x, y l`
a nhung s thu. c, d
a du l`
a biu thu
i`u kin
. c`n v`
,
,
,
,
,c 21 42 0. a
ng thu
ng
c
an khng m, ngha l`
a phai thoa m
an bt da
,
,,
,c 21 42 = 0 ngha l`
thu
a nghim
ung nhau
. cua phuong tr`nh bc
. hai tr`
x = y.
,
Nu nhung s x, y khng m, th` d thy 21 42 0, ngo`
ai ra c`
on c
o
,
,,
,
, 2

ng thuc 1 42 0, 1 0
1 0 v`
a 2 0. Nguo. c lai,
. cho nhung bt da
,
,,
,
, ,
`
`
,c thu
, nht suy
ng thu
v`
a 2 0. Nhu chung minh o phn trn, tu bt da
,
,
,
ra x, y l`
a nhung s thu. c. T`u 2 0, suy ra x, y c
o c`
ung du; v`
a cui c`
ung

1 0 suy ra x, y khng m.
,
, ,
ng dung
,ng minh bt da
,c d
,ng ta
ng thu
u
dinh
i xu
. l trn trong chu
.
,
tin h`
anh nhu sau:
,
,
`ng cho mt
,c di xu
,ng P(x, y) v`
,ng minh
Gia thit ra
a phai chu
a thu
. d
,
,
`ng vo
,i nhu,ng gi
,i gi
. c l`
ra
a tri. thu. c bt k` cua x, y (hoa
a vo
a tri. khng m bt
,
,
c n`
i x + y a, ...) d
. c vo
k` hoa
ay nhn
a tri. khng m: P(x, y) 0.
a thu
. gi
, ,
,
,
,
ng minh, ta chuyn da thu
c P(x, y) th`
,c phu. thuc
chu
anh biu thu
ao
. v`
,
,
,
,
,
c n`
1 v`
a 2 . Trong da thu
ay ta thay 2 boi 1 v`
a d
ai
. luo. ng khng m
,
1
,,
z = 21 42 , ngha l`
a thay 2 = (21 z). Trong kt qua, ta nhn
uo. c
. d
4
`ng vo
,c hai bin 1 v`
,ng minh ra
,i nhu,ng gi
da thu
a z v`
a c`n chu
a tri. khng
,
,
,
,

c c`n chu
,ng
i han
i 1 da
t buc
cho, th` da thu
m z v`
a nhung gio
i v o
. ba
. d
,
,
,c ban d`u d
ng thu
ng.
minh nhn
a tri. khng m. T`u d
y suy ra bt da
u
. gi

Ta xet mt
. s v du:
.
,
,
,ng s thu.,c, thoa m
an d
V du. 2.36. Chung minh ra
`ng nu a v`
a b l`
a nhu
i`u

,,
,c d
,ng hai bin
Chuong 2. a thu
i xu

84

,
,
,ng bt d
ng
ng thuc sau dy d
kin
a
u
. a + b c, th` nhu

c4
c8
c2
; a4 + b4 ; a8 + b8
.
2
8
128
,
`,i giai. Trong tru,`o,ng ho.,p b`
Lo
ai n`
ay 1 = a + b v`
a 2 = ab. Ta c
o
1
1
1
S2 = a2 + b2 = 21 22 = 21 2. (21 z) = 21 + z.
4
2
2
,
1 2
S2 c .
Theo dinh
o
. l 2.3 th` z 0, theo gia thit 1 c, do d
2
,,
, ,
`ng c
,c (2.2) v`
Ho`
an to`
an tuong tu. vo
i S4 = a4 + b4 ba
ach tra cng thu
a
,,
t`m d
uo. c

2
1 1 2
1
a4 + b4
c
= c4 .
2
2
8
,
,,
,
,c c`
ng thu
Bt d
on lai
ung l`
am tuong tu. .
a
. c
,
,
,ng s thu.,c, th` bt d
V du. 2.37. Chung minh ra
`ng nu x v`
a y l`
a nhu
ng
a
,
ng
thuc sau d
y d
u
x6 + y6 x5 y + xy5 .
a2 + b2

,
,
`,i giai. a thu
,c di xu
,ng co, so, l`
,c
Lo
a 1 = x + y v`
a 2 = xy v`
a cng thu
1 2
(2.2) cho S4 , S6 v`
a 2 = (1 z). Ta c
o:
4
x6 + y6 x5 y xy5 = S6 2 S4 =
= (61 641 2 + 921 22 232 ) 2 (41 421 2 + 222 ) =

= 61 741 2 + 1321 22 432 =


1
1
1
= 61 741 . (21 z) + 1321 . (21 z)2 4. (21 z)3 =
4
16
64
5 4
5
1
=
z + 21 z2 + z3 0.
16 1
8
16
,
,ng s thu.,c du,o,ng, th` bt
V du. 2.38. Chung minh ra
`ng nu x v`
a y l`
a nhu
,
,
ng
d
ng thuc sau d
a
y d
u
s
r

x2
y2
+
x + y.
y
x

,
,ng minh bt da
,c di xu
,ng
ng thu
2.5. Chu

85

,
,

,,
,
`,i giai. Ta d
,c d
ng thu
. t u = x, v = y. Khi do
bt d
Lo
a
a
uo. c dua v` dang
.
u2 v 2
+
u + v, hay u3 + v3 uv(u + v).
v
u
,
,
,ng minh bt d
,c sau c`
,i u > 0 v`
ng thu
Ta phai chu
ung vo
a v > 0. D`
ung
a
S3 = 31 31 2 . Ta c
o
u3 + v3 uv(u + v) = 31 31 2 1 2 = 31 41 2 = 1 (21 42 ).
,
2
,c sau c`
Theo d
ung khng m,
inh
. l 2.3 1 0, 1 42 0, nn biu thu
,
,
ng minh.
suy ra kt qua c`n chu

,
,
,
,
, `
V du. 2.39. T`m gi
a tri. nho nht cua biu thuc xy(x y)2 v
oi d
iu kin
.
x + y = a.
,
1
`,i giai. Vo
,i 1 = x + y v`
o
Lo
a 2 = xy v`
a 2 = (21 z) , ta c
4
1
xy(x y)2 = 2 (21 42 ) = 2 z = (21 z)z =
4
1
1 2
= (a z)z = (z2 + a2 z)
4
4
!


2 2
1
a
a4
=
z
+
.
4
2
4
,
,
a4
a4
,
,c trn khng th vu,o.,t qu
suy ra biu thu
T`u do
a
v`
a nhn
l`
a gi
a
.
16
16
2
,
a
,n nht khi z
,i 1 = a (gia thit da
cho) v`
tri. lo
= 0. Ngha l`
a vo
a
2
2
,
a
,
,,
,,
21 42 =
, t`u dy d t`m duo. c x, y l`
a nghim
. cua phuong tr`nh bc
.
2
2
a
hai z2 az +
= 0.
8
,
,
,ng s du,o,ng thoa m
V du. 2.40. Chung minh ra
`ng nu x v`
a y l`
a nhu
an

x + y = 1, th`

x+

1
x

2


2
1
25
+ y+

.
y
2

,,
,c d
,ng hai bin
Chuong 2. a thu
i xu

86

,
,
,
,
,
`,i giai. Ta bin di biu thu
,c v` dang
,c cua 1 v`
Lo
a 2 (trong bin
. biu thu
,
,
,
c ta ch
di biu thu
u y 1 = 1):

2 
2
1
1
25
x2 + y2 17
x+
+ y+

= x2 + y2 + 2 2
=
x
y
2
x y
2
= (21 22 ) +

21 22 17
1 22 17

= 1 22 +

=
2
2
22
22

1
(432 1522 42 + 2).
222
,
,ng minh biu thu
,c trong du ngoa
. c khng m, ngha l`
Ta c`n chu
a
=

432 + 1522 + 42 2.

(2.5)

V` x, y > 0, nn 2 > 0; ngo`


ai ra z = 21 42 0 (ngha l`
a 1 42 0),
1
1
,
,
,
c 432 + 1522 + 42 c
suy ra 2 . Nhu vy
o tt
t`u do
. 0 < 2 4 . a thu
4
,
,
1
,,
,n nht trn khoang 0 < 2 l`
ca h. s d`u duong, n
a2
o nhn
a tri. lo
. gi
4
1
,
,ng minh du,o.,c (2.5).
chu
tai
a
. ta d
. 2 = 4 . Nhu vy

` tp
Bai
.
, ,
,
Mt
a tra l`oi c
ac b`
ai tp
. s go. i y v`
. sau tai
. trang 119.
,
,
,
,
`ng vo
ng minh ra
i nhung s thu. c bt k` x v`
Chu
a y:
. 2.41. 5x2 6xy + 5y2 0;

. 2.43. x4 + y4 x3 y + xy3 ;

. 2.42. 8(x4 + y4 ) (x + y)4 ;

. 2.44. x2 + y2 + 1 xy + x + y.

`ng vo
,ng minh ra
,i nhu,ng s khng m bt k` x v`
Chu
a y:

. 2.45. ( x + y)8 64xy(x + y)2 ;


. 2.46.

x3 + y3

x+y
2

3
;

,c bc
2.6. B`
ai to
an v` tam thu
. hai

87

. 2.47. x4 + 2x3 y + 2xy3 + y4 6x2 y2 .


`ng vo
,ng minh ra
,i nhu,ng s du,o,ng bt k`
Chu
. 2.48.

x y
+ 2.
y x

,
` toan
v` tam th
2.6. Bai
uc bc
. hai
,
,
,c c
c`n phai tnh to
Rt nhi`u b`
ai to
an trong d
an mt
o
o
. s biu thu
,
,
,
,
,
,
,
a nghim
cho. Ta c
chu
o th giai nhung b`
ai
. cua phuong tr`nh bc
. hai da
,
,
`
c d
ng.
ng c
to
an n`
ay ba
ac da thu
i xu
,,
,,
2
V du. 2.49. Cho phuong tr`nh bc
ay lp
. hai x + 6x + 10 = 0. H
. phuong
,
,,
,,
,
,ng nghim
tr`nh bc
oi c
o nghim
`ng b`nh phuong nhu
. hai m
. ba
. cua phuong
cho.
tr`nh d
a
,
,
,,
`,i giai. K hiu
cho; y1 v`
Lo
a x2 l`
a nghim
a y2 l`
a
a
. x1 v`
. cua phuong tr`nh d
,
,
, ,,
,,
,
nghim
a gia su b v`
a c l`
a nhung h.
. cua phuong tr`nh bc
. hai ta phai t`m v`
,
,,
,
2

c Vi`ete1
s cua phuong tr`nh bc
. hai: y + by + c = 0. Theo cng thu
1 = x1 + x2 = 6,

2 = x1 x2 = 10

c
v`
a ta d
o
a
y1 + y2 = b, y1 y2 = c.
,
,

Nhung theo gia thit y1 = x21 , y2 = x22 , do d


o
b = (y1 + y2 ) = (x21 + x22 ) = S2 = (21 22 ) = 16,
c = y1 y2 = x21 x22 = 22 = 100.
,
,
,,
Nhu vy
o dang
. phuong tr`nh phai t`m c
.
y2 16y + 100 = 0.
1 Francois

Vi`ete (1540-1603): Nh`


a to
an hoc
ap
. Ph

,,
,c d
,ng hai bin
Chuong 2. a thu
i xu

88

,,
2
V du. 2.50. H
ay lp
. hai z + pz + q = 0 sao cho nghim
.
. phuong tr`nh bc
,
,
ng s
cua n
o l`
a nhu
z1 = x61 2x22 , z2 = x62 2x21 ,
,
,,
,,
2
od
a nghim
y x1 , x2 l`
. cua phuong tr`nh bc
. hai x x 3 = 0.
,
`,i giai. Theo cng thu
,c Vi`ete
Lo
1 = x1 + x2 = 1,

2 = x1 x2 = 3.

. t kh
Ma
ac ta lai
o
. c
p = z1 + z2 = (x61 2x22 ) + (x62 2x21 ),
q = z1 z2 = (x61 2x22 )(x62 2x21 ).
,
,c (2.2) d d`
Ta d`
ung cng thu
ang biu din p v`
a q theo c
ac bin 1 v`
a 2 ,
,,
,
thay gi
sau d
a tri. 1 = 1 v`
a 2 = 3, tnh d
a q.
o
uo. c nhung h. s p v`
Ta c
o
p = (x61 + x62 ) 2(x21 + x22 ) = S6 2S2 =
= (61 641 2 + 921 22 232 ) 2(21 22 ) =
= (16 6.14 (3) + 9.12 (3)2 2(3)3 ) 2(12 2(3)) = 140;
q = (x61 2x22 )(x62 2x21 ) = x61 x62 2(x81 + x82 ) + 4x21 x22 =
=62 2S8 + 422 = 62 2(81 861 2 + 2041 22 1621 32 + 242 ) + 422
=(3)6 2(18 8.16 (3) + 20.14 .(3)2 16.12 .(3)3 + 2(3)4 ) + 4(3)2
= 833.
,
,
,,
Nhu vy
a phuong tr`nh bc
a
. p = 140, q = 833 v`
. hai phai t`m l`
2
z 140z 833 = 0.
,
,,
V du. 2.51. Cho x1 v`
a x2 l`
a nghim
cua phuong tr`nh bc
.
. hai
,
,
,
,
,
2
k
k
ng gi
x + px + q = 0. H
ay tnh nhu
a tri. cua biu thuc x1 + x2 v
oi k =

1, 2, 3, 4, 5.

,c bc
2.6. B`
ai to
an v` tam thu
. hai

89

,
`,i giai. Theo cng thu
,c Vi`ete ta c
Lo
o 1 = x1 + x2 = p, 2 = x1 x2 = q.
,
,
,
c o (2.2) c
Theo c
ac cng thu
o
x1 + x2 = S1 = p;
x21 + x22 = S2 = 21 22 = p2 2q;
x31 + x32 = S3 = 31 31 2 = p3 + 3pq;
x41 + x42 = S4 = 41 421 2 + 222 = p4 4p2 q + 2q2 ;
x51 + x52 = S5 = 51 531 2 + 51 22 = p5 + 5p3 q 5pq2 .
v`
a
1
x1
1 + x2 =
2
x2
1 + x2 =
3
x3
1 + x2 =
4
x4
1 + x2 =
5
x5
1 + x2 =

1
(x1 + x2 ) =
x1 x2
1
(x2 + x22 ) =
x21 x22 1
1
(x3 + x32 ) =
x31 x32 1
1
(x4 + x42 ) =
x41 x42 1
1
(x5 + x52 ) =
x51 x52 1

1
S1
2
1
S2
22
1
S3
32
1
S4
42
1
S5
52

1
= p;
q
1
= 2 (p2 2q);
q
1
= 3 (p3 + 3pq);
q
1
= 4 (p4 4p2 q + 2q2 );
q
1
= 5 (p5 + 5p3 q 5pq2 ).
q

,
,
,,
,ng nghim
V du. 2.52. Chung minh ra
`ng nu x1 v`
a x2 l`
a nhu
. cua phuong
,
,
,
2
tr`nh bc
oi c
ac h. s nguyn p v`
a q, th` v
oi s tu. nhin
. hai x + px + q = 0 v
n
kh
ac 0 bt k` n, s xn
a s nguyn.
1 + x2 l`
,
`,i giai. Ta c
,c trong
Lo
o 1 = x1 + x2 = p, 2 = x1 x2 = q . Theo cng thu
mnh
d
` 2.2 ta vit lai:
.
.
n
n1
xn
+ xn1
) q(xn2
+ xn2
).
1 + x2 = p(x1
2
1
2

90

,,
,c d
,ng hai bin
Chuong 2. a thu
i xu

,
,
,
`ng
,c n`
,ng minh kha
ng dinh
Ta d`
ung cng thu
ay d chu
cua b`
ai to
an ba
.
,,
,i n = 1, 2 ta c
phuong ph
ap quy nap.
o
. Vo
x11 + x12 = x1 + x2 = 1 = p Z,
x21 + x22 = S2 = 21 22 = p2 2q Z.

, ,,
,
,
,c o,
Gia su Sn1 v`
a Sn2 l`
a nhung s nguyn (n 3), th` theo cng thu
n
,i
trn ta suy ra Sn = xn
ung l`
a s nguyn. Suy ra Sn l`
a s nguyn vo
1 + x2 c

moi
. n.

,
,
,
,ng s x, y, z thoa m
V du. 2.53. Chung minh ra
`ng tt ca nhu
an x + y + z = 5


,
7
v`
a xy + yz + zx = 8, na
`m trong khoang 1, .
3


,
,
7
,
,
`
`i giai. Ta se chu
ng minh s z na
m trong khoang 1, , c`
Lo
on c
ac s kh
ac
3
,
,,
,
,
,c ta tnh du,o.,c 1 = x + y = 5 z,
ng thu
ho`
an to`
an tuong tu. . T`u hai da
2 = xy = 8 z(x + y) = 8 z(5 z) = z2 5z + 8. Suy ra x, y l`
a nghim
.
,
,,
cua phuong tr`nh
u2 (5 z)u + z2 5z + 8 = 0.
,
,
,
,
,,
V` x, y l`
a nhung s thu. c nn bit
. s cua phuong tr`nh phai khng m:
(5 z)2 4(z2 5z + 8) 0.

,
,,
,,
Ta c
o th vit phuong tr`nh cui c`
ung duo
i dang
a t`m
. (z 1)(3z 7) 0 v`
7
,,
duo. c 1 z .
3
, ,
,
` tp
Bai
(Mt
a tra l`oi b`
ai tp
. s go. i y v`
. sau tai
. trang 120)
.

,
,,
,
. 2.54. Lp
a c
ac nghim
o l`
a l
uy th`ua bc
. phuong tr`nh bc
. hai m`
. cua n
.
,
,
,,
2
ba cua c
ac nghim
c
ua
phu
o
ng
tr`
nh
x
+
6x
+
10
=
0.
.
,
,,
,
. 2.55. Lp
a c
ac nghim
o l`
a l
uy th`ua bc
. phuong tr`nh bc
. hai m`
. cua n
.
,
,
,`,
,,
2
muoi cua c
ac nghim
. cua phuong tr`nh x + x 3 = 0.

,c di xu
,ng ra thu
`,a s
2.7. Phn tch da thu

91

,
,,
. 2.56. Lp
a c
ac nghim
an x51 +x52 =
. phuong tr`nh bc
. hai m`
. x1 , x2 thoa m
31, x1 + x2 = 1.
,
,
,
,
,,
,i gi
. 2.57. Vo
a tri. thu. c n`
ao cua a th` tng b`nh phuong c
ac nghim
. cua
,
,,
phuong tr`nh x2 (a 2)x a 1 = 0 nhn
a tri. nho nht?
. gi
,
,,
`ng nu x1 , x2 l`
,ng minh ra
. 2.58. Chu
a nghim
. cua phuong tr`nh bc
. hai
,
,
2
n
n ,

x 6x + 1 = 0, th` tng x1 + x2 voi n l`


a s tu. nhin bt k` khng chia ht
cho 5.
,
,,
. 2.59. Cho x2 + px + q = 0 l`
a phuong tr`nh bc
o
. hai, nghim
. , cua n
,
,

,
,,
,
,,
l`
a nhung s duong. H
ay biu din 4 + 4 theo nhung h. s cua phuong
tr`nh.

,
,
,
2.7. Phn tch da
uc d
ung ra th`ua s
th
i x
,
,
,
`ng
,c d
,ng bc
Ta c
o th phn tch ra th`ua s nhung da thu
i xu
. cao ba
,
,,
,,
,c c
c
ach chuyn th`
anh c
ac d
ac bin 1 v`
a 2 . C
ac buo
cd
anh:
a thu
uo. c tin h`
,
,
,
,
,
c di xu
ng th`
c cua 1 v`
1. Chuyn d
anh c
ac d
a 2 v`
a ta phn
a thu
a thu
,
,

c n`
tch d
ay ra th`ua s.
a thu
,
, ,
,
,,
,c theo bc
a nho hon bc
2. Ta phn tch d
a thu
. cua da
. cua 2 , thu`ong l`
,c ban d`u.
thu
,
,
, ,
,c v`
3. Thay gi
a tri. cua 1 = x + y v`
a 2 = xy v`
ao biu thu
a bin d
i dua
,
,
,
,
,c bc
. c ba theo x, y v`
v` nhung tch cua nhung da thu
a t`u d
y
. hai hoa
,
,
,

`
`
ta tnh nghim
a phn tch tip.
. cua tung thua s v`
,c bc
Ta ly v du. c
ac da thu
. bn:
,
`,a s:
V du. 2.60. H
ay phn tch d
a thuc ra thu
P(x, y) = 10x4 27x3 y 110x2 y2 27xy3 + 10y4 .

,,
,c d
,ng hai bin
Chuong 2. a thu
i xu

92
,
`,i giai. Ta c
o
Lo

P(x, y) = 10(x4 + y4 ) 27xy(x2 + y2 ) 110x2 y2 = 10S4 272 S2 11022 .


,,
,c (2.2) ta t`m d
Theo cng thu
uo. c
P(x, y) = 1041 6721 2 3622 .
,c n`
,c bc
,c tam
,i kin thu
Ta coi da thu
ay l`
a da thu
. hai theo bin 2 , th` vo
,
,
,,
,c bc
,c
thu
ang phn tch ra th`ua s. Giai phuong tr`nh d
a thu
. hai ta d d`
5 2
,i 2 ta c
di vo
o c
ac nghim:
2 = 221 v`
, vy
a 2 =
.
36 1 .
5
P(x, y) = 36(2 + 221 )(2 21 ) = (221 + 2 )(521 362 ).
36
,
Thay c
ac gi
a tri. cua 1 v`
a 2 ta c
o
P(x, y) = (2(x + y)2 + xy)(5(x + y)2 36xy) =
= (2x2 + 5xy + 2y2 )(5x2 26xy + 5y2 ).
,,
th`u,a s lai
,c bc
,c
ac d
Mi
a tam thu
a thu
. l`
. hai nn ta lai
. l`n luo. t phn tch c
,
,
,c bc
n`
ay. V du. xem 2x2 + 5xy + 2y2 nhu tam thu
o
. hai cua bin x, ta se c
1
nghim
a v` th
. x = 2 y, x = 2y v`
1
2x2 + 5xy + 2y2 = 2(x + y)(x + 2y) = (2x + y)(x + 2y).
2
,,
,
Tuong tu. ta c
ung c
o
5x2 26xy + 5y2 = (x 5y)(5x y).
,,
Cui c`
ung ta nhn
uo. c
. d
P(x, y) = (2x + y)(x + 2y)(x 5y)(5x y).
,
`,a s:
V du. 2.61. H
ay phn tch da thuc ra thu
P(x, y) = 6x4 11x3 y 18x2 y2 11xy3 + 6y4 .

,c di xu
,ng ra thu
`,a s
2.7. Phn tch da thu

93

,
,
`,i giai. Ta biu din d
,c di xu
,ng P(x, y) qua 1 v`
Lo
a 2 :
a thu
P(x, y) = 6(x4 + y4 ) 11xy(x2 + y2 ) 18x2 y2
= 6S4 112 S2 1822
= 641 3521 2 + 1622 .
,
,c n`
,c bc
,i bin 2 . Giai ra ta c
Ta coi da thu
ay l`
a tam thu
o nghim
. hai di vo
.
3
,
,
,
,
,
2
2
c d
v`
a da thu
2 = 21 v`
a 2 =
uo. c phn tch ra th`ua s nhu sau:
16 1
3
P(x, y) = 16(2 221 )(2 21 ) = (221 2 )(321 162 ).
16
,,
,
,
`
`
c ban d
T`u dy tro v d
a thu
u
P(x, y) = (2(x + y)2 xy)(3(x + y)2 16xy) =
= (2x2 + 3xy + 2y2 )(3x2 10xy + 3y2 ).
,
,,
, nht c
,c nn khng phn tch d
Th`ua s thu
o nghim
a s phu
anh
uo. c th`
. l`
,
,
,
,
,
c c
hai d
tch c
ac da thu
o h. s l`
a thu. c, nn ta giu nguyn. Th`ua s thu
,
,
d`
ang phn tch th`
anh (x 3y)(3x y). Cui c`
ung ta nhn
. duo. c
P(x, y) = (2x2 + 3xy + 2y2 )(x 3y)3x y).
,
`,a s:
V du. 2.62. H
ay phn tch d
a thuc ra thu
P(x, y) = 2x4 x3 y + x2 y2 xy3 + 2y4 .
,
`,i giai. Tu,o,ng tu., nhu, hai b`
ai trn ta c
o
Lo
P(x, y) = 2(x4 + y4 ) xy(x2 + y2 ) + x2 y2 = 2S4 2 S2 + 22 =
= 2(41 421 2 + 222 ) 2 (21 22 ) + 22 =
= 241 921 2 + 722 = (21 2 )(221 72 ) =
= ((x + y)2 xy)(2(x + y)2 7xy) =
= (x2 + xy + y2 )(2x2 3xy + 2y2 ).

94

,,
,c d
,ng hai bin
Chuong 2. a thu
i xu

,
,,
,
,
,c c
Hai th`ua s trn khng phn tch duo. c ra tch nhung d
o h. s thu. c.
a thu

,
,,
,
,c di xu
,ng chuyn sang da thu
,c phu.
C
o nhi`u tru`ong ho. p khi d
a thu
,
,
,
,,
,i n 2 th` phu,o,ng
thuc
ao 1 v`
a 2 , nhung khi giai phuong tr`nh d
i vo
. v`
,
,
,
tr`nh khng c
o nghim
ach phn tch theo kiu n`
ay khng
. thu. c. Nhu vy
. c
,
,
,
,
,
,
,

cho kt qua. Voi nhung da thuc nhu vy,


o th phn tch ra th`ua s
. ta c
,
,,
, ,,
,
`ng c
ng han
ba
ach d`
ung phuong ph
ap h. s bt dinh,
cha
.
. nhu tru`ong ho. p
,
bc
o phn tch
c
. bn, d
P(x, y) = (ax2 + bxy + cy2 )(cx2 + bxy + ay2 )
,
,
,
,,
,
,
od
a nhung n s c`n phai t`m. Ta c
o th l`
am nhu v du. sau:
y a, b, c l`
,
`,a s:
V du. 2.63. H
ay phn tch d
a thuc ra thu
P(x, y) = 2x4 + 3x3 y + 6x2 y2 + 3xy3 + 2y4 .
,
,
,
`,i giai. Ta c
,c v` dang
Lo
o th chuyn d
ac bin 1 v`
a 2 :
a thu
. c
P(x, y) = 241 521 2 + 422 .
,
,,
,
,i 2 khng c
Phuong tr`nh n`
ay d
o nghim
ung c
ach
i vo
. thu. c. Vy
. ta phai d`
kh
ac. Ta c
o
2x4 + 3x3 y + 6x2 y2 + 3xy3 + 2y4 = (ax2 + bxy + cy2 )(cx2 + bxy + ay2 ).
,
,
, ,
,c du
,i moi
ng thu
ng vo
a tri. x, y. V` th ta c
o th
Ta phai t`m a, b, c d da
. gi
,
,,
p dung
a
phuong ph
ap gi
a tri. ring cua x, y, ngha l`
a cho mt
a tri. cu.
.
. s gi
, ,
,
2

th d
ac h. s. Ly x = y = 1 th` 16 = (a + b + c) , t`u dy c
o
t`m c
,
,
`
a + b + c = 4. Tuy c
o hai gi
a tri. nhung ta ch cn chon
. mt
.
a + b + c = 4.
,,
Cho x = 1, y = 1 ta nhn
a ab+c = 2, ta
uo. c 4 = (ab+c)2 , ngha l`
. d
,
,
,,

c
ung ch ly du cng
a du. Cui c`
ung x = 0, y = 1, ta nhn
uo. c ac = 2.
. l`
. d

,ng b`
2.8. Nhu
ai to
an kh
ac

95

,
,
,
,,
Nhu vy
ac d
x
inh
. a, b, c ta giai h. phuong tr`nh
. d

a + b + c = 4,
a b + c = 2,

ac = 2.
,,
,
,
,
, ,
T`u hai phuong tr`nh d`u ta t`m d
a a + c = 3. Kt ho. p vo
i
uo. c b = 1 v`
,,
,
,
,
ba ta t`m duo. c a = 1, c = 2 (hoa
. c c = 1, a = 2). Cui
phuong tr`nh thu
,,
c`
ung ta nhn
. duo. c
2x4 + 3x3 y + 6x2 y2 + 3xy3 + 2y4 = (x2 + xy + 2y2 )(2x2 + xy + y2 ).
,
, ,
`ng c
Ta c
o th kim tra lai
ach phn tch n`
ay ba
ach khai trin v`
a nh
om
. c
,
,
`

c ban du. Mt
lai
anh biu thu
u y l`
a nu ly a b + c = 2 th` h.
. th`
. ch
,,
,
,
,
c v`
phuong tr`nh se c
o nghim
a nhu vy
. phu
. khng dua dn kt lun.
.

,
,
` tp
(Mt
a tra l`oi b`
ai tp
Bai
. s go. i y v`
. sau tai
. trang 123)
.
,
,c sau ra th`u,a s
Phn tch nhung da thu
. 2.64. 2x4 + 7x3 y + 9x2 y2 + 7xy3 + 2y4 ;
. 2.65. 18x4 21x3 y 94x2 y2 21xy3 + 18y4 ;
. 2.66. 3x4 8x3 y + 14x2 y2 8xy3 + 3y4 ;
. 2.67. 2x4 x3 y + 3x2 y2 xy3 + 2y4 ;
. 2.68. (x + y)5 x5 y5 .

,
` toan
khac

2.8. Nhung bai


,c d
,ng c`
C
ac dang
ai to
an lin quan dn d
on rt nhi`u. Khi ra
a thu
i xu
. b`
,
,
,
,
,
c hoa
. c nhung kt lun
chon
d
`, ngu`oi ra d` d
a
ep,
. nhung cng thu
. d
. ph`n
,
,
ng cua c
. Muc
n`
ao c
o ph`n d
ac bin trong do
ay ta xet mt
i xu
. n`
. s loai
.
,
,
,
,
,

p dung
b`
ai to
an m`
a c
o kha n
ang a
d
a
th
u
c
d

i
x
u
ng
d
u
o
c.

.
.
,
,
c. Ta xet v du.
1. R
ut gon
. biu thu

,,
,c d
,ng hai bin
Chuong 2. a thu
i xu

96

,
,
V du. 2.69. R
ut gon
. biu thuc sau:






1
1
1
1
1
3
1
6
1
P(p, q) =
+
+
+
.
+
+
(p + q)3 p3 q3
(p + q)4 p2 q2
(p + q)5 p q
,
,,
`,i giai. Ta d
,c (2.2), ta c
. t 1 = p + q v`
Lo
a 2 = pq. Su dung
cng thu
o
a
.
1 S3
3 S2
6 1
P(p, q) = 3 3 + 4 2 + 5
=
1 2
1 2
1 2
=

31 31 2 3(21 22 )
6
+
+ 4 =
31 32
41 22
1 2

4
1
1
1 (31 31 2 ) + 32 (21 22 ) + 622
= 413 = 3 = 3 3 .
4
3
p q
1 2
1 2
2

,
,
,ng minh d
,c (hoa
,ng minh bt da
,c). D`
ng thu
. c chu
ng thu
2. Chu
ung da
a
,
,
,
,
,
,
,
,

c d
ng ta c
ng minh d
c (hoa
c) mt
. c bt d
ng thu
ng thu
thu
o th chu
i xu
a
a
.
,
,
,

c
ach rt tu. nhin v`
a ch thng qua bin di.
,
,
,
V du. 2.70. Chung minh d
ng thuc sau
a
(x + y)3 + 3xy(1 x y) 1 = (x + y 1)(x2 + y2 xy + x + y + 1);
,
,
,
`,i giai. Ta bin di v tr
Lo
ai cua mi lin h.
(x + y)3 + 3xy(1 x y) 1 = 31 + 32 (1 1 ) 1 = 31 + 32 31 2 1.
,
,
, ,
,c
ng thu
Ta bin d
i v phai cua d
a
(x + y 1)(x2 + y2 xy + x + y + 1) = (1 1)(21 32 + 1 + 1)
= 31 31 2 + 21 + 1 21 + 32 1 1 =
= 31 31 2 + 32 1.
,
,
,c.
ng thu
Hai v d`u c
o kt qua tr`
ung nhau, ta c
o da
,
,,
3. Giai phuong tr`nh v dinh.
.
,
,,
,,
V du. 2.71. H
ay t`m nghim
. nguyn duong cua phuong tr`nh sau
x3 + y3 + 1 = 3xy.

,ng b`
2.8. Nhu
ai to
an kh
ac

97

,
,
,
,,
`,i giai. Ta chuyn c
Lo
ac s hang
ad
ua phuong tr`nh v` dang
. sang mt
. v v`
.
31 31 2 + 1 32 = 0, suy ra (1 + 1)(21 1 + 1 32 ) = 0.
,
,
,
,
T`u x > 0, y > 0 suy ra 1 > 0, nhu vy
a nghim
. 1 = 1 khng phai l`
. cua
,
,
,,
,,
phuong tr`nh trn. Ta ch c`
on giai phuong tr`nh
21 1 + 1 32 = 0.
,,
,
Suy ra x v`
ayd
ac dinh
uo. c x
. t`u h.

x + y = 1
1 2
( 1 + 1).
3 1
,
,
,,
,,
,,
,i vic
Theo dinh
. l 2.2, giai h. trn tuong duong vo
. giai phuong tr`nh sau
1
z2 1 z + (21 1 + 1) = 0.
3
,
,,
Nghim
a
. cua phuong tr`nh bc
. hai l`
r
1
21 1 2
z1,2 =

(1 1 + 1) =
2
3
r4
r
1
1
1 2
1
=
(1 41 + 4) =
(1 2)2 .
2
12
2
12
,
,
,
,
,,
,
,c du,o
,i du c
nghim
a s thu. c, th` biu thu
an phai
. cua phuong tr`nh l`
, ,
,i x, y nguyn
khng m, d
ay ch xay ra khi 1 = 2 hay x + y = 2 vo
i`u n`
,
,
,
,,
,
,,
duong. i`u n`
ay ch xay ra khi x = y = 1. Ta thu lai
. phuong tr`nh, th`
,
,
,,
x=y=1d
a nghim
ch thu. c l`
. cua phuong tr`nh.
,c cho mt
,c kh
,c
,i nhu,ng d
4. B`
ai to
an chia ht mt
ac vo
a thu
a thu
. d
. da thu
,,
,,
,
,ng theo c
,ng minh gon
di xu
ac bin duo. c chu
ang hon c
ac phuong ph
ap
. g`
,`,
thng thuong.
xy = 2 =

,
, ,
V du. 2.72. Chung minh ra
`ng mt
o
i xung hai bin x, y khng c
. da thuc d
,
,
,
,
,
,
2
2

h. s tu. do chia ht cho x + xy + y khi v`


a ch khi d
a thuc d
uo. c biu din
,
,
,
`

theo 1 v`
a 2 nhu mt
o tng c
ac h. s ba
ng khng.
a thuc hai bin c
. d

,,
,c d
,ng hai bin
Chuong 2. a thu
i xu

98

,
, ,
`,i giai. Gia su, da thu
,c di xu
,ng P(x, y) chia ht cho x2 + xy + y2 , ngha
Lo
,,
,c di xu
,ng. Ta
l`
a P(x, y) = (x2 + xy + y2 )Q(x, y), o d
a da thu
y Q(x, y) l`
,
,
,
,
c
o x2 + xy + y2 = 21 2 . Gia su Q(x, y) = (1 , 2 ) biu din qua 1 v`
a
P(x, y) c
2 . Khi do
o dang
.
P(x, y) = (x2 + xy + y2 )Q(x, y) = (21 2 )(1 , 2 ).
,
,,
,
,c trn ta nhn
Thay 1 = 2 = 1 v`
ao biu thu
uo. c (1 1)(1, 1) = 0. Nhu
. d
,
vy
a 2 v`
a P(x, y) = (21 2 )(1 , 2 ) nhn
a
. P(x, y) biu din qua 1 v`
. gi
,
,
2
tri. 0 khi 1 = 2 = 1. i`u n`
ay ngha l`
a tng h. s cua (1 2 )(1 , 2 )
`
ng khng.
ba
,,
Nguo. c lai,
. cho
n2
P(x, y) = n
2 + b2 n4
22 + b3 n6
32 +
1 + b1 1
1
1

,
,
, ,, ,
,
,
l`
a biu din cua P(x, y) qua 1 v`
a 2 v`
a gia su tng h. s cua c
ach biu
,,
`ng khng. u,a n ra ngo`
din trn ba
ai, th` P(x, y) duo. c vit lai
.
1

P(x, y) =

n
1


1 + b1

2
21


+ b2

2
21

2


+ b3

2
21

3
+

,
2
,
,,
,
,c trong du ngoa
. c (theo bin z = 2 ) khng vuo. t
Hon nua bc
. cua da thu
1
n
qu
a v`
a k hiu
o l`
a k.
. n
2
,
,
,
,
`ng 0. Ngha
,c trong du ngoa
. c ba
Tng cua nhung h. s trong biu thu
,
,
2
3
`ng 0 vo
c 1 + b1 z + b2 z + b3 z + b a
i z = 1. Suy ra theo dinh
l`
ad
a thu
.
,
,
2
c n`
l Bezout d
ay chia ht cho 1 z. Nhu vy
a thu
.
1 + b1 z + b2 z2 + b3 z3 + = (1 z)H(z),
2 Etienne

,,
Bezout (1730-1783): Nh`
a to
an hoc
ap.
. ngu`oi Ph

,ng b`
2.8. Nhu
ai to
an kh
ac

99

,
,,
`ng k 1. V` th
od
y bc
. cua H(z) ba
!
 2
 3
 
2
2
2
n
+ b2
+ b3
+ =
P(x, y) = 1 1 + b1
21
21
21

  
 
2
2
2
n
2
n2
= 1 1 2 H
= (1 2 )1 H
.
2
1
1
21
,
,c H(z) khng vu,o.,t
Nu n = 2m + 1, th` k m, ngha l`
a bc
. cua da thu
,
, ,
,c
qu
a m 1, c`
on s n 2 = 2m 1 khng nho hon hai l`n bc
a thu
. cua d
H(z).
,
,,
Nu n = 2m, th` k m, ngha l`
a bc
am1
. xua H(z) khng vuo. t qu
, ,
,
,
v`
a v` th s n 2 = 2m 2 khng nho hon hai l`n bc
. cua H(z). Nhu vy,
.
,
,
,`,
,
,
`

trong moi
tru
o
ng
h
o
p
n

2
khng
nh
o
ho
n
hai
l
n
b
c
c
ua
H(z),
v`

th
. 
.
.
,
,

2
ngha l`
,a 1 o, mu,
,c cua 1 .2 .
khng chu
a n
o l`
a mt
n2
H
. da thu
1
21
,
Ta c
o th vit
P(x,
y) =(21 2 )(1 , 2 ),
,,
2
n2
,c n`
,c
. Suy ra da thu
o dy (1 , 2 ) = 1 H
l`
a mt
ao d
a thu
o
. d
21
P(x, y) chia ht cho 21 2 = x2 + xy + y2 .

,
,
,,
,
. c nhung thng s
5. T`m nhung d
o ngha hoa
i`u kin
h. phuong tr`nh c
. d
,
,
,,
,,
trong phuong tr`nh thoa m
an di`u kin
o nghim.
h. phuong tr`nh c
. d
.
,
,
V du. 2.73. T`m d
a nghim
i`u kin
(x, y) l`
. cho a, b, c d
. cua h.

= a,
x + y
x2 + y2 = b,

3
x + y3 = c.
,
, ,
,
,
,
`,i giai. B`
ai to
an d
ai to
an loai
on lai
Lo
ua v` b`
ch c`
. tr`u x, y d
. su. phu. thuc
.
,
,
,i 1 = x + y v`
a, b, c. Vo
a 2 = xy ta c
o th chuyn h. trn v` dang
.

= a,
1
21 22
= b,

3
1 31 2 = c.

,,
,c d
,ng hai bin
Chuong 2. a thu
i xu

100

1
,,
,
,,
ao phuong tr`nh
T`u hai phuong tr`nh: 1 = a, 2 = (a2 b), thay v`
2
3
`ng
, ba ta c
thu
o a3 a(a2 b) = c, ngha l`
a a3 3ab + 2c = 0. D thy ra
2
3
3
nu a 3ab + 2c = 0 th` h. c
o nghim.
vy
a
.
. a 3ab + 2c = 0 chnh l`
,
,
`i hoi mi lin quan giua a, b, c.
di`u kin
. do

,
,
` tp
Bai
(Mt
a tra l`oi c
ac b`
ai tp
. s go. i y v`
. sau tai
. trang 125)
.
,
,
,c sau
R
ut gon
. nhung biu thu
(x + y)7 x7 y7
;
(x + y)5 x5 y5




1
1
1
1
2
1
. 2.75.
+
+
.
+
(a + b)2 a2 b2
(a + b)3 a b
. 2.74.

`ng
,ng minh ra
Chu
. 2.76. (x + y)4 + x4 + y4 = 2(x2 + xy + y2 )2 ;
. 2.77. (x + y)7 x7 y7 = 7xy(x + y)(x2 + xy + y2 )2 .
`ng vo
,ng minh ra
,c (x + y)n xn yn chia
,i n = 6k 1, d
. 2.78. Chu
a thu
ht cho x2 + xy + y2 .
,c x2n + xn + 1 chia ht cho
,i nhu,ng di`u kin
. 2.79. Vo
ao th` d
a thu
. n`
x2 + x + 1?
,c (x + 1)n + xn + 1 chia ht cho
,i nhu,ng di`u kin
. 2.80. Vo
ao th` da thu
. n`
x2 + x + 1?
,
`ng nu nhu,ng s u, v, x, y thoa m
,ng minh ra
. 2.81. Chu
an u + v = x +
,
,
,
,
2
2
2
2

c sau d
i moi
ng thu
ng
y, u + v = x + y , th` vo
u
. s tu. nhin n da
un + vn = xn + yn .
,
,,
. 2.82. Giai phuong tr`nh trong tp
. s nguyn
x + y = x2 xy + y2 .

,,
,ng
2.9. Chuyn d
` v` phuong tr`nh h. s di xu

101

,
,
,
,
`ng nu n l`
,ng minh ra
,c
. 2.83. Chu
a nhung s le, l`
a bi
. cua 3, th` biu thu
(a + b)n an bn 3(ab)

n1
2

(a + b)

chia ht cho a2 + ab + b2 .

,,
,
2.9. Chuyn d
ung
` v` phuong tr`nh h. s d
i x
, ,
,
,,
,c di xu
,ng c
ng dung
a thu
o th u
giai mt
.
. s phuong tr`nh bc
. cao.
,
,
,

. c bit
. Trong tit
Tt nhin nhung phuong tr`nh bc
o dang
ao do
a
. cao c
. d
. n`
,
,
,
,
ng:
i phuong tr`nh h. s d
n`
ay ta quan tm to
i xu
,c mt
inh
ngha 2.3. Mt
. da thu
. bin
.
P(z) = a0 zn + a1 zn1 + + an (a0 6= 0)
(2.6)
,
,
,
an a0 = an , a1 = an1 , a2 =
goi
a h. s di xung, nu nhung h. s thoa m
. l`
an2 , ....
,
,
,c n`
ng han
V du. v` loai
ay rt nhi`u cha
a thu
. d
. nhu:
. c zn + 1 d`u l`
z5 3z4 + 2z3 + 2z2 3z + 1; 2z8 6z5 6z3 + 2; hoa
a
,
,
,

c h. s d
ng.
nhung da thu
i xu
,,
,c h. s di xu
,ng goi
Mt
ai l`
a mt
a
. phuong tr`nh P(z) = 0, v tr
. da thu
. l`
,,
,
phuong tr`nh h. s d
o dinh
i xung. Ta c
. l
,
,
,
, ,
2.9.1. .inh l co ban cua da
uc h. s d
ung
th
i x
,
,
inh
l 2.4. Moi
n 2k
a thuc h. s di xung bc
. d
. cha
.
P(z) = a0 z2k + a1 z2k1 + + a2k

, ,
du,o,i dang
c
o th biu din
.

P(z) = zk Q(),
,
,,
1
,
od
a Q() l`
a mt
d
k cua .
a thuc bc
y = z + v`
.
.
z

,,
,c d
,ng hai bin
Chuong 2. a thu
i xu

102

,
,
,
a cho
Moi
`u chia ht cho (z + 1) v`
i xung P(z) bc
a thuc h. s d
. le d
. d
,,
,
,

thuong l`
a mt
n.
i xung bc
. da thuc h. s d
. cha
n 2k: P(z). Ta d
,ng minh. 1) Ta xet d
,c bc
. t s hang
Chu
chung
a thu
a
. cha
.
zk ra ngo`
ai


1
1
P(z) = zk a0 zk + a1 zk1 + + a2k1 k1 + a2k k .
z
z
`ng a0 = a2k , a1 = a2k1 , ...
Ta ch
u y ra




 
1
1
P(z) = zk a0 zk + k + a1 zk1 + k1 + + ak .
z
z
,
,
,
1
,
,ng minh zi + i , i = 1, 2, ...k biu din thng
Nhu vy
on phai chu
. ta ch c`
z
,
1
,
,
,c (2.2) vo
,i
qua = z + . Nhung b`
ai to
an n`
ay l`
a h. cua nhung cng thu
z
,
,
tng Sk = xk + yk biu din thng qua 1 = x + y v`
a 2 = xy. Trong
,
1
1
,`,
,
k
k
truong ho. p cu. th n`
ay x = z, y = th` Sk = x + y chnh l`
a zk + k v`
a
z
z
1
,c (2.2) vo
,i 1 = , 2 = 1 du,o.,c vit lai
1 = z + = , 2 = 1. Cng thu
.
z
,
nhu sau:
z2 + z2 = 2 2,
z3 + z3 = 3 3,
z4 + z4 = 4 42 + 2,
z5 + z5 = 5 53 + 5,
z6 + z6 = 6 64 + 92 2,
z7 + z7 = 7 75 + 143 7,
z8 + z8 = 8 86 + 204 162 + 2,
z9 + z9 = 9 97 + 275 303 + 9,
z10 + z10 = 10 108 + 356 504 + 252 2,

,,
,ng
2.9. Chuyn d
` v` phuong tr`nh h. s di xu

103

,
,c h. s d
,ng bc
2) Ta xet da thu
i xu
. le 2k + 1:
P(z) = a0 z2k+1 + a1 z2k + + a2k z + a2k+1 .

,
,,
ta c
Ta c
o a0 = a2k+1 , a1 = a2k , a2 = a2k1 , ... khi do
o th vit duo
i dang
.
P(z) = a0 (z2k+1 + 1) + a1 (z2k + z) + a2 (z2k1 + z2 ) + + ak (zk+1 + zk )
= a0 (z2k+1 + 1) + a1 z(z2k1 + 1) + a2 z2 (z2k3 + 1) + + ak zk (z + 1).

,
s hang,
`ng c
,c trong du ngoa
. c c
Trong mi
biu thu
o chung z + 1, ba
ach
.
,
c sau
p dung
a
cng thu
.
z2m+1 + 1 = (z + 1)(z2m z2m1 + z2m2 + z2 z + 1).
,,
Ta nhn
. duo. c
a0 (z2k+1 + 1) = a0 (z + 1)(z2k z2k1 + z2k2 + + z2 z + 1),
a1 z(z2k1 + 1) = a1 z(z + 1)(z2k2 z2k3 + z + 1) =
= a1 (z + 1)(z2k1 z2k2 + z2 + z),
a2 z2 (z2k3 + 1) = a2 z2 (z + 1)(z2k4 + 1) =
= a2 (z + 1)(z2k2 + z2 ),

ak zk (z + 1) = ak (z + 1)zk .

,,
,,
`ng c
Ba
ach nh
om z + 1 ra ngo`
ai ta duo. c P(z) = (z + 1)Q(z), o dy Q(z) l`
a
,
, ,
,
,
, ,
,
,
,

d
a ta c
o th kim tra tng cua nhung d
a thuc tng cua nhung da thuc v`
a
,
,
,

thuc n`
ay l`
a mt
i xung.
. da thuc h. s d

,
,
,
,
Mnh
d` 2.1. a thuc P(z) bc
ad
a ch khi
a thuc h. s d
i xung khi v`
. n l`
.
,
n
o thoa m
an
 
1
= P(z).
zn P
z

,,
,c d
,ng hai bin
Chuong 2. a thu
i xu

104

,ng minh. Cho P(z) = a0 zn + a1 zn1 + + an (a0 6= 0). Ta c


Chu
o
!
 
 n
 n1
1
1
1
1
zn P
= zn a0
+ a1
+ + an1 + an =
z
z
z
z
= a0 + a1 z + + an1 zn1 + an zn =
= an zn + an1 zn1 + + a1 z + a0 .

,
,
, , ,
,c P(z) = zn P 1z du
ng thu
ng khi v`
a
a ch khi nu c
ac h. s tuong u
ng
,
,
`
c sau ba
ng nhau, ngha l`
cua hai da thu
a
a0 = an , a1 = an1 , ....
 , ,
,
,
,c
,c P(z) = zn P 1z ch xay ra khi v`
ng thu
a ch khi da thu
N
oi c
ach kh
ac da
,c h. s d
,ng.
P(z) l`
a da thu
i xu

,
,
,ng d
a P(z)
Mnh
d
a Q(z) l`
a nhu
i xung v`
a thuc h. s d
` 2.2. Nu P(z) v`
.
P(z)
,
,
chia ht cho Q(z), th` H(z) =
c
ung l`
ad
a thuc h. s d
i xung.
Q(z)
,
,
,
,,
,ng minh. Ta k hiu
,c P(z) v`
Chu
a Q(z) tuong
. bc
. cua cua nhung da thu
,,
,
,ng l`
,c H(z) c
u
a m v`
a n th` da thu
o bc
a Q(z) l`
a nhung
. m n. Boi v` P(z) v`
,
,c h. s d
,ng nn theo kt qua mnh
da thu
d` 2.1 th`
i xu
.
 
 
1
1
n
m
, Q(z) = z Q
.
P(z) = z P
z
z
,
,,
,c trn, ta nhn
ng thu
Chia theo v hai da
. duo. c


1
zm P 1z
P(z)
mn P z
 =z
.
= n
Q(z)
z Q 1z
Q 1z
,
,,
,,
Nhu vy
i dang
. H(z) vit duo. c duo
.
 
1
.
z
,c h. s di xu
,ng.
i`u n`
ay ngha l`
a H(z) l`
ad
a thu
H(z) = zmn H

,,
,ng
2.9. Chuyn d
` v` phuong tr`nh h. s di xu

105

,
,
,,
2.9.2. Nhung v du. giai phuong tr`nh bc
. cao
,ng dung
Ta xet mt
dinh
. l trn:
. s v du. u
.
,
,,
V du. 2.84. Giai phuong tr`nh 6z4 13z3 + 12z2 13z + 6 = 0.
,
,,
n) o,, trn l`
`,i giai. Phu,o,ng tr`nh bc
Lo
a phuong tr`nh h. s d
i
. bn (bc
. cha
,
,
,
,
,
,
,
ng. Theo dinh
xu
ai cua phuong tr`nh duo. c bin d
a
i l`
. l 2.4 v tr


1
1
4
3
2
2
2
6z 13z + 12z 13z + 6 = z 6z 13z + 12 13 + 6 2 =
z
z
 




1
1
= z2 6 z2 + 2 13 z +
+ 12 =
z
z
= z2 (6(2 2) 13 + 12) = z2 (62 13)
,
,
,
,
,,
cho, ta ch c`
V` z = 0 khng phai l`
a nghim
on giai
a
. cua phuong tr`nh d
,,
phuong tr`nh
62 13 = 0.
,
13
,
,,
cho
D thy = 0 v`
a=
. Nhu vy
t`m nghim
cua phuong tr`nh da
.
.
6
,
,,
,,
,
,
,
i giai hai phuong tr`nh
tuong duong vo
1
13
1
.
z + = 0, z + =
z
z
6
,,
,
,,
, nht khng c
, hai c
Phuong tr`nh thu
o nghim
o hai
. thu. c, phuong tr`nh thu
3
2
nghim
a z1 = v`
a z2 = .
. l`
2
3
,
,,
V du. 2.85. Giai phuong tr`nh

4z11 +4z10 21z9 21z8 +17z7 +17z6 +17z5 +17z4 21z3 21z2 +4z+4 = 0
,
,
,,
`,i giai. y l`
,ng bc
a phuong tr`nh h. s d
Lo
i xu
inh
. l 2.4 v
. le. Theo d
,
,
,,
tr
ai cua phuong tr`nh chia ht cho z + 1 v`
a cho kt qua
4z11 +4z10 21z9 21z8 +17z7 +17z6 +17z5 +17z4 21z3 21z2 +4z+4 =
= (z + 1)(4z10 21z8 + 17z6 + 17z4 21z2 + 4).

,,
,c d
,ng hai bin
Chuong 2. a thu
i xu

106

,
,
,
,,
,,
,,
,i giai hai phu,o,ng tr`nh
Nhu vy,
`u tuong d
uong vo
. giai phuong tr`nh ban d
4z10 21z8 + 17z6 + 17z4 21z2 + 4 = 0.
,
,,
,,
, nht cho ta nghim
Phuong tr`nh thu
ai cua phuong tr`nh
. z1 = 1. V tr
,
n. Ta lai
, hai lai
,c h. s di xu
,ng bc
thu
ad
a thu
. l`
. cha
. bin di
z + 1 = 0;

4z10 21z8 + 17z6 + 17z4 21z2 + 4 =




1
1
1
5
5
3
= z 4z 21z + 17z + 17 21 3 + 4 5 =
z
z
z
 





1
1
1
5
5
3
=
= z 4 z + 5 21 z + 3 + 17 z +
z
z
z
= z5 (4(5 53 + 5) 21(3 3) + 17) = z5 (45 413 + 100).
,
,
,
,,
,,
cho, nn ta ch c`
Boi v` z = 0 khng phai l`
a nghim
on
a
. cua phuong tr`nh d
,
,
,,
phai giai phuong tr`nh
(44 412 + 100) = 0.
,,
Suy ra ta c
o nghim
a thm c
ac nghim
ung
. = 0 v`
. trong phuong tr`nh tr`
,,
phuong
44 412 + 100 = 0.
,
,
Cui c`
ung ta nhn
am nghim
uo. c n
. d
.
5
5
= 0, = , = , = 2, = 2.
2
2
,
,
,
,
,,
,,
,,
t`m d
am phuong
uo. c nghim
. cua phuong tr`nh ban d`u ta phai giai n
tr`nh
1
1
5
1
5
1
1
= 0, z + = , z + = , z + = 2, z + = 2.
z
z
2
z
2
z
z
,,
,,
,,
,i mt

C`
ung vo
nghi
m
d
a
t`
m
d
u
o
c
tru
o
c
d
y
z
=
1,
ta
t`
m
thm d
.

uo. c 6
1
.
.
,
,
nghim
o 2 nghim
a
. nua (c
. kep), tng cng
. l`
1
z1 = 1, z2 = 2, z3 = ,
2
1
z4 = 2, z5 = , z6 = z7 = 1, z8 = z9 = 1.
2
z+

,,
,ng
2.9. Chuyn d
` v` phuong tr`nh h. s di xu

107

,
,,
V du. 2.86. Giai phuong tr`nh
9z6 18z5 73z4 + 164z3 73z2 18z + 9 = 0.
,
,,
n, ta c
`,i giai. Ta nhn
,ng bc
Lo
a phuong tr`nh h. s di xu
o
. thy dy l`
. cha
9z6 18z5 73z4 + 164z3 73z2 18z + 9 =
 






1
1
1
3
3
2
= z 9 z + 3 18 z + 2 73 z +
+ 164 =
z
z
z
= z3 (9(3 3) 18(2 2) 73 + 164) =
= z3 (93 182 100 + 200).
,
,
,
,
,,
cho, ta ch c`
V` z = 0 khng phai l`
a nghim
on giai
a
. cua phuong tr`nh d
,,
,i :
phuong tr`nh bc
i vo
. ba d
93 182 100 + 200 = 0.
,
,
,
,,
,
p
V tr
ai cua phuong tr`nh c
o th phn tch th`
anh c
ac th`ua s (c
o th a
dung
d
inh
. l Bezout)
.
( 2)(92 100) = 0.
,,
Ta t`m d
uo. c ba nghim
.
= 2,

10
,
3

10
.
3

,
,,
,
,,
`ng c
Nhu vy,
ach giai ba phuong
`u ba
. ta t`m nghim
. phuong tr`nh ban d
tr`nh:
1
= 2,
z
,
,,
Giai ra ta d
uo. c 6 nghim:
.
z+

z1 = z2 = 1,

z+

z3 = 3,

1
10
=
,
z
3

z4 =

1
,
3

z+

1
10
= .
z
3

z5 = 3,

1
z6 = .
3

,
,,
V du. 2.87. Giai phuong tr`nh
z7 + 2z6 5z5 13z4 13z3 5z2 + 2z + 1 = 0.

,,
,c d
,ng hai bin
Chuong 2. a thu
i xu

108

,
,
,,
`ng c
`,i giai. y l`
,ng bc
a phuong tr`nh h. s di xu
Lo
ach chia v tr
ai
. le. Ba
,
, `
3
cho z + 1 (c
o th d`
ung so d Horner )
P(z) = (z + 1)(z6 + z5 6z4 7z3 6z2 + z + 1).
,
,,
,,
Phuong tr`nh ban d`u c
o mt
a ta giai phuong tr`nh
. nghim
. z = 1 v`
Q(z) = z6 + z5 6z4 7z3 6z2 + z + 1 = 0.
Ta c
o
Q(z) = z3



z3 +

1
z3






1
1
+ z2 + 2 6 z +
7 =
z
z

= z3 ((3 3) + (2 2) 6 7) = z3 (3 + 2 9 9).
,
,
,
,
,,
V` z = 0 khng phai l`
a nghim
on giai
. cua phuong tr`nh ban d`u. Ta ch c`
,,
phuong tr`nh
3 + 2 9 9 = 0.
,
,,
Phuong tr`nh n`
ay c
o th vit
3 + 2 9 9 = 2 ( + 1) 9( + 1) = ( + 1)(2 9) = 0.
,
nghim
Ta c
o ba nghim
a
o
. 1 = 1, 2 = 3, 3 = 3. Khi d
. cua Q(z) l`
,
,,
nghim
. cua ba phuong tr`nh
z+

1
= 1,
z

z+

1
= 3,
z

z+

1
= 3.
z

,
,
,
,,
,,
Giai phuong tr`nh trn cho ta nghim
a tt ca s nghim
. v`
. cua phuong tr`nh
ban d
a
`u l`

3 5
3 5
z1 = 1, z2,3 =
, z4,5 =
.
2
2
,
,,
V du. 2.88. Giai phuong tr`nh

P(z) = 12z5 56z4 + 107z3 107z2 + 56z 12 = 0.


3 William

,,
George Horner (1786-1837): Nh`
a to
an hoc
. ngu`oi Anh.

,,
,ng
2.9. Chuyn d
` v` phuong tr`nh h. s di xu

109

,
,
,,
`,i giai. y khng phai l`
,ng, nhu,ng ta nhn
Lo
a phuong tr`nh h. s di xu
.
,
,
ng v`
thy c
ac h. s g`n d
a c
o nghim
o th phn tch ra
i xu
. z = 1. Ta c
,
th`ua s
P(z) = (z 1)(12z4 44z3 + 63z2 44z + 12).
,
,,
,,
Phuong tr`nh ban d`u c
o mt
a z = 1 v`
a ta giai phuong tr`nh
. nghim
. l`
Q(z) = 12z4 44z3 + 63z2 44z + 12 = 0.
,
,
,
,,
,,
,ng, ta c
Nhung phuong tr`nh n`
ay l`
a phuong tr`nh h. s d
o th bin di
i xu




 
1
1
2
2
+ 63 =
Q(z) = z 12 z + 2 44 z +
z
z
= z2 (12(2 2) 44 + 63) = z2 (122 44 + 39).
,
,
,
,
,
z = 0 khng phai l`
a nghim
on phai giai
. cua Q(z) = 0. Ta ch c`
122 44 + 39 = 0.
,
13
3
,,
,,
a 2 =
. Giai hai phuong tr`nh
Phuong tr`nh n`
ay c
o hai nghim
. 1 = 2 v`
6
1
3
1
13
z+ = ; z+ =
z
2
z
6
,
,,
, ,

ta duo. c 2 nghim.
K
t
h
o
p
v
o
i
nghi
m
ban
d
`u z = 1, suy ra tng s
.
.
.
,
,,
cho l`
nghim
a:
. cua phuong tr`nh da
3
2
z1 = 1, z2 = , z3 = .
3
2

` tp
Bai
.
, ,
,
(Mt
a tra l`oi b`
ai tp
. s go. i y v`
. sau tai
. trang 129).
,
,,
. 2.89. Giai phuong tr`nh: 10z6 + z5 47z4 47z3 + z2 + 10z = 0.
,
,
,,
`ng tt ca nhu,ng nghim
,ng minh ra
. 2.90. Chu
. cua phuong tr`nh h. s di
,ng bc
xu
. bn
az4 + bz3 + cz2 + bz + a = 0,

(a 6= 0)
,
,,
,
,c.
c
o th tnh duo. c nh`o bn phep tnh s hoc
a c
an thu
. v`

,,
,c d
,ng hai bin
Chuong 2. a thu
i xu

110

,
,
,,
`ng tt ca nhu,ng nghim
,ng minh ra
. 2.91. Chu
. cua phuong tr`nh h. s di
,ng bc
xu
am
. n
az5 + bz4 + cz3 + cz2 + bz + a = 0,

(a 6= 0)

,
,,
,
,c.
c
o th tnh duo. c nh`o bn phep tnh s hoc
a c
an thu
. v`

,
,,
`ng nu bit mt
,ng minh ra
. 2.92. Chu
. nghim
. cua phuong tr`nh h. s di
,
,, `
,ng bc
ng bn phep tnh s
xu
au, th` nhung nghim
on lai
. s
. c`
. t`m duo. c ba
,
c.
hoc
a c
an thu
. v`
,
,
,ng minh kh
. 2.93. H
ay d`
ung d
l Bezout d
ach chu
ac cua kt
inh
ua ra c
.
,
,
,c h. s di xu
,ng bc
lun:
a thu
. Nhung d
. le chia ht cho z + 1.

, ,
,,
,
` tra l`oi bai
` tp
2.10. Go. i y
va
. chuong 2
,
,,
. t 1 = x+ y, 2 = xy v`
cho th`
. 2.6. a
a bin di h. phuong tr`nh d
anh
a
1 = 5,
,,
,
h. phuong tr`nh
H. n`
ay c
o nghim
. 1 = 5, 2 = 6. T`u
2
1 32 = 7.
suy ra h. da
cho c
do
o nghim
.

x1 = 2, x2 = 3,
y1 = 3; y2 = 2.

1 = 5,
,,
H. phuong tr`nh
31 31 2 = 65.
,,
cho c
n`
ay c
o nghim
o nghim
. 1 = 5, 2 = 4. H. phuong tr`nh da
.


x1 = 1,
x2 = 4,
y1 = 4;
y2 = 1.
,,
,
. 2.7. H. phuong tr`nh tro. gi
up l`
a


41 = 32 ,
1 + 21 22 = 26.
,
,
,,
,,
2
, hai, ta nhn
Loai
. 2 t`u phuong tr`nh thu
. duo. c 31 51 78 = 0. Giai
,,
,
. 2.8. H. phuong tr`nh tro. gi
up l`
a

, ,
,,
,
`i b`
2.10. Go. i y v`
a tra lo
ai tp
. chuong 2

111

,,
,,
phuong tr`nh bc
uo. c
. hai trn ta t`m d


1 = 6,
1 = 13
3 ,
2 = 8;
2 = 52
9 .
,
,
,,
T`u dy cho nghim
`u
. cua h. phuong tr`nh ban d





x1 = 2,
x2 = 4,
x3 = 13+6377 ,
x4 = 136377 ,
y1 = 4;
y2 = 2;
y3 = 136 377 ;
y4 = 13+6 377 .

21 22 + 1 = 32,
,,
,
. 2.9. H. phuong tr`nh tro. gi
up l`
a
121 = 72 .
,
,
,
,,
,,
,
2
nht, ta nhn
Loai
. 2 t`u phuong tr`nh thu
. duo. c phuong tr`nh 71 171
,
,,
,,
224 = 0. Giai phuong tr`nh bc
. hai trn, ta t`m duo. c


1 = 7,
1 = 32
7 ,
2 = 384
2 = 12;
49 .
,
,
,,
T`u dy cho nghim
`u
. cua h. phuong tr`nh ban d





10
10
x3 = 16+8
x2 = 4,
x1 = 3,
x4 = 168
7 ,
7 ,
10
10
168
16+8
y2 = 3;
y1 = 4;
y3 =
;
y4 =
.
7
7

2 = 15,
,,
,
. 2.10. H. phuong tr`nh tro. gi
up l`
a
1 + 21 22 = 42.
,
,
,
,,
,,
,
hai, ta nhn
Loai
uo. c phuong tr`nh 21 +1 72 = 0.
. 2 t`u phuong tr`nh thu
. d
,
,,
,,
Giai phuong tr`nh bc
. hai trn, ta t`m duo. c


1 = 8,
1 = 9,
2 = 15;
2 = 15.
,
,
,,
T`u dy cho nghim
`u
. cua h. phuong tr`nh ban d





x1 = 3,
x2 = 5,
x3 = 9+221 ,
x3 = 9221 ,
y1 = 5;
y2 = 3;
y3 = 92 21 ;
y3 = 9+2 21 .
,
,
,,
s v`
,i c
. 2.11. Quy d
ad
ac n 1 , 2 ta nhn
`ng mu
ua v` h. vo
. duo. c:

31 31 2 = 182 ,
1 = 12.

112

,,
,c d
,ng hai bin
Chuong 2. a thu
i xu

,
,
,,
,
,,
T`u dy t`m duo. c 1 = 12, 2 = 32. T`u dy cho hai nghim
. cua h. phuong
tr`nh ban d
`u


x1 = 4,
x2 = 8,
y1 = 8;
y2 = 4.
,
,
,,
s v`
,i c
. 2.12. Quy d
ad
ac n 1 , 2 ta nhn
`ng mu
ua v` h. vo
. duo. c:

1 2 = 30,
61 = 52 .
,,
H. phuong tr`nh n`
ay c
o nghim:
.


1 = 5,
1 = 5,
2 = 6;
2 = 6.
,
,
,,
T`u dy cho bn nghim
`u
. cua h. phuong tr`nh ban d




x1 = 2,
x2 = 3,
x3 = 6,
x4 = 1,
y1 = 3;
y2 = 2;
y3 = 1;
y4 = 6.
,
,
,,
,i c
. 2.13. ua v` h. vo
ac n 1 , 2 ta nhn
. duo. c:

1 = a,
31 31 2 = b(21 22 ).
,
,,
,,
Bin di h. phuong tr`nh n`
ay, ta duo. c (3a 2b)2 = a3 a2 b. Nu s
a3 a2 b
,,
,,
3a 2b 6= 0, ta nhn
. duo. c nghim
. duy nht 2 = 3a 2b . Trong tru`ong
a3 + 2a2 b
,,
,
cho c
ho. p n`
ay h. phuong tr`nh da
o nghim
. (nu 4(3a 2b) 0)

s
s
3
2

a
a
+
2a
b
a
a3 + 2a2 b

x1 = +
x1 =
,
,

2
4(3a 2b)
2
4(3a 2b)
s
s

a
a3 + 2a2 b
a
a3 + 2a2 b

;
y1 =
.
y1 = 2

4(3a 2b)
2
4(3a 2b)
,
Nu 3a 2b = 0, nhung a3 a2 b 6= 0, th` h. khng c
o nghim.
ung
. Cui c`
,
3
2

i 1 = 0 v`
nu 3a 2b = 0 v`
a a a b = 0, ngha l`
a a = b = 0, th` vo
a
,
,`,
,
,

2 l`
a mt
a nghim
up. Suy ra trong truong ho. p n`
ay
. s bt k` l`
. cua h. tro. gi

, ,
,,
,
`i b`
2.10. Go. i y v`
a tra lo
ai tp
. chuong 2

113

,
,
,,
. p s x, y thoa m
moi
an x + y = 0 d`u l`
a nghim
. ca
. cua h. phuong tr`nh
ban d
`u.

21 22 + 21 = 23,
,,
,
. 2.14. H. phuong tr`nh tro. gi
up l`
a
21 2 = 19.
,
,,
,,
,,
, nht, ta nhn
Loai
. 2 t`u phuong tr`nh thu
. duo. c phuong tr`nh
,
,,
,
,
t`m duo. c hai h. nghim
21 21 15 = 0. T`u d
up:
o
. cua h. tro. gi


1 = 3,
1 = 5,
2 = 10;
2 = 6.
,
,
,
cho c
h. phuong tr`nh da
T`u d
o nghim
o
.




x1 = 5,
x2 = 2,
x3 = 2,
x4 = 3,
y1 = 2;
y2 = 5;
y3 = 3;
y4 = 2.
,,
,
. 2.15. H. phuong tr`nh tro. gi
up


1 2 = 20,
1 = 5,
c
o nghim
.
41 = 52
2 = 4;
,
,
,,
, nghim
T`u d
a
o
. cua h. phuong tr`nh ban d`u l`




x1 = 1,
x2 = 4,
x3 = 5+241 ,
y1 = 4;
y2 = 1;
y3 = 52 41 ;


1 = 5,
2 = 4.

x4 =
y4 =

5 41
2 ,
5+ 41
.
2

41 421 2 + 22 = 1153,
21 32 = 33.
,
,,
,,
,
, nht, ta nhn

Loai
uo. c 22 332 + 32 = 0. T`u d
o
. 1 t`u phuong tr`nh thu
. d
,
,,
,
t`m duo. c hai h. nghim
up:
. cua h. tro. gi



1 = 6,
1 = 129,
2 = 32.
2 = 1;
,
,,
h. phuong tr`nh d
cho c
T`u do
o nghim
a
.




x2 = 3 8,
x3 = 3 + 8,
x1 = 3 + 8,

y1 = 3 8;
y2 = 3 + 8;
y3 = 3 8;
,,
,
. 2.16. H. phuong tr`nh tro. gi
up l`
a

114

,,
,c d
,ng hai bin
Chuong 2. a thu
i xu

129
+
1
129 1

x
=
x
=
,
,
5
6
x4 = 3 8,
2
2

y4 = 3 + 8;

129

y5 =
y6 = 129 + 1 ;
;
2
2

129 + 1
x = 129 1 ,

,
8
x7 =
2
2

y8 = 129 + 1 .
y7 = 129 1 ;
2
2
x + z = 2,
,
,
,ng
. t y = z dua h. da
cho v` h. d
. 2.22. a
ua v`
i xu
x3 + z3 = 8.

1 = 2,
,
,

h. tro. gi
up
H. n`
ay c
o nghim
o
. 1 = 2, 2 = 0. T`u d
3
1 31 2 = 8.
,,
ta nhn
uo. c hai nghim
.
. d


x1 = 2,
x2 = 0,
z1 = 0;
z2 = 2.
,
Vy
a
`u l`
. nghim
. cua h. ban d


x1 = 2,
x2 = 0,
y1 = 0;
y2 = 2.
,
,,
,ng
. t x2 = z v`
cho th`
.
a bin di h. da
anh h. phuong tr`nh d
i xu
2.23. a
,
y + z = 5,
`ng phu,o,ng ph
,ng v`
Giai h. n`
ay ba
ap h. di xu
a cui c`
ung
3
3
y + z = 65.
,
ta c
o nghim
`u
. cua h. ban d




x1 = 1,
x2 = 1,
x3 = 2,
x4 = 2,
y1 = 4;
y2 = 4.
y3 = 1;
y4 = 1.
,
x
y
,,
. t
cho th`
. 2.24. a
= u, = v v`
a bin d
anh h. phuong tr`nh
i h. da
b
a
u + v = 1,
,
,,
,ng 1
,i 1 = u + v, 2 = uv, ta nhn
. t n mo
di xu
a
uo. c
1
. d
+ = 4.
u v

, ,
,,
,
`i b`
2.10. Go. i y v`
a tra lo
ai tp
115
. chuong 2

1 = 1,
1
,
h. 1
suy ra 1 = 1, 2 = . T`u dy cho ta nghim
. duy nht

4
= 4,
2
a
b
1
,,
h. phuong tr`nh ban d
o nghim
x = ,y = .
u = v = . Vy
`u c
.
.
2
2
2
,

,,
. t x = u, y = v v`
cho th`
. 2.25. a
a bin d
anh h. phuong
i h. da
,
u + v = 2uv,
,ng
,i 1 = u + v, 2 = uv, ta nhn
. t n mo
tr`nh di xu
a
.
2
2
u + v = 20.

,
1 = 22 ,
,,
duo. c h.
giai h. n`
ay cho nghim
.
21 22 = 20,


1 = 4,
1 = 5,
2 = 2;
2 = 52 .
,

,
V` 0 x = u, 0 y = v, nn phai c
o 2 = uv 0, v` th nghim
. thu
,
,
, nht cho hai nghim
,ng
hai khng thoa m
an. Nghim
. thu
. cua h. di xu



u2 = 2 6,
u1 = 2 + 6,

v1 = 2 6;
v2 = 2 + 6.
,
,
,
, hai khng phai l`
H. thu
a nghim,
v` n
o khng thoa m
an u 0, v 0. Ch
.
,
,
, nht v`
cho ta nghim
c`
on nghim
a t`u do
. thu
. cua h. ban d`u:


x = u21 = (2 + 6)2 = 10 + 4 6,

y = u22 = (2 6)2 = 10 4 6.
,
1
1
,,

cho th`
. 2.26. a
a bin d
anh h. phuong tr`nh
i h. d
a
. t x 4 = u, y 5 = v v`
,
u3 + v3 = 35,
,,
,ng
giai trong b`
di xu
y l`
a h. phuong tr`nh da
ai 2.7
u + v = 5.
v`
a c
o nghim
.


u1 = 2,
u2 = 3,
v1 = 3;
v2 = 2.
,
T`u dy suy ra nghim
.


x1 = 16,
x2 = 81,
y1 = 243;
y2 = 32.

,,
,c d
,ng hai bin
Chuong 2. a thu
i xu

116
r

r
1
1
1
1
u5 = + x, v5 = x v`
+ x = u, 5 x = v. Khi d
a
o
2
2
2
2

u + v = 1,
,,
,ng
H. n`
ay c
o nghim
ta nhn
uo. c h. di xu
.
. d
u5 + v5 = 1.


u1 = 1,
u2 = 0,
v1 = 0;
v2 = 1.
. t
. 2.27. a

,
1
,,
,
cua phuong tr`nh ban d`u
T`u u5 = + x cho ta hai nghim
.
2
1
1
x1 = , x2 = .
2
2

u2 + v2 = 1,
,,
,
ng
. t sin x = u, cos x = v ta c
. 2.28. a
o h. phuong tr`nh di xu
u3 + v3 = 1.

21 22 = 1,
,, 3
,
Ta c
o h. tro. gi
up
Loai
. 2 ta nhn
. duo. c 1 31 +2 = 0.
3
1 31 2 = 1.
,,
,,
,,
Phuong tr`nh n`
ay c
o nghim
d
ac
inh
uo. c c
. l Bzu tnh d
. 1 = 1. Su dung
.
,,
. p nghim
nghim
on lai.
uo. c hai ca
. c`
. Ta nhn
. d
.


1 = 2,
1 = 1,
3
2 = .;
2 = 0;
2
,
,i 1 = 1, 2 = 0 ta c
. p cui c`
. p
Ca
ung khng cho nghim
o ca
. (u, v) thu. c. Vo
nghim
.


u1 = 1,
u2 = 0,
v1 = 0;
v2 = 1.
,,
,,
Ta nhn
. duo. c hai h. phuong tr`nh


sin x = 1,
sin x = 0,
cos x = 0;
cos x = 1.
,,
phuong tr`nh da
cho c
Do d
o nghim
o
.
x=

+ 2k,
2

x = 2k

(k = 0, 1, 2, ...).

, ,
,,
,
`i b`
2.10. Go. i y v`
a tra lo
ai tp
. chuong 2

117

,ng sau
. t 4 629 x = u, 4 77 + x = v. Khi d
ta c
. 2.29. a
o h. d
o
i xu

,
u + v = 8,
,,
,,
Giai h. phuong tr`nh n`
ay ta nhn
. duo. c
4
4
u + v = 706.

u1 = 3,
v1 = 5;

u2 = 5,
v2 = 3.

,
,,
T`u 4 629 x = u ta nhn
uo. c hai nghim
. d
. x1 = 548, x2 = 4.

,ng sau
. t 3 8 + x = u, 3 8 x = v. Khi d
ta c
o
o h. d
. 2.30. a
i xu

,
u + v = 1,
1 = 1,
,
Bin di v` h. tro. gi
up
H. n`
ay c
o
u3 + v3 = 16.
31 31 2 = 16.

1 = 1,
,
,,
ta nhn
nghim
T`u do
uo. c hai nghim
.
. d
.
2 = 5.

1
1
21
21

u1 = 2 + 2 ,
u2 = 2 2 ,

v1 = 1 21 ;
v2 = 1 + 21 .
2
2
2
2

,
,
,
,,
,
T`u 3 8 + x = u ta nhn
cua phuong tr`nh ban d
uo. c nghim
`u
. d
.

x1 = 3 21, x2 = 3 21.

,ng
. t 1 x2 = y. Khi d
ta c
. 2.31. a
o h. di xu
o

x2 + y2 = 1,
1
35
1
+ =
.
x y
12

,
21 22 = 1,
,
Bin di v` h. tro. gi
up
H. n`
ay c
o nghim
.
121 = 352 .

1 = 5 ,

2 = 12 .
25

1 = 7 ,

2 = 12 .
49

118

,,
,c d
,ng hai bin
Chuong 2. a thu
i xu

,
,
,
,
T`u dy ta t`m ra bn nghim,
nhung ch c
o ba nghim
an y 0:
.
. thoa m

5
4
3
73

x3 =
x1 = ,
x2 = ,
,

5
5
14
14

3
4

y1 = ;
y2 = ;
y3 = 5 + 73 .
5
5
14
14
,
,,
Nhu vy
o nghim
`u c
. phuong tr`nh ban d
.

4
3
5
73
x1 = , x2 = , x3 =
.
5
5
14
14

,ng sau
. t 3 10 x = u, 3 3 x = v. Khi do
ta c
. 2.32. a
o h. d
i xu

,
u + v = 1,
1 = 1,
,
Bin di v` h. tro. gi
up
H. n`
ay c
o
3
3
u + v = 7.
31 31 2 = 7.

1 = 1,
,
,,
ta nhn
nghim
T`u d
o
.
. duo. c hai nghim
.
2 = 2.


u2 = 1,
u1 = 2,
v2 = 2.
v1 = 1;

,
,
,,
,,
T`u 3 10 x = u ta nhn
cua phuong tr`nh ban d
uo. c nghim
`u
. d
.
x1 = 2, x2 = 11.
19 x
,,
. t
. 2.33. a
= y, ngha l`
a 19 x = xy + y, ta nhn
uo. c h.
. d
x+1


,
x + y + xy = 19,
1 + 2 = 19,
,
Bin di v` h. tro. gi
up
H. n`
ay c
o
xy(x + y) = 84.
1 2 = 84.


1 = 7,
1 = 12,
h. cho hai ca
. p nghim
nghim
Mi
.
.
2 = 12;
2 = 7.





x1 = 3,
x2 = 4,
x3 = 6 + 29,
x4 = 6 29,

y1 = 4;
y2 = 3;
y3 = 6 29;
y4 = 6 + 29.
,,
Phuong tr`nh ban d`u c
o bn nghim
.

x1 = 3, x2 = 4, x3 = 6 + 29, x4 = 6 29.


x2 + y2 = 17,
,
2
ng
. t y = 17 x . Khi do
ta c
. 2.34. a
o h. d
i xu
x + y + xy = 9.

, ,
,,
,
`i b`
2.10. Go. i y v`
a tra lo
ai tp
119
. chuong 2

,
21 22 = 17,
,

`
Bin di v h. tro. gi
up
H. n`
ay c
o nghim
.
1 + 2 = 9.


1 = 5,
1 = 7,
2 = 4;
2 = 16.
,
nht cho nghim
, hai khng cho c
H. thu
ac nghim
. x1 = 1, x2 = 4. H. thu
.
,
thu. c x, y.


x3 + y3 = 35,
,
3
3

ng
. t y = 35 x . Khi do
ta c
o h. di xu
. 2.35. a
(x + y)xy = 30.
,
,,
,
,

t`m ra hai nghim


Giai h. phuong tr`nh voi h. tro. gi
up v`
a sau d
o
.
x1 = 2, x2 = 3.
. 2.41. Ta c
o
5x2 6xy + 5y2 = 5S2 62 = 5(21 22 ) 62 = 521 162 =
1
= 521 16. (21 z) = 21 + 4z 0.
4
. 2.42. Ta c
o
8(x4 + y4 ) (x + y)4 = 8S4 41 = 8(41 421 2 + 222 ) 41 =
1
1
= 741 3221 (21 z) + 16 (21 z)2 = 621 z + z2 0.
4
16
. 2.43. Ta c
o
x4 + y4 x3 y xy3 = x4 + y4 xy(x2 + y2 ) = S4 2 S2 =
= 41 421 2 + 222 2 (21 22 ) = 41 521 2 + 422 =
1
1
3
1
= 41 521 . (21 z) + 4. (21 z)2 = 21 z + z2 0.
4
16
4
4
. 2.44. Ta c
o
x2 + y2 + 1 xy x y = 21 32 1 + 1 =
1
= 21 3. (21 z) 1 + 1 =
4
1 2
3
1
3
= 1 1 + 1 + z = ( 1 1)2 + z 0.
4
4
2
4

,,
,c d
,ng hai bin
Chuong 2. a thu
i xu

120
. t
. 2.45. a

x = u,

,
,c c
ng thu
bt d
y = v. Khi do
o dang
a
.

(u + v)8 64u2 v2 (u2 + v2 )2 .


,
,
,ng minh
V` u, v l`
a nhung s khng m nn ta ch c`n chu
(u + v)4 8uv(u2 + v2 ).
Tht
. vy,
.
(u + v)4 8uv(u2 + v2 ) =
= 41 82 (21 22 ) = 41 821 2 + 1622 =
1
1
= 41 821 . (21 z) + 16 (21 z)2 = z2 0.
4
16
. 2.46. Ta c
o
x 3 + y3

x+y
2

3

1
1 3
( 31 2 ) 31 =
2 1
8
3
3
3
= 31 1 2 = 1 z 0.
8
2
8
=

. 2.47. Ta c
o
x4 + 2x3 y + 2xy3 + y4 6x2 y2 = S4 + 22 S2 622 =
= 41 421 2 + 222 + 22 (21 22 ) 622 = 41 221 2 822 =
= (z + 42 )2 2(z + 42 )2 822 = z2 + 62 z.
,
,
,c hin nhin du
ng thu
ng.
V` x, y 0, nn 2 0 v`
a z 0. Bt d
a
,
,
,
2
2
. 2.48. Bt phuong tr`nh d
a 21 22
ua v` dang
. x + y 2xy, ngha l`
,
,
,
,
,
ng minh 21 42 , nhung d
ng hin nhin
22 . T`u d
ay du
y phai chu
i`u n`
,c d
tnh trong v du.
theo cng thu
a
.
,,
. 2.54. Ta c
o 1 = x1 + x2 = 6, 2 = x1 x2 = 10. Phuong tr`nh mun lp
.
3
3

z2 + pz + q = 0 c
o c
ac nghim
z
=
x
,
z
=
x
.
V`

th
1
.
1 2
2
p = z1 + z2 = x31 + x32 = 31 31 2 = (6)3 3(6)10 = 36,
q = z1 z2 = x31 x32 = 32 = 1000.

, ,
,,
,
`i b`
2.10. Go. i y v`
a tra lo
ai tp
. chuong 2

121

,,
phuong tr`nh c`n lp
Do d
a z2 + 36z + 1000 = 0.
o
. l`
,,
. 2.55. Ta c
o 1 = x1 + x2 = 1, 2 = x1 x2 = 3. Phuong tr`nh mun lp
.
2
10
10

z + pz + q = 0 c
o c
ac nghim
. z1 = x1 , z2 = x2 . V` th
10
p = z1 + z2 = x10
1 + x2 =
8
6 2
4 3
2 4
5
= 10
1 101 2 + 351 2 501 2 + 251 2 22 = 4207,
10
10
q = z1 z2 = x10
1 x2 = 2 = 59409.

,,
phuong tr`nh c`n lp
Do d
a z2 4207z + 59049 = 0.
o
. l`
,
,,
ta c
. 2.56. Phuong tr`nh phai t`m c
o dang
x2 + px + q = 0. Khi do
o
.
,
5
5
cho x1 +x2 = 31, x1 +x2 = 1
p = x1 +x2 = 1 , q = x1 x2 = 2 . Gia thit d
a
,
v`
a bin d
ung v` dang
i ch
.


51 531 2 + 51 22 = 31,
p5 + 5p3 q 5pq2 = 31,
ngha l`
a
1 = 1.
p = 1.
,
,,
,,
,,
2
ta nhn
T`u do
a phuong tr`nh n`
ay
. duo. c phuong tr`nh 5q 5q 30 = 0 v`
,
,
,,
c
o nghim
o hai phuong tr`nh thoa m
an di`u
. q1 = 3, q2 = 2. Nhu vy
. ta c
kin
ai.
. d`u b`
x2 x + 3 = 0,

x2 x 2 = 0.

,
. 2.57. Ta c
o 1 = x1 + x2 = a 2, 2 = x1 x2 = (a + 1). Tng b`nh
,
,,
phuong cua c
ac nghim
o dang
. c
.
x21 + x22 = S2 = 21 22 = (a 2)2 + 2(a + 1) =
= a2 2a + 6 = (a 1)2 + 5.
,
,
,
,
`ng tng b`nh phu,o,ng cua c
T`u dy thy ra
ac nghim
o gi
a tri. nho nht khi
. c
a = 1.
,
,
`ng xn + xn l`
,i moi
. 2.58. Ta ch ra ra
. s nguyn vo
. s tu. nhin n.
1
2 a mt
Tht
. vy,
. v`
1 = x1 + x2 = 6, 2 = x1 x2 = 1,

122

,,
,c d
,ng hai bin
Chuong 2. a thu
i xu

,c (2.1) c
th` cng thu
o dang
a Sn1 = 6Sn2 Sn3 .
. Sn = 6Sn1 Sn2 v`
,,
Ta tnh d
uo. c Sn :
Sn = 35Sn2 6Sn3 = Sn3 + 5(7Sn2 Sn3 ).
,
`ng Sn v`
. c khng chia ht
Ta thy ra
a Sn3 d`ng th`oi chia ht cho 5 hoa
,
cho 5. Nhu vy
ac s Sn3 , Sn6 , Sn9 , ... d`u
. nu Sn chia ht cho 5 th` c
,
. c S3
chia ht cho 5. Cui c`
ung cua d
ay trn l`
a mt
. trong ba s S1 , S2 hoa
,
,
,

chia ht cho 5. Nhung ta d tnh duo. c S1 = 6, S2 = 34, S3 = 198, ngha l`


a
,
,
n
n
i moi
khng mt
ao chia ht cho 5, v l. Suy ra tng Sn = x1 + x2 vo
. s n`
.
,
s tu. nhin n khng chia ht cho 5.

. t 4 = u, 4 = v. Khi do
theo di`u kin
cho
. 2.59. a
a
. d
p = + = u4 + v4 , q = = u4 v4 ,
,
,

. t 1 = u + v, 2 = uv, ta bin di mi
v`
a ta phai tnh 4 + 4 = u + v. a
quan h. n`
ay v` dang
.
p = 41 421 2 + 222 ,

q = 42 .

,,
,,
,,
Loai
ung phuong
. 2 ta t`m duo. c phuong tr`nh tr`

41 4 4 q21 + 2 q + p = 0.
,,
,,
Phuong tr`nh tr`
ung phuong n`
ay t nht c
o hai nghim
a
. l`
r
q

24q+ 2 qp

(do 2 q p 0) v`
a nhi`u nht c
o bn nghim
a
. l`
r
q

24q 2 qp
p

,,
(khi 2 4 q 2 q p 0). V` 1 l`
a mt
. s duong, nn trong bn nghim
.
,
,
,,
,`,
,

l`
trn ch c
o mt
an do
a truong ho. p c
ac du cng.
Tht
. s duong thoa m
.
.

,
2

vy,
a p 4q 0. V` th |p| 2 q
. t`u > 0, > 0 suy ra p < 0, q > 0 v`

, ,
,,
,
`i b`
2.10. Go. i y v`
a tra lo
ai tp
. chuong 2

123

p
p

,
ngha l`
a p 2 q v`
a 2 q p 4 q = 2 4 q. Nhu vy
. trong du
,
c
an phai ly du (+):
r
q
p

4
4
+ = 2 4 q + 2 q p.
. 2.64. Ta c
o
2x4 + 7x3 y + 9x2 y2 + 7xy3 + 2y4 =
= 2S4 + 72 S2 + 922 = 2(41 421 2 + 222 ) + 72 (21 22 ) + 922 =
= 241 21 2 22 = (2 + 221 )(21 2 ) =
= (xy + 2(x + y)2 )((x + y)2 xy) =
= (2x2 + 5xy + 2y2 )(x2 + xy + y2 ) =
= (x + 2y)(2x + y)(x2 + xy + y2 ).
,
,,
,
(Th`ua s cui c`
ung khng phn tch d
uo. c trn tp
. s thu. c).
. 2.65. Ta c
o
18x4 21x3 y 94x2 y2 21xy3 + 18y4 =
= 18S4 212 S2 9422 =
= 18(41 421 2 + 222 ) 212 (21 22 ) 9422 =
= 1841 9321 2 1622 = (321 162 )(621 + 2 ) =
= (3(x + y)2 16xy)(6(x + y)2 + xy) =
= (3x2 10xy + 3y2 )(6x2 + 13xy + 6y2 ) =
= (x 3y)(3x y)(2x + 3y)(3x + 2y).
. 2.66. Ta c
o
3x4 8x3 y + 14x2 y2 8xy3 + 3y4 = 3S4 82 S2 + 1422 =
= 3(41 421 2 + 222 ) 82 (21 22 ) + 1422 = 341 2021 2 + 3622 .
,,
,
,c bc
,i 2 khng c
Ta nhn
o nghim
. duo. c tam thu
. hai di vo
. thu. c. V` th ta
,
,,
, hai d phn tch ra th`u,a s (phu,o,ng ph
p dung
a
phuong ph
ap thu
ap h. s
.

,,
,c d
,ng hai bin
Chuong 2. a thu
i xu

124
. t
bt dinh).
a
.

3x4 8x3 y + 14x2 y2 8xy3 + 3y4 =


= (Ax2 + Bxy + Cy2 )(Cx2 + Bxy + Ay2 )
,,
2
a A+B+C =
Cho gi
a tri. x = y = 1, ta nhn
. duo. c (A + B + C) = 4, ngha l`
,
`ng A + B + C = 2.
2. Theo l lun
o th cho ra
. trong ph`n v du. ta c

,,
,i x = 1, y = 1 ta
Cho x = 0, y = 1 ta t`m duo. c A.C = 3. Cui c`
ung vo
,,
,
nhn
a A + B + C = 6. Nhu vy
o h.
uo. c (A B + C)2 = 36 ngha l`
. d
. ta c
,,
phuong tr`nh

A + B + C = 2,
A B + C = 6,

A.C = 3.

,
,,
, hai ta ly du (+), th` t`u, phu,o,ng
Nu trong v phai phuong tr`nh thu
,,
, nht B = 2. T`u, d
. c l`
d tnh d
tr`nh thu
aC =
o
uo. c A = 1, C = 3 (hoa
,
,
,
1, A = 3). Cui c`
ung ta nhn
uo. c su. phn tch
. d
3x4 8x3 y + 14x2 y2 8xy3 + 3y4 = (x2 2xy + 3y2 )(3x2 2xy + y2 ).
. 2.67. Ta c
o
2x4 x3 y + 3x2 y2 xy3 + 2y4 = 2S4 2 S2 + 322 =
= 2(41 421 2 + 222 ) 2 (21 22 ) + 322 =
= 241 921 2 + 922 = (21 32 )(221 32 ) =
= ((x + y)2 3xy)(2(x + y)2 3xy) =
= (x2 xy + y2 )(2x2 + xy + 2y2 ).
,
,,
,
,c sau c`
Nhung tam thu
ung khng phn tch duo. c trong tp
ac s thu. c.
. c

, ,
,,
,
`i b`
2.10. Go. i y v`
a tra lo
ai tp
. chuong 2

125

. 2.68. Ta c
o
(x + y)5 x5 y5 = 51 S5 = 51 51 + 531 2 51 22 =
= 51 2 (21 2 ) =
= 5xy(x + y)(x2 + xy + y2 ).
,
,c da
cho
. 2.74. Ta bin d
i theo cng thu
(x + y)7 x7 y7
7 S7
75 2 1431 22 + 71 32
= 51
= 1
=
5
5
5
(x + y) x y
1 S5
531 2 51 22
71 2 (41 221 2 + 22 )
71 2 (21 2 )2
7
=
= (21 2 ) =
5
51 2 (21 2 )
51 2 (21 2 )
7
7
= ((x + y)2 xy) = (x2 + xy + y2 ).
5
5

,
,c da
cho
. 2.75. Ta bin d
i theo cng thu




1
1
1
1
2
1
+
+
+
=
(a + b)2 a2 b2
(a + b)3 a b
=

2 1
2
2 22
1 S2
+ 3
= 1 2 2 + 2 =
2
2
1 2
1 2
1 2
1 2

21 22 + 22
21
1
1
=
= 2 = 2 2.
2
2
2
2
a b
1 2
1 2
2
,
,
c da
cho
. 2.76. Ta bin di theo cng thu
=

(x + y)4 + x4 + y4 = 41 + S4 = 41 + 41 421 2 + 222 =


= 241 421 2 + 222 = 2(41 221 2 + 22 ) =
= 2(21 2 )2 = 2(x2 + xy + y2 )2 .
,
,c da
cho
. 2.77. Ta bin di theo cng thu
(x + y)7 x7 y7 = 71 S7 = 71 (71 751 2 + 1431 22 71 32 ) =
= 751 2 1431 22 + 71 32 = 71 2 (41 221 2 + 22 ) =
= 71 2 (21 2 )2 = 7xy(x + y)(x2 + xy + y2 )2 .

126

,,
,c d
,ng hai bin
Chuong 2. a thu
i xu

,
,
,i 1 = 2 = 1. Theo cng
. 2.78. Ta k hiu
a gi
a tri. cua tng Sn vo
. an l`
,,
,c (2.1) vo
,i gi
thu
a tri. 1 = 2 = 1, ta nhn
uo. c
. d
an = an1 an2 v`
a an1 = an2 an3 .
,
,
,c trn cho kt qua an = an3 . Tu,o,ng tu.,,
ng thu
Cng
theo v hai da
.
,,
an3 = an6 . V` th ta nhn
. duo. c
an = an6

(n 6).

,
2
2

,c (x + y)n xn yn = n
,i
d
a thu
1 Sn chia ht cho x + xy + y , vo
,
,
,
,c (theo
1 = 2 = 1 th` di`u kin
a du l`
a tng c
ac h. s cua d
a thu
. c`n v`
,
`ng khng (xem v du. 2.72), ngha l`
1 , 2 ) phai ba
a 1 an = 0, hay n
oi
c
ach kh
ac an = 1.
V` an = an6 = an12 = ... v`
a 1 = 2 = 1 ta c
o:
a6k+1 = a1 = S1 = 1,
a6k+2 = a2 = S2 = 1,
a6k+3 = a3 = S3 = 2,
a6k+4 = a4 = a1 = 1,
a6k+5 = a2 = 1,
a6k = a6 = a3 = 2.
,
,
. c
Nhu vy,
o khi v`
a ch khi nu n c
o dang
. quan h. trn, an = 1 c
. 6k + 1 hoa
l`
a 6k + 5 (ngha l`
a 6k 1).
2
,c P(x) bc
. 2.79. Cho da thu
a
. m chia ht cho x + x + 1, ngha l`
,
,
,
,
2
c Q(x) l`
c bc

P(x) = (x + x + 1)Q(x), o dy da thu


a da thu
o
. m 2. Khi d
   2
  
x
x
x
x
P
=
+ +1 Q
.
2
y
y
y
y
,,
,i ym , nhn
Nhn hai v vo
. duo. c
 
 
x
x
2
2 m2
m
= (x + xy + y )y
Q
y P
y
y

, ,
,,
,
`i b`
2.10. Go. i y v`
a tra lo
ai tp
127
. chuong 2
 
 
,
,
,c ym P yx v`
,a y o, mu
Biu thu
a ym2 Q yx khng chu
s, ngha l`
a hai da
 
 
,,
x
x
m
2
2
m

,c. Nhu, vy
thu
y
P
chia
h
t
cho
x
+
xy
+
y
.
Ngu
o
c
l
ai
n
u
y
P
.
. .
y
y
,
,,
,i y = 1 ta nhn
vo
chia ht cho x2 + xy + y2 , th` t`u d
o
. duo. c P(x) chia ht
,
2
,c P(x) bc
cho x2 + x + 1. Nhu vy,
a
. da thu
. m chia ht cho x + x + 1 khi v`
,
,
x
m
2
2
c y P y chia ht cho x + xy + y .
ch khi da thu
,
,
,
,c x2n + xn yn + y2n chia ht
Nhu vy
a tri. n d
da thu
. ta c`n phai t`m gi
,
cho x2 + xy + y2 . Nhung
x2n + xn yn + y2n = S2n + n
2,
,
,
,
,c n`
v`
a d da thu
ay chia ht cho x2 + xy + y2 th` d
ad
a tng
i`u kin
u l`
. c`n v`
,
`ng 0, vo
,c n`
,i 1 = 2 = 1, ngha l`
c
ac h. s cua da thu
ay (theo 1 , 2 ) ba
a
, ,
,
a2n + 1 = 0, hay l`
a a2n = 1. Theo kt qua cua b`
ai 2.78 suy ra phai c
o
,
. c l`
. c
2n = 6k + 2 hoa
a 2n = 6k + 4. N
oi c
ach kh
ac, phai c
o n = 3k + 1 hoa
,
,
2n
n
2
c x + x + 1 chia ht cho x + x + 1 khi v`
n = 3k + 2. a thu
a ch khi n
,
,
khng phai bi
. s cua 3.
,
,
,
,
,c (x + y)n + xn + yn chia
. 2.80. Ta phai t`m nhung gi
a tri. cua n d
d
a thu
,
,
,
ng dinh
ht cho x2 + xy + y2 (xem b`
ai 2.79). T`u kt qua b`
ai s 2.78, kha
.
,
, , ,
cua b`
ai to
an c
o duo. c voi n = 6k 2, n = 6k 4.
,
, ,
,
,c co, so, cua x, y l`
. 2.81. Ta k hiu
a 1 , 2 , cua u, v l`
a 1 , 2 :
a thu
. nhung d
1 = x + y,

2 = xy,

1 = u + v, 2 = uv.

Theo d
ai ra
i`u kin
. b`
1 = 1 v`
a 21 22 = 21 22 .
,
,,
,
,c bt k`
,i da thu
vo
T`u dy ta nhn
uo. c 1 = 1 , 2 = 2 . Nhung khi d
o
. d
(z1 , z2 ), ta c
o (1 , 2 ) = (1 , 2 ). Theo d
a mt
inh
. l 2.1, nu P(x, y) l`
.
,
,
,
,

c d
ng bt k` v`
da thu
a P(x, y) = (1 , 2 ) l`
a biu din cua n
o theo c
ac
i xu
,,
,
,
,

c d
ng co so, th`
da thu
i xu
P(x, y) = (1 , 2 ) = (1 , 2 ) = P(u, v).

,,
,c d
,ng hai bin
Chuong 2. a thu
i xu

128
,,
,
,i moi
Tru`ong ho. p ring vo
. n

xn + yn = un + vn .
,
,,
,,
cho c
. 2.82. Phuong tr`nh d
o th vit duo
i dang
a
.
1 = 21 32 .
,
,,
,
Boi v` x, y phai l`
a s thu. c, nn 42 21 v`
a v` th
21 1 = 32

3 2
,
4 1

ngha l`
a
1 2
1 0 hay 1 (1 4) 0.
4 1
,
,,
`ng 32 = 2 1 , ta nhn
`
, ra
Tu d
y suy ra 0 1 4. Nn nho
uo. c
. d
1
,
,
nhung kha n
ang sau dy
1 = 0,

2 = 0;

1 = 1,

2 = 0;

2
1 = 2, 2 = ; 1 = 3, 2 = 2; 1 = 4, 2 = 4.
3
,
,
,,
,
,
, ba bi. loai
Tru`ong ho. p thu
a 1 , 2 phai
. tr`u do x, y phai nguyn, ngha l`
,
,,
,,
,
nguyn. Nhung tru`ong ho. p c`
on lai
ac h. phuong tr`nh sau:
. cho ta c




x + y = 0,
x + y = 1,
x + y = 3,
x + y = 4,
xy = 0;
xy = 0;
xy = 2;
xy = 4.
,
,,
. p nghim
Giai c
ac h. phuong tr`nh cho ta c
ac ca
.






x1 = 0,
x2 = 1,
x3 = 0,
x4 = 2,
x5 = 1,
x6 = 2,
y1 = 0;
y2 = 0;
y3 = 1;
y4 = 1;
y5 = 2;
y6 = 2.
,
,
, ,
,,
`ng c
. p trn thu. c su. l`
ac ca
a nghim
Ba
ach kim tra lai,
. ta thy c
. cua phuong
cho.
tr`nh da
,
`ng da thu
,ng minh ra
,c di xu
,ng
. 2.83. Ta phai chu
(a + b)n an bn 3(ab)

n1
2

(a + b)

, ,
,,
,
`i b`
2.10. Go. i y v`
a tra lo
ai tp
. chuong 2

129

,
`ng 0 (xem v
,c (theo 1 , 2 ) c
,i 1 = 2 = 1 l`
vo
ad
o tng c
ac h. s ba
a thu
,
giai), ngha l`
n = 6k + 3 (b`
du. d
a 1 an 3 = 0 hay an = 2. Do d
ai
a
o
,
,
,
ng minh.
tp
i`u c`n chu
. 2.78). T`u gia thit suy ra d
. 2.89. Ta c
o
10z6 + z5 47z4 47z3 + z2 + 10z =
= z(10z5 + z4 47z3 47z2 + z + 10).
,
, ,
,,
,c trong ngoa
,c bc
i h. s tu., do kh
. c o v phai l`
a thu
a da thu
ac khng,
. le vo
,

,c (theo
nn theo dinh
o chia ht cho z + 1. Ta thu. c hin
. l 2.4 n
. chia da thu
,,
,
so d` Horner) v`
ad
uo. c:
10z6 + z5 47z4 47z3 + z2 + 10z =
= z(z + 1)(10z4 9z3 38z2 9z + 10) =




 
1
1
3
2
38 =
= z (z + 1) 10 z + 2 9 z +
z
z
= z3 (z + 1)(10(2 2) 9 38) =
= z3 (z + 1)(102 9 58).
,
`ng khng ta t`m du,o.,c hai nghim
,c sau c`
ng thu
Cho d
ung ba
a
. z1 = z2 =
,,
z3 = 0, z4 = 1 v`
a phuong tr`nh bc
. hai
102 9 58 = 0.
,
,,
Nghim
ay l`
a = 2, =
. cua phuong tr`nh n`

29
10 .

,
dn phu,o,ng
T`u dy dn

tr`nh
1
1
29
= 2, z + =
.
z
z
10
,
,,
,,
,i hai nghim
nhn
, ta nhn
C`
ung vo
cd
a
o
uo. c tt ca 6 nghim
. d
. truo
. d
.
,,
cho phuong tr`nh ban d`u:
z+

z1 = z2 = z3 = 0, z4 = z5 = z6 = 1, z7 =

5
2
, z8 = .
2
5

,,
,c d
,ng hai bin
Chuong 2. a thu
i xu

130
. 2.90. Ta c
o

 




1
1
+c =
az4 + bz3 + cz2 + bz + a = z2 a z2 + 2 + b z +
z
z
= z2 (a(2 2) + b + c) = z2 (a2 + b + (c 2a)).
,
,
,,
V` a 6= 0 nn z khng l`
a nghim
a ta giai
`u v`
. cua phuong tr`nh ban d
,,
,i :
phuong tr`nh di vo
a2 + b + (c 2a) = 0.
,
,
,,
bit thoa m
Nghim
an bn phep tnh s hoc
a c
an
a
. cua phuong tr`nh d
. v`
,
,
,
,
,
,
,
,
,
`ng hai phuong tr`nh
c. Nghim
thu
`u duo. c giai ba
. cua phuong tr`nh ban d
z+

1
= ,
z

,
,,
, nht hoa
, hai cua phu,o,ng tr`nh trn.
. c nghim
od
a nghim
y l`
. thu
. thu
,
,
,,
,,
,
Nghim
ay c
ung t`m duo. c nh`o bn phep tnh v`
a
. cua nhung phuong tr`nh n`
,
c.
c
an thu
,
,,
,,
. 2.91. V tr
ai cua phuong tr`nh chia ht cho z + 1 (dinh
a thuong
. l 2.4) v`
,
,c h. s d
,ng bc
,i h. s cua s hang
,n
l`
a mt
c
o bc
i xu
. da thu
. bn (vo
.
. lo
,,
,c n`
p dung
nht l`
a a). V` th b`
ai tp
cho da thu
ay v`
a nghim
. 2.90 duo. c a
.
.
,
,
,,
,
,

c.
cho biu din boi bn phep tnh v`
phuong tr`nh da
a c
an thu
,
,ng c
,i = z + 1z , phu,o,ng tr`nh h. s d
o s hang
. 2.92. Ta dua bin mo
i xu
.
,
,,
, `
,,
,

i . Nu z1 l`
tu. do kh
ac khng d
a
uo. c d
ua v phuong tr`nh bc
. ba di vo
,
,,
1

`
bit cua phuong tr`nh ban d
mt
a
u (z1 6= 0), th` = z1 + z1 l`
. nghim
. da
,
,,
,,
,
,

nghim
dinh
ua
. l Bezout ta d
. cua phuong tr`nh bc
. ba di voi . Su dung
.
,,
,,
,

phuong tr`nh bc
a n
o c
o nghim
. ba di voi v phuong tr`nh bc
. hai m`
.
,
,
,

biu din nhu bn phep tnh s hoc


a c
an thuc.
. v`
,
,
,
,
,
c h. s d
ng bc
,i s hang
. 2.93. a thu
tu. do a0 6= 0) c
o th
i xu
. le (vo
.
,,
vit duo
i dang
.
P(z) = a0 (z2k+1 + 1) + a1 (z2k + z) + a2 (z2k1 + z2 ) +

, ,
,,
,
`i b`
2.10. Go. i y v`
a tra lo
ai tp
. chuong 2

131

,
,c tai
Tnh gi
a tri. cua d
a thu
. z = 1:
P(1) = a0 (1 + 1) + a1 (1 1) + a2 (1 + 1) + = 0.

,
,c d
cho chia ht cho z + 1.
Nhu vy
inh
a thu
a
. l Bezout d
. P(1) = 0. Theo d

, ,
CHUONG

,
,
,

c cua cac
da
y s
ng thu
Bt da
, ,
tu.
d
`ng thu

, ,
. p th
. c ba s . . . . . . . . . . . . . . . . . . . . . . . . . . . . . . . . . . . . . . . . . . . . .
u tu. hai hoa
3.1. Ca
,
hoa
..........................................................
3.2. Tng quat
,,
, ,
d
3.3. Su dung
cac
ay s d
u tu. . . . . . . . . . . . . . . . . . . . . . . . . . . . . . . . . . . . . . . .
`ng th
.
,
,
ng th
3.4. Chuyn d
uc Karamata . . . . . . . . . . . . . . . . . . . . . . . . . . . . . .
` v` bt da

,
,c Karamata . . . . . . . . . . . . . . .
ng thu
3.4.1. H`
am l`i v`
a bt d
a
,
,,
,
c Karamata . . . . . . . . . . . . . . . . .
ng thu
3.4.2. Su dung
bt da
.
,
,
c kh
ng thu
ac . . . . . . . . . . . . . . . . . . . . . . . .
3.4.3. Mt
. s bt da
,
,
,c trong tam gi
ng thu
3.4.4. Nhung bt da
ac . . . . . . . . . . . . . .

, ,
,,
,
` tra l`oi bai
` tp
3.5. Go. i y va
. chuong 3 . . . . . . . . . . . . . . . . . . . . . . . . . . . . . . . . . . . . . .

134
143
149
156

156
163
166
168
171

,,
,,
,,
,,
su dung
c, ta da
rt nhi`u dai
Trong hai chuong truo
.
. luo. ng bt bin
,
,
,,
,
,p b`
. c
ra v`
ta giai duo. c nhung lo
trong b`
ai to
an d
a du. a v`
ao d
ai to
an da
a
o
,
,
,
,
`

c m`
ng minh mt
ng thu

a trong do
trung. Chuyn d
ay ta chu
n`
a
. s bt d
,
,
,

`
o nhiu b`
c
ac bin tham gia c
o vai tr`
o nhu nhau. C
ai to
an c
o nhung biu
,
,
,
,
,
,,
,
,

c khng di d
i su. chuyn d
thu
ac bin cho nhau. Trong tru`ong ho. p
i vo
i c
,
,,
,,
,
p xp theo mt
, tu., n`

nhu vy,
ao do
. ngu`oi ta thu`ong cho nhung bin sa
. thu
,
,
,
,
,,
,

i kt lun
m`
a khng anh huong g` to
ai to
an. Vic
ac b`
ai to
an bt
. cua b`
. giai c
,
,
,
,
,
,
,

c c
tu. n`
ng thu
p xp theo mt
l`
da
o nhung bin s duo. c sa
ao do
a rt
. thu

133
, ,
,,
thun
a c
o th tng qu
ay, ta nghin
at h
oa duo. c. Trong chuyn d
` n`
. tin
. v`
,
,
,
,
,,
,
u vic
ng minh bt d
c vo
i k thut
ng thu
p
cu
ac b. s d
a
uo. c sa
. chu
. tao
. ra c
,
,
,
,
,
,
,
tu. v`
ng minh mt
c ni
ng thu
nh`o k thut
thu
a sau do
ay chu
. n`
. s bt da
,
ting nhu : Cauchy, Chebyshev, ...
,
,,
,,
,,
ao
Ta ly mt
o tnh cht thu. c t: Mu`oi ngu`oi xp h`
ang ly nuoc v`
. v du. c
,
,,
,

`
`
`i hoi th`oi gian ly d
c
ac th`
ung. Mi th`
ung d
ac nhau. Cn sa
p xp
o
y nuoc kh
, , ,
,
,,
,
,
,
,
,
,
,
,
,
a
thu tu. h`
ang d
ao d
o. i nhu th n`
t ho. p th`oi gian ch`o d
o. i cua mu`oi ngu`oi l`
,

nho nht?
,
,
,
,
, tu., t
Mt
ac cho ta l`oi khuyn l`
a xp theo thu
ang cua th`oi gian
. tru. c gi
,
,
,
,,
,,
l`
am d`y nuo
c c
ac th`
ung nuo
c. kim nghim
ac n`
ay ta xem xet
. cam gi
,
,,
,
nhu sau: K hiu
ac th`oi gian c`n thit l`
am d`y nuo
c
. T1 < T2 < ... < T10 c
c
ac th`
ung.
, ,
,,
,,
,,
, ,
,
p nhu l`oi khuyn trn th` t ho. p th`oi gian
Nu ngu`oi ly nuo
c duo. c sa
,
, ,
,
, tu., h`
,i nhu,ng thu
ch`o d
a T = 10T1 + 9T2 + + T10 . Vo
ang do. i
o. i phai cho l`
, , ,
,
,
, ,
,
kh
ac, t ho. p cua th`oi gian ch`o do. i se l`
a S = 10S1 + 9S2 + + S10 , o d
y
,
(S1 , S2 , ..., S10 ) l`
a mt
an vi. cua (T1 , T2 , ..., T10 ).
. ho
,
Hai b. 10 s kh
ac nhau lun lun t`n tai
. mt
. ch s i sao cho Si 6= Ti .
0
0
,i s j > i n`
Sj = Ti < Si vo
. Ta g
Khi do
ao do
an lai
a
. Si = Sj , Sj = Si v`
,
0
0
0
0
0

. t S = 10S1 + 9S2 + + S10 . Khi d

Sk = Sk voi k 6= i, j. a
o
S S 0 = (11 i)(Si Si0 ) + (11 j)(Sj Sj0 ) =
= (Si Sj )(j i) > 0.
,
,
,
,
, ,
, tu., theo c
su. thay d
Do d
ach trn cho kt qua th`oi gian ch`o giam d
o
i thu
i
, ,
thu. c su. .
,
0
, tu., trn
Nu (S10 , S20 , ..., S10
) 6= (T1 , T2 , ..., T10 ), th` qu
a tr`nh thay d
i thu
, ,
,,
,i (T1 , T2 , ..., T10 ) nhi`u nht l`
. p lai.
duo. c la
a sau 9 l`n. V` t ho. p
. Ta se dat
. to
,
,
,
, , `
bu,o
, tu.,, nn T thu.,c
,c thu.,c hin
th`oi gian ch`o do. i d
u giam sau mi
. di thu
,
,
,
, ,
su. l`
a gi
a tri. nho nht cho th`oi gian ch`o d
o. i.

134

,
,
,,
,c cua c
, tu.,
ng thu
Chuong 3. Bt da
ac d
ay s d
`ng thu

,
,,
Truo
c khi di v`
ao tng qu
at h
oa b`
ai to
an n`
ay ta xet b`
ai to
an c
o s bin
,
,
,
ng dung
t hon v`
a c
ac u
cua n
o.
.

, ,
. p th
. c ba s
3.1. Ca
u tu. hai hoa
,
,
,
,
,
,
,c d
ng thu
p
Xut ph
at t`u mt
a d hiu ch
ung ta c
o th a
a
on gian v`
. bt d
,
dung
v`
a giai h`
ang loat
ai to
an.
.
. b`
,ng s thu.,c. Nu a1 a2 v`
Mnh
d
a nhu
a
` 3.1. Cho a1 , a2 , b1 , b2 l`
.
b1 b2 , th`

a1 b1 + a2 b2 a1 b2 + a2 b1 .
,
,
,
,
Bt d
ng thuc xay ra du b
ang khi v`
a ch khi a1 = a2 hoa
. c b1 = b2 .
a
,
,ng minh. Chu
,ng minh du,o.,c suy ra t`u, bt d
,c (a1 a2 )(b1
ng thu
Chu
a
,
,
,
,c xay ra khi v`
ng thu
. c b1 = b2 .
b2 ) 0. D thy da
a ch khi a1 = a2 hoa

,
,i a1 a2 v`
ng vo
Mnh
d` trn c`
on du
a b1 b2 . thun
.
. tin,
. hai b.
,
,
,
,
s (a1 , a2 ) v`
a (b1 , b2 ) goi
ad
ung thoa m
an d`ng th`oi
`ng thu tu. , nu ch
. l`
,
. c d
a1 a2 v`
a b1 b2 hoa
a b1 b 2 .
`ng th`oi a1 a2 v`
,
,
,i nhu,ng b. (a1 , a2 ) hoa
. c (b1 , b2 ) d
T`u mnh
d` 3.1 cho thy vo
`ng thu
.
,
,
,
,
,
`ng c
tu. , ta thay di nhung s hang
ach thay d
i
. trong tng a1 b1 + a2 b2 ba
,
,
, ,
,
,,
,
, ,,
,
,
c
ac ph`n tu c`
ung b. th` tng se nho di. Ch t`u nhung y tuong don gian nhu
,
,
,
p dung
vy,
o th a
giai c
ac b`
ai to
an.
. nhung ta c
.
,
,
,
V du. 3.1. Chung minh bt d
ng thuc
a
a3 + b3 a2 b + b2 a,

,,
,ng s du,o,ng.
a nhu
od
y a, b l`

,
,
,
`ng s c`
`,i giai. Mt
,n ho,n hoa
. c ba
a b se lo
on lai.
Lo
. trong nhung s a v`
. Biu
,
, ,
,
,c o, v tr
,c da
,ng vo
,i bin
ng thu
cho d`u d
thu
ai v`
a v phai cua bt da
i xu

, tu., hai hoa


. p thu
. c ba s
3.1. Ca

135

,
,
, ,
,c khng thay d
ng thu
(ngha l`
a bt da
cua a v`
a b cho
i khi ta thay di ch
,
,
,
,
2
2
a b v`
nhau), nn ta ch c`n xet tru`ong ho. p a b. Khi do
a suy ra b.
,,
,
,
,,
2
2
tu. . Su dung
s (a, b) v`
a (a , b ) l`
a d`ng thu
mnh
d
a ta nhn
` 3.1 v`
uo. c
.
.
. d

a3 + b3 = a2 a + b2 b a2 b + b2 a.

,ng s du,o,ng. Chu,ng minh bt


V du. 3.2 (IMO 1964). Cho a, b, c l`
a nhu
,
,
da
ng thuc
a3 + b3 + c3 + 3abc a2 b + ab2 + b2 c + bc2 + c2 a + ca2 .
,
,
`,i giai. Bt da
,c d
,ng theo c
ng thu
Lo
ac thng s a, b, c (ngha l`
a bt
i xu
,
,
, ,
,,
,
c khng bin di khi ta thay d
ng thu
da
ac bin). Boi th ta c
o
i ch cua c
,
`
ng a b c. Khi d

th cho ra
o
a3 + b3 + c3 + 3abc = a(a2 + bc) + b(b2 + ac) + c(c2 + ab).
, tu.,(a, b) v`
p dung
Ta a
mnh
d` 3.1 cho c
ac b. s d`ng thu
a (a2 + bc, b2 +
.
.
,
, ,
,
,
c trn l`
ng thu
ac) c
ung nhu (c, a) v`
a (c, b). V bn phai cua da
a
a(a2 +bc)+b(b2 +ac)+c(c2 +ab) a2 b+ca2 +ab2 +b2 c+bc2 +c2 a.

,
V du. 3.3 (IMO 1984). Cho a, b, c l`
ad
ai c
ac canh
cua mt
ac bt
. d`
.
. tam gi
,
k`. Chung minh ra
`ng
a2 b(a b) + b2 c(b c) + c2 a(c a) 0.
,,
, ,
H
ay x
ac d
`ng.
inh
. tru`ong ho. p xay ra du ba
,
,
,
,,
,,
`,i giai. Bt da
,c d`u b`
,c
,i bt da
ng thu
ng thu
ai tuong duong vo
Lo
a3 b + b3 c + c3 a a2 b2 + b2 c2 + c2 a2 .

(3.1)

,
,
,c n`
,c
ng thu
ng thu
Bt da
ay l`
a v`
ong quanh theo c
ac bin (ngha l`
a bt d
a
,
,
v`
khng thay d
ong tr`
on theo c
ac bin). Do vy,
i khi di ch
. khng mt
,
,
`ng trong b. ba (a, b, c) s a l`
,n nht. T`u,
tnh tng qu
at ta c
o th cho ra
a lo

,
,
,,
,c cua c
, tu.,
ng thu
Chuong 3. Bt da
ac d
ay s d
`ng thu

136

,
d thy trong b. (bc, ac, ab) s nho nht l`
do
a bc. Trong b. (a2 + bc, b2 +
,
,
,,
,n nht l`
ac, c2 + ab) s lo
a a2 + bc, di`u n`
ay d
uo. c suy ra t`u nhung bt
,
,
,
,c tam gi
,i ch
ng thu
da
ac (vo
u y a, b, c l`
a nhung d. d`
ai c
ac canh
cua mt
.
.
tam gi
ac)
a2 + bc b2 + ac (a b)(a + b c) 0,

a2 + bc c2 + ab (a c)(a + c b) 0.
,
,c
Ta xet biu thu

A = bc(bc + a2 ) + ac(ac + b2 ) + ab(ab + c2 ).


,
, ,
,
,
,
,c o, s hang
, hai v`
, ba ta di ch
Nu trong biu thu
a thu
cua nhung th`ua
. thu
,
,,
,n ho,n, v` b. s
s ac v`
a ab, th` theo mnh
d` 3.1 ta se nhn
.
. duo. c tng lo
,
, tu., (d
(ac, ab) v`
a (ab + c2 , ac + b2 ) l`
a d`ng thu
ay d kim tra, ban
i`u n`
.
,

doc
am). Khi d
o
. tu. l`
A bc(bc + a2 ) + ab(ac + b2 ) + ac(ab + c2 ).
,
,
,
,
, nht v`
, hai
By gi`o ta lai
thu
a thu
. thay di nhung th`ua s trong s hang
.
,
,c v`
,i nhu,ng b. s (bc, ab) v`
ng thu
p dung
trong d
a a
mnh
d` 3.1 vo
a
a
.
.
,
,
2
2
tu. :
(ac + b , bc + a ) l`
a d`ng thu
A ab(bc + a2 ) + bc(ac + b2 ) + ac(ab + c2 ).

,,
Ta nhn
uo. c
. d

A = b2 c2 + c2 a2 + a2 b2 + a2 bc + ab2 c + abc2
a3 b + b3 c + c3 a + a2 bc + ab2 c + abc2 .
,
,
,c (3.1).
ng thu
T`u dy suy ra bt d
a
,
, ,
, , ,
,
,c ch xay ra
ng thu
thu. c hin
Su. di ch
nhung s hang
da
.
. hai l`n. a
,
,,
,
. c l`
trong nhung tru`ong ho. p: 1) ac = ab hoa
a ac + b2 = ab + c2 v`
a 2)
. c l`
ab = bc hoa
a bc + a2 = ac + b2 .
,
, ,
,
,
,c ch xay ra khi v`
ng thu
suy ra a = b = c, ngha l`
T`u do
ad
a ch khi
a
cho l`
tam gi
ac da
a d`u.

, tu., hai hoa


. p thu
. c ba s
3.1. Ca

137

,
,ng d
ai c
ac canh
cua tam gi
ac bt k`. H
ay
V du. 3.4. Nu a, b, c l`
a nhu
. d`
.
,
,
,
chung minh bt d
ng thuc
a


a b
c
b
a c
2
+ +
+ + + 3.
b
c a
c
b a
,
,
,
,
`,i giai. a
,c l`
. t v tr
ng thu
Lo
ai cua bt da
a B v`
a ta bin d
anh dang:
i th`
.


2
2
2
a b
c
a + bc c + ab b + ac
B=2
+ +
+
+
.
=
b
c a
ab
ac
bc
,
,
,c c
ng thu
Bt da
o tnh v`
ong quanh theo c
ac bin. Ta c
o th cho a l`
a canh
.
,
,
,
,
,
n nht. Vo
i b, c ta xet hai tru`ong ho. p: b c v`
lo
a b c.


1 1
,`,
,
,

ng nhung b.
Truong ho. p b c: Ta d thy ra
,
v`
a (c2 +
ac bc
,
1
1
,
,
, tu.,. Khi do
ta thay di nhung th`ua s
v`
a
ab, b2 + ac) l`
a d`ng thu
ac
bc
,
,
, ,
d
, hai v`
, ba trong tng B se dn
a thu
trong s hang
n tng nho hon.
. thu


c a a
bc
b2
a2 + bc c2 + ab b2 + ac
+
+
= + + +1+
+
B
.
ab
bc
ac
b
c
b
ab ac


,,
1 1
,
,

, tu., v`
`
By gio lai
a (bc, b2 ) l`
a d`ng thu
a su dung
. xet nhung b. ab , ac v`
.
a b
,,
,i ch
mnh
d
ung vo
u y + 2, ta nhn
` 3.1 c`
. duo. c
.
b a
c a a
b
a c
b
B + + + 1 + 2 + + + 3.
b
c
b
a
c
b a
,
,`,
,
,
,
,
Truong ho. p b c: Thay d
i nhung th`ua s c`n thu. c hin
. trong s hang
.
,
,
,
,
nht v`
hai cua tng B, ta c
,ng minh.
thu
a thu
ung suy ra di`u c`n chu
,
,,
,
`ng mnh
Ta thy ra
d` 3.1 d
at biu cho hai bin s, nhung n
o c
o
uo. c ph
.
,
,
,
,
,
,
,
c c
ng thu
p dung
o ba bin s. By gi`o ta xet tuong
th a
cho nhung bt da
.
,
tu. mnh
d
` 3.1 cho b. ba s.
.
, ,
Hai b. ba s = (a1 , a2 , a3 ) v`
a = (b1 , b2 , b3 ) goi
ad
`ng thu tu. ,
. l`
,
,
. c d`ng
nu ch
ung thoa m
an d
a b1 b2 b3 hoa
`ng th`oi a1 a2 a3 v`
,
th`oi a1 a2 a3 v`
a b1 b2 b3 . Hai b. ba s = (a1 , a2 , a3 ) v`
a=

138

,
,
,,
,c cua c
, tu.,
ng thu
Chuong 3. Bt da
ac d
ay s d
`ng thu

,
,
, ,
ung thoa m
an a1 a2 a3
(b1 , b2 , b3 ) goi
a nghich
ao thu tu. , nu ch
. d
. l`

. c a1 a2 a3 v`
v`
a b1 b2 b3 hoa
a b1 b2 b3 .
,
,
,
p xp ch
V du:
o th sa
ung
. Cho ba s nguyn duong a, b, c bt k` th` ta c
,
,

. c a b c). Khi do
hai b. ba
theo thu tu. a b c (hoa
 c)
 s (a, b,
1 1 1
,
,
3
3 3
tu. , c`
, ,
l`
a
v`
a (a , b , c ) l`
a d`ng thu
on hai b. ba s (a, b, c) v`
a
a b c
,
, tu.,.
nghich
ao thu
. d
,
Mnh
d` 3.2. Cho hai b. ba s thu. c (a1 , a2 , a3 ) v`
a (b1 , b2 , b3 ). K hiu
.
.
,
,,
S(i1 ,i2 ,i3 ) = a1 bi1 + a2 bi2 + a3 bi3 , o d
y
(i
,
i
,
i
)
l`
a
ho
a
n
v
i
c
ua
(1,
2,
3).

1 2 3
.

Khi d
o

,
, ,
,
ng
(a) Nu hai b. ba s l`
ad
ac bt d
ng thuc sau d
`ng thu tu. th` c
a
u
S(1,2,3) S(i1 ,i2 ,i3 ) S(3,2,1) .
,
,
, ,
,
ng
(b) Nu hai b. ba s l`
a nghich
ac bt d
ng thuc sau d
ao thu tu. th` c
a
u
. d
S(1,2,3) S(i1 ,i2 ,i3 ) S(3,2,1) .
,
,
,
,
C
ac bt d
ng thuc trn xay ra du ba
`ng khi v`
a ch khi a1 = a2 = a3 hoa
. c
a
,
,
,
(i1 , i2 , i3 ) = (1, 2, 3) (d
oi bt d
ng thuc v tr
ai) v`
a (i1 , i2 , i3 ) = (3, 2, 1)
i v
a
,
,
,
,
(d
oi bt d
ng thuc v phai).
i v
a
,
Ch
u y: Ta hiu hai b. s (x1 , x2 , ..., xn ) = (y1 , y2 , ..., yn ) khi xi = yi

,i i = 1, 2, ..., n.
vo
, ,
,ng minh. (a) Gia su, hai b. s d
, tu.,: a1 a2 a3 v`
Chu
a b1
`ng thu
,
,,
,
`
`
`
`

, tu.,:
p dung
b2 b3 . Tu mnh
d 3.1, ta a
ln luo. t cho nhung b. d
ng thu
.
.
,
(a2 , a3 ) v`
a (b2 , b3 ); (a1 , a2 ) v`
a (b1 , b3 ); (a2 , a3 ) v`
a (b1 , b2 ), suy ra nhung
,
,c
ng thu
bt da
S(1,2,3) = a1 b1 + a2 b2 + a3 b3 a1 b1 + a2 b3 + a3 b2
a1 b3 + a2 b1 + a3 b2 a1 b3 + a2 b2 + a3 b1 = S(3,2,1) ,
S(1,2,3) S(1,3,2) S(3,1,2) S(3,2,1) .

, tu., hai hoa


. p thu
. c ba s
3.1. Ca

139

,
,,
,
,,
,
,c sau
ng thu
Tuong tu. ta c
ung nhn
uo. c nhung bt da
. d
S(1,2,3) S(2,1,3) S(2,3,1) S(3,2,1) .

,
,
,,
`ng c
,i s
Nhu vy
au tng duo. c tao
ac b. a1 a2 a3 v`
a b1 b2 b3
. ba
. vo
,
,
,
,
n nht l`
ta kt lun
a S(1,2,3) = a1 b1 + a2 b2 + a3 b3 , tng nho nht
. tng lo
l`
a S(3,2,1) = a1 b3 + a2 b2 + a3 b1 . Suy ra
S(1,2,3) S(i1 ,i2 ,i3 ) S(3,2,1) .
,
,
,
,c xay khi v`
ng thu
Ban
a ch khi (i1 , i2 , i3 ) = (1, 2, 3) (di
a
. doc
. d thy d
,
,
,c v tr
,c
,i bt d
,i bt da
ng thu
ng thu
vo
ai) v`
a (i1 , i2 , i3 ) = (3, 2, 1) (di vo
a
,
v phai).
,
,
,
(b) Nu nhung b. ba s (a1 , a2 , a3 ) v`
a (b1 , b2 , b3 ) l`
a nghich
. dao thu
,
,
,
tu. , th` (a1 , a2 , a3 ) v`
a (b10 , b20 , b30 ) = (b3 , b2 , b1 ) l`
a nhung b. ba d
`ng thu
,
,
tu. . Theo ph`n (a), tng
S(1,2,3) = a1 b1 + a2 b2 + a3 b3 = a1 b30 + a2 b20 + a3 b10
,
,
l`
a nho nht v`
a tng
S(3,2,1) = a1 b3 + a2 b2 + a3 b1 = a1 b10 + a2 b20 + a3 b30

,ng minh.
,n nht. Tu,o,ng tu., ph`n (a) suy ra di`u c`n chu
l`
a lo
,
,
, tu.,
,i b. ba d
T`u mnh
d
ut ra nhung ch
u y sau dy: Vo
` 3.2 ta r
`ng thu
.
,
,n nht.
(a1 , a2 , a3 ) v`
a (b1 , b2 , b3 ), tng S(1,2,3) = a1 b1 + a2 b2 + a3 b3 l`
a lo
,
,
,
,
,
,
dn
Su. chuyn d
a nhung s trong c`
ung mt
i nhung th`ua s l`
. b. ba dn
,
, ,
tng nho hon.
,
,
p dung
xet:
mnh
d` 3.2 cho mt
By gi`o ta c
o th a
.
.
. s v du. da
,
,
, hai cho v du. 3.2: Vit lai
,c c`n chu
,ng minh
ng thu
C
ach giai thu
. bt da
,,
duo
i dang:
.
a2 (b + c a) + b2 (c + a b) + c2 (a + b c) 3abc

,
,
,,
,c cua c
, tu.,
ng thu
Chuong 3. Bt da
ac d
ay s d
`ng thu

140

, ,,

v`
a gia su a b c. Khi d
o
c(a + b c) b(c + a b) a(b + c a),

,
,c thu
,
ng thu
v` c(a + b c) b(c + a b) = (b c)(b + c a) 0, bt da
,ng minh tu,o,ng tu.,. Nhu, vy
hai chu
a (c(a + b c), b(c +
. hai b. s (a, b, c) v`
,
,
`
`
tu. . Theo mnh
a b), a(b + c a)) d
d
o
ng thu
3.2, ta c
.
a2 (b + c a) + b2 (c + a b) + c2 (a + b c)
ba(b + c a) + cb(c + a b) + ac(a + b c),
2

a (b + c a) + b2 (c + a b) + c2 (a + b c)
ca(b + c a) + ab(c + a b) + bc(a + b c).

,
,
,
,
,c trn, v phai r
ng thu
,
Cng
ut gon
on 6abc. T`u do
. theo v cua hai bt da
. c`
,
,
,
c c`n chu
ng minh.
ng thu
suy ra bt d
a
,
,
,
hai cho v du. 3.3: T`u, a, b v`
C
ach giai thu
a c l`
a c
ac canh
cua tam gi
ac, ta
.
, , ,,
c
o th gia su a b c.
,ng
a(b + c a) b(c + a b) c(a + b c). i`u n`
Khi do
ay chu
1
1
1
,
,

. Nhu vy,
minh nhu ph`n trn. Ta c
ung c
o
. hai b. s (a(b +
a b
c
1 1 1
, tu.,. Theo mnh
c a), b(c + a b), c(a + b c)) v`
a
, ,
l`
a d`ng thu
.
a b c
d` 3.2, ta c
o

1
1
1
a(b + c a) + b(c + a b) + c(a + b c)
c
a
b
1
1
1
a(b + c a) + b(c + a b) + c(a + b c)
a
b
c
= a + b + c.
,
1
1
1
,c trn th`
ng thu
R
ut gon
anh a(b a) + b(c b) + c(a c) 0,
. bt da
c
a
b
,
,
,,
,,
,c n`
,c c`n chu
,ng minh.
,i bt da
ng thu
ng thu
bt da
ay tuong duong vo

Ta xet v du. sau:

, tu., hai hoa


. p thu
. c ba s
3.1. Ca

141

,
,,
,
`ng v
oi moi
V du. 3.5. Chung minh ra
. s duong a, b, c th`
a4 + b4 + c4 a3 b + b3 c + c3 a.

,
,
,
, ,
`,i giai. Do tnh di xu
,ng cua c
,c, ta c
ng thu
Lo
ac bin trong bt d
o th gia
a
, tu., v`
b. ba (a, b, c) v`
thit l`
a a b c. Khi do
a (a3 , b3 , c3 ) l`
ad
a
`ng thu
,
,
,
,

theo mnh
d` 3.2 su. thay d
ung mt
i nhung th`ua s trong c`
.
. b. dn dn
,
, ,
tng nho hon. Suy ra
a4 + b4 + c4 = a3 a + b3 b + c3 c a3 b + b3 c + c3 a.
,
,
,
,c xay ra khi v`
ng thu
a
a ch khi a = b = c.

,
,,
,
V du. 3.6. Chung minh ra
`ng v
oi moi
. s duong a, b, c th`
a5 b + b5 c + c5 a a4 bc + ab4 c + abc4 .

,
,
,
,
`,i giai. Do tnh d
,ng cua c
,c, ta c
ng thu
Lo
ac bin trong bt da
o th
i xu
,
,
nhung b. ba (a4 , b4 , c4 ) v`
gia thit l`
a a b c. Khi do
a (bc, ac, ab) l`
a
,
,
,
,
4
4
4
tu. . Theo mnh
nghich
d
o tng S = a bc + ab c + abc l`
a
ao thu
` 3.2 c
. d
.
,
nho nht. Suy ra
a4 bc + ab4 c + abc4 a4 ab + b4 bc + c4 ac = a5 b + b5 c + c5 a.
,
,
,
,c xay ra khi v`
ng thu
a
a ch khi a = b = c.

,
,,
,
V du. 3.7. Chung minh ra
`ng v
oi moi
. s duong x, y, z th`
xy3 + yz3 + zx3 x2 yz + xy2 z + xyz2 .

,
,
, ,
,
`,i giai. Do tnh di xu
,ng cua c
,c, ta c
ng thu
Lo
ac bin trong bt d
o th gia
a
,
nhung b. ba (x2 , y2 , z2 ) v`
thit x y z. Khi do
a (yz, xz, xy) l`
a nghich
.
,
,
,
,
,
2
2
2
`
tu. . Theo mnh
dao thu
o tng S = x yz + y xz + z xy l`
a nho nht.
. d 3.2 c
,
,
,
,
Mt
ach chuyn th`ua s cho ta tng xy3 + yz3 + zx3 . Suy ra
. trong nhung c
xy3 + yz3 + zx3 x2 yz + y2 xz + z2 xy.
,
,
,
,c xay ra khi v`
ng thu
a
a ch khi x = y = z.

,
,
,,
,c cua c
, tu.,
ng thu
Chuong 3. Bt da
ac d
ay s d
`ng thu

142

,
,,
,
`ng v
oi moi
V du. 3.8. Chung minh ra
. s duong a, b, c th`
a b
c
a 2 b2
c2
+ + 2 + 2 + 2.
b
c a
b
c
a


,
,
1 1 1
`,i giai. Nhu,ng b. ba (a, b, c) v`
, tu.,. Theo
Lo
a
, ,
l`
a nghich
. dao thu
a b c
mnh
d
` 3.2 suy ra
.
a

1
1
a b
c
1
+b +c + + ,
a
b
c
b
c a

ngha l`
a
Ngo`
ai ra


a b
c
+ + 3.
b
c a
a b
c
+ +
b
c a

2

a2 b 2
c2
= 2 + 2 + 2 +2
b
c
a

(3.2)


a c
b
+ +
c
b a

`ng c
, tu.,
,i nhu,ng b. d`ng thu
p dung
Ba
ach a
mnh
d
` 3.2 di vo
.
.


a b c
,,
v`
a
, ,
ta nhn
uo. c
. d
b c a

(3.3)


a b c
, ,
b c a

c2
a b b c
c a
a c
b
a2 b2
+ 2 + 2 + + = + + .
(3.4)
2
b
c
a
b c
c a a b
c
b a
,
T`u (3.2), (3.3) v`
a (3.4) suy ra

 
2
 2

a
c
b
a
c
b
a
b2
c2
3
+ +

+ +
3
+
+
.
b a c
b a c
b2
c2
a2
,
, ,,
,,
,c c`n chu
,ng minh.
c hai v cho 3 ta nhn
ng thu
Sau khi gian uo
a
. duo. c bt d

` tp
Bai
.
,
,
`ng phu,o,ng ph
, tu.,, h
Ba
ap nhung b. d
ay s d`ng thu
ay l`
am nhung b`
ai
,
,,
,
,
,
,
,
,
i a, b, c l`
tp
a nhung s duong (l`oi giai v`
a go. i y o trang 171):
. sau vo
`ng aa bb ab ba .
,ng minh ra
. 3.9. Chu

,
3.2. Tng qu
at h
oa

143

`ng
,ng minh ra
. 3.10. Chu
b
a
a+b
a a+b b a+b .
2

`ng 3(a3 + b3 + c3 ) (a + b + c)(a2 + b2 + c2 ).


,ng minh ra
. 3.11. Chu
`ng 2a15 + 3b10 5a6 b6 .
,ng minh ra
. 3.12. Chu
`ng 4a21 + 3b14 7a12 b6 .
,ng minh ra
. 3.13. Chu

,
hoa

3.2. Tng quat


,
Trong ph`n n`
ay ta tng qu
at mnh
d
a mnh
d` 3.2 cho hai d
ay
` 3.1 v`
.
.
,
,
,
,
,

huu han
ao dinh
ua v`
. ngha:
. s thu. c. Truoc tin ta d
,
inh
ngha 3.1. Hai b. s thu. c (a1 , a2 , ..., an ) v`
a (b1 , b2 , ..., bn ) (n 2)
.
,
,
,
,

`
`
goi
ad
ung thoa m
an d
a
ng thu tu. , nu ch
ng th`oi a1 a2 an v`
. l`
,
`
`
. c d
b1 b2 bn hoa
a b1 b2
ng thoi a1 a2 an v`

bn .

,
Hai b. s thu. c (a1 , a2 , ..., an ) v`
a (b1 , b2 , ..., bn ) (n 2) goi
a nghich
.
. l`
,
,
,
,
,
d
ung thoa m
an d`ng th`oi a1 a2 an v`
a b1
ao thu tu. , nu ch
,
. c d
b2 bn hoa
a b1 b 2 bn .
`ng th`oi a1 a2 an v`
,,
K hiu
y b. s
. S(i1 ,i2 ,...,in ) = a1 bi1 + a2 bi2 + + an bin , o d
,
(i1 , i2 , ..., in ) l`
a ho
an vi. cua b. s (1, 2, ..., n).
,
inh
a (b1 , b2 , ..., bn1 ,
. l 3.1. (a) Cho hai b. sthu. c (a1 , a2 , ..., an1 , an ) v`
,
,
,
,
ta c
bn ) (n 2) d
o c
ac bt da
ng thuc sau
`ng thu tu. . Khi d
o
S(1,2,...,n) S(i1 ,i2 ,...,in ) S(n,n1,...,1) ,
,
,
v
oi moi
a ho
an vi. cua (1, 2, ..., n).
. (i1 , i2 , ..., in ) l`
,
(b) Cho hai b. s thu. c (a1 , a2 , ..., an1 , an ) v`
a (b1 , b2 , ..., bn1 , bn )

144

,
,
,,
,c cua c
, tu.,
ng thu
Chuong 3. Bt da
ac d
ay s d
`ng thu

,
,
, ,
,
ta c
ng thuc sau
(n 2) nghich
o c
ac bt d
o
a
. dao thu tu. . Khi d
S(1,2,...,n) S(i1 ,i2 ,...,in ) S(n,n1,...,1) ,
,
,
v
oi moi
a ho
an vi. cua (1, 2, ..., n).
. (i1 , i2 , ..., in ) l`
,
,
,
, ,,
C
ac bt da
ng thuc o (a) v`
a (b) xay ra du ba
`ng khi v`
a ch khi a1 = a2 =
,
,
,
... = an hoa
. c (i1 , i2 , ..., in ) = (1, 2, ..., n) (d
oi bt d
ng thuc v tr
ai) v`
a
i v
a
,
,
,
,

(i1 , i2 , ..., in ) = (n, n 1, ..., 2, 1) (d


oi bt da
ng thuc v phai).
i v
,ng minh. Ta chu
,ng minh cho tru,`o,ng ho.,p (a) vo
,
,i hai b. s d
Chu
`ng thu
,
,,
,
tu. a1 a2 an v`
a b1 b2 bn , c`
on c
ac tru`ong ho. p kh
ac
,
,
,
,
,
,
,
`
ng minh tuong tu. . Ta se chu
ng minh dinh
ng phuong ph
chu
l ba
ap quy
.
,
,
,
,
,
ng minh trong mnh
i n = 2, kha
ng dinh
nap
d
uo. c chu
` 3.1.
. d
. theo n. Vo
.
,
,
i n 1 (n 3).
ng vo
Ta gia thit d
inh
u
. l d
,
,
ng minh tng lo
,n nht l`
Ta se chu
a
S(1,2,...,n) = a1 b1 + a2 b2 + + an bn .
,
Tht
o
. vy,
. nu trong tng S(i1 ,i2 ,...,in ) = a1 bi1 + a2 bi2 + + an bin c
,
, `, ,

ng dinh
a
i1 = 1, th` kha
. suy ra tru. c tip tu gia thit quy nap.
. Cho i1 6= 1 v`
,
,,
`
`
tu mnh
d 3.1 ta nhn
uo. c
.
. d
,i j1 = 2, 3, 4, .., n.
a1 bi1 + aj1 b1 a1 b1 + aj1 bi1 vo
,
,
,
,c xay ra khi v`
ng thu
. c (bi1 , b1 ) = (b1 , bi1 ). Theo
a
a ch khi a1 = aj1 hoa
,
,

gia thit quy nap


ay s a2 ... an1 an v`
a
. cho nhung d
b2 ... bn1 bn ta c
o

a2 bi2 + + aj1 bi1 + + an bin a2 b2 + a3 b3 + + an bn .


,
,
,
,c xay ra khi v`
ng thu
. c (bi2 , ..., bin ) =
a
a ch khi a2 = a3 = ... = an hoa

,
3.2. Tng qu
at h
oa

145

(b2 , b3 , ..., bn ). Khi d


o
a1 bi1 + a2 bi2 + + aj1 b1 + + an bin
a1 b1 + a2 bi2 + + aj1 bi1 + + an bin
a1 b1 + a2 b2 + + an bn = S(1,2,...,n) .

,
,
,,
,
,ng minh tng lo
,n nht l`
chu
Nhu vy
a S(1,2,...,n) . Kt ho. p c
ac tru`ong
. ta da
,
,
,
,
,c xay ra khi v`
ng thu
. c
ho. p da
a ch khi a1 = a2 = ... = an hoa
(bi1 , bi2 , ..., bin ) = (b1 , b2 , ..., bn ).
,
,
,,
,
,ng minh du,o.,c tng nho nht l`
Ho`
an to`
an tuong tu. ta c
ung chu
a

S(n,n1,...,1) = a1 bn + a2 bn1 + + an b1 .
,
,ng s thu.,c v`
H. qua 1: Cho a1 , a2 , ..., an l`
a nhu
a (a10 , a20 , ..., an0 ) l`
a ho
an vi.
,

cua (a1 , a2 , ..., an ). Khi d


o
a21 + a22 + + a2n a1 a10 + a2 a20 + + an an0 .
,
,ng s thu.,c du,o,ng v`
a
H. qua 2: Cho a1 , a2 , ..., an l`
a nhu
a (a10 , a20 , ..., an0 ) l`
,

ho
an vi. cua (a1 , a2 , ..., an ). Khi d
o
a0
a0
a10
+ 2 + + n n.
a1 a2
an
,
V du. 3.14. Cho x1 x2 ... xn v`
a y1 y2 ... yn l`
a hai b. s thu. c.
,
,
Chung minh ra
`ng nu z1 , z2 , ..., zn l`
a ho
an vi. bt k` cua c
ac s y1 , y2 , ..., yn ,
th`
n
n
X
X
(xi yi )2
(xi zi )2 .
i=1

i=1

,
,
`,i giai. Theo gia thit ta c
Lo
o
y21 + y22 + + y2n = z21 + z22 + + z2n .

146

,
,
,,
,c cua c
, tu.,
ng thu
Chuong 3. Bt da
ac d
ay s d
`ng thu

,
,
,,
,,
,c da
,i
ng thu
cho tuong d
a r
ut gon,
Sau khi khai trin v`
uong vo
. bt da
n
X
i=1

xi yi

n
X

xi zi

i=1

,
,c trn du
ng thu
ng.
v`
a theo dinh
. l 3.1 bt da

,
,,
,
V du. 3.15. Cho 2n s duong a1 < a2 < ... < a2n . Chung minh ra
`ng tng
S = a1 a2n + a2 a2n1 + ... + an an+1
,
,
,ng ca
,ng s d
cho.
c
o gi
a tri. nho nht trong s tng nhu
. p trong nhu
a

,
,,
,
`,i giai. 1) C
, tu.,
,i hai b. s d`ng thu
Lo
ach thu nht: Su dung
dinh
. l 3.1 vo
.
,
,
,ng minh.
(a1 , a2 , ..., an ) v`
a (an+1 , ..., a2n ) cho kt qua phai chu
, ,
,,
,
2) C
ach thu hai: C
ung nhu chu
ng minh d
l 3.1, ta d`
ung phuong
inh
.
ph
ap quy nap
an hoc:
. to
.
,
,
,
,
i n = 1 kha
ng dinh
ng l`
Vo
a hin nhin v` khng c
o ho
an vi. kh
ac cua
. du
c
ac s.
,
,
,
`ng kha
,ng minh kha
,i 2n s. Ta se chu
ng dinh
ng
ng vo
Gia thit ra
du
.
,i 2(n + 1) s: a0 < a1 < a2 < ... < a2n < a2n+1 .
ng vo
dinh
ung du
. c
, ,
,,
,
,ng minh d
D chu
a a2n+1 nhn
a tri. nho hon l`
a
uo. c kt ho. p hai s a0 v`
. gi
, ,

ch
ung kt ho. p voi c
ac s kh
ac. Tht
. vy,
. a0 a2n+1 +ak a` < a0 ak +a2n+1 a`
,,
boi v` (a` a0 )(a2n+1 ak ) > 0.
,
,
,
,
V` th tng nho nht cua nhung tch hai s c
o dang
a0 a2n+1 + ....
.
,
,
,
,
,
i 2n s th` tng cua tch c
. p s c
Nhung theo gia thit quy nap:
ac ca
o gi
a
. Vo
,
,
,
ng d
ng
tri. nho nht l`
a a1 a2n + a2 a2n1 + ... + an an+1 . Nhu vy
inh
u
. d
. kha
,
i moi
vo
. n.
`ng c
,ng minh tu,o,ng tu., hoa
,ng
. c d`
: Ba
Ch
uy
ach chu
ung d
l 3.1 ta chu
inh
.
,
,
,
,,
,
,
,,
. p s duong c
ng dinh:
minh duo. c nhung kha
Tng cua nhung tch c
ac ca
o
.
,
n nht l`
gi
a tri. lo
a

S = a1 a2 + a3 a4 + ... + a2n1 a2n .

,
3.2. Tng qu
at h
oa

147

,
,
, ,
`ng phu,o,ng ph
. p tng
Ba
ap nhu chu
ng minh b`
ai trn cho tch cua c
ac ca
,
hai s cua d
ay s.
,
,,
,
V du. 3.16. Cho 2n s duong a1 < a2 < ... < a2n . Chung minh ra
`ng tng
S = (a1 + a2n )(a2 + a2n1 )...(an + an+1 )
,
,ng ca
,ng s d
cho.
c
o gi
a tri. l
on nht trong s tch nhu
. p trong nhu
a
,
,
,,
`,i giai. D`
,i n = 1, kha
ng dinh
ng
Lo
ung phuong ph
ap quy nap
u
. d
. theo n. Vo
,
, ,
,
,
l`
a hin nhin v` khng c
o t ho. p kh
ac cua tng.
,
, ,, ,
,ng minh kha
,i 2n s. Ta se chu
ng dinh
ng d
ng vo
Gia su kha
ung
inh
. du
. c
,

i 2(n + 1) s: a0 < a1 < a2 < ... < a2n < a2n+1 .


ng vo
du
,
,ng minh kt ho.,p tng hai s a0 v`
,n ho,n l`
D chu
a a2n+1 nhn
a tri. lo
a
. gi
, ,
,

ch
ung kt ho. p voi c
ac s kh
ac. Tht
o
. vy,
. voi k, ` {1, 2, ..., 2n} ta c
(a0 + a2n+1 )(ak + a` ) (a0 + ak )(a2n+1 + a` )
= (a0 ak + a0 a` + a2n+1 ak + a2n+1 a` )
(a0 a2n+1 + a0 a` + ak a2n+1 + ak a` )
= a0 (ak a2n+1 ) + a` (a2n+1 ak ) = (a2n+1 ak )(a` a0 ) 0.
,
,
,n nht cua nhu,ng tng hai s c
V` th tch lo
o dang
. (a0 +a2n+1 )...(an +
,
,
,
,
,
i 2n s th` tch cua tng c
. p
an+1 ). Nhung theo gia thit quy nap:
ac ca
. Vo
,
,
n nht l`
s c
o gi
a tri. lo
a (a1 + a2n )(a2 + a2n1 )...(an + an+1 ). Nhu vy
.
,
,
i moi
ng dinh
ng vo
kha
u
. d
. n.
,
,
,
,
`ng c
ng minh tu,o,ng tu., cho kha
ng d
: Ba
ach chu
Tch cua nhung
Ch
uy
inh:
.
,
,
,,
. p s duong c
tng theo c
ac ca
o gi
a tri. nho nht l`
a

S = (a1 + a2 )(a3 + a4 )...(a2n1 + a2n ).


,ng s nguyn du,o,ng kh
V du. 3.17 (IMO 1978). Cho c1 , c2 , ..., cn l`
a nhu
ac
,
nhau. Chung minh ra
`ng
c2
cn
1
1
c1 + 2 + + 2 1 + + + .
2
n
2
n

148

,
,
,,
,c cua c
, tu.,
ng thu
Chuong 3. Bt da
ac d
ay s d
`ng thu

,
,
,,
`,i giai. Cho a1 , a2 , ..., an l`
, tu., t
a nhung s ci d
Lo
ang d`n. V`
uo. c xp theo thu
,
,
,
ai l`
a nhung s nguyn duong kh
ac nhau, ta c
o a1 1, a
n
2 2, ..., an 
1
1
1
1
,
a 1, 2 , , 2
v`
a 1 > 2 > > 2 . Nhu vy
. hai b. (a1 , a2 , ..., an ) v`
2
n
2
n
,
, ,

l`
a nghich
d
ao
th
u
t
u
.
Theo
d
inh
l

3.1
ta
c
o

.
.
.

c2
cn
a2
an
2
n
+ + 2 a1 + 2 + + 2 1 + 2 + + 2 .
22
n
2
n
2
n
,
,
,ng s du,o,ng, th` bt d
V du. 3.18 (My 1974). Nu a, b, c l`
a nhu
ng thuc
a
ng
sau d
u
c1 +

aa bb cc (abc)

a+b+c
3
.

,
,
,
`,i giai. Bt da
,c d
,ng theo c
ng thu
ta c
Lo
ac thng s a, b, c, do do
o th
i xu
`ng a b c, khi do
ln a ln b ln c. Theo dinh
cho ra
. l 3.1:
a ln a + b ln b + c ln c a ln a + b ln b + c ln c,
a ln a + b ln b + c ln c a ln b + b ln c + c ln a,
a ln a + b ln b + c ln c a ln c + b ln a + c ln b.
,
,
,,
,c trn v`
ng thu
Cng
ac bt d
a ta nhn
a
. theo v cua c
. duo. c:
(a + b + c)(ln a + ln b + ln c)
.
3
,
,
,
,,
am s m
u se nhn
T`u dy bin d
i theo h`
. duo. c kt qua.
a ln a + b ln b + c ln c

` tp
Bai
.

, , ,
(Goi
a tra l`oi t`u trang 172)
. y v`

`ng nu a, b, c > 0, th`


,ng minh ra
. 3.19. Chu
a3
b3
c3
a 2 + b2 + c 2
+
+

.
b+c c+a a+b
2
,
,,
`ng
,ng minh ra
. 3.20. Cho a, b, c l`
a nhung s duong. Chu
bc
ca
ab
1
+
+
(a + b + c).
b+c c+a a+b
2

,,
, tu.,
3.3. Su dung
c
ac d
ay s d`ng thu
.

149

,
,,
`ng
,ng minh ra
. 3.21. Cho x1 , x2 , ..., xn l`
a nhung s duong. Chu
x1 + x2 + + xn

x21 x22
x2
+
+ + n.
x2 x3
xn

,
,,
,,
,ng
. 3.22. Cho a, b, c l`
a nhung s duong v`
a n l`
a s nguyn duong, chu
`ng
minh ra
bn
cn
an1 + bn1 + cn1
an
+
+

.
b+c c+a a+b
2
,
,
. 3.23. (My 1999) Cho a1 , a2 , a3 , ... (n > 3) l`
a nhung s thu. c sao cho
a1 + a2 + + an n v`
a a21 + a22 + + a2n n2 .

`ng max(a1 , a1 , ..., an ) 2.


,ng minh ra
Chu

,
`ng radian) cua mt
. 3.24. Cho A, B, C l`
a d. do g
oc (ba
ac, a, b, c
. tam gi
,
a+b+c
,
`ng
ng minh ra
. t p =
l`
ad
ai c
ac canh
cua tam gi
ac v`
ad
. Chu
. d`
a
.
2
B
C
3
A
+
+

.
pa pb pc
p

,,
, ,
d
3.3. Su dung
cac
ay s d
u tu.
`ng th
.
,
,,
,,
, tu., da
giai duo. c rt nhi`u b`
Phuong ph
ap d`
ung d
ay s d
ai tp
`ng thu
. v`
,
,
,
,
,
,
c. Nhung n
ng minh mt
ng thu
o c`
on l`
a mt
ap chu
bt d
a
. phuong ph
. s bt
,
,
,
,
,
,
,
,
,

c Chebyshev v`
c Cauchy.
c co ban nhu bt da
ng thu
ng thu
ng thu
a bt d
da
a
,
,,
,
inh
ngha 3.2. Mt
ay s thu. c a1 , a2 , ..., an duo. c goi
a d
ay d
o n d
iu
.
. d
. l`
.
,,
,

t
ang, nu a1 a2 ... an . D
ay s a1 , a2 , ..., an duo. c goi
a d
ay d
o n
. l`
,
d
iu
. giam, nu a1 a2 ... an .

,
, ,
,c Chebyshev). Cho hai d
ng thu
inh
l 3.2 (Bt d
ay s thu. c hu
u han
a
.
.
a1 , a2 , ..., an v`
a b1 , b2 , ..., bn . a
. t A = a1 b1 + a2 b2 + + an bn v`
a
B = a1 bn + a2 bn1 + + an b1 ..

,
,
,,
,c cua c
, tu.,
ng thu
Chuong 3. Bt da
ac d
ay s d
`ng thu

150

,
ang (hoa
. c
(a) Nu hai d
ay a1 , a2 , ..., an v`
a b1 , b2 , ..., bn c`
ung d
iu
o n d
. t
,
,
c`
ung d
on d
iu
. giam), th`
(a1 + + an )(b1 + + bn )
B;
n
,
,
(b) Nu d
ay a1 , a2 , ..., an l`
ad
ang v`
a b1 , b2 , ..., bn l`
ad
on diu
o n d
iu
. t
.
,
,
,
,
giam (hoa
. c a1 , a2 , ..., an d
a b1 , b2 , ..., bn d
ang), th`
o n d
iu
on diu
. giam v`
. t
(a1 + + an )(b1 + + bn )
A
B.
n
,
,
,
,,
,
,
C
ac bt d
ng thuc trong (a) v`
a (b) tro th`
anh d
ng thuc khi v`
a ch khi
a
a
A

a1 = a2 = ... = an hoa
. c b1 = b2 = ... = bn .
,ng minh. Ta chu
,ng minh cho tru,`o,ng ho.,p (a) vo
,i hai d
cho l`
Chu
ay da
a c`
ung
,`,
,
,
,
,
,
,
ng minh ho`
don diu
ang, c`
on c
ac truong ho. p kh
ac chu
an to`
an tuong tu. .
. t
,
,,
ng
Theo dinh
l 3.1 ho
an vi. v`
ong quanh theo bi , ta nhn
uo. c n bt da
.
. d
,
c
thu
A a1 b1 + a2 b2 + + an bn B,
A a1 b2 + a2 b3 + + an b1 B,

A a1 bn + a2 b1 + + an bn1 B.
,
,
,
,
,
,c trn v`
,c
ng thu
ng thu
a bin d
Cng
theo v cua nhung bt da
i v` bt d
a
.
Chebyshev.
,
,
,
,
,
,c khi v`
,c tro, th`
ng thu
ng thu
a ch khi
anh da
C
ung t`u d
inh
. l 3.1 bt da
. c b1 = b2 = ... = bn .
a1 = a2 = ... = an hoa
,
,c Chauchy). Nu nhu
,ng s a1 , a2 , ..., an l`
,ng
ng thu
inh
a nhu
. l 3.3 (Bt da
,
s thu. c khng m, th`

a1 + a2 + + an n n a1 a2 ...an .
,
,
, ,
a
ng thuc xay ra khi v`
a ch khi a1 = a2 = ... = an .

,,
, tu.,
3.3. Su dung
c
ac d
ay s d`ng thu
.

151

,ng minh. a
,c l`
. t G = n a1 a2 ...an . Nu G = 0 th` bt da
ng thu
Chu
a
,
a1 a2
a1
. t b1 =
d
, b2 =
, ..., bn =
hin nhin. Xet G 6= 0, khi d
o
a
G
G2
,
a1 a2 ...an
= 1. Theo h. qua 2 ta c
o
Gn
an
b1
b2
bn
a1 a2
+
+ +
,
n
+
+ +
=
b n b1
bn1
G
G
G
,
,
a + a2 + + an
,,
,,
,c xay ra khi
,i 1
ng thu
di`u n`
ay tuong duong vo
G. a
n
,
v`
a ch khi b1 = b2 = ... = bn , suy ra a1 = a2 = ... = an .

,
,ng s du,o,ng, th`
V du. 3.25. Chung minh ra
`ng nu a1 , a2 , ..., an l`
a nhu

a1 a2 ...an

n
.
1
1
1
+
+ +
a1 a2
an

,
,
, ,
a
ng thuc xay ra khi v`
a ch khi a1 = a2 = ... = an .
,
a1
a1 a2
a1 a2 ...an

`,i giai. a
. t G = n a1 a2 ...an , b1 =
, b2 =
, ..., bn =
=
Lo
2
G
G
Gn
,
1. Theo h. qua 2 ta c
o
b1 b2
bn
G
G
G
+
+ +
=
+
+ +
,
b2 b3
b1
a1 a2
an
,,
,,
,i
di`u n`
ay tuong d
uong vo
n
G
.
1
1
1
+
+ +
a1 a2
an
,
,
,
,c xay ra khi v`
ng thu
a
a ch khi b1 = b2 = ... = bn , suy ra a1 = a2 =
n

... = an .
,
,ng s du,o,ng, th`
V du. 3.26. Chung minh ra
`ng nu a1 , a2 , ..., an l`
a nhu
r
a21 + a22 + + a2n
a1 + a2 + + an

.
n
n
,
,
, ,
a
ng thuc xay ra khi v`
a ch khi a1 = a2 = ... = an .

152

,
,
,,
,c cua c
, tu.,
ng thu
Chuong 3. Bt da
ac d
ay s d
`ng thu

,
,
`,i giai. Theo h. qua 1, ta c
Lo
o
a21 + a22 + + a2n a1 a2 + a2 a3 + + an a1 ,
a21 + a22 + + a2n a1 a3 + a2 a4 + + an a2 ,

a21 + a22 + + a2n a1 an + a2 a1 + + an an1 .
,
,c trn v`
ng thu
Cng
theo v c
ac bt da
a cng
thm v`
ao hai v c`
ung dai
.
.
.
,,
,,
2
2
2
luo. ng a1 + a2 + + an , ta nhn
. duo. c
n(a21 + a22 + + a2n ) (a1 + a2 + + an )2 ,
,
,
,
,,
,,
,ng minh. a
,c xay ra khi
,i kt qua c`n chu
ng thu
d
ay tuong d
i`u n`
uong vo
,
v`
a ch khi a1 = a2 = ... = an .
,
,c Cauchy-Schwarz). Cho a1 , a2 , ..., an , b1 , b2 , ..., bn
ng thu
V du. 3.27 (Bt d
a
,ng s thu.,c. Khi d

l`
a nhu
o

(a1 b1 + a2 b2 + + an bn )2 (a21 + a22 + + a2n )(b21 + b22 + + b2n ),


,
,
, ,
sao cho ai = kbi
d
ng thuc xay ra khi v`
a ch khi t`n tai
`ng s k n`
ao d
a
o
. mt
. ha
,
hoa
. c bi = kai v
oi i = 1, n.
,
`,i giai. Nu a1 = a2 = ... = an = 0 hoa
. c b1q= b2 = ... = bn = 0
Lo
,
,,

. t S = a21 + a22 + + a2n v`


th` kt lun
a hin nhin. Nguo. c lai
a
a
. l`
. ta d
q
,
,
,
T = b21 + b22 + + b2n . V` hai dai
ac khng, ta c
o th
. luo. ng trn d`u kh
,
ai
bi ,
i i = 1, n. Theo h. qua 1,
. t xi =
da
v`
a xn+i =
vo
S
T
a2 + a22 + + a2n b21 + b22 + + b2n
2= 1
+
= x21 + x22 + + x22n
S2
T2
x1 xn+1 + x2 xn+2 + + xn x2n + xn+1 x1 + xn+2 x2 + + x2n xn =
2(a1 b1 + a2 b2 + + an bn )
,
ST
,
,
,
,,
,,
,c c`n chu
,ng minh. a
,c xay
,i bt da
ng thu
ng thu
di`u n`
ay tuong d
uong vo
,
,i i = 1, n, hay l`
,i i = 1, n.
ra khi v`
a ch khi xi = xn+i vo
a ai T = bi S vo
=

,,
, tu.,
3.3. Su dung
c
ac d
ay s d`ng thu
.

153

,
,ng s du,o,ng v`
a abc = 1. Chung
V du. 3.28 (IMO 1995). Cho a, b, c l`
a nhu
minh ra
`ng
1
1
1
3
+
+
.
a3 (b + c) b3 (c + a) c3 (b + a)
2
,
,
1
1
1
`,i giai. a
,c c`n
. t x = bc = , y = ca = , z = ab = . Bt da
ng thu
Lo
a
b
c
,
,ng minh tro, th`
chu
anh
y2
z2
3
x2
+
+
.
z+y x+z y+x
2
,
,
,,
,
,
`ng x
c l`
ng theo c
ng thu
Boi bt d
ad
ac bin, nn ta gia thit ra
a
i xu
1
1
1
,
x2 y2 z2 v`
y z, khi d
a

. Nhu vy
o
. hai b.
z+y
x+z
y+x
(x2 , y2 , z2 ) v`
a


1
1
1
, tu.,. Theo d
,
,
l`
a d`ng thu
o
inh
. l 3.1 ta c
z+y x+z y+x
y2
z2
x2
y2
z2
x2
+
+

+
+
,
z+y x+z y+x
y+x z+y x+z
x2
y2
z2
x2
y2
z2
+
+

+
+
.
z+y x+z y+x
x+z y+x z+y
,
,,
,c n`
ng thu
Cng
ac bt d
ay theo v v`
a chia cho 2, ta nhn
a
uo. c
. c
. d


x2
y2
z2
1 y2 + x2 z2 + y2 x2 + z2
+
+

+
+
.
z+y x+z y+x
2
y+x
z+y
x+z
,
,
(a + b)2
,c a2 + b2
,c Cauchy, ta
ng thu
ng thu
p dung
Ta a
bt da
v`
a bt da
.
2
,,
nhn
. duo. c


x2
y2
z2
1 y+x z+y x+z
x+y+z
+
+

+
+
=
z+y x+z y+x
2
2
2
2
2

3 3 xyz
3

= .
2
2
,
V du. 3.29. Cho 0 ak < 1 v
oi k = 1, 2, ..., n v`
a S = a1 + a2 + + an .
,
`
H
ay chung minh ra
ng
n
X
ak
nS

.
1 ak
nS

k=1

,
,
,,
,c cua c
, tu.,
ng thu
Chuong 3. Bt da
ac d
ay s d
`ng thu

154

,
,
, , ,,
`,i giai. Khng mt tnh tng qu
Lo
at ta c
o th gia su l`
a
a1 a2 an 0.

Khi d
o
0 < 1 a1 1 a2 1 an v`
a

a1
a2
an


.
1 a1
1 a2
1 an

,
,c Chebyshev:
ng thu
Theo bt da
a2
an
a1
(1 a1 ) +
(1 a2 ) + +
(1 an )
1 a1
1 a2
1 an
n
n
n
1 X ak X
n S X ak

(1 ak ) =
.
n
1 ak
n
1 ak

S=

k=1

k=1

k=1

,ng s du,o,ng bt k`. Chu,ng minh bt


V du. 3.30. Cho x1 , x2 , ..., xn l`
a nhu
,
,
d
ng thuc
a
(x1 + x2 + ... + xn )3 n2 (x31 + x32 + ... + x3n ).
,
,
, ,
`,i giai. Do bt d
,c l`
,ng theo c
ng thu
Lo
a di xu
ac bin nn ta c
o th gia thit
a
`ng
ra
,
,c Chebyshev ta c
ng thu
x1 x2 ... xn . Theo bt d
o
a
! n
!
n
n
X
X
X
n
x3i
xi
x2i
i=1

i=1

i=1

v`
a
n

n
X
i=1

x2i

n
X

!2
xi

i=1

,
,
,
,
,c trn ta du,o.,c bt d
,c c`n phai chu
,ng
ng thu
ng thu
Nhn theo v cua hai d
a
a

minh.
,
,ng s du,o,ng, th`
V du. 3.31. Chung minh ra
`ng nu x1 , x2 , ..., xn l`
a nhu
(x1 x2 ...xn )

x1 +x2 +...+xn
n

xx11 xx22 ...xxnn .

,,
, tu.,
3.3. Su dung
c
ac d
ay s d`ng thu
.

155

,
,
,,
,,
`,i giai. Bt da
,c da
,i
ng thu
cho tuong duong vo
Lo
x1 + x2 + ... + xn lg x1 + lg x2 + ... + lg xn

n
n
x1 lg x1 + x2 lg x2 + ... + xn lg xn

.
n
,
,
,,
,
,c n`
,c Chebyshev (hoa
,ng
ng thu
ng thu
. c ta chu
Bt d
ay duo. c suy ra t`u bt da
a
,
,ng minh d
minh nhu trong chu
inh
. l Chebyshev).

,
V du. 3.32. Cho a, b, c > 0, n N. Chung minh ra
`ng

an+1 + bn+1 + cn+1


a+b+c

.
an + bn + cn
3

,
,
, ,
`,i giai. Gia su, 0 < a b c, suy ra an bn cn . Theo bt da
,c
ng thu
Lo

Chebyshev

(a + b + c)(an + bn + cn ) 3(an+1 + bn+1 + cn+1 ).


,
,
,ng minh.
T`u dy suy ra d
i`u phai chu

,
V du. 3.33. Cho tam gi
ac nhon
`ng
. ABC. Chung minh ra
sin 2A + sin 2B + sin 2C sin A + sin B + sin C.
,
, ,
`,i giai. Gia su, A B C. Ta xet hai b. s (sin A, sin B, sin C) v`
Lo
a
,

(cos A, cos B, cos C), do tam gi


ac l`
a nhon
. nn A, B, C (0, 2 ), trong khoang
,
,
,
n`
ay h`
am sin d
ang, c`
on cos don diu
on diu
. t
. giam, suy ra sin A sin B
,
,c Chebyshev
ng thu
a
p dung
sin C v`
a cos A cos B cos C. Sau do
bt da
.
,
cho kt qua.

` tp
Bai
.

,
, , ,,
(Go. i y v`
a l`oi giai o trang 174).

,
,,
,ng
. 3.34. Cho a, b, c, d l`
a nhung s duong v`
a S = a2 + b2 + c2 + d2 . Chu
`ng
minh ra
a3 + b3 + c3 a3 + b3 + d3 a3 + c3 + d3 b3 + c3 + d3
+
+
+
S.
a+b+c
a+b+d
a+c+d
b+c+d

156

,
,
,,
,c cua c
, tu.,
ng thu
Chuong 3. Bt da
ac d
ay s d
`ng thu

,
,,
. 3.35. Cho a1 , a2 , ..., an l`
a nhung s duong v`
a a = a1 + a2 + + an .
,
`
ng minh ra
ng
Chu
n
X
ai
n
.

2a ai
2n 1
i=1

,,
`ng vo
,ng minh ra
,i moi
,i moi
. 3.36. Chu
a vo
. s nguyn duong n v`
. x R,
,
,
c sau d
ng thu
ng
bt da
u
sin2n x + cos2n x

1
2n1

,
. 3.37. Cho ai v`
a bi (i = 1, 2, ..., n) l`
a c
ac s thu. c sao cho a1 a2 ...
,ng minh
an > 0 v`
a b1 a1 , b1 b2 a1 a2 , ..., b1 b2 ...bn a1 a2 ...an . Chu
`ng
ra
b1 + b2 + ... + bn a1 + a2 + ... + an .
`ng
,ng minh ra
. 3.38. Cho tam gi
ac nhon
. ABC. Chu
sin A + sin B + sin C
1
(tgA + tgB + tgC).
cos A + cos B + cos C
3

,
,
ng th
3.4. Chuyn d
uc Karamata
` v` bt da
,
,
` l`i va
` bt da
ng th
3.4.1. Ham
uc Karamata
, ,
,,
,
,,
Chuyn d` n`
ay mo rng
nhung kt qua cua ph`n truo
c, trong chuyn
.
c lai
ai nim
am l`i v`
ad
ac dinh
am l`i,
d` ta c
ung nha
inh
. l x
. mt
. kh
. v` h`
. h`
,,
,
,
,
dua ra c
ac v du. mt
am co so l`
a nhung h`
am l`i.
. s h`
,
,,
inh
ngha 3.3. H`
am s gi
a tri. thu. c f(x) x
ac d
inh
. trn doan
. [a, b] duo. c
.
goi
a h`
am l`i nu
. l`
f(x + (1 )y) f(x) + (1 )f(y)
,i moi
vo
a [0, 1].
. x, y thuc
. [a, b] v`

,
,c Karamata
ng thu
3.4. Chuyn d
` v` bt d
a

157

,,
H`
am s f(x) duo. c goi
a h`
am l`i cha
. t nu
. l`
f(x + (1 )y) < f(x) + (1 )f(y)
,i moi
vo
a (0, 1).
. x, y thuc
. [a, b], x 6= y v`
: 1. H`
Ch
uy
am s f(x) goi
a h`
am l
om (h`
am l
om cha
. t) trn [a, b] nu f(x)
. l`
,
,
. t). H`nh 3.1 th hin
l`
a h`
am l`i (h`
am l`i cha
am l`i, c`
on h`nh 3.2 th hin
. h`
.
h`
am l
om.

y
C

B
A
A
C
0

H`nh 3.1. H`
am l`i

H`nh 3.2. H`
am l
om

,
. t c
. t
2. H`
am l`i cha
o y ngha h`nh hoc
am s f(x) l`
a l`i cha
. nhu sau: H`
,
,
,
i moi
khi v`
a ch khi vo
a B = (y, f(y)) trn d` thi.
. hai dim A = (x, f(x)) v`
,
,
,,
`
`m giu,a x v`
,i moi
m duoi doan
ay
cua f th` d
im C = (z, f(z)) na
. AB vo
. z na
(h`nh 3.1).
,,
,ng
3. Nu h`
am s f(x) l`
a h`
am lin tuc
. trn doan
. [a, b], th` ngu`oi ta chu
,
,, `
,i moi
ng f(x) l`
minh d
a h`
am l`i khi v`
a ch khi vo
uo. c ra
. x, y [a, b]


x+y
f(x) + f(y)
f

2
2
,
. t khi v`
v`
a n
o l`
a h`
am l`i cha
a ch khi


x+y
f(x) + f(y)
f
<
2
2

158

,
,
,,
,c cua c
, tu.,
ng thu
Chuong 3. Bt da
ac d
ay s d
`ng thu

,
,i tt ca x, y [a, b], x 6= y.
vo
l`
C
o rt nhi`u t`
ai liu
oi v` h`
am l`i, mt
a cun s
ach [11]
. n
. trong s do
,
,

c
o b`
an toi v`
a lit
o.
. k mt
. s tnh cht cua n
,u tnh cht v`
Ta khng di su nghin cu
a c
ac d
am
inh
. l lin quan dn h`
,
,
n nhu sau:
l`i, ta ch d`
ung mt
o sa
inh
. l c
. d
inh
l 3.4. Cho h`
am s f(x) c
od
am bc
ao
. h`
. hai trn [a, b].
.
,
00
l`
Nu d
am bc
oi moi
am d
a h`
am l`i;
ao
o
. h`
. hai f (x) 0 v
. x [a, b], th` h`
,
00
l`
Nu dao
am bc
oi moi
am d
a h`
am l`i
o
. h`
. hai f (x) > 0 v
. x (a, b), th` h`
cha
. t.
: 1. Mt
Ch
uy
am tuyn tnh f(x) = ax + b (x R) l`
a mt
am l`i v`
a
. h`
. h`

c
ung l`
a h`
am l
om.
,
2. Tng hai h`
am l`i (h`
am l
om) l`
a mt
am l`i (h`
am l
om).
. h`
,
,
,
. c l
T`u dinh
o th kim tra c
ac h`
am sau dy l`
a l`i hoa
om:
. l trn ta c
,i x > 0.
1. H`
am s f(x) = x vo
,i 0 < < 1 h`
Ta c
o f 00 (x) = ( 1)x2 , vy
am s n`
ay l
om, c`
on
. vo
,
i < 0 v`
vo
a > 1 h`
am s n`
ay l`i.
,
x
i a > 0, a 6= 1.
2. H`
am s f(x) = a vo
2
00
x
,i moi
cho l`i.
Ta c
o f (x) = a ln a > 0 vo
am s d
a
. x, suy ra h`
,
i x > 0, a > 0, a 6= 1.
3. H`
am s f(x) = lga x vo
1
00
,i a < 1, f(x) l`
,i a > 1
Ta c
o f (x) = 2
, nn vo
a h`
am l`i, c`
on vo
x ln a
n
o l`
a h`
am l
om.
,i x > 0.
4. H`
am s f(x) = x ln x vo
1
Ta c
o f 00 (x) = > 0, nn h`
am f(x) l`
a h`
am l`i.
x
,
,
,,
, tu., v`
,ng dung
Nhung ph`n truo
c ta xet c
ac b. s d
a c
ac u
cua
`ng thu
.
,
n
o. Ph`n n`
ay ta xet c
ac b. s c
o tnh cht g`n nhu vy:
.
,
,
inh
ngha 3.4. Cho hai b. s g`m nhung s thu. c (a1 , a2 , ..., an ) v`
a
.

,
,c Karamata
ng thu
3.4. Chuyn d
` v` bt d
a

159

,i i = 1, 2, ..., n 1. Ta
(b1 , b2 , ..., bn ) sao cho ai ai+1 , bi bi+1 vo
,
`ng b. s (a1 , a2 , ..., an ) tri
n
oi ra
. hon b. s (b1 , b2 , ..., bn ), nu

a 1 b1 ,

a
1 + a2 b1 + b2 ,

(3.5)
...

a
+
a
+

+
a

b
+
b
+

+
b
,

1
2
n1
1
2
n1

a + a + + a = b + b + + b .
1
2
n
1
2
n
. c l`
K hiu
a (a1 , a2 , ..., an )  (b1 , b2 , ..., bn ) hoa
a
. l`

(b1 , b2 , ..., bn ) (a1 , a2 , ..., an ).


Ta xet mt
. s v du:
.

1 1
1 1
1
,i n 2, (1, 0, ..., 0)  ( , , 0, ..., 0)   ( , , ..., ).
1. D thy vo
| {z } | 2 2 {z
n
n
n
}
{z
}
|
n s

n s
n s
,,
a khng b. s
2. B. s (5, 5, 0) v`
a (6, 2, 2) khng so s
anh d
uo. c, ngha l`
,
n`
ao tri
ao.
. hon b. s n`
,
, ,
3. Cho a, b, c l`
a d. d`
ai c
ac canh
cua mt
ac bt k`. Ta c
o th gia
.
. tam gi
,,
ta lun lun c
su a b c. Khi d
o
o


a+b b+c c+a
(a, b, c) 
,
,
.
2
2
2
x1 + x2 + + xn
4. Nu x1 x2 xn v`
ax=
, th`
n
(x1 , x2 , ..., xn )  (x,
| x,{z..., x}).
n s
,
,
`ng x1 + x2 +
,ng minh ra
i k = 1, 2, ..., n 1 ta ch c`n chu
Tht
. vy,
. vo

+ xk kx. Ta c
o
(nk)(x1 + +xk ) (nk)kxk k(nk)xk+1 k(xk+1 + +xn ),

,
nhu vy
. (nk)(x1 + +xk ) k(xk+1 + +xn ). Cng
. k(x1 + +xk )
,
,
,
,
,

c trn, ta nhn
ng thu
v`
ao hai v cua bt da
uo. c n(x1 + + xk )
. d
x1 + + xk kx.
k(x1 + + xn ) = knx. Do d
o

,
,
,,
,c cua c
, tu.,
ng thu
Chuong 3. Bt da
ac d
ay s d
`ng thu

160

inh
l 3.5. Cho f(t) l`
a mt
am l`i trn (a, b) v`
a x1 , x2 , ..., xn , y1 , y2 , ...,
. h`
.

yn (a, b) sao cho (x1 , x2 , ..., xn )  (y1 , y2 , ..., yn ). Khi d


o
f(x1 ) + f(x2 ) + + f(xn ) f(y1 ) + f(y2 ) + + f(yn ).

(3.6)

Nu f(x) l`
a h`
am l
om th`
f(x1 ) + f(x2 ) + + f(xn ) f(y1 ) + f(y2 ) + + f(yn ).
,
,
,c 3.6 goi
,c Karamata1 . Chu
,ng minh bt
ng thu
ng thu
Bt da
a bt da
. l`
,
,
,
,c n`
ng thu
d
ay du. a v`
ao hai mnh
d` phu. tro. sau:
a
.
,
Mnh
d` 3.3. Nu f(t) l`
a h`
am l`i v`
a x1 + x2 = y1 + y2 v
oi (y1 x1
.

x2 y2 ), th` f(y1 ) + f(y2 ) f(x1 ) + f(x2 ).


,ng minh. Mnh
Chu
d
ay l`
a
` n`
.
,
,
ng han
hin nhin, cha
. ta xet
,`,
,
,
truong ho. p nhu h`nh 3.3. Ta thy
`m trn CD, suy ra
doan
. AB na
,
,
trung dim cua AB l`
a E c
ung
,
,
`
m trn trung dim cua CD l`
na
a
,
,
F. Tung d
ay
. cua nhung doan
. n`

A
C

y1 x 1 G

x2 y2

l`
a
H`nh 3.3
f(y1 ) + f(y2 )
,
2
f(x1 ) + f(x2 )
FG =
.
2
,
,
,c trn suy ra di`u c`n chu
,ng minh.
ng thu
T`u c
ac d
a
EG =

, ,
,
inh
ngha 3.5. Phep dich
a vic
. chuyn cua b. s (x1 , x2 , ..., xn ) l`
. thu. c
.
,
,
,,
,
bao to`
hin
ang xi v`
a giam xj mt
an
`ng th`oi t
ai
. d
. d
. luo. ng sao cho vn
,
,
tng cua hai s n`
ay, khi xi xj .
1 Jovan

Karamata (1902-1967): Vin


an hoc
. s to
. Serbia-Montenegro.

,
,c Karamata
ng thu
3.4. Chuyn d
` v` bt d
a

161

,
,n ho,n s 5 v`
s 7 lo
V du:
a ta thu. c hin
o
. Cho b. s (3, 7, 8, 9, 5, 4) khi d
.
,
,
,
phep dich
a giam: 7 2 v`
a t
ang: 5 + 2. Sau khi dich
. chuyn l`
. chuyn b. s
,,
tro th`
anh (3, 5, 8, 9, 7, 4).
, ,
,i phep dich
Theo mnh
d
` 3.3 th` vo
. chuyn cua b. s (x1 , x2 , ..., xn ),
.
,,
dai
ang.
. luo. ng f(x1 ) + f(x2 ) + + f(xn ) khng t
,
,
`, mt
,ng phep dich
Mnh
d
oi nhu
` 3.4. Tu
. b. s (b1 , b2 , ..., bn ) v
. chuyn lin
.
, , ,
,
tip c
o th d
oi b. s (a1 , a2 , ..., an ) khi v`
a ch khi
ua t
(a1 , a2 , ..., an )  (b1 , b2 , ..., bn ).
,
,ng minh. Phu,o,ng ph
,ng minh cua J. Karamata, G.H. Hardy, J.E.
Chu
ap chu
Littlewood v`
a G. Polya.
,
1. i`u kin
a hin
. c`n: i`u kin
. (a1 , a2 , ..., an )  (b1 , b2 , ..., bn ) l`
,
,
,
,
,
nhin sau nhung phep dich
chuyn. Tht
chuyn t`u
.
.
. vy,
. su. dich
,
l`n thu.,c hin
,i (a1 , a2 , ..., an ), sau mi
(b1 , b2 , ..., bn ) to
. chuyn
. phep dich
,,
,,
,
,
, ngha l`
ta nhn
c do
a tri
`u
. duo. c b. s tri
. hon b. s truo
. hon b. s d
tin (b1 , b2 , ..., bn ).
,
2. i`u kin
u:
. d
,
`ng nu (a1 , a2 , ..., an )  (b1 , b2 , ..., bn ), th` t`u, b.
,ng minh ra
Ta phai chu
,
`ng
,ng minh ba
,i (a1 , a2 , ..., an ). Ta chu
s (b1 , b2 , ..., bn ) ta c
o th tin to
,
,
,
quy nap
an hoc
. to
. theo s luo. ng cua b. s.
,
,
,i n = 1, kha
ng d
a hin nhin.
a) Vo
inh
. l`
,i d
c
ng: vo
b) Cho mt
od
i`u sau du
i`u kin
. s bt k` k, k n 1 da
.
,
,
`

ng
(a1 , a2 , ..., ak )  (b1 , b2 , ..., bk ) thoa m
an, th` t`u b. s (b1 , b2 , ..., bk ) ba
,
,
,
,

,ng
i b. s (a1 , a2 , ..., ak ). Ta phai chu
c
ac phep dich
o th tin to
. chuyn c
,
,i k = n. Trong b. s (b1 , b2 , ..., bn ) ta
ng dinh
ng vo
minh kha
ung du
. trn c
,
,
,i b1 , bn (b1 l`
,n nht trong
lin tuc
a s lo
. chuyn di vo
. thu. c hin
. phep dich
,
, ,
,
,
v bn phai cua cua tt ca
b. s, bn l`
a s nho nht trong b. s). Khi d
o

,
,
,,
,c cua c
, tu.,
ng thu
Chuong 3. Bt da
ac d
ay s d
`ng thu

162

,
,
,
,c trong (3.5) d`u t
,i mt
ng thu

bt da
ang, ngha l`
a tin to
ao d
im n`
o
. th`oi d
,
,
,,
,
,
c tro th`
c
ng thu
ng thu
c
o mt
anh da
a
. bt d
a1 + a2 + + a` = b1 + b2 + + b` ,

,
,,
,i cua b1 .
od
a mt
y b1 l`
. bin mo
,,
, ,
,c n`
ng thu
T`u da
ay h. (3.5) tro th`
anh hai ph`n

a`+1 b`+1 ,
a1 b 1 ,

a
a + a b + b ,
`+1 + a`+2 b`+1 + b`+2 ,
1
2
2
1
...
...

a`+1 + + an = b`+1 + + bn .
a1 + + a` = b1 + + b`
,
,
,

,i (a1 , a2 , ..., a` )
Theo gia thit quy nap,
o th tin to
. t`u b. s (b1 , b2 , ..., b` ) c
,
,
,i (a`+1 , ..., an ) (v` ta lun c
o th tin to
o
v`
a t`u (b`+1 , b`+2 , ..., bn ) c
(a1 , a2 , ..., a` )  (b1 , b2 , ..., b` ) v`
a (a`+1 , ..., an )  (b`+1 , b`+2 , ..., bn )). V`
,
,
,i
nhung di`u trn ta thy ngay b. s (b1 , b2 , ..., bn ) c
o th tin to
,ng minh.
(a1 , a2 , ..., an ).
o l`
ad
i`u c`n chu
, ,
,,
ng minh dinh
chu
hai mnh
d` trn.
. l 3.5, ta su dung
.
.
, ,
,
V` (a1 , a2 , ..., an )  (b1 , b2 , ..., bn ) nn ta c
o th thu. c hin
. phep chuyn
,
,i b. s (a1 , a2 , ..., an ) (mnh
dich
d
` 3.4),
. t`u b. s (b1 , b2 , ..., bn ) tin to
.
,
,
,
i c
Vo
ac phep chuyn dich
ang (mnh
. tng f(x1 ) + f(x2 ) + + f(xn ) t
.

d
` 3.3).

,
,
,c Fuchs). Cho f(x) l`
ng thu
H. qua 3.1 (Bt d
a h`
am l`i, p1 , p2 , ..., pn l`
a
a
,
,
,
ng s duong, x1 x2 xn , y1 y2 yn sao cho
nhu
k
X
i=1

pi x i

k
X
i=1

n
n
X
X
,
pi yi v
oi 1 k n 1 v`
a
pi x i =
pi y i .
i=1

i=1

Khi d
o
p1 f(x1 ) + p2 f(x2 ) + + pn f(xn ) p1 f(y1 ) + p2 f(y2 ) + + pn f(yn ).

,
,c Karamata
ng thu
3.4. Chuyn d
` v` bt d
a

163

. t p1 + p2 + + pn = 1 v`
Nu ta d
a
a
y1 = y2 = ... = yn = p1 x1 + p2 x2 + + pn xn ,
,
,
,
,c Fuchs2 tro, th`
,c Jensen3 .
ng thu
ng thu
th` bt da
anh bt d
a
,
,
,c Jensen). Cho f(x) l`
ng thu
H. qua 3.2 (Bt d
a mt
am l`i trn d
oan
a
. h`
.
,
,
ng s trn d

[a, b], x1 , x2 , ..., xn l`


a nhu
o
an
[a,
b],
p
,
p
,
...,
p
l`
a
nh
u
ng
s
.
1
2
n
,
,,

duong bt k` c
o tng ba
`ng 1. Khi d
o
f(p1 x1 + + pn xn ) p1 f(x1 ) + + pn f(xn ).

,
,
,
,
,
,c trn suy ra h`
,c ni
ng thu
ng thu
T`u nhung bt da
ang loat
ac bt da
. c
,
,
,c Cauchy, Chebyshev, ...
ng thu
ting kh
ac nhu bt da
,
,,
,
ng th
3.4.2. Su dung
bt da
uc Karamata
.
,
,
,
,ng s du,o,ng bt k` a, b v`
V du. 3.39. V
oi nhu
a c ta c
o bt d
ng thuc
a
1
1
1
1
1
1
+
+

+
+ .
a+b b+c c+a
2a 2b 2c
,
, ,
,
,
`,i giai. V` vai tr`
Lo
o cua a, b, c nhu nhau, nn ta c
o th gia thit a b c.
,,
,
d thy nhung b. s (2a, 2b, 2c)  (a + b, a + c, b + c). Su dung
Khi do
.
,
1 ,
,c Karamata cho h`
i hai b. s trn suy ra di`u c`n
ng thu
bt da
am l`i vo
x
,ng minh.
chu

,
,,
V du. 3.40 (` thi Olympic to
an khu vu. c Th
ai B`nh Duong 1996). Cho
,
,
,ng d
a, b v`
a c l`
a nhu
ai c
ac canh
cua mt
ac. Chung minh ra
`ng khi
. d`
.
. tam gi
,
,
ta c
ng
d
o bt d
ng thuc sau d
o
a
u

a + b c + b + c a + c + a b a + b + c.

,
,
, ,
,
`,i giai. Do vai tr`
,c, nn ta c
ng thu
Lo
o a, b, c nhu nhau trong bt da
o th gia
,
suy ra (a + b c, c + a b, b + c a)  (a, b, c).
thit a b c. T`u d
o
2 Lazarus
3 Johan

Immanuel Fuchs (1833-1902): Nh`


a to
an hoc
. Ba Lan.
Ludwig William Valdemar Jensen (1859-1925): Nh`
a to
an hoc
. an Mach.
.

,
,
,,
,c cua c
, tu.,
ng thu
Chuong 3. Bt da
ac d
ay s d
`ng thu

164

,,
,c Karamata cho h`
ng thu
Su dung
a hai b. s trn se cho
am l
om x v`
a
. bt d
,
kt qua.

,
,,
, ,
V du. 3.41 (IMO 2000). V
oi nhu
ng s duong a, b, c > 0 thoa m
an d
i`u kin
.
,
,
,
`
ng
abc = 1. Chung minh ra
ng bt d
ng thuc sau du
a




1
1
1
b1+
c1+
1.
a1+
b
c
a
,
,,
`,i giai. T`u, di`u kin
Lo
ac s duong x, y, z sao cho
. abc = 1, suy ra t`n tai
. c
,
x
y
z
,c da
ng thu
bt da
cho c
a = , b = , c = . Khi do
o dang
.
y
z
x
(x y + z)(y z + x)(z x + y) xyz.

(3.7)

,,
,,
,
, ,
Xet tru`ong ho. p ca ba th`ua s x y + z, y z + x, z x + y d`u duong.
Ta ly lgarit hai v
ln(x y + z) + ln(y z + x) + ln(z x + y) ln x + ln y + ln z.
,
,
,
,
,c nn ta gia thit x y z.
ng thu
Vai tr`
o cua x, y, z nhu nhau trong bt da
,,
ta c
Khi do
o c
ac b. s (y z + x, x y + z, z x + y)  (x, y, z). Su dung
.
,
,
,
,

ng
ng thuc Karamata voi h`
am l
om ln x v`
a hai b. s trn suy ra bt da
bt da
,
, ,
,,
,
,c (3.7) vo
,c (3.7). Ban
,i c
ng thu
ac tru`ong ho. p
thu
ng minh bt d
oc
a
. d
. tu. chu

c`
on lai.
.

,
,c Szeg). Cho (x) l`
ng thu
V du. 3.42 (Bt da
a h`
am l`i v`
a a1 a2 ...

a2n1 , khi d
o

(a1 ) (a2 ) + (a3 ) + (a2n1 ) (a1 a2 + a3 + a2n1 ).


,
,
`,i giai. Ta d
,c Szeg4
. t a = a1 a2 + a3 a4 + + a2n1 . Bt da
ng thu
Lo
a
,
dua v` dang
.
(a1 ) + (a3 ) + + (a2n1 ) (a) + (a2 ) + + (a2n2 ).
4 G
abor

,,
Szeg (1895-1985): Nh`
a to
an hoc
. ngu`oi Hungari.

,
,c Karamata
ng thu
3.4. Chuyn d
` v` bt d
a

165

,
,
,ng minh
Nhu vy
on chu
. ta ch c`
(a1 , a3 , ..., a2n3 , a2n1 )  (a2 , a4 , ..., a2n4 , a2n2 , a),
,
,
,
`ng nhau v`
,i
l`
nhung d
a hin nhin v` tng c
ac s ba
a a2k1 a2k vo
i`u do

moi
. k = 1, 2, ..., n.

,
V du. 3.43 (IMO 1999). Cho n 2. H
ay x
ac d
`ng s C nho nht sao
inh
. ha
,
,
cho bt da
ng thuc
!4
n
X
X
2
2
xi xj (xi + xj ) C
xi
1i<jn

i=1

,
,
,
ng v
du
oi tt ca c
ac s thu. c x1 , x2 , ..., xn 0.

,
`,i giai. Ta xet tru,`o,ng ho.,p n = 2. a
. t x1 = m + h v`
Lo
a x2 = m h (ngha
x1 x2
x1 + x2

v`
ah=
), khi do
l`
am=
2
2
1
x1 x2 (x21 + x22 ) = 2(m4 h4 ) 2m4 = (x1 + x2 )4
8
,
,
,
,
c xay ra khi v`
ng thu
da
a ch khi h = 0, ngha l`
a x1 = x2 .
,`,
,
Xet truong ho. p n > 2, ta c
o x1 + x2 + + xn > 0 (v` nu x1 + x2 + +
,
,
,c l`
ng thu
bt da
a hin nhin).
xn = 0 th` moi
. xi = 0, i = 1, 2, ..., n. Khi do
xi
,i i = 1, 2, ..., n, khi do
. t ai =
a1 +a2 + +an = 1.
vo
a
x1 + x2 + + xn
,
,c c`n chu
,ng minh v`
ng thu
Thay xi = ai (x1 + x2 + + xn ) v`
ao bt da
a r
ut

gon
anh dang:
. th`
.

ai aj (a2i + a2j ) C.
1i<jn
,
,
,,
,c trn, ta nhn
ng thu
Khai trin v tr
ai v`
a nh
om lai
. trong bt da
. duo. c
n
X
i=1

a3i (a1 + + ai1 + ai+1 + + an) =

n
X

a3i (1 ai ).

i=1



1
`
. t) trn 0, , v`
Xet h`
am f(x) = x (1 x) = x x l`
a h`
am li (nga
2


1
f 00 (x) = 6x 12x2 = 6x(1 2x) > 0 trn 0,
.
2
3

166

,
,
,,
,c cua c
, tu.,
ng thu
Chuong 3. Bt da
ac d
ay s d
`ng thu

,
,
,
,c nhu, nhau, khng mt tnh tng qu
ng thu
at
V` vai tr`
o cua ai trong bt d
a
, ,
ta c
o th gia thit a1 a2 an .


,
1
1 1
ng
Nu a1 , th` ta c
o
, , 0, ..., 0  (a1 , a2 , ..., an ). Theo bt da
2
2 2
,c Karamata, ta c
thu
o
 
 
1
1
1
+f
+ f(0) + + f(0) = .
f(a1 ) + f(a2 ) + + f(an ) f
2
2
8


1
Nu a1 > , th` 1 a1 , a2 , ..., an 0, 21 . Ta xet hai b. s g`m n 1
2
,
,,
,c Karamata
ng thu
ph`n tu v`
a (1 a1 , 0, ..., 0)  (a2 , a3 , ..., an ), theo bt d
a

suy ra

f(a2 ) + + f(an ) f(1 a1 ) + f(0) + + f(0).


,
,,
,,
,
,c trn vo
,i f(a1 ) v`
ng thu
Cng
ao hai v bt da
a su dung
. v`
. tru`ong ho. p n = 2
ta c
o
1
f(a1 ) + f(a2 ) + + f(an ) f(a1 ) + f(1 a1 ) .
8
,
,
,
`ng nhau v`
,c xay ra khi v`
ng thu
a
a ch khi hai bin ba
a n 2 bin kh
ac d`u
`
ng 0.
ba

,
,

ng th
3.4.3. Mt
uc khac
. s bt da
,
,
,
, 12
12
12
V du. 3.44. T`m gi
a tri. cu. c d
ai
. cua biu thuc a + b + c , nu
1
1 a, b, c 1 v`
aa+b+c= .
2
,
,
`ng h`
`,i giai. Ta ch
Lo
u y ra
am f(x) = x12 l`
a lin tuc
a l`i trn khoang [1, 1],
. v`
,
,
,
v` f 00 (x) = 132x10 0 trn (1, 1). Vai tr`
o cua a, b, c nhu nhau trong biu
, ,
,
,c, nn khng mt tnh tng qu
thu
at ta c
o th gia thit 1 c b a 1,
1
`ng (1, 1 , 1)  (a, b, c).
,ng minh ra
,i a + b + c = , ta se chu
c`
ung vo
2
2
,
,
`ng nhau, ho,n nu,a
Tht
. vy,
. d thy tng cua hai b. ba d`u ba
1 a v`
a

1
1
1
= 1 c = a + b.
2
2
2

,
,c Karamata
ng thu
3.4. Chuyn d
` v` bt d
a

167

,
,c Karamata ta c
ng thu
Theo bt da
o
1
1
a12 + b12 + c12 = f(a) + f(b) + f(c) f(1) + f( ) + f(1) = 2 + 12 .
2
2
1
1
,
,,
Vy
a tri. cu. c dai
a 2 + 12 d
a c = 1.
at
uo. c khi a = 1, b = v`
. gi
. l`
. d
2
2
,
,
,
,c tap
,ng minh thoa m
Nhung v du. sau d
ut khi chu
an
y phu
. hon mt
. ch
,
,
,
,
c Karamata.
ng thu
nhung d
i`u kin
. cua bt da

,
V du. 3.45. Cho x1 , x2 , ..., xn [ 6 , 6 ]. Chung minh ra
`ng
cos(2x1 x2 ) + cos(2x2 x3 ) + + cos(2xn x1 ) cos x1 + + cos xn .

,
,
,

`,i giai. V` h`
Lo
am cos x l
om trn doan
i`u kin
.
. [ 2 , 2 ], ta ch c`n kim tra d
, ,

(3.5) l`
ad
u
v
o
i
hai
b

s
(2x

x
,
...,
2x

x
)
v`
a
(x
,
x
,
...,
x
).
Ta
s
a
p

1
2
n
1
1 2
n
.
xp lai
ay th`
anh c
ac b. s (2xm1 xm1 +1 , ..., 2xmn xmn +1 )
. hai b. s n`
,
,
,
,c:
ng thu
v`
a (xk1 , xk2 , ..., xkn ) thoa m
an nhung bt da
2xm1 xm1 +1 2xm2 xm2 +1 ... 2xmn xmn +1 ,
xk1 xk2 ... xkn
,
,,
,
,
,
,i quy u,o
,c xn+1 = x1 ). i`u n`
(vo
ay thu. c hin
. duo. c. Hon nua nhung ta thy
`ng 2xm1 xm1 +1 2xk1 xk1 +1 xk1 v`
a
ra
(2xm1 xm1 +1 ) + (2xm2 xm2 +1 ) (2xk1 xk1 +1 ) + (2xk2 xk2 +1 )
xk1 + xk2 .
,
,
,
,,
, ,
, nht khng nho ho,n tng `
Tuong tu. tng ` s d
`u tin cua b. s thu
,
, ,
, hai. Thu.,c t n
s cua b. s thu
o khng nho hon (2xk1 xk1 +1 ) + +
,
, ,
,
,
(2xk` xk` +1 ), nhung tng n`
ay khng nho hon xk1 +xk2 + +xk` (c
o th
,
`ng quy nap).
,ng minh ba
Nhu vy,
chu
.
. (2xm1 xm1 +1 , ..., 2xmn xmn +1 ) 
(xk1 , xk2 , ..., xkn ).

,ng s du,o,ng. Chu,ng minh ra


V du. 3.46. Cho a1 , a2 , ..., an l`
a nhu
`ng
a31 a32
a3
+
+ + n a21 + a22 + + a2n .
a2 a3
a1

168

,
,
,,
,c cua c
, tu.,
ng thu
Chuong 3. Bt da
ac d
ay s d
`ng thu

,
,
,
`,i giai. Thu.,c hin
,c c`n chu
,ng
ng thu
Lo
a vit lai
i bin xi = ln ai v`
a
. d
. bt d
minh
e3x1 x2 + e3x2 x3 + + e3xn x1 e2x1 + e2x2 + + e2xn .

,
,,
,
,
,, ,
T`u d
an to`
an tuong tu. nhu b`
ai to
an truo
c voi h`
am l`i f(x) = ex
y l lun
. ho`
v`
a b. s (3x1 x2 , ..., 3xn x1 )  (2x1 , 2x2 , ..., 2xn ).

,
,
,

ng th
3.4.4. Nhung bt da
uc trong tam giac
,
,
,
,c Karamata ta c
,c
,i bt da
ng thu
ng thu
Vo
o th s
ang tao
ang loat
. h`
. bt da
,,
,
,i nhau, sau dy ch
v` c
ac ph`n tu trong mt
ac vo
ung ta dua ra mt
. tam gi
.
,
,
,
,`,
,
`

ng thuc m`
ng c
t`m ra ba
s bt d
a nguoi ta da
ach l`
am nhu trn. cho
a
,
,

thng nht ta k hiu


a nhung g
oc cua tam gi
ac; a, b, c l`
a d. d`
ai
. , , l`
,
,,
a+b+c
, , ,
,
i g
l`
a nua chu vi cua tam
c
ac canh
tuong ung vo
oc trn; p =
.
2
gi
ac; x = p a, y = p b, z = p c.
,
V du. 3.47. Cho tam gi
ac nhon
`ng
. ABC, chung minh ra
1 < cos + cos + cos

,
,
v`
a x
ac d
ao th` c
od
ng thuc?
inh
a
. khi n`

3
2

,
,
,

`,i giai. Khng mt tnh tng qu

Lo
at ta gia thit . Khi d
o
3
,

2
,
,
,

c > , > + (= )
ng thu
v`
a . T`u nhung bt d
,
a
3
2
3
3
suy ra
 
 
, , 0  (, , ) 
, ,
.
h i
2 2
3 3,3

,c Karamata ta c
ng thu
o
V` f(x) = cos x l`
a h`
am l
om trn 0, , theo bt da
2


1=f
+f
+ f(0)
2
2
f() + f() + f() = cos + cos + cos


 3
f
+f
+f
= .
3
3
3
2

,
,c Karamata
ng thu
3.4. Chuyn d
` v` bt d
a

169

,
,
`ng xay ra v` trong mt
,c v tr
ng thu
Bt da
ai khng c
o du ba
ac
. tam gi
,
,
,
,
,
`
c v phai du ba
ng thu
ng xay ra khi v`
khng th c
o hai g
oc vung. Bt d
a
a
,
ch khi ABC l`
a tam gi
ac d`u.

,
,
,c v` c
ng thu
I. Nh
ung bt da
ac canh:
.
,
,
, ,
,
1. Vai tr`
o cua c
ac canh
cua mt
ac nhu nhau, nn ta c
o th gia thit
.
. tam gi
,
nhung quan h. sau du
ng cho tam gi
a b c. Khi do
ac bt k` (ban
. doc
.
,
,
tu. kim tra).


p p p
2p 2p 2p
, ,
, ,
.
(p, p, 0)  (a, b, c) 
, (p, 0, 0)  (z, y, x) 
3 3 3
3 3 3
,
,c sau:
ng thu
Ta c
o c
ac bt da
1
a 2 + b 2 + c2
1
a)
< ;
3
(a + b + c)2
2
1
bc + ca + ab
1
b) <
;
4
(a + b + c)2
3

c) p < p a + p b + p c < 3p;


p
p
p

d) a(p a) + b(p b) + c(p c) 2p;


1
(b + c)(c + a)(a + b)
8
e) <

;
4
(a + b + c)3
27
9
1
1
1
f)
+
+
.
p
pa pb pc


,
`ng (a, b, c)  a + b , c + a , b + c , ta c
2. D kim tra thy ra
o
2
2
2
8abc (b + c)(c + a)(a + b).
,i (2x, 2y, 2z) (a, b, c) ta c
3. Vo
o
a) 8xyz abc;
2xyz
abc
b)

;
xy + yz + zx
ab + bc + ca
c) 32xyz(xy + yz + zx) abc(ab + bc + ca);
,
,c v` c
,ng bt d
ng thu
II. Nhu
ac g
oc:
a

170

,
,
,,
,c cua c
, tu.,
ng thu
Chuong 3. Bt da
ac d
ay s d
`ng thu

, ,
ta c
Ta c
o th gia thit . Khi d
o quan h.
o
 
(, 0, 0)  (, , ) 
, ,
.
3 3 3
,,
,i k < 0; l`
,i 0 < k 1 trn
1. Su dung
h`
am f(x) = sink x l`
a l`i vo
a l
om vo
.
,

,c sau du
,i k 2 trn [0, ]. Nhu,ng bt d
ng thu
ng
[0, ] v`
a l`i vo
a
4
k
31+ 2
,i k < 0, sink + sink + sink
;
a) Vo
2k
k
31+ 2
,
k
k
k
i 0 < k 1, 0 < sin + sin + sin
b) Vo
;
2k
3

,i k < 0, k sink + sink + sink ;


c) Vo
2
2
2
2

3
,
k
k
i 0 < k 1, 1 < sin
d) Vo
+ sin
+ sink k ;
2
2
2
2
,,
2. H`
am s f(x) = lg sin x l`
a h`
am l
om trn (0, ), o dy lg l`
a k hiu
am
. h`
,
,
,
,
c sau d
ng thu
ng:
lgarit co s 10. Nhung bt da
y d
u

3 3
0 < sin sin sin
.
8

,i k < 0; l`
,i 0 < k 1 trn [0, ] v`
a
3. H`
am s f(x) = cosk x l`
a l`i vo
a l
om vo
2
,

,c sau du
,i k 2 trn [0, ]. Nhu,ng bt da
ng thu
ng
l
om vo
4
k
31+ 2

,i k < 0,
a) Vo
cosk + cosk + cosk ;
k
2
2
2
2
k

31+ 2
,i 0 < k 1, 2 < cosk + cosk + cosk
b) Vo
;
2
2
2
2k
,

,c sau dy
ng thu
4. H`
am s f(x) = lg cos x l`
a h`
am l
om trn (0, ). Bt da
2
ng:
du

3 3
0 < cos cos cos
.
2
2
2
8

` tp
Bai
.
. 3.48. Nu n 2 l`
a mt
a 0 < a1 < a2 < ... < a2n+1 l`
a
. s nguyn v`

, ,
,,
,
`i b`
3.5. Go. i y v`
a tra lo
ai tp
. chuong 3

171

,
,
`ng
,ng minh ra
nhung s thu. c, chu

a1

a2 +

a3

a2n + n a2n+1 <

< n a1 a2 + a3 a2n + a2n+1 .

,
,
,,
,ng minh bt d
,c sau d
,i x, y, z l`
ng thu
ng vo
. 3.49. Chu
a nhung s duong
a
u
r
r
r
xy
xz
zy
xy xz zy
+
+

+
+
.
z2
y2
x2
z2
y2
x2
,
,,
`ng
,ng minh ra
. 3.50. Cho a, b, c, d l`
a nhung s duong. Chu
a4 + b4 + c4 + d4 + 2abcd a2 b2 + a2 c2 + a2 d2 + b2 c2 + b2 d2 + c2 d2 .
,
,,
`ng
,ng minh ra
. 3.51. Cho a1 , a2 , ..., an l`
a nhung s duong. Chu






a22
a2n
a21
1+
1 +
.
(1 + a1 )(1 + a2 ) (1 + an ) 1 +
a2
a3
a1
,
,ng minh bt d
,c Karamata c
ng thu
. 3.52. Chu
o trong
a
. s:

m1 f(x1 ) + m2 f(x2 ) + + mn f(xn ) m1 f(y1 ) + m2 f(y2 ) + + mn f(yn ),


,,
o dy f l`
a mt
am l`i v`
a hai b. s (x1 , x2 , ..., xn ) v`
a (y1 , y2 , ..., yn ) c`
ung
. h`
,
,
,
don diu
a thoa m
an h. sau
. giam v`

m1 x1 m1 y1 ,

m1 x1 + m2 x2 m1 y1 + m2 y2 ,
...

m1 x1 + + mn1 xn1 m1 y1 + + mn1 yn1 ,

m x + + m x = m y + + m y .
1 1
n n
1 1
n n

(mi R).

, ,
,,
,
` tra l`oi bai
` tp
3.5. Go. i y
va
. chuong 3
, ,,
,
ln a ln b. Nhu vy
. 3.9. Gia su a b, khi do
a (ln a, ln b) d`ng
. (a, b) v`
,
, tu.,. T`u, mnh
suy ra
thu
d
o a ln a + b ln b a ln b + b ln a. T`u do
` 3.1, ta c
.
a b
b a
a b a b .

,
,
,,
,c cua c
, tu.,
ng thu
Chuong 3. Bt da
ac d
ay s d
`ng thu

172

,
,
,,
,c c`n chu
,ng minh tu,o,ng
ng thu
. 3.10. Do a, b l`
a nhung s duong, bt d
a
,,
,i
duong vo
a+b

a+b
a b ba .
2
,
, ,
,
a+b
ng minh bt da
,c trn ta ch c`n
. t kh
ng thu
ab, nn d
Ma
ac,
chu
2
,
,ng minh bt d
,c sau:
ng thu
chu
a
 a+b
ab
a b ba .
,
,
,,
,,
,ng minh bt d
,c aa bb ab ba , d
,i chu
ng thu
Tuong duong vo
a kt qua
a
y l`
,
cua b`
ai 3.9.
, ,,
hai b. s (a, b, c) v`
. 3.11. Gia su a b c. Khi do
a (a2 , b2 , c2 ) l`
a d`ng
,,
,
,
tu. . Su dung
thu
mnh
d` 3.2 ta c
o
.
.
a3 + b3 + c3 aa2 + bb2 + cc2 ,
a3 + b3 + c3 ab2 + bc2 + ca2 ,
a3 + b3 + c3 ac2 + ba2 + cb2 .

,
,
,,
,c trn ta nhn
,ng minh.
ng thu
Cng
ac bt da
uo. c kt qua c`n chu
. theo v c
. d
,
,,
,c Cauchy cho 5 s du,o,ng:
ng thu
. 3.12. Su dung
bt da
.
a15 , a15 , b10 , b10 , b10 .
,
,,
,c Cauchy cho 7 s du,o,ng: a21 , a21 , a21 ,
ng thu
. 3.13. Su dung
bt da
.
a21 , b14 , b14 , b14 .
,
, , ,,
,c di xu
,ng theo a, b, c nn ta c
ng thu
. 3.19. Do bt d
o th gia su
a
1
1
1
a + b c + a b + c. Do do

a b c. Khi do

.
b+c
c+a
a+b
Theo dinh
. l 3.1:
a3
b3
c3
a3
b3
c3
+
+

+
+
,
a+b b+c c+a
b+c c+a a+b
a3
b3
c3
a3
b3
c3
+
+

+
+
.
c+a a+b b+c
b+c c+a a+b

, ,
,,
,
`i b`
3.5. Go. i y v`
a tra lo
ai tp
. chuong 3

173

,
,,
,c trn, r`i chia cho 2, ta nhn
ng thu
Cng
a
. theo v hai bt d
. duo. c


b3
c3
a3
1 a3 + b3 b3 + c3 c3 + a3
+
+
+
+
.

2
a+b
b+c
c+a
b+c c+a a+b
,
x2 + y2
x 3 + y3
,c
ng thu
= x2 xy + y2
, ta c
o
Do bt d
a
x+y
2


a2 + b2 + c2
1 a2 + b2 b2 + c2 c2 + a2
=
+
+
2
2
2
2
2
 3

3
3
3
3
1 a +b
b +c
c + a3

+
+
2
a+b
b+c
c+a

a3
b3
c3
+
+
.
b+c c+a a+b

, ,,
1
1
1

. 3.20. Gia su 0 < a b c. Khi d

v`
a bc
o
b
+
c
c
+
a
a
+
b


1
1
1
hai b. ba
ca ab. Do do
,
,
v`
a (bc, ca, ab) l`
a nghich
.
b+c c+a a+b
,
,
,
tu. . Theo d
dao thu
inh
. l 3.1
ca
ab
ab
bc
ca
bc
+
+

+
+
,
b+c c+a a+b
b+c c+a a+b
bc
ca
ab
ca
ab
bc
+
+

+
+
.
b+c c+a a+b
b+c c+a a+b
,
,,
,c trn, ta nhn
ng thu
Cng
a
. theo v hai bt d
. duo. c


bc
ca
ab
2
+
+
a + b + c.
b+c c+a a+b
, ,,
. 3.21. Gia su x1 x2 ... xn > 0, suy ra x21 x22 ... x2n v`
a

1
1
1
1
1
1
hai b. s (x21 , x22 , ..., x2n ) v`

...
. Do do
a
, , ...,
x1
x2
xn
x x
xn
,
, 1 2
,
,

p dung
l`
a nghich
aa
d
inh
. dao thu tu. v`
. l 3.1 se cho kt qua.
.
, ,,
hai b. s (an , bn , cn ) v`
. 3.22. Gia su a b
a

 c. Khi do
1
1
1
, tu.,.
,
,
l`
a d`ng thu
b+c c+a a+b
, ,
. 3.23. Ban
oc
. d
. tu. giai.

174

,
,
,,
,c cua c
, tu.,
ng thu
Chuong 3. Bt da
ac d
ay s d
`ng thu

, ,,
hai b. s (A, B, C) v`
. 3.24. Gia su A B 
C. Khi d
a
o

1
1
1
,
,
tu. v`
p dung
,
,
d`ng thu
aa
mt
. l 3.1 se
.
. s l`n dinh
pa pb pc
,
cho kt qua.
,
, ,,
,c Chebyshev cho
ng thu
p dung
. 3.34. Gia su a b c d v`
aa
bt d
a
.
,
b. ba s do,n diu
mi
ac s trn r`i cng
ung lai.
. giam trong c
. ch
.
, , ,,
1
1

. 3.35. Ta c
o th gia su a1 a2 an . Khi do
2a a1
2a a2
1
,i quy u,o
,c an+i = ai . Vo
,i m = 0, 1, 2, ..., n 1, theo dinh

. Vo
.
2a an , ,
l 3.1, ta nhn
d
u
o
c

.
.
n
n
X
X
am+i
ai

.
2a ai
2a ai
i=1

i=1

,
Pn
nai
ai
, , Pn
,c n`
ng thu
Cng
n bt d
i=1
.
ay, ta nhn
a
uo. c i=1
.
. d
2a ai
2a ai
a
1 1 ai
,
,,
T`u
= +
, ta nhn
uo. c
. d
2a ai
2 2 2a ai
n
n
X
n 1 X ai
ai
+
n
.
2
2
2a ai
2a ai
i=1
i=1
,
,
,c c`n chu
,ng minh.
ng thu
T`u dy suy ra bt d
a
,,
,i a, b 0 v`
. p s (a, b) v`
. 3.36. Vo
a n l`
a s nguyn duong, ca
a (an1 , bn1 )
,
,
,
,
,,
c Chebyshev, ta nhn
tu. . Do d
ng thu
, theo bt da
l`
a d`ng thu
uo. c
o
. d

1
(a + b)(an1 + bn1 ).
2
,
,,
,c Chebyshev, cui c`
. p lai
ng thu
p dung
ung nhn
La
bt d
a
. duo. c
. vic
. a
.
an + bn

an + bn

1
(a + b)n .
2n1

,
`ng c
,c trn se
. t a = sin2 x v`
ng thu
Ba
ach da
a b = cos2 x v`
a thay v`
ao bt d
a
,
,ng minh.
cho kt qua c`n chu
bk
,i
. t ck =
. 3.37. a
v`
a dk = (c1 1) + (c2 1) + + (ck 1) vo
ak

, ,
,,
,
`i b`
3.5. Go. i y v`
a tra lo
ai tp
. chuong 3

175

,
,
,c Cauchy v`
ng thu
cho, ta c
1 k n. Theo bt d
a gia thit d
o
a
a

c1 + c2 + + ck
k c1 c2 ...ck 1,
k
,
suy ra dk 0. Cui c`
t`u do
ung ta c
o
(b1 + b2 + + bn ) (a1 + a2 + + an )
= (c1 1)a1 + (c2 1)a2 + + (cn 1)an
= d1 a1 + (d2 d1 )a2 + + (dn dn1 )an
= d1 (a1 a2 ) + d2 (a2 a3 ) + + dn an 0.
, ,,
,
. 3.38. Gia su 0 < A B C < 2 . Xet hai b. (tgA, tgB, tgC) d
on
,
,
,
,c
ng thu
p dung
diu
ang, (cos A, cos B, cos C) l`
ad
aa
bt d
on diu
a
. t
. giam v`
.
Chebyshev.

, ,
CHUONG

,,
`
Phuong tr`nh ham

,,
,
th nhung gia
tr.i d
4.1. Phuong phap
i s . . . . . . . . . . . . . . . . . . . . . . . . . . . . . . . . .
,
,,
di
4.2. Phuong phap
..........................................
m bt dng
.
,
da
................................
4.3. Chuyn d
uc giao hoan
` v` cac
th

4.3.1. inh
. ngha . . . . . . . . . . . . . . . . . . . . . . . . . . . . . . . . . . . . . . .
,c Chebyshev . . . . . . . . . . . . . . . . . . . . . . . . . . . . . . .
4.3.2. a thu
,
,c giao ho
4.3.3. B`
ai to
an tng qu
at v` c
ac da thu
an . . . . . . .

, ,
,,
,
` tra l`oi bai
` tp
4.4. Go. i y va
. chuong 4 . . . . . . . . . . . . . . . . . . . . . . . . . . . . . . . . . . . . . .

176
187
194

194
197
200
203

,
,
,,
,,
,,
Phuong tr`nh h`
am l`
a phuong tr`nh m`
a n s l`
a c
ac h`
am s. Giai phuong
,
,
,
,,
cho
tr`nh h`
am ngha l`
a t`m tt ca nhung h`
am s thoa m
an phuong tr`nh da
,,
,,
,

truoc. C
o rt nhiu dang
ac nhau nhu trong cun s
ach [5]
. phuong tr`nh kh
lit
da
. k.

,,
,
th nhung gia
tr.i d
4.1. Phuong phap
i s
, ,
,,
,,
,
Phuong ph
ap chung d giai c
ac phuong tr`nh h`
am l`
a khng c
o. Nhung
,
,
,,
,,
,
c
o phuong ph
ap th c
ac gi
a tri. bin v`
ao phuong tr`nh d giai, thu. c cht
,
,,
cho khi ta thay gi
dy c
ung l`
a dang
a tri. bin
. bt bin cua phuong tr`nh da
,
,,
,
,,
,,
d
ng. C
kh
ac nhau phuong tr`nh vn
ung nhu chuong truo
c, khi ta thay di
u
,
,
,
,c d
,ng khng d
vai tr`
o cua c
ac bin cho nhau th` d
a tri. v`
a t`u
a thu
i xu
i gi

,,
,ng gi
4.1. Phuong ph
ap th nhu
a tri. di s

177

, ,
,c d
,ng v`
da
t`m ra nhung u
do
ng dung
a cho tnh bt
i xu
. rt hay cho da thu
,
,
,
,
c di xu
ng. Trong muc
i da thu
bin di vo
ay ta c
ung lo. i dung
tnh bt
. n`
.
,
,
,
, ,
,
c khi thay c
ng thu
bin cua da
ac gi
a tri. cua bin kh
ac nhau d
giai.
,
,,
V du. 4.1. Giai phuong tr`nh h`
am


1
= x,
f(x) + f
1x

(x 6= 0, x 6= 1).

(4.1)

,
`ng
`,i giai. Trong (4.1) ta thay x ba
Lo

1
,,
, ta nhn
. duo. c
1x




1
x1
1
f
+f
=
.
1x
x
1x

1
,,
, ta nhn
uo. c
. d
1x


x1
x1
f
+ f(x) =
.
x
x
,
,,
Ly (4.3) tr`u di (4.2), ta nhn
uo. c
. d


x2 x + 1
1
f(x) f
=
.
1x
x(x 1)

(4.2)

,
`ng
T`u (4.2) ta lai
. thay x ba

(4.3)

(4.4)

Cng
a (4.4) ta c
o
. (4.1) v`
f(x) =

x3 x + 1
.
2x(x 1)

,, ,
,,
,,
,
`ng c
cho, h`
Ba
ach thu tru. c tip v`
ao phuong tr`nh h`
am d
am t`m duo. c thu. c
a
,
,
su. l`
a nghim
. cua (4.1).
,,
`ng c
,i c
ach th v`
ao phuong tr`nh h`
am vo
ac gi
a tri. kh
ac nhau v`
a sau
Ba
,
,
,
,
c n`
ng thu
kt ho. p c
do
ac da
ay se t`m ra nghim.
Xet mt
in h`nh:
.
. v du. d

`, phu,o,ng tr`nh sau


V du. 4.2. H
ay t`m h`
am f(x) tu
af(x + ) + bf(x ) = cx,

( 6= 0).

(4.5)

,,
,,
Chuong 4. Phuong tr`nh h`
am

178

,
`ng x th` phu,o,ng tr`nh (4.5) c
`,i giai. Ta th x + ba
Lo
o dang
.
af(x) + bf(x) =

c
(x ).

(4.6)

`ng x th` phu,o,ng tr`nh (4.5) c


Ta th x ba
o dang
.
c
af(x) + bf(x) = (x + ).
(4.7)

, , ,
,
tr`u di cho nhau cho ta
Nhn (4.6) v`
a (4.7) l`n luo. t vo
i a v`
a b v`
a sau do
,
kt qua
c
(a2 b2 )f(x) = [(a + b)x (a b)].

,
,,
T`u dy ta nhn
uo. c:
. d


,
c
1

,
a) Nu a2 6= b2 , th` f(x) =
x
. Ta kim tra tru. c tip
ab
a+b
,
,,
cho.
thy h`
am trn l`
a nghim
a
. cua phuong tr`nh d
,
,
cho khng c
b) Nu a2 b2 = 0, c 6= 0, phuong tr`nh d
o nghim.
a
.
,
,
,
Nu c 6= 0, a = b = 0, th` phuong tr`nh (4.5) c
o nghim
a tt ca c
ac
. l`
h`
am.

,,
Nu c 6= 0, a = b 6= 0, th` phuong tr`nh (4.5) khng c
o nghim.
.
,
,
c) Nu c = 0, a = b 6= 0, th` phuong tr`nh (4.5) c
o nghim
.
,

f(x) = f(x), nghim


l`
a
m
oi
h`
a
m
s
l
e.
.
.
,,

d) Nu c = 0, a = b 6= 0, th` phuong tr`nh (4.5) c


o nghim
.

n.
f(x) = f(x), nghim
a moi
am s cha
. l`
. h`
,
,
,
e) Nu c = a = b = 0, th` phuong tr`nh (4.5) c
o nghim
a tt ca c
ac
. l`
h`
am.
,
,,
xet hai v du. din h`nh d`
Ta da
ung phuong ph
ap th c
ac gi
a tri. kh
ac
,
,
,
`ng h`
c. L`
ng thu
. c nhin l`
cho ra
nhau d`u cho d
am nhu vy
a ta d
am s
a
a
. ma
,
,
,
t`n tai,
nghim
o c
o thu. c
a
. d
. nn khi t`m ra nghim
. r`i phai kim tra lai
. n
,
su. l`
a nghim
. hay khng.
,
,
,
,i moi
V du. sau d
ao tnh bt bin cua h`
am s l`
a vo
a tri. cua
y du. a v`
. gi

,,
,ng gi
4.1. Phuong ph
ap th nhu
a tri. di s

179

`ng s (bt bin) v`


l`
bin s, h`
am s d`u nhn
a tri,
a tri. do
a ha
a ta
. gi
. mt
. gi
,
,, `
,
,
ng s n`
l`
t`m duo. c ha
ay, sau do
a t`m ra nghim
am.
. cua phuong tr`nh h`
,
,
,
,ng h`
V du. 4.3. H
ay t`m tt ca nhu
am s x
ac d
oi moi
a thoa m
an
inh
. v
. x v`
,,
phuong tr`nh sau d
y
(x y)f(x + y) (x + y)f(x y) = 4xy(x2 y2 ).

(4.8)

,
,,
`,i giai. a
. t u = x + y, v = x y v`
Lo
a thay v`
ao (4.8), ta nhn
uo. c
. d
vf(u) uf(v) = uv(u2 v2 ).
(4.9)
, , f(u) f(v)
2
2
,i u 6= 0, v 6= 0, t`u, (4.9) ta nhn
Vo
. duo. c u v = u v , hay
f(v)
f(u)
u2 =
v2 .
(4.10)
u
v
f(x)
,
,i
T`u (4.10) suy ra h`
am s
x2 nhn
ung mt
a tri. (bt bin) vo
. c`
. gi
x
,
f(x)
,

a tri. kh
ac nhau cua di s, ngha l`
a
x2 = c v`
a khi d
nhung gi
o
x
3
f(x) = x + cx,
(4.11)
,,
`ng s bt k`.
od
a mt
y c l`
. ha
,,
. c u = 0, v 6= 0) ta nhn
Nu trong (4.9) thay v = 0, u 6= 0 (hoa
uo. c
. d
,
,i x = 0. Suy
f(0) = 0. Nhung moi
am c
o dang
a tri. 0 vo
. h`
. (4.11) d`u nhn
. gi
,
,
,
ra nu h`
am f(x) thoa m
an (4.8) n
o c`n thoa m
an dang
(4.11). D kim
.
,
,,
tra thy moi
am c
o dang
an phuong tr`nh (4.8).
. h`
. (4.11) thoa m
,
,
,
,
,i y tu,o,ng cua b`
l`
Vo
ai trn ta c
o th cho mt
a tri. cua h`
am s sau do
a
. gi
,
,,
giai phuong tr`nh:
,
,,
V du. 4.4. H
ay giai phuong tr`nh h`
am sau

f(x + y) 2f(xy) 3f(x) + (2x2 1)f(y) = 2x(xy 1) 5.

(4.12)

,
, ,
,
`,i giai. Gia su, f(x) l`
. t f(0) = c. Trong (4.12)
Lo
a nghim
a ta da
. cua (4.12) v`

ta cho y = 0, khi d
o
2f(x) = 2cx2 + 2x + 5 3c.

,,
,,
Chuong 4. Phuong tr`nh h`
am

180

,
T`u d
o 2c = 5 3c, suy ra c = 1. Suy ra f(x) = x2 + x + 1.
y cho x = 0, ta c
,
,
,,
, , ,
Kim tra tru. c tip h`
am t`m d
an (4.12).
uo. c thu. c su. thoa m
,
V du. 4.5 (IMO 1999). T`m tt ca h`
am f : R R sao cho

f(x f(y)) = f(f(y)) + xf(y) + f(x) 1


,
v
oi moi
. x, y R.
,
,,
,,
`,i giai. a
. t f(0) = c. Thay x = y = 0 v`
Lo
ao phuong tr`nh trn v`
a nhn
uo. c
. d
f(c) = f(c) + c 1.
,
,
,i x = f(y) A, ta c
Nhu vy
a mi`n gi
a tri. cua f, th` vo
o
. c 6= 0. Cho A l`
c = f(0) = f(x) + x2 + f(x) 1.
,
c + 1 x2
,,
,,
.
Giai phuong tr`nh trn ta t`m d
uo. c f(x) =
2
2
,,
,
i moi
ng vo
Ta d
am du
i t`m c. Phuong tr`nh h`
. x, y, nn ta thay x = y 6= 0
,,
,,
cho, ta duo. c:
v`
ao phuong tr`nh da
f(x f(x)) = f(f(x)) + xf(x) + f(x) 1.
,
c + 1 x2
,,

v`
ao phuong tr`nh trn cho kt qua:
2
2


c + 1 (x f(x))2
c + 1 (f(x))2
c + 1 x2
c + 1 x2

+x

1.
2
2
2
2
2
2
2
2

Thay f(x) =

(x f(x))2 = (f(x))2 + x(c + 1 x2 ) + c + 1 x2 2.


x2 + 2xf(x) (f(x))2 = (f(x))2 + x(c + 1 x2 ) + c + 1 x2 2.
2xf(x) = x(c + 1 x2 ) + c 1.


c + 1 x2
= x(c + 1 x2 ) + c 1.
2x

2
2
x2
, ,
,c trn suy ra c = 1. Vy
,i x A, f(x) = 1
ng thu
T`u da
. H`
am t`m
. vo
2
2
,
x
,,
,i moi
duo. c x
ac d
o th thay h`
am f(x) = 1
x
ac dinh
inh
. vo
.
. x R. Ta c
2

,,
,ng gi
4.1. Phuong ph
ap th nhu
a tri. di s

181

,
,,
, ,
,i moi
cho v`
vo
ao phuong tr`nh da
a thy n
o thu. c su. l`
a nghim
. x R v`
. cua
,,
phuong tr`nh.
, ,
,,
,
,,
,,
l`
C`
on phuong ph
ap nua d giai phuong tr`nh h`
am do
a phuong ph
ap
,
,
,,
,,
`
Cauchy: Phuong ph
ap n`
ay giai phuong tr`nh h`
am trn min x
ac dinh
. cua
,,
,
,
`ng c
n
o ba
ach l`n luo. t xet c
ac c
ac bin l`
a s tu. nhin, s nguyn, s huu
,
,
,
,
,
,,
d`
t v`
a nhung s thu. c. Trong khi giai nhung phuong tr`nh n`
ay ta vn
ung
,
,
,

c
ach th c
ac gi
a tri. kh
ac nhau cua d
am s.
i s d t`m ra cng thuc h`

,
,,
V du. 4.6. H
ay giai phuong tr`nh h`
am
f(x + y) = f(x) + f(y)

(4.13)

,,
,,
,,
,
o dy f(x) l`
a h`
am lin tuc
oi moi
a phuong tr`nh
. v
. x (phuong tr`nh trn goi
. l`
Cauchy).
,
,,
`,i giai. Th x = y = 0 v`
Lo
ao (4.13) ta nhn
a
. duo. c f(0) = 2f(0), ngha l`
f(0) = 0.
,,
Th y = x v`
ao (4.13) ta nhn
a nu f(x)
uo. c f(x) = f(x) ngha l`
. d
,
,
,
l`
a nghim
o phai l`
a mt
am le.
. cua (4.13), th` n
. h`
,,
,i y = 2x, ta
Th y = x v`
ao (4.13) ta nhn
on vo
uo. c f(2x) = 2f(x), c`
. d
,,
nhn
. duo. c
f(3x) = f(x) + f(2x) = f(x) + 2f(x) = 3f(x).
`ng phu,o,ng ph
,ng minh du,o.,c f(nx) = nf(x) vo
,i moi
Ba
ap quy nap
. ta chu
. s
,
tu. nhin n.
,, ,
t`m duo. c cua h`
Nu m l`
a s nguyn m, do c
ac tnh cht d
am f(x) ta c
o
a
f(mx) = f(mx) = (m)f(x) = mf(x).
,
,
,ng minh da
,c
ng thu
chu
Nhu vy
a
. ta d
f(`x) = `f(x)

(4.14)

,,
,,
Chuong 4. Phuong tr`nh h`
am

182
,i moi
ng vo
du
. s nguyn `.

,, ,
,c
,i x = 1, t`u, (4.14) ta nhn
ng thu
Nu ta ky hiu
a
. f(1) = c vo
. duo. c d
f(`) = c`

(4.15)

,i moi
ng vo
du
. s nguyn `.
,
Nu p l`
a mt
on q l`
a mt
ac 0 th` theo (4.14)
. s nguyn, c`
. s tu. nhin kh
,,
v`
a (4.15) ta nhn
uo. c
. d
 
 
p
p
qf
=f q
= f(p) = cp
q
q
 
,
p
p
,
,c
ng thu
ngha l`
af
= c. . T`u dy suy ra da
q
q
f(x) = cx
(4.16)
, ,
,
,
, ,
,
i moi
ng vo
du
a tri. huu t cua x. Cho mt
a d
ay s huu t
. gi
. s v t x v`
a1 , a2 , a3 , ..., an , ... sao cho lim an = x. Theo (4.16) ta c
o
n

f(an ) = can .
(4.17)
,
,
,,
Khi n v`
a v` tnh lin tuc
am f(x), t`u (4.17) ta nhn
. cua h`
. duo. c
f(x) = cx.
,
,
,
`ng s bt k`, l`
`ng h`
am f(x) = cx, o dy c l`
a mt
a nghim
D thy ra
. ha
. cua
,,
phuong tr`nh (4.13).
,
,
,,
, ,
Hai v du. sau dy l`
a nhung b`
ai to
an phuong tr`nh h`
am co ban c
o th
,
,
,
,
`ng c
dua v` dang
ai to
an v`ua giai ba
ac phep th thch ho. p.
. b`
,
,,
V du. 4.7. H
ay giai phuong tr`nh h`
am

f(xy) = f(x) + f(y),


,,
,
od
am f(x) l`
a h`
am lin tuc
oi moi
y h`
. v
. x > 0.

(4.18)

,
,,
`,i giai. Ta th x = cu v`
Lo
a y = cv v`
ao (4.18). Ta nhn
. duo. c
f(cu+v ) = f(cu ) + f(cv ), ngha l`
a
g(u + v) = g(u) + g(v),

(4.19)

,,
,ng gi
4.1. Phuong ph
ap th nhu
a tri. di s

183

,,
,,
od
a dang
a mt
am lin tuc
y g(u) = f(cu ) l`
. (4.19) l`
.
. h`
. theo u. Boi v` dang
,
,,
giai b`
(4.13) da
ai truoc, nn
g(u) = f(cu ) = au.
,,
V` x = cu , ngha l`
a u = lgc x nn ta t`m d
uo. c
f(x) = a.lgc x,
,,
,
`ng s bt k`, c > 0, c 6= 1.
od
a nhung ha
y a, c l`
,
,,
an (4.18).
D thy h`
am t`m duo. c thoa m
,
,,
V du. 4.8. Giai phuong tr`nh h`
am


f(x) + f(y)
x+y
=
,
f
2
2
,,
,
od
am f(x) l`
a h`
am lin tuc
oi moi
y h`
. v
. x.

(4.20)

J
(4.21)

,
,,
`,i giai. Th x = u + v v`
Lo
a y = 0 ta nhn
. duo. c


u+v
f(u + v) + f(0)
f
=
,
2
2
m`
a v tr
ai theo (4.21) l`
a
f(u + v) + f(0)
f(u) + f(v)
=
,
2
2
hay
f(u + v) f(0) = (f(u) f(0)) + (f(v) f(0)),
,,
,i g(x) = f(x) f(0).
dy chnh l`
a dang
. phuong tr`nh (4.13) vo
g(x) = ax v`
Khi d
a suy ra
o
f(x) = ax + f(0) = ax + b,
,,
`ng s bt k` v`
od
a ha
a v`
a b = f(0).
y a l`
,
,
,
Kim tra tru. c tip ta thy f(x) = ax + b l`
a nghim
. cua (4.21).
,
,,
,,
Mt
ap kh
ac giai phuong tr`nh h`
am l`
a t`m ra mt
. phuong ph
. nghim
.
,
,
`

ng c
ba
am phai t`m f(x). Sau d
ach th (x) = f(x)(x)
ring (x) cua h`
o

,,
,,
Chuong 4. Phuong tr`nh h`
am

184

,
,,
,,
,,
,i
cho v`
v`
ao phuong tr`nh h`
am da
a giai phuong tr`nh h`
am nhn
uo. c di vo
. d
,
,
,
,,
,
phuong tr`nh h`
(x). Khi do
am phai giai se d
on gian rt nhi`u.
,
,,
V du. 4.9. Giai phuong tr`nh h`
am
f(x + y) = f(x) + f(y) + xy,
,
,,
od
am f(x) l`
a h`
am lin tuc
y h`
im x = 0.
. tai
. d

(4.22)

,
,,
`,i giai. Vo
,i y = 0, t`u, (4.22) ta nhn
,i moi
vo
Lo
a tri.
o
. duo. c f(0) = 0. Khi d
. gi
, ,
thu. c cua x ta c
o
lim [f(x) + f(y) + xy] = f(x) + lim f(y) + 0 = f(x) + f(0) + 0 = f(x),

y0

y0

,
`ng f(x) (nu
ngha l`
a theo (4.22) lim f(x + y) = f(x), di`u n`
ay ch ra ra
y0
,i moi
t`n tai)
a h`
am lin tuc
. l`
. di vo
. x.
, ,
,
1
,,
Ta c
o th kim tra h`
am (x) = (x2 + x) l`
a nghim
cua phuong
.
2
tr`nh (4.22).
,i moi
. t (x) = f(x) (x). Khi d
h`
a
am (x) l`
a h`
am lin tuc
a
o
. vo
. x v`
(x + y) = f(x + y) (x + y) =
= [f(x) + f(y) + xy] [(x) + (y) + xy] =
= [f(x) (x)] + [f(y) (y)] =
= (x) + (y).
,,
,,
y l`
a dang
. phuong tr`nh (4.13), ta t`m duo. c nghim
. (x) = ax. Suy ra
1
1
1
f(x) = (x) + (x) = ax + (x2 + x) = x2 + (a + )x,
2
2
2
,,
1 2

`ng s bt k`. D thy ra


`ng d
ngha l`
a f(x) = 2 x + bx, o d
a ha
y b l`
y chnh
,
,,
cho.
l`
a nghim
. chung cua phuong tr`nh da
,
,
,
V du. 4.10. H
ay t`m tt ca c
ac h`
am f(x) thoa m
an tt ca c
ac d
i`u kin
. sau

d
y:

,,
,ng gi
4.1. Phuong ph
ap th nhu
a tri. di s

185

,
1) f(x) = f(x), v
oi moi
. x;
,
2) f(x + 1) = f(x) + 1, v
oi moi
. x;
 
1
1
,
3) f
oi moi
= 2 f(x), v
. x 6= 0.
x
x
,
,
,
,
`ng c
`ng (x) = x thoa m
`,i giai. Ba
Lo
ach kim tra tru. c tip ta thy ra
an tt
,
,
,
ca c
ac d
ai to
an, suy ra l`
a mt
ai to
an.
i`u kin
. cua b`
. nghim
. ring cua b`
. t (x) = f(x) x. Khi d

a
o
(x) = f(x) + x = f(x) + x = (x),
(x + 1) = f(x + 1) (x + 1) =

(4.23)
(4.24)

= f(x) + 1 (x + 1) = f(x) x = (x),


 
 
1
1
1
1
1
=f
= 2 f(x) =
(4.25)

x
x
x
x
x
1
1
= 2 [f(x) x] = 2 (x).
x
x
,,
,i x = 0, t`u, (4.23) ta nhn
a (0) = 0.
Vo
. duo. c (0) = (0), ngha l`
,
,
,
,
i x = 1, t`u (4.24) ta nhn
Vo
. duo. c (1) = (0) = 0.
,,
,i x 6= 0 v`
Vo
a x 6= 1, theo (4.23), (4.24) v`
a (4.25) ta nhn
. duo. c




1
1
(x) = (x + 1) = (x + 1)2
= (x + 1)2
x+1
x+1




x
1
+ 1 = (x + 1)2
= (x + 1)2
x+1
x+1




2
x
x+1
1
2
= (x + 1)2

=
x

1
+
(x + 1)2
x
x
 
1
1
= x2
= x2 2 (x) = (x),
x
x
ngha l`
a 2(x) = 0 hay l`
a (x) = 0.
,
,
,i moi
T`u (x) = 0 vo
am s f(x) = x l`
a nghim
. x, suy ra h`
. duy nht cua
,,
cho.
phuong tr`nh da

,,
,,
Chuong 4. Phuong tr`nh h`
am

186

,
,,
,
a thoa m
an phuong tr`nh
V du. 4.11. T`m c
ac h`
am x
ac dinh
oi moi
. x v`
. v
xf(y) + yf(x) = (x + y).f(x)f(y)

(4.26)

,
v
oi x, y bt k`.
,
,,
,
`ng x, ta t`m d
`,i giai. T`u, (4.26) thay y ba
Lo
uo. c 2xf(x) = 2x(f(x))2 . T`u d
y
,
,
,
2
i x 6= 0 ta t`m d
. c l`
vo
a f(x) = 0 hoa
a f(x) = 1.
uo. c f(x) = (f(x)) , ngha l`
,
,
,
,
i x = a 6= 0. Khi do
t`u (4.26) ta nhn
Cho f(a) = 1 vo
uo. c af(y) + y =
. d
,

(a + y)f(y) hay y = yf(y), ngha l`


a f(y) = 1 voi y 6= 0.
,
,
,,
tt ca nghim
Ta k hiu
a:
. f(0) = c. Khi do
. cua phuong tr`nh (4.26) l`
a) H`
am s f(x) = 0;
,
b) Nhung h`
am s


f(x) =

,i x 6= 0
1 vo
,i x = 0.
c vo

,
,
` tp
Bai
(Go. i y v`
a tra l`oi b`
ai tp
. sau tai
. trang 203).
.
,
,,
. 4.12. Giai phuong tr`nh h`
am sau
 
1
f(x) + af
= x,
x
,
,,
. 4.13. Giai phuong tr`nh h`
am

a 6= 1.

f(x + y) + f(x y) = 2f(x) cos y.


,
,,
. 4.14. Giai phuong tr`nh h`
am
f(x + y) f(x y) = 4xy.
,
,,
. 4.15. Giai phuong tr`nh h`
am
f(x + y) 2f(x y) + f(x) 2f(y) = y 2.

,
,,
4.2. Phuong ph
ap dim bt dng
.

187

,
,,
. 4.16. Giai phuong tr`nh h`
am
f(x + y) = f(x)f(y).
,
,,
. 4.17. Giai phuong tr`nh h`
am
f(xy) = f(x)f(y).
,
,,
. 4.18. Giai phuong tr`nh h`
am
f(x + y) + f(x y) = 2[f(x) + f(y)].
,
,,
. 4.19. Giai phuong tr`nh h`
am
f(x) +

x
1
= f( ).
sin x
2

,
,,
. 4.20. Giai phuong tr`nh h`
am
f(x + y) = f(x)f(y) + xy xf(y) yf(x) + x + y.

,
,,
di
4.2. Phuong phap
m bt dng
.
,
, , ,
duo. c u
L thuyt dim bt dng
da
ng dung
rt nhi`u trong to
an cao
.
.
,
,
,
,
,
,

cp nhu giai tch thu. c, giai tch phuc v`


a giai tch h`
am,... trong muc
ay ta
. n`
,
, ,
,,
,
,,
,

lo. i dung
ai
. dim bt dng
. nhu mt
. d
. luo. ng bt bin d giai mt
. s phuong
tr`nh h`
am.
,
,
,
Mt
ad
am f(x) nu f(x0 ) = x0 .
im bt dng
im x0 goi
. cua h`
. d
. l`
,i n = 2, 3, 4, ..., h`
. t f1 (x) = f(x) v`
a
a fn (x) = f(fn1 (x)) vo
am s fn (x)
,
,
goi
a phep la
. p thu n cua h`
am f(x).
. l`
,
, n
K hiu
a tp
o tnh cht sau dy:
im bt d
ng
. Sn l`
. d
. cua f (x). Ta c
,
,
,
1. Nu x l`
a mt
cua fn (x), th` f(x) l`
a c
ung l`
ad
ng
im bt
. dim bt d
.
, n
dng
. cua f (x). Tht
. vy
.
fn (f(x)) = fn+1 (x) = f(fn (x)) = f(x).

,,
,,
Chuong 4. Phuong tr`nh h`
am

188

,
nh xa. Sn v`
Nhu vy,
ao chnh Sn .
. f(x) a
,
,i a, b Sn , th`
nh trn Sn . V` nu f(a) = f(b) vo
2. f l`
ad
on a
a = fn (a) = fn1 (f(a)) = fn1 (f(b)) = fn (b) = b.
,
,
T`u hai tnh cht trn suy ra nu Sn l`
a tp
a phep ho
an
. huu han
. th` f(Sn ) l`
vi. trong Sn .

,
,
V` f(x) = x keo theo f2 (x) = f(f(x)) = f(x) = x, nhu vy
im bt dng
. d
.
,
,
, 2
, , ,
2 4 8
cua f c
ung l`
a dim bt d
ng
c
ua
f
.
Ta
x
e
t
d
a
y
f,
f
,
f
,
f
,
...
tu
o
ng
u
ng
ta
.
,,
nhn
uo. c
. d
S1 S2 S4 S8 ...
,
,
V du. 4.21. T`m tt ca c
ac h`
am f : R R sao cho f(f(x)) = x2 2 v
oi moi
.
x R.
,
, ,
,
`,i giai. Gia su, h`
Lo
am f(x) t`n tai.
ac tp
a S4 nhu sau:
. Ta tnh c
. S2 v`
,
,
,
,
,,
Nhung d
cua h`
am f2 l`
a nghim
im bt dng
.
. cua phuong tr`nh x =
,
, 4
,
x2 2, ngha l`
a S2 = {1, 2}. Nhung d
a nghi
m
im bt d
ng
.  cua f (x) l`
.
,
1 5
,,
cua phuong tr`nh x = x4 4x2 + 2, ngha l`
a S4 = 1, 2,
. Ta
2

1 + 5
1 5
. t c =
da
,d =
. V` f l`
a ho
an vi. trong S2 v`
a c, d S4 \ S2 ,
2
2
. c f(c) = d (do f a
nh xa. S4 v`
nn f(c) = c hoa
ao S4 ).
,
,
Nu f(c) = c, th` f2 (c) = c keo theo c l`
a dim bt dng
cua f2 , di`u
.
n`
ay v ly.
c = f(d) = f(f(c)) = f2 (c), c
Nu f(c) = d, th` f(d) = c. Khi do
ung v
,
,
,
,
`
ly nhu trn. Nhu vy
am f n`
ao thoa m
an b`
ai to
an.
. khng th tn tai
. mt
. h`

,
V du. 4.22 (IMO 1983). T`m tt ca c
ac h`
am f : R+ R+ sao cho
,
+
f(xf(y)) = yf(x) v
oi moi
a f(x) 0 khi x +.
. x, y R v`

,
,,
4.2. Phuong ph
ap dim bt dng
.

189

,
,
,,
`,i giai. 1. Thay x = y = 1 v`
,c da
ng thu
cho ta nhn
Lo
ao d
a
uo. c
. d
,
,,
,c d
ng thu
cho ta nhn
f(f(1)) = f(1). Thay x = 1, y = f(1) v`
ao da
a
uo. c
. d

f(f(f(1))) = [f(1)]2 . Khi do


[f(1)]2 = f(f(f(1))) = f(f(1)) = f(1) f(1) = 1 (v` f(1) R+ ).
,
,
,
Nhu vy
a dim bt d
ng
. 1 l`
. cua f.
,,
,
2. Ly y = x, nhn
ung l`
a
. duo. c f(xf(x)) = xf(x). Nhu vy
. w = xf(x) c
,
,
,
+

i moi
d
im bt d
ng
. cua f vo
. xR .
, ,,
,
,,
3. Gia su f c
od
x > 1. Theo buo
c 2, xf(x) = x2 c
ung l`
a
im bt d
ng
.
,
,
,
2
2
4
2n

d
x f(x ) = x c
ung l`
a dim bt d
... Nhu vy
a
im bt d
ng,
ng,
.
.
. x l`
,
,
,
2n
2n
2n

nhung dim bt d
V` x > 1 nn x +, nhung f(x ) = x
ng.
.
,
,
, ,

chu khng phai l`


a 0 nhu gia thit. i`u n`
ay v l.
, ,,
,

4. Gia su f c
o dim bt dng
. x (0, 1). Khi do


 
 
 
1
1
1
1
1
x =f
f(x) = xf
f
= ,
1=f
x
x
x
x
x
,
,
1
,i ph`n 3 o, trn. Nhu, vy
ngha l`
a f c
o dim bt d
> 1, tr
ai vo
ng
.
. f khng
x ,
,
c
o dim bt dng
ao trong khoang (0, 1).
. n`
,
,
,
,,
5. Sau c
ac buoc trn ch c`
on lai
cua f l`
a 1. Theo ph`n 2
im bt d
ng
. d
.
1 ,
+
i moi
ta c
o xf(x) = 1 suy ra f(x) = vo
. xR .
x
 
,,
1
x
y
1
,

Thu lai:
V
o
i
f(x)
=
,
f(xf(y))
=
f
= = yf(x) v`
a f(x) = 0
.
x
y
x
x
,
1
,,
cho.
a nghim
am da
khi x +. Vy
. cua phuong tr`nh h`
. f(x) = x l`
,
,
V du. 4.23 (IMO 1996). T`m tt ca c
ac h`
am thoa m
an f : N0 N0 sao cho
,
f(m + f(n)) = f(f(m)) + f(n) v
oi moi
. m, n N0 .

,
,,
`,i giai. 1. Cho m = n = 0, nhn
Lo
. duo. c f(f(0)) = f(f(0)) + f(0), suy
,,
ra f(0) = 0. Lai
a f(n)
uo. c f(f(n)) = f(n), ngha l`
. ly m = 0, ta nhn
. d

190

,,
,,
Chuong 4. Phuong tr`nh h`
am

,
,
,,
,,
,i moi
phuong tr`nh tro th`
l`
a dim bt d
anh
cua f vo
ng
. n N0 . Khi do
.

f(m + f(n)) = f(m) + f(n).


,
,
,
`ng kw c
ta ch ra ra
2. Nu w l`
a mt
cua f, khi do
ung l`
a
. dim bt dng
.
,
,
,
,
`

ng quy nap
dim bt dng
cua f voi moi
an
.
. k N0 . Ta chung minh ba
. to
, ,,
,
,`,
,

bit d
ng. Gia su kw l`
hoc:
a dim bt dng,
khi
u
. Truong ho. p k = 0 ta da
.
,
f(kw + w) = f(kw + f(w)) = f(f(kw)) + f(w) = kw + w v`
do
a nhu vy
.
,
,

(k + 1)w c
ung l`
a dim bt dng
. cua f.
,
,
,i moi
f(n) = 0 vo
3. Nu 0 l`
a dim bt dng
duy nht cua f, khi do
.
.
,
,
,,
,,
`ng khng l`
n N0 theo buo
c 1. Hin nhin, h`
am ha
a nghim
c
ua
phu
o
ng
.
cho.
tr`nh da
,
,
,
`ng w l`
Nu f c
o t nht mt
w > 0 v`
a gia thit ra
ad
ng
im
. dim bt d
.
,
,
,
,
,,
,
`ng ch c
bt d
duong nho nht. Ta se ch ra ra
o nhung dim bt dng
ng
.
.
, ,,
,

kw, k N0 l`
a nghim.
Th
t
v
y,
Gi
a
s
u
x
l`
a
m
t
d
i
m
b
t
d
ng
n`
a
o
d

o
.
. .
.
.
,,
,,
duong. Theo thut
an chia, x = kw + r, o dy 0 r < w. Ta c
o
. to
x = f(x) = f(r + kw) = f(r + f(kw)) =
= f(r) + f(kw) = f(r) + kw.
,
,
,
,
,,
Nhu vy
a dim bt dng
duong nho nht, nn
. f(r) = x kw = r. T`u w l`
.
r = 0 v`
a x = kw.
,
,
,,
,i moi
T`u f(n) l`
ad
vo
c 1), nn f(n) =
im bt d
ng
.
. n N0 (theo buo
,
i cn N0 . Ta c
cn w vo
o c0 = 0.
,i n N0 , theo thut
4. Vo
an chia, ta c
o n = kw + r, 0 r < w. Vy
. to
.
f(n) = f(r + kw) = f(r + f(kw)) = f(r) + f(kw) =
hni
)w.
= cr w + kw = (cr + k)w = (cr +
w
,,
w > 0, ly c0 = 0 v`
,i mi
Thu lai:
a c1 , c2 , ..., cw1 N0 bt k`. Xet h`
am
. vo
,
,
, ,
,
n
`
f(n) = (cr + [ w ])w, o dy r l`
a phn du cua phep chia n cho w (v`
a ca h`
am

,
,,
4.2. Phuong ph
ap dim bt dng
.

191

`ng khng). Ta vit m = kw + r, n = `w + s vo


,i 0 r, s < w. Khi d

ha
o
f(m + f(n)) = f(r + kw + (cs + `)w) =
= cr w + kw + cs w + `w = f(f(m)) + f(n).

,
V du. 4.24. T`m tt ca c
ac h`
am f : N0 N0 sao cho f(f(m)+f(n)) = m+n
,
v
oi moi
. m, n N0 .
,
,
`ng f(x) = x l`
`ng quy nap,
`,i giai. D thy ra
Lo
a mt
Ba
. nghim.
.
. ta se ch ra
`ng b`
ra
ai to
an khng c
o nghim
ac.
. kh
, ,,
,
ng minh f(1) = 1: Gia su f(1) = t > 1. Cho s = f(t1) > 0. Thy
Ta chu
`ng nu f(m) = n, th` f(2n) = f(f(m) + f(m)) = 2m. Nhu, vy
ra
. f(2t) = 2
2s + 2t = f(f(2s) + f(2t)) = f(2t) = 2, suy ra
v`
a f(2s) = 2t 2. Khi d
o
f(1) = 1.
t < 1, di`u n`
ay v l. Do do
, ,,
f(n + 1) = f(f(n) + f(1)) = n + 1. Do d

Gia su f(n) = n. Khi do


o
,
i moi
f(n) = n vo
. n N0 .

,
V du. 4.25 (IMO 1987). Chung minh ra
`ng khng t`n tai
am f : N0 N0
. h`
sao cho f(f(n)) = n + 1987.
,
, ,
,
,
`,i giai. Gia su, t`n tai
f l`
nh, ngha l`
Lo
am f nhu vy.
a d
a
on a
. h`
. Khi do
f(a) = f(b), suy ra a + 1987 = f(f(a)) = f(f(b)) = b + 1987, keo theo
a = b.
, ,,
,i k gi
Gia su f(n) kh
ac vo
a tri. kh
ac nhau c1 , ..., ck trong N0 , ngha l`
a
,
,
i moi
i 2k gi
f(f(n)) kh
f(n) 6= c1 , ..., ck vo
ac vo
a tri. kh
ac
o
. n N0 . Khi d
,
nhau c1 , ..., ck v`
a f(c1 ), ..., f(ck ) trong N0 (nhung gi
a tri. f(cj ) kh
ac nhau
,,
,
,

nh). By gi`o nu w 6= c1 , c2 , ..., ck , f(c1 ), f(c2 ), ..., f(ck ),


boi v` f l`
a don a
`
tn tai

khi d
o
. m N0 sao cho f(f(m)) = w. V` w 6= f(cj ), m 6= cj , do do
,
`
`
`
ng
f(f(n)) = w. iu n`
tn tai
ay ch ra ra
. n N0 sao cho f(n) = m, khi do
,
f(f(n)) kh
ac ch c
o 2k gi
a tri. c1 , c2 , ..., ck , f(c1 ), ..., f(ck ) v`
a khng kh
ac

,,
,,
Chuong 4. Phuong tr`nh h`
am

192

,
gi
a tri. n`
ao nua. V` n + 1987 kh
ac 1987 gi
a tri:
a 2k 6= 1987,
. 0, 1, ..., 1986 v`
di`u n`
ay v l.

,
,
,
,,
V du. 4.26. H
ay t`m tt ca c
ac d
a nghim
a thuc P(x) l`
. cua phuong tr`nh
P(Q(x)) = Q(P(x)),
(4.27)
,,
,
,
od
a mt
am x
ac dinh
oi moi
a Q(x) > x v
oi moi
a
y Q(x) l`
. h`
. v
. x v`
. x 0 v`
P(0) = 0.
,
`,i giai. Cho Q(x) l`
,i moi
,i moi
Lo
a h`
am x
ac dinh
a Q(x) > x vo
. vo
. x v`
. x 0.
Ta tao
ay
. d

a0 , a1 , a2 , ..., an , ...

(4.28)

theo c
ach sau:
a0 = 0, a1 = Q(0), a2 = Q(a1 ), ..., ak = Q(ak1 ), ...
,,
Ta nhn
a a1 > a0 ; Q(a1 ) > a1 ngha l`
a a2 > a1 ;
uo. c Q(0) > 0, ngha l`
. d
,
`
`
ng d
Q(a2 ) > a2 ngha l`
a a3 > a2 ; ... diu n`
ay ch ra ra
ay (4.28) t
ang v`
a
,

suy ra n
o c
o v han
ac nhau.
. nhung s hang
. kh
,
,
,
,
,
,
c P(x) phai t`m phai thoa m
,c:
ng thu
a thu
an nhung da
P(a0 ) = P(0) = 0 = a0 ,
P(a1 ) = P(Q(0)) = a1
P(a2 ) = P(Q(a1 )) = Q(P(a1 )) = Q(a1 ) = a2 ,
P(a3 ) = P(Q(a2 )) = Q(P(a2 )) = Q(a2 ) = a3 ,

,
,
,
Tt ca c
ac s hang
ay (4.28) thoa m
an P(an ) = an .
. cua d
,c H(x) = x c
,i d
Vo
ung c
o H(an ) = an .
a thu
,
,c tr`
,i v han
Khi hai da thu
ung nhau vo
ac gi
a tri. kh
ac nhau cua d
i s
. c
th` ch
ung tr`
ung nhau.

,
,,
4.2. Phuong ph
ap dim bt dng
.

193

,
,
,,
,,
Nhu vy
a nghim
uo. c P(x) = x l`
. ta t`m d
. duy nht cua phuong tr`nh

(4.27).

` tp
Bai
.
, ,
,
Mt
a tra l`oi b`
ai tp
. s go. i y v`
. sau tai
. trang 204.
,
. 4.27. T`m tt ca c
ac h`
am f : R R sao cho
,i moi
a) f(x + y) = f(x) + f(y) + 2xy vo
a
. x, y R v`
f(x)
b) limx0
= 1.
x
,
. 4.28. (1986 IMO). T`m tt ca c
ac h`
am f : [0, ) [0, ) sao cho
,i x, y 0 v`
a) f(xf(y))f(y) = f(x + y) vo
a
,
i 0 x < 2.
b) f(2) = 0 v`
a f(x) 6= 0 vo

, ,,
. 4.29. Gia su f : R R+ sao cho
p
,i moi
f( x2 + y2 ) = f(x)f(y) vo
. x, y R.
,
,i x Q trong do
c
T`m tt ca f(x) vo
o s hang
. f(1).
, , ,,
,
. 4.30. (1990 IMO). Cho Q+ l`
a tp
ac s huu t duong. H
ay xy du. ng
. c
h`
am s f : Q+ Q+ sao cho
f(x) ,
+
i moi
vo
. x, y Q .
y
,
, ,
,
. 4.31. (1994 IMO). Cho S l`
a tp
n hon 1. T`m tt ca c
ac h`
am
. s thu. c lo
f(xf(y)) =

f : S S sao cho
,i moi
a) f(x + f(y)) = y + f(x) + yf(x) vo
a
. x, y S v`
f(x)
,i 1 < x < 0 v`
,i 0 < x.
. t vo
b)
t
ang nga
a vo
x
,
. 4.32. (1992 IMO). T`m tt ca c
ac h`
am f : R R sao cho
,i moi
f(x2 + f(y)) = y + (f(x))2 vo
. x, y R.

194

,,
,,
Chuong 4. Phuong tr`nh h`
am

,
. 4.33. T`m tt ca c
ac h`
am f : Q Q sao cho f(2) = 2 v`
a


x+y
f(x) + f(y) ,
i x 6= y.
f
vo
=
xy
f(x) f(y)

,
da

4.3. Chuyn d
uc giao hoan
` v` cac
th
4.3.1. .inh ngha
,
,
,
,,
,c da
d
ach cua t
ac gia [8] c
o nhi`u
Nhung d
a thu
uo. c xet trong cun s
,
,,
,
,
,
,
,
ng dung
c o chuo,ng truo
,c l`
u
ai tp
an. Nhung d
a da
a thu
. trong vic
. giai b`
. to
,
,
,
,
,

c hai bin di xu
ng, ngha l`
c khng thay di khi ta d
thu
a da thu
i hai bin
,
,
c giao ho
cho nhau. Ta xet hai d
an nhu d
a thu
inh
. ngha sau dy.
,c mt
inh
ngha 4.1. Hai da thu
a Q(x) goi
a giao ho
an, nu
. bin P(x) v`
. l`
.
P(Q(x)) = Q(P(x)).
,
, ,
,c trong d
,i moi
ng thu
ng vo
a
a tri. thu. c cua x, ngha l`
a
inh
. ngha du
. gi
,
,
,

c P(Q(a)) = Q(P(a)).
ng thu
nu a l`
a mt
o da
. s thu. c bt k` th` ta lun c
,
,
,
,
c, t`m d
c giao ho
B`
ai to
an cho mt
an cua n
o khng phai
a thu
. da thu
,
,,
,
don gian, ta di xet l`n luo. t c
ac v du. sau:
,
,
V du. 4.34. Cho l`
a s bt k`, h
ay t`m tt ca d
an c
o bc
a thuc giao ho
.
,
,
,
,
2

khng l
on hon 3 d
oi d
i v
a thuc P(x) = x .
,
`,i giai. T`m da thu
,c Q(x) = x3 + ax2 + bx + c. T`u, d
cho
Lo
i`u kin
. da
P(Q(x)) = Q(P(x)) ta c
o:
(x3 + ax2 + bx + c)2 = (x2 )3 + a(x2 )2 + b(x2 ) + c.
,
,
,
`ng v
Khai trin l
uy th`ua hai v v`
a nh
om lai
anh c
ac h. s: Ta thy ra
. d so s
,
,
,
,
,, 5
,
n, c`
,a l
phai ch chu
uy th`ua bc
on o v tr
ai, h. s truo
c x l`
a 2a. Ngha
. cha
,,
,, 3 `

ng 2c, ngha l`
c
l`
a a = 0. Sau do
ung o v tr
ai, h. s truoc x ba
a c = 0. V`

,c giao ho
4.3. Chuyn d
ac da thu
an
` v` c

195

,
. c v`
th c`
on Q(x) = x3 + bx. Khai trin ngoa
a so s
anh c
ac h. s c`
on lai
. ta
,,
nhn
. duo. c

2b = 3,
b2 = 32 + b,

= 3 + b.
,
,,
, hai trong h. ta c
. t = 2, khi d
b = 3. T`u phuong tr`nh thu
a
o
o
. c l`
92 = 122 3, hay l`
a 2 = 0. Ngha l`
a = 0 hoa
a = 1. Suy ra
,

= 0 hoa. c = 2. D kim tra hai gi


a tri. n`
ay l`
a nghim
ung.
. d
,
,c bc
,c P(x) = x2
,i d
Nhu vy,
an vo
a thu
. t`n tai
. da thu
. ba Q(x) giao ho
,i = 0: P(x) = x2 , Q(x) = x3 ; vo
,i = 2:
. c = 2. Vo
nu = 0 hoa
P(x) = x2 2, Q(x) = x3 3x.
,,
,
,ng minh du,o.,c nu Q(x) l`
,c bc
Tuong tu. c
ung chu
a da thu
. hai, th`
,
c bc
P(x) = Q(x), c`
on nu Q(x) l`
ad
a thu
. mt,
. th` Q(x) = x.

,
,
V du. 4.35. Chung minh ra
`ng trong c
ac da thuc bc
a
. k, t`n tai
. khng qu
,
,
,

cho P(x).
mt
an v
oi da thuc bc
a thuc giao ho
a
. d
. hai d
,
,
,
`,i giai. Cho da thu
,c P(x) = x2 + px + q, phai t`m c
Lo
ac h. s a1 , ..., ak cua
,c sau:
da thu

,
,c
ng thu
Ta c
o da

Q(x) = xk + a1 xk1 + a2 xk2 + + ak .

(xk + a1 xk1 + + ak )2 + p(xk + a1 xk1 + + ak ) + q


(x2 + px + q)k a1 (x2 + px + q)k1 ak = 0.
,,
`ng c
`ng 0 nhu,ng h. s tru,o
,c x2k , x2k1 , ..., x, x0 . Ta nhn
Ba
ach cho ba
uo. c
. d
,
,,
,i a1 , a2 , ..., ak , p, q. i giai h. phu,o,ng tr`nh
mt
i vo
. h. phuong tr`nh d
,
, ,,
,
`ng c
n`
ay khng phai l`
a d. Ta tin h`
anh ba
ach kh
ac: Gia su nhung h. s
,, 2k1 2k2
,,
,,
,c
truo
cx
,x
, ..., xk l`n luo. t l`
a b1 , b2 , ..., bk , ta nhn
. duo. c cng thu

,,
,,
Chuong 4. Phuong tr`nh h`
am

196
sau
b1 = 2a1 + R1 (p, q) = 0,

b2 = 2a2 + R2 (p, q, a1 ) = 0,
...
bk = 2ak + Rk (p, q, a1 , ..., ak1 ) = 0,
,
,
,,
,c d
od
a mt
y Ri l`
ai
. biu thu
. s cua p, q, a1 , ..., ai1 .
,
,
, ,
,c d`u tin cua h. trn suy ra a1 biu din qua p, q, T`u,
ng thu
T`u da
,
,
,
,c thu
, hai suy ra a2 biu din qua p, q v`
ng thu
da
a a1 , ngha l`
a n
o c
ung ch
,
,
, ,
,c thu
, ba a3 c
ng thu
biu din qua p, q. T`u d
ung biu din qua p, q, a1 , a2 ,
a
,
,
,
,,
ngha l`
a n
o c
ung biu din ch qua p, q, ..., tip tuc
. nhu vy
. se nhn
. duo. c
,
,
,
,
c
ac h. s a1 , a2 , ..., ak d`u biu din ch qua p, q. Nhu vy,
ac h. s cua
. c
, , ,
,c Q(x) giao ho
,i P(x) x
da thu
an vo
ac dinh
ng thng qua p, q.
o l`
a
. tuong u
,
ng minh.
di`u c`n chu

,
,ng da thu,c bc
V du. 4.36. H
ay t`m tt ca nhu
a bc
a n
o giao ho
an
. 4 v`
. 8 m`
,
,
cho P(x).
v
oi d
a thuc bc
a
. hai d
,
`ng da thu
`,i giai. Ta se chu
,c Q(x) = P(P(x)) giao ho
,ng minh ra
,i
an vo
Lo
,c Q(x) c
P(x). Tht
o bc
a
. vy,
. Q(P(x)) = P(P(P(x))) = P(Q(x)). a thu
. l`
,
,
,
,
c giao ho
i P(x). Tuong
4 v`
a theo v du. 4.35 th` n
o l`
a da thu
an duy nht vo
,
,
,
ng minh cho da thu
c R(x) = P(P(P(x))) duy nht bc
tu. ta chu
an
. 8 giao ho
,
i P(x).
vo

,
,
,
V du. 4.37. Chung minh ra
`ng hai d
a R(x) giao ho
an v
oi da
a thuc Q(x) v`
,
,
,
thuc thu ba P(x) bc
ung giao ho
an v
oi nhau.
. 2, th` ch
,
`,i giai. a
,i
. t S(x) = Q(R(x)) v`
Lo
a T (x) = R(Q(x)). V` P(x) giao ho
an vo
Q(x) v`
a R(x), nn
P(S(x)) = P(Q(R(x))) = Q(P(R(x))) = Q(R(P(x))) = S(P(x)).

,c giao ho
4.3. Chuyn d
ac da thu
an
` v` c

197

,c S(x). Tu,o,ng tu., P(x) c


,i d
Ngha l`
a P(x) giao ho
an vo
ung giao ho
an
a thu
,
,
,

`
c c`
i T (x). D thy ra
ng S(x) v`
vo
a T (x) l`
a nhung da thu
ung bc,
. (nu Q(x)
,
v`
a R(x) c
o bc
a k v`
a `, th` bc
a T (x) l`
a k`). Theo v du. 4.35
. l`
. cua S(x) v`
suy ra S(x) = T (x), ngha l`
a Q(R(x)) = R(Q(x)).

,
4.3.2. a th
uc Chebyshev
,
,
inh
`ng t`n tai
ay d
a thuc P1 , P2 , ..., Pk , ... giao
. mt
. d
. l 4.1. Chung minh ra
,
ho
an c`
ung nhau sao cho bc
a k v`
a P2 (x) = x2 2.
. cua Pk (x) l`
,
,
,,
`ng mt
`,i giai. Ta giai b`
Lo
ai to
an ba
ap.
. s phuong ph
,
,,
. t x = t + t1 . Khi do
d d`
Phuong ph
ap 1: a
ang kim tra xk c
o dang
.
xk = (t + t1 )k =
= (tk + tk ) + a1 (tk1 + t(k1) ) + a2 (tk2 + t(k2) )+
+ + ak1 (t + t1 ) + ak ,

,,
,
`ng quy nap
od
a nhung s c dinh.
Ba
an hoc
y a1 , a2 , ..., ak l`
.
. to
. theo k suy
,
,
,
,
k
k

i dang
ra t + t c
o th biu din duo
.
xk + b1 xk1 + + bk1 x + bk ,

,,
,
. K hiu
od
a nhung s c d
ao d
y b1 , b2 , ..., bk l`
inh
o
. n`
.
Pk (x) = xk + b1 xk1 + + bk .
1
k
k
Theo dinh
. ngha th` Pk (t + t ) = t + t sao cho

Pk (P` (t + t1 )) = Pk (t` + t` ) = tk` + tk` = Pk` (t + t1 ).


,
,c bt k`
. p d
mt
Nhu vy,
a thu
. Pk (P` (x)) = Pk` (x) = P` (Pk (x)). Do do
. ca
,
,
,,
xy du. ng duo. c d
t`u d
ay Pk (x) d`u giao ho
an. V` P2 (x) = x2 2, ta da
ay
,
,
c c`n t`m.
nhung da thu
,
,,
,,
,c Pk (x) thng qua d
Phuong ph
ap 2: C
o th nhn
ay d
ay d
uo. c d
a thu
a
. d
,
,
,
,
c Chebyshev Tk (x) (c
c Chebyshev trong
thu
ac ban
o th tham khao da thu
. c

,,
,,
Chuong 4. Phuong tr`nh h`
am

198

,
,
,
,
,c n`
[12]). Ta se khng dua ra d
ngha tru. c tip d
ay, nhung d
inh
a thu
ua
.
,
,,
,c n`
dn phuong tr`nh h`
am m`
ad
ay thoa m
an: Tk (cos t) = cos kt. Ta
a thu
,
,
,
,
,

`
ng minh ra
c bc
,c
ng Tk l`
c
o th chu
a da thu
a thu
. k. D kim tra nhung d
,i nhau: Tk (Tm (x)) = Tkm (x) = Tm (Tk (x)) (do su.,
Chebyshev giao ho
an vo
, ,
,
,
bin di cua cos k(mt) = cos kmt = cos m(kt)). Nhung Tk khng phai l`
a
,
,,
,
,
,

c duo. c goi
a unita, nu h. s o s hang
unita (mt
. l`
. bc
. cao nht cua
. da thu
,
,,
,
k1
`

ng 1), v` h. s o s hang
n
o ba
a2
. Nhung d
ay c
o th
i`u n`
. bc
. cao nht l`
,
,,
,
c phuc
,i c
. t n`
. t Pk (x) = 2Tk ( x2 ). Vo
ach da
ay
kha
a da
uo. c nh`o vic
. d
. di bin v`
,
,
,
,
,
,

`
c vn duo. c bao to`
ng d
th` tnh giao ho
an cua c
ac da thu
an. Nhu vy
ay
. ba
,
,
,
,
`
c Chebyshev ta c
da thu
ung du. ng d
ay theo yu cu. Mt
u y quan
uo. c d
. ch
,
,
,

`
c Chebyshev c
ng dung
trong
l`
a nhung da thu
ou
rt nhiu trong l thuyt
.
.
,
,
,
`ng da thu
c.
xp x nhung h`
am ba
, ,,
,
,
,
,
: Nhung phuong ph
Ch
uy
ap trn xut ph
at t`u y tuong sau dy: Cho mt
.
,
,

h`
am s f nhn
v
h
an
c
a
c
gi
a
tr
i
v`
a
sao
cho
v
o
i
m
oi
s
t
u
nhin
n,
f(nt)
=
.
.
.
.
.
,,
,

,i nhau.

Qn (f(t)), o dy Qn (x) l`
ad
a
th
u
c.
Khi
d
o
Q
v`
a
Q
giao
ho
an vo

m
n
,,
,c giao ho
V du,
am f(t) = et th` ta nhn
ay c
ac da thu
an
uo. c d
. nu cho h`
. d
,
,
,
c Pn nhu,
Qn (x) = xn . Nu f(t) = et + et , th` ta nhn
ay da thu
uo. c d
. d
,
`ng
trong trong v du. d
ay (ch c
o kh
ac l`
a thay t ba
`u tin trong chuyn d
` n`
,,
,
,
,
,
t
c Tn nhu o trn.
e ). Nu f(t) = cos t, th` ta nhn
ay da thu
uo. c d
. d
,
,,
,c Pk bc
Phuong ph
ap 3: T`u v du. 4.35, suy ra t`n tai
a thu
. duy nht d
. k
,
,
,
2
c nhu vy:
i P2 (x) = x 2. Ta vit mt
giao ho
an vo
. s da thu
.
P1 (x) = x,
P2 (x) = x2 2,
P3 (x) = x3 3x,
P4 (x) = x4 4x2 + 2,
P5 (x) = x5 5x3 + 5x,
P6 (x) = x6 6x4 + 9x2 2,

,c giao ho
4.3. Chuyn d
ac da thu
an
` v` c

199

,,
,
`ng c
od
a P5 (x) t`m ba
ach nhu
y P4 (x) = P2 (P2 (x)), P6 (x) = P2 (P3 (x)) v`
,
,
,
,
,
,c trn ta c
,c
ng thu
trong v du. 4.34. T`u nhung d
o th gia thit d
ay d
a
a thu
,
,c h`i quy sau:
thoa m
an cng thu
Pk+1 (x) = xPk (x) Pk1 (x).
,
,ng minh cng thu
,c n`
,i moi
ng vo
Ta c
o th chu
ay d
ac dinh
ay
u
. d
. k > 1. X
,
2
`
c P1 , P2 , ..., Pn , ... ba
,i
ng quy nap:
da thu
on vo
. P1 (x) = x, P2 (x) = x 2, c`
,
`ng tt ca da thu
,ng minh ra
,c
k > 1, Pk+1 (x) = xPk (x) Pk1 (x). Ta se chu
,
c P2 ; khi do
,i d
ch
Pk giao ho
an vo
ung giao ho
an c`
ung nhau (do v du.
a thu
4.37).
`ng phu,o,ng ph
,ng minh ba
Ta chu
ap quy nap.
a P2 (x) giao
. D thy P1 (x) v`
, ,,
,
,
,

ho
an voi P2 (x). Gia su tt ca c
ac d
an
a thuc P1 (x), P2 (x), ..., Pk (x) giao ho
,
,ng minh Pk+1 (x) c
,i P2 (x), ta phai chu
,i P2 (x). Ngha l`
vo
ung giao ho
an vo
a
,
,
,
,
ng minh cng thu
c giao ho
ta phai chu
an giua Pk+1 (x) v`
a P2 (x) = x2 2
l`
a:
Pk+1 (x2 2) = (Pk+1 (x))2 2.
,
,,
,i Pk+1 v`
Tht
ao v`
a bin d
i v` dang
. vy,
. ta thay phuong tr`nh h`i quy vo
.
,
ng minh l`
ta c`n chu
a
(x2 2)Pk (x2 2) Pk1 (x2 2) = (xPk (x) Pk1 (x))2 2.
,
,
,,
`ng c
Ba
ach khai trin v`
a dua v` c
ac s hang
. chung ta nhn
. duo. c
2([Pk (x)]2 xPk (x)Pk1 (x) + [Pk1 (x)]2 ) = 2x2 8
ngha l`
a
[Pk (x)]2 xPk (x)Pk1 (x) + [Pk1 (x)]2 = 4 x2 .

(4.29)

,
,
,ng minh Sk (x) khng
Ta k hiu
ai cua (4.29) l`
a Sk (x). Ta ch c`
on chu
. bn tr

,,
,,
Chuong 4. Phuong tr`nh h`
am

200

`ng 4 x2 . Tht

phu. thuc
ao k v`
a ba
o
. v`
. vy,
. cho k > 2. Khi d
Sk (x) = [Pk (x)]2 xPk (x)Pk1 (x) + [Pk1 (x)]2 =
= Pk (x)[Pk (x) xPk1 (x)] + [Pk1 (x)]2 =
= Pk (x)Pk2 (x) + [Pk1 (x)]2 =
= [Pk2 (x)]2 xPk1 (x)Pk2 (x) + [Pk1 (x)]2 = Sk1 (x),
,i moi
ngha l`
a Sk (x) = Sk1 (x). V` S2 (x) = 4 x2 , nn Sk (x) = 4 x2 vo
.
,
,
,,
,
,

an voi P2 (x) voi


k 2. Nhu vy
a chung minh d
uo. c Pk (x) giao ho
. ta d
moi
. k.

,
,
v` cac
da

` toan
tng quat
uc giao hoan
4.3.3. Bai
th
,
,
,c P(x) (khng nht thit phai l`
Khi cho da thu
a unita) b`
ai to
an tng
,
,
,c Q(x) (khng nht thit phai l`
. t ra l`
qu
at da
a t`m nhung d
a unita)
a thu
,
,`,
,
,
,
,

giao ho
an voi n
o. Truong ho. p d
a c
ac d
o bc
a 1 th` ban
on gian l`
a thuc c
. l`
.
,
,

doc
ay giai b`
ai tp
a b`
ai tp
a Q(x)
. h
. 4.38 v`
. 4.39. Tip tuc,
. ta gia thit P(x) v`
,
,
`

du c
o bc
. lon hon 1.
Cho
P(x) = a0 xk + a1 xk1 + + ak1 x + ak

v`
a

Q(x) = b0 x` + b1 x`1 + + b`1 + b` .


, ,
,
`ng 0,
,c trn v` dang
Ta c
o th dua nhung d
c
ac h. s a1 v`
a b1 b a
a thu
.
,
,,
,c nhu, vy
,c tinh
nhung d
a da thu
a thu
uo. c tin
. tin d
. tin. Vic
. goi
. l`
. tinh
,
,
,c mo
,i n
,i
h`
anh nhu sau: Ta c dinh
a vo
o ta xy du. ng mt
a thu
. mt
. s a v`
. d
,
,
,c da
,c: R(a) (x) = R(x a) + a.
cho R(x) theo cng thu
R(a) t`u da thu
,c R(x) t`u, R(a) (x) , ta d`
,c R(x) = R(a) (x + a) a.
ly lai
ung cng thu
. da thu
V du. R(x) = 2x2 + 3x + 5 th` R(a) (x) = 2(x a)2 + 3(x a) + 5 + a =
,
,
2x2 4ax + 2a2 + 3x 3a + 5 + a, d a1 = 3 4a = 0 suy ra ta phai c
dinh
. a=

3
4

v`
a R(a) (x) = 2x2 +

37
8 .

,c giao ho
4.3. Chuyn d
ac da thu
an
` v` c

201

`ng nu P(x) v`
,ng minh du,o.,c ra
,c giao ho
D chu
a Q(x) l`
a hai da thu
an
th` P(a) (x) v`
a Q(a) (x) c
ung giao ho
an.
,
`ng P(x) v`
,c tinh
Ph`n sau d
a Q(x) l`
a nhung da thu
tin,
y ta cho ra
.
,,
ngo`
ai ra ch
ung c`
on l`
a unita, ngha l`
a ch
ung c
o h. s o s hang
bc
.
. cao
,
,
,
,,
,
`
`

`
ng 1. Ba
ng c
nht ba
ac l lun
an to`
an tuong tu. nhu phn truoc ta t`m
. ho`
,,
,
,
`ng 1. Ta lit

duo. c d
ay d
an voi h. s bc
a thuc giao ho
. cao nht ba
. k lai
.
mt
. s v du:
.

,c bc
,c P0 (x) = x, P1 (x) =
A. Cho R(x) l`
a mt
ay d
a thu
. da thu
. r. Xet d
R(x), P2 (x) = R(R(x)), P3 (x) = R(R(R(x))), ... Pi+1 (x) = Pi (R(x)).
,
`ng c
,c Pi (x) giao ho
,i nhau v`
D thy ra
ac d
an vo
a bc
a thu
. cua Pi (x)
`ng ri .
ba
,
,c Fk (x) = xk . Tt ca d
,c Fk (x) giao ho
B. Cho d
ay d
an di mt
a thu
a thu
.
,
`
ng k.
v`
a bc
. cua Fk (x) ba
,
1
k
k
c dang
C. D
ay d
an d
a bc
a thu
i mt
. Pk (t + t ) = t + t giao ho
. v`
.
,
`ng k.
cua Pk (t) ba
`ng cng thu
,c Hk (t t1 ) = tk + tk (k
D. inh
ay H1 , H2 , .... ba
. ngha d
,
,
,ng minh du,o.,c d
,c trn t`n tai
l`
a s le). D chu
ay d
a x
ac d
Nhung
a thu
inh.
.
. v`
,
,c n`
da thu
ay c
ung giao ho
an di mt
a bc
a k.
. v`
. cua Hk (x) l`
,u v` da thu
,c ngu,`o,i ta c
E. Trong dai
ung hay nghin
. s, khi nghin cu
,
,
,
,
,
,
,

u nhung d
c c
c (ban
cu
o h. s l`
a nhung s phu
o th tham khao
a thu
. doc
. c
,
,c sao
trong cun s
ach [8]). Ta c dinh
a mt
. mt
. s tu. nhin m v`
. s phu
,
,c th` vo
,i moi
cho m = 1 (theo tnh cht cua s phu
. m, t`n tai
. m 1 s
,
nhu vy
ac 1).
. kh
,
,
. p s (u, v) sao cho uv = , v`
. t x = u + v. D kim
Ta xet nhung ca
a da
,,
tra duo. c
uk + vk = xk + a1 xk1 + + ak ,
,,
,
. t Pk (x) =
phu. thuc
o dy a1 , a2 , ..., ak l`
a nhung s n`
ao do
ao . a
. v`

202

,,
,,
Chuong 4. Phuong tr`nh h`
am

,
`ng d
,c Pk (x) d
. c trung ba
xk +a1 xk1 + +ak . a thu
a
i`u kin:
. Pk (u+v) =
k
k
u + v nu uv = .
,
u` v` = (uv)` = ` = ,
Cho s ` sao cho n
o chia cho m c`
on du 1. Khi do
,
`
`
kl
kl
vy
a ` chia cho m du 1 th`
. Pk (u + v ) = u + v . Vy
. th` nu k v`
Pk (P` (u + v)) = ukl + vkl = P` (Pk (u + v)), ngha l`
a Pk (x) v`
a P` (x) giao
,
,,
,
,

,c giao

xy du. ng duo. c theo s d


ho
an voi nhau. Ta da
ay nhung da thu
,
,
,c n`
ho
an P1 (x), Pm+1 (x), P2m+1 (x), ... tt ca nhung da thu
ay l`
a unita, tinh
.
,

tin v`
a bc
a k.
. cua Pk (x) l`
, ,
,
i m = 1, = 1 ta c
,i m = 2, = 1
. c bit
a
o kt qua cua ph`n A, vo
. vo
, ,
ta c
o kt qua cua ph`n D.
,
,
,
C
ac v du. t`u ph`n A d
ay bao h`
am tt ca nhung
n ph`n E trong ph`n n`
,
,
,
`ng nu P(x) v`
,c giao ho
da thu
an. Ta c
o th gia thit ra
a Q(x) l`
a nhung da
,
,c giao ho
,c,
,n ho,n 1 vo
,i h. s l`
thu
an, unita, tinh
a nhung s phu
. tin, bc
. lo
,
,,
,
,
,,
,
,
`m o, mt
th` ch
ung na
ay d
uo. c xy du. ng o ph`n A, B, E o
. trong nhung d
trn.

` tp
Bai
.
`ng hai da thu
,ng minh ra
,c bc
,i
. 4.38. Chu
a Q(x) giao ho
an vo
. nht P(x) v`
,
. c l`
. c l`
nhau khi v`
a ch khi hoa
a P(x) = x + , Q(x) = x + , hoa
a t`n tai
. mt
.
,
x0 sao cho P(x0 ) = Q(x0 ) = x0 (d
im bt d
ng).
.
,
,c bc
,c Q(x) giao
. 4.39. Cho P(x) l`
a da thu
ay t`m tt ca c
ac d
a thu
. 1, h
,i P(x).
ho
an vo
`ng nu hai da thu
,ng minh ra
,c P(x) v`
. 4.40. Chu
a Q(x) giao ho
an th`
,
(a)
(a)
c giao ho
P (x) v`
a Q (x) c
ung l`
a hai d
an.
a thu

, ,
,,
,
`i b`
4.4. Go. i y v`
a tra lo
ai tp
. chuong 4

203

, ,
,,
,
` tra l`oi bai
` tp
4.4. Go. i y
va
. chuong 4
,
x2 a
`ng 1 . H`
am kt qua l`
a f(x) =
.
. 4.12. Thay x ba
x
(1 a2 )x
,,
`ng: x = 0, y = z;
. 4.13. Thay l`n luo. t c
ac gi
a tri. x v`
a y ba

,,
,,
x = + z, y = ; x = , y = + z v`
a nhn
uo. c phuong tr`nh
. d
2
2
2
2

f(z) = f(0) cos z + f( ) sin z.


2
,
,,
,,
Giai phuong tr`nh h`
am n`
ay, ta nhn
. duo. c nghim
. f(x) = a cos x + b sin x,
,,
,
`

od
a nhung hang s bt k`.
y a, b l`
,,
,
. 4.14. D`
ung phuong ph
ap bt bin nhu trong b`
ai 4.3. Nghim
a f(x) =
. l`
2
x + c.
,i: x = 0, y = z; x = 0, y = z; x = z, y = 0
. 4.15. Thay x v`
a y lin tip vo
,,
,,
,
v`
a phuong tr`nh nhn
ua v` f(z) = z + 1.
. duo. c d
h x i2
x
x
,
am f1 (x) = 0
. 4.16. T`u f(x) = f( + ) = f( ) suy ra f(x) 0. H`
2
, 2,, 2
,
i f1 (x) = 0. Khi d

l`
a mt
Gia su c
o nghim
ac f(x) kh
ac vo
o
. nghim.
.
. kh
,

f(x) > 0 voi moi


o mt
a tri. x = x0 sao cho f(x0 ) = 0,
. x. Tht
. vy,
. nu c
. gi
,
,,
,,
`
cho ta nhn
tu phuong tr`nh da
uo. c
. d
f(x0 + y) = f(x0 ).f(y) = 0.f(y) = 0
, ,
,c cui c`
,i moi
,i moi
ng thu
vo
ung suy ra f(x) = 0 vo
a
a
. y. T`u d
. x, ngha l`
,
`
i d
. t ra.
f(x) = f1 (x) v l vo
iu ta da
,,
,
cho v`
Lgarit co s 10 hai v phuong tr`nh h`
am d
a h`
am s lgf(x)
a
,
,
,,
,,
thoa m
an phuong tr`nh (4.13). Nghim
a f(x) = 0 v`
a
. cua phuong tr`nh l`
f(x) = ax .
,
,,
. 4.17. H`
am lgf(x) thoa m
an phuong tr`nh (4.18) v`
a c
o nghim
. f(x) = 0
a
v`
a f(x) = x .
,,
. 4.18. D`
ung phuong ph
ap Cauchy v`
a nghim
a f(x) = a.x2 .
. l`

204

,,
,,
Chuong 4. Phuong tr`nh h`
am

,
,
,,
. 4.19. Mt
a (x) = cotgx. Kt qua l`
a
. nghim
. ring cua phuong tr`nh l`
f(x) = a + cotgx.
,
,
,,
. 4.20. Mt
a (x) = x. Kt qua f(x) =
. nghim
. ring cua phuong tr`nh l`
x + ax .
,i n R, xet y = x, y = 2x, ..., y = (n 1)x.
. 4.27. Vo
,
,
,
. 4.31. H
ay ch ra f ch c
o dim bt dng
. duy nht tai
. 0.
, ,,
,
ch ra x > 0 keo theo f(x) > 0 v`
. 4.32. Gia su f(0) = 0, khi d
a f t
ang.
o
,, ,
,
i y = cx vo
i c kh
. 4.33. Thu vo
ac nhau thuc
a y = x 2.
. Q v`

, ,
CHUONG

,
,

`
HOC
NHUNG TRO CHOI TOAN
.

5.1. Chin thut


. bt bin . . . . . . . . . . . . . . . . . . . . . . . . . . . . . . . . . . . . . . . . . . . . . . . . . . .
,
,
,
vo
` choi . . . . . . . . . . . . . . . . . . . . . . . . . . . . . . . . . . . . . . . . . . . . .
i tro
5.2. Kin th
uc toan
,
,
` toan
thi hoc
5.3. Nhung bai
. sinh gioi . . . . . . . . . . . . . . . . . . . . . . . . . . . . . . . . . . . . . . .
,
` choi Nim . . . . . . . . . . . . . . . . . . . . . . . . . . . . . . . . . . .
5.4. Chuyn d
` chin thut
. tro

5.4.1. inh
a v du. . . . . . . . . . . . . . . . . . . . . . . . . . . . . . . .
. ngha v`
,
5.4.2. Chin thut
trong
tr`
o choi Nim . . . . . . . . . . . . . . . . . . .
.
,
,ng minh bang chin thut
,ng dung
5.4.3. Chu
au
n
o ....
. v`
.

206
210
214
225

225
229
236

,
` liu
Tai
. tham khao . . . . . . . . . . . . . . . . . . . . . . . . . . . . . . . . . . . . . . . . . . . . . . . . . . . . . . . . . .
,
Nhung k hiu
. ...............................................................

241

Muc
. luc
. .....................................................................

242

240

,
,
,
,
ng hoa
. c
`i hoi nhung chin thut
C
o rt nhi`u tr`
o choi to
an hoc
o
. d
. d tha
, ,
,
,
,,
,
,
`i hoi mt
d bao v. buoc di trong cuc.
o choi n`
ay c
o th d
o
. Nhung tr`
. s
,
,
,
,
. c nhung phep suy ra cua
tnh cht l thuyt cua tp
` hoa
. ho. p, lgic mnh
. d
, ,
,
,,
,
,
to
an t ho. p. Mt
o choi c
o th c
o v han
c di, lai
o nhung tr`
o
. s tr`
. buo
. c
,
,
,
,
,
,
,

`
c. Mt
ng l`
choi phai d`ung sau huu han
a mt
. buo
. chin thut
. tha
. so d cho
,,
,
,,
,`,
,
,,
ng lun lun thu. c hin
a buo
c
phep nguoi tha
i, trong thu. c t l`
. duo. c buoc d
,
,
,
,,

. n buoc d
ng cua d
di cui c`
ung. Mt
o th t`m
i tha
i thu c
. chin thut
. cha
,
,
,
`

`
c to
ng kin thu
thy ba
an hoc.
o choi to
an hoc
. ngha v tr`
. Mt
. dinh
. nhu
sau:

,,
,
,ng tr`
Chuong 5. Nhu
o choi to
an hoc
.

206

,,
,,
,
,
an hoc
inh
o choi to
. thu`ong g`m mt
. s ngu`oi choi sao
. tr`
. ngha 5.1. Mt
cho:
,
,,
,
,
,,
,
1) Nhung ngu`oi choi thay phin nhau thu. c hin
c di trn
. nhung buo
,
,,
,,
,,

c cho truoc;
huu han
a theo mt
i tuo. ng dm duo. c v`
. d
. quy ta
,`,
,
,
,
,,
,
,
c di l`
ng cuc.
2) Nguoi cui c`
ung thu. c hin
a ngu`oi tha
uo. c buo
. d
.
,
,
,
,u hai kha
Nhu vy,
ai to
an tr`
o choi ta phai nghin cu
. khi xem xet mt
. b`
c cho,i nhu, th n`
, nht l`
, hai l`
canh
quan trong:
Thu
a quy ta
ao; thu
a chin
.
.
,
,
,
,
,
,

i mi ngu`oi choi. C
ng han
thut
ao d
o rt nhi`u v du,
i vo
. nhu th n`
. cha
.
,
,
nhu tr`
o choi sau:
,
,
,
,
C
o ba d
ng soi g`m dng 1 vin soi, d
ng 3 vin soi, d
ng 5 vin soi. Hai
,
, ,
,,
,,
.
ngu`oi l`n luo. t ly di mt
. c mt
ao d
o
. dng soi hoa
. s vin soi cua mt
. dng n`
,
,`,
,
,

Nguoi n`
ao ly vin soi cui c`
ung l`
a ngu`oi y chin tha
ng.
, ,
,
, , ,
,
Tr`
o choi c
o th thu. c hin
a ngu`oi thu
nht v`
a II l`
a
. nhu sau: K hiu
. I l`
,`, ,
nguoi thu hai.
I

II

II

(1, 3, 5) (1, 3, 2) (1, 1, 2) (1, 1, 0) (1, 0, 0) (0, 0, 0),

,
, , ,
,,
,
,
ng. y l`
a dang
o choi Nim d
oi k
v`
a ngu`oi thu
nht tha
on gian se duo. c n
. tr`
,
,
,
trong chuyn d
ay.
` cua chuong n`
,,
, , ,
. t ra l`
Thu`ong mt
ai to
an da
a t`m mt
nht
. b`
. chin thut
. cho ngu`oi thu
,`,
,
,
,
,
ng. Ph`n thu
hai tha
nht ta xet nhung b`
. c nguoi thu
hoa
ai to
an tr`
o choi
,
,,
,
tuong di don gian.

5.1. Chin thut


. bt bin
,
,
,
Mt
ac lut
a thu. c hin
am sao
. chin thut
. trong khi thu. c hin
. c
. choi l`
. l`
, ` ,
,
,
,
,
,
,
,

dua v nhung du kin


o lo. i cho ngu`oi choi.
o c
ung l`
a nhung l`oi
. bt bin c
, ,
,
giai cua c
ac b`
ai to
an tr`
o choi to
an hoc.
.

5.1. Chin thut


. bt bin

207

,,
, ,
oi 2001 hat
a ly
V du. 5.1. A v`
a B tin h`
anh tr`
o choi v
u.
. d
. Mt
. nuoc di l`
,
,
,,
,
khoi d
. c 3 hat
a thay phin nhau. Ngu`oi
ng hat
u
. d
. 1, 2 hoa
. du.
. A di truoc v`
,,
,,
,,
n`
ao ly d
ung l`
a ngu`oi chin tha
ng. Vy
ao c
o chin
uo. c hat
u
. d
. cui c`
. ngu`oi n`
,
,
,

nhu th n`
thut
ng trong cuc
a chin thut
ao?
lun tha
o
. d
. choi v`
. d
,
,,
ng nu A thu.,c hin
`,i giai. A lun lun tha
Lo
. chin thut
. sau: Khoi d`u A
,,
,,
ly 1 hat
c tip theo A se ly d
a s hat
i 4 x hat,
y x l`
u
. du,
. nuo
. od
. d
. B
,, , ,
,
,

`
`
c d
c d
. Tht
ly o nuo
da
on lai
i truo
o
. vy,
. sau khi A di ln du tin, c`
. 2000

`
`

hat
u.
. d
. Tip theo, theo chin thut
. trn th` sau mi ln B ri dn A di,
,
`

ng bi
dng hat
on lai
ung d
n
. du
. lun c`
. s hat
. ba
. s cua 4. Do vy,
. cui c`
,,
,
,,
,,
c
on lai
u B thu. c hin
ach n`
ao th` A c
ung d
luo. t B th` c`
i d
uo. c nuo
. 4 hat.
. D`
. c

ng cuc.
cui se ly di ht s hat
a A tha
. du
. v`
.

,
,
,
V du. 5.2 (My 1999). Tr`
o choi Y2K trn dai b
ang c
o vung kch co 12000.
,,
,,
, `
,
,ng trng hoa
ao nhu
. c l`
a S hoa
. c l`
a O. Ngu`oi choi d
Hai ngu`oi choi vit v`
u
,
,

SOS se tha

tin tao
anh chu
ng cuc.
ac d
a
. ra ba lin tip th`
. Nu tt ca c
,
,,
,
`
`
d
a khng tao
o choi goi
a h`
oa. Chung minh
in d
y m`
uo. c SOS th` tr`
. ra d
. l`
,`,
, ,
`

ra
ng nguoi choi thu hai c
o mt
ng.
. chin thut
. tha
,
`,i giai. Goi
. t S hoa
. c O v`
Lo
a xu nu khi d
ao n`
ay se tao
a
. mt
. trng l`
. cho
,`,
,,
,,
,

nguoi choi tip theo nhn


i tip theo.
. duo. c SOS trong luo. t d
,
,
B d`: Mt
a xu khi v`
a ch khi n
o na
`m trong khi 4 vung
. vung l`
,,
lin tip c
o dang
a mt
y l`
. S S, o d
. trng.
,
,,
Tht
a xu. Nguo. c lai,
. vy,
. d thy nhung vung trong S S l`
. nu
vung l`
a xu, th` khi ta vit O se tao
. di`u kin
. vit SOS trong l`n tip
,
,
,,
,`,
,
,
theo cua nguoi choi kh
ac. Nhu vy
o mt
v`
a mt
. xu phai c
. S o bn canh
.
.
,,
,, , ,

trng o pha bn canh


kia.
Khi
ta
vi
t
S
s
e
b
i
thua
ngay
o
bu
o
c
ti
p
theo,
.
.
,
,
,
. t k tu. S kh
ngha l`
a ta phai d
ac bn canh
kia
cua trng, tao
a
.
. ra dang
.
,
S S l`
a nhung xu.
,,
,
,
ng
, hai c
By gi`o ta quay lai
ai to
an, ngu`oi choi thu
o chin thut
. b`
. tha

208

,,
,
,ng tr`
Chuong 5. Nhu
o choi to
an hoc
.

,
, , ,
, , ,
,
choi, ngu`oi thu
nhu sau: Sau khi ngu`oi thu
nht da
hai vit S tai
ach
. vi. tr c
,
,
,
,
. c t`u pha d`u tin m`
xa t nht 4 trng t`u pha d
ang hoa
a
`u cua dai b
,`, ,
,
,
,
,
,
,
,
hai, ngu`oi thu
hai se da
c thu
. t S c
choi. n buo
nguoi thu nht da
ach xa
,
,, ,
,
,
,,
,
,
`

c do
sao cho o giua l`
choi cua m`nh ln truo
ba t`u d
a trng. (Nu buo
c
a
,
,
,
,
,
,
,
,
,

`
hai cua ngu`oi thu
nht o bn canh
. c c
di thu
hoa
ach mt
.
. t`u choi ln
,
,
,`, ,
,`, ,
,
`
hai o, canh
. t S thu
du cua nguoi thu hai, th` nguoi thu hai se d
bn kia.)
a
.
,
,
,
,,
,
,
,
,
`

hai cua ngu`oi thu


hai, tn tai
Sau buoc thu
ang. Nhu
. 2 xu trn giai b
,
,,
,
,
trong hai ngu`oi choi phai di`n v`
vy
ung ai d
ao nhung n`
ay v`
a tr`
o
o
. cui c`
,
choi se khng h`
oa.
,,
,,
,
,
, hai dn lu,o.,t, lun lun
Tai
c choi kh
ac, khi ngu`oi choi thu
. bt k` buo
,
,
,,
, ,
n xu, nhu, vy
c
o s le trng v`
a s cha
hai c
o th lun
. ngu`oi choi thu
,
,
lun choi v`
ao khng phai l`
a xu.

,
,,
,
V du. 5.3. C
o mt
an vung v
oi s vung trn c
ac canh
l`
a le. Hai ngu`oi
. b`
.
,
,,
,,
,
,
,ng d
ao
choi l`n luo. t d
. t nhu
ao c
ac vung cua b`
an. Ngu`oi choi n`
a
`ng xu v`
,
,
,
,,
khng c`
on ch d
. t d
a thua trn.
ay ch ra ra
`ng ngu`oi
da
`ng xu coi nhu l`
. H
,
,
,
choi thu nht c
o th lun lun chin tha
ng.
,
,
,
`,i giai. V` c
ac canh
c
o s le vung nn t`n tai
Lo
.
. mt
. vung tai
. tm cua
,,
,,
,,
, ,
, ,
. t d`ng xu tai
b`
an. Ngu`oi choi thu
nht d
an. Nu ngu`oi choi thu
a
. o tm b`
,
,, `
, , ,
. t d
. t d`ng xu
nht c
o th d
hai c`
on da
ao mt
a
uo. c d
ng xu v`
. , th` ngu`oi thu
,
,
,
,
ng qua d`ng xu o tm b`
, hai
i vi. tr ngu,`o,i thu
tai
an (di din
. vi. tr di xu
. vo
,
,
,
,
. t d`ng xu). V` din
. t b`
d
d
an c`
on trng giam, tr`
o choi phai
a
a
. tch cua ma
,,
, ,
ng.
kt th
uc. Ngu`oi choi thu
nht se tha

,,
,
, ,
V du. 5.4 (Tr`
o choi Bachet). Khoi d
an c
o n qun c`o v
oi n > 0.
`u, trn b`
,`,
, `
,
,

`
Hai nguoi choi mi ln choi ly d
a nhiu nht k qun
i t nht 1 qun c`o v`
,
,
,,
,,
,,
,
,
c`o. Ngu`oi cui c`
ung c
o th ly d
on lai
an se d
uo. c tt ca qun c`o c`
uo. c
. trn b`
,
,,
,
,,
,
,
,ng gi
goi
a ngu`oi tha
ng cuc.
oi nhu
a tri. n`
ao cua n ngu`oi choi thu nht c
o
. l`
. V
,
,
,
,
,
,
,

ng gi
mt
ng cuc?
oi nhu
a tri. n`
ao cua n ngu`oi choi thu hai
. chin thut
. tha
. V

5.1. Chin thut


. bt bin

209

c
o mt
ng cuc?
. chin thut
. tha
.
,
, ,, ,
,
,
,
`ng
`,i giai. Ta c
i nhu,ng tru,`o,ng ho.,p d
Lo
o th thu vo
on gian cua n, ta thy ra
,
,
,
,
ng,
nht c
ngu`oi choi thu
nu n 6 0 (mod k + 1), khi do
o chin thut
. tha
, ,
,,
,`, ,
ng. chu
ng minh d
nguo. c lai
o chin thut
ay,
i`u n`
. nguoi thu hai se c
. tha
,
cho nt l`
a s qun c`o trn b`
an.
, nht c
,i 0 < rt < k + 1, th` ngu,`o,i cho,i thu
Nu nt = (k + 1)qt + rt vo
o
,
,

`
`
ng ba
ng c
ng do rt > 0 nn
th tha
ach mi l`n ly di rt qun c`o. (Ch
u y ra
,
,
,
,
,
,
,
,

nht ly nhu vy
hai xut ph
,i bi
mi l`n ngu`oi thu
at vo
. th` ngu`oi thu
. cua
,
,
,
, , ,
,
,
k + 1 qun c`o v`
a l`n n`
ao c
ung nhu vy
hai ch c
o th ly d
i t`u 1
. ngu`oi thu
,
dn k qun c`o).
,,
,
, nht c
Nu n = (k + 1)q, th` ngu`oi thu
o ly bao nhiu qun c`o di,
,
,,
,
ng ba
`ng
, hai c
. ng han
cha
o th tha
. m qun c`o (1 m k) th` ngu`oi thu
,
,
l`n choi sao cho m + q = k + 1.
c
ach ly q qun c`o mi

` tp
Bai
.
,
,
,
ngu,`o,i cho,i c
. 5.5. Trong tr`
o choi Bachet, nu lut
o
. choi thay di cho mi
,
,
`

. c 2 hoa
. c 4 hoa
. c 8 hoa
. c 16 hoa
. c ... (mt
th mi ln ly s qun c`o l`
a 1 hoa
.
,
,
,

ng?
s l
uy th`ua cua 2), th` ai c
o th tha
,
,
,
,
ngu,`o,i c
. 5.6. Trong tr`
o choi Bachet, nu lut
o th
. choi thay di cho mi
,
, `
ng?
. c mt
ly d
ln, th` ai c
o th tha
i mt
. hoa
. s nguyn t qun c`o mi
,,
,
,
. 5.7. Khoi d
o mt
oc trn b`
an c`o n n vung. Hai
`u c
. qun c`o tai
. mt
. g
,
,,
,
,
ngu`oi choi A v`
a B thay nhau di chuyn qun c`o sang (mt
. vung) bn
,
,
,
,,
,`,
,

. c xung duoi. Nguoi choi khng th di chuyn


tr
ai, bn phai, ln trn hoa
,
,,
,,
,i vung da
di qua r`i. Ngu`oi thua cuc
to
a ngu`oi cui c`
ung khng th
. l`
,,
,
,,
ng khi n l`
n? Ngu,`o,i n`
ng khi
d
ao se tha
a s cha
ao se tha
i duo. c nua. Ngu`oi n`
,
,
,,
ng nu qun c`o, kho,i d`u t`u, mt
n l`
a s le? ngu`oi n`
ao se tha
. vung bn
,,
,
canh
vung o g
oc b`
an c`o?
.

210

,,
,
,ng tr`
Chuong 5. Nhu
o choi to
an hoc
.

,,
, , , ,
,i hai dng qun c`o, g`m p v`
. 5.8. Khoi d`u vo
a q qun c`o tuong u
ng. A
,
,
,
,
,
,,
,
,
,
c di. Ngu`oi choi thu. c hin
v`
a B thay nhau thu. c hin
cd
i
. nhung buo
. buo
,
,
,
,
v`
bao g`m ly di mt
ao d
a chia nhung qun c`o cua dng
ng qun c`o n`
o
. d
, `

ng nhau).
kia th`
anh hai dng (khng nht thit hai d
o s qun c`o ba
ng c
,
,`,
,`, , ,
,

Nguoi thua cuc


a nguoi dn luo. t khng c`
on dng qun c`o c
o th chia
. l`
,
,,
,,
,
ng? (Go.,i y: Kt qua phu. thuc
n
duo. c. Ngu`oi n`
ao se tha
ao nhung tnh cha
. v`
, ,
le cua p v`
a q).

,
,
,
vo
` choi
i tro
5.2. Kin th
uc toan
,
,,
,
, s d
V du. 5.9. Hai ngu`oi choi thay phin nhau vit lin tip c
ac chu
cui
,
,,
,
, s, theo quy d
c`
ung ta c
o mt
o 10 chu
Ngu`oi thu nht vit chu
inh:
. con s c
.
,
,
,
,,
,
,,
, s thu, hai, lai
s thu nht, ngu`oi thu hai vit chu
. ngu`oi thu nht vit s thu
,
,,
,,
,
ba,... Ngu`oi thu hai mun c
o mt
on ngu`oi thu nht chng
. s chia ht cho 7, c`
,
,
,,
,
,
lai
ay. Vy
a choi nhu th n`
ao d
d
at
uo. c y mun cua m`nh?
. y mun n`
. ai v`
. d
,
,
`,i giai. Ngu,`o,i thu
, hai l`
Lo
a vit s cui c`
ung quyt d
o
inh
. kt qua s y c
,
,
,
,
,
nht vit chu s cui c`
chia ht cho 7 khng. Khi ngu`oi thu
ung cua m`nh,
,
,`, ,
,,
,
, mu,`o,i, ngu,`o,i
th` nguoi thu hai nhn
o 9 chu s. t`m s thu
. duo. c mt
. s c
, , ,
, hai thm s 0 v`
(ngha l`

thu
ao s d
a nhn s thu duo. c vo
i 10), sau do
o
,
,
,
d
o s du l`
a r, th` s c`n
em chia cho 7. Kt qua thu d
uo. c sau phep chia c
,
,
,,
,
,
hai l`
vit thm v`
ao cua ngu`oi thu
a 7 r. y c
ung l`
a chin thut
. ngu`oi di
ng.
sau lun lun tha

,
,,
cho sa
V du. 5.10. Trn bang d
n phuong tr`nh
a
x2 + x + = 0.

,,
,
,,
, ,,
Ngu`oi thu nht trong hai ngu`oi choi d
uy y v`
a sau
uo. c quy`n chon
. ra ba s t`
,,
,`, ,

nguoi thu hai d


d
d
ac s d
ao c
ac du trong phuong tr`nh trn
o
em c
o
in v`
,
,,
,`, ,

mt
ach t`
uy y. Nguoi thu nht se tha
ng cuc
uo. c mong mun
. c
. nu thu. c hin
. d

,
,
,c to
5.2. Kin thu
an voi tr`
o choi

211

,
,,
,,
,u t phn
o hai nghim
l`
a cui c`
ung nhn
uo. c mt
. phuong tr`nh bc
. hai c
. hu
. d
,
,,
,
,,
,
,,
cua ngu`oi thu nht. Ngu`oi
bit.
ach chng lai
inh
o
. Ngu`oi thu hai t`m c
. y d
. d
,
,
,
,
,
,
,,
thu nht se phai chon
ac s nhu th n`
ao d
thu. c hin
. c
. duo. c mong mun cua
m`nh?
,
,
, , ,
`,i giai. Goi
Lo
a c
ac s m`
a ngu`oi thu
nht chon,
. a, b, c l`
. th th` anh ta phai
`ng 0 th` bi.
,i a, b, c 6= 0, (v` nu mt
chon
. sao cho a + b + c = 0 vo
. s ba
,`,
,
,
,
,
hai ly s do
nht v`
g
nguoi thu
an v`
ao h. s thu
a phuong tr`nh khng
,,
,
,
,
,
nht se thua). Nhu, vy,
c`
on l`
a phuong tr`nh bc
a ngu`oi thu
. hai nua v`
.
, , ,
,,
nhn
, du n`
ngu`oi thu
hai d`
u c
o th a, b, c v`
ao bt cu
ao th` ta vn
d
u
. o. c
, ,
,,
mt
a theo h. qua cua d
l Viete th` lun c
o
inh
.
. phuong tr`nh bc
. hai m`
,
c
,
hai c
. c nghim
hai nghim
o th l`
a
. phn bit
. x1 = 1, x2 = a (hoa
. thu
b
c
a
, , ,
x2 = , x2 = , x2 = ,... phu. thuc
ao ngu`oi thu
hai g
an a, b, c v`
ao vi.
. v`
a
b
b
tr n`
ao).

,
,
V du. 5.11. Trong mt
od
a trong mt
u. ng 15 qua c`u xanh, v`
. chic hp
. c
.
,
,`,
,,
,

hp
ac c
o 12 qua cu tra
ng. Hai nguoi choi, trong mi luo. t d
i, ho. buc
. kh
. ly
,
,
,`,
,`, , ,

`
`
d
. c 2 qua cu tra
ng. Nguoi tha
ng cuc
a nguoi ly d
i 3 qua cu xanh hoa
uo. c
. l`
,
,
,
,`, ,
,
`

nhung qua cu cui c`


ung. Vy
ao
. nguoi thu nht phai choi theo chin thut
. n`
,

d
ng cuc?
tha
.

,
,
`,i giai. Ngu,`o,i thu
, nht trong lu,o.,t cho,i d
Lo
`u tin, c`n ly ra 2 qua c`u
,
,
,
,,
,
ng t`u, hp
, hai, v`
tra
a sau buo
c n`
ay th` t l. giua qua c`u xanh v`
a qua
. thu
ng l`
c`u tra
a 15 : 10 = 3 : 2.
,
,,
, hai ly 3 qua c`u xanh t`u, hp
, nht hoa
. c 2
Nu l
uc y, ngu`oi thu
. thu
,
,
,, , ,
,
,
,
,
,
,
,
,

hai th` ngu`oi thu


nht phai ly nguo. c lai
c
ng o hp
qua m`u tra
. thu
. o buo
,
,
,,
,,
,

hai hoa
,
ng o hp
. c 3 qua c`u xanh o hp
tip theo, 2 qua c`u tra
. thu
. thu
, ,
,
,
,
,
,
nht d
dam bao su. bt bin cua t l. 3 : 2 d
n cui cuc
. choi.

,,
,
,
V du. 5.12. Hai muoi c g
ai ng`i quanh mt
an v`
a choi mt
o choi
. chic b`
. tr`

212

,,
,
,ng tr`
Chuong 5. Nhu
o choi to
an hoc
.

,
,,
,
ac qun b`
ai. Trong mt
ai. Khoi d
v
oi n qun b`
`u, mt
. c c`m tt ca c
. l`n
,
c
choi, nu l
uc d
o t nht mt
ai c`m t nht hai qun b`
ai, mt
o
. c g
. trong
,
,
,
,
,

ng c g
nhu
ai n`
ay phai chuyn mt
ai cho mi ngu`oi ng`i bn canh
.
. qun b`
,
,

`
`

m`nh. Tr`
o choi kt th
uc khi v`
a ch khi mi c g
ai cm nhiu nht mt
. qun
b`
ai.
, ,
,
,
` ng.
a) Chung minh ra
`ng nu n 20 th` tr`
o choi khng th du
,
,
,
b) Chung minh ra
`ng nu n < 20 th` tr`
o choi phai c
o kt th
uc.
,
`,i giai. a) Nu n > 20 th` theo nguyn l Dirichlet1 , t`n tai
Lo
. t nht mt
. c
,
,
,
g
ai c`m t nht hai qun b`
ai tai
o choi khng kt th
uc.
. moi
. th`oi dim. Vy
. tr`
Nu n = 20, th` g
an nh
an cho c
ac c g
ai l`
a G1 , G2 , ..., G20 theo chi`u
,
,
a G1 l`
a c g
ai d
ac qun b`
ai. inh
kim d`ng h` v`
`u tin giu tt ca c
. ngha
,
,
`
m trong tay c g
gi
a tri. hin
ai l`
a i nu n
o na
ai Gi (gi
a tri.
. th`oi cua qun b`
,
,
,
,
`
tu. ta g
ng chnh s thu
. t S tng gi
qun b`
ai ba
an cho c
ac c g
ai). a
a tri. cua
,
,,
,,
,
nhung qun b`
ai. Khoi d
ai,
`u S = 20 (v` G1 khoi d`u c`m tt ca 20 qun b`

`
ng 1).
mi qun b`
ai d`u c
o g
ai tri. ba
,
,,
ngu,`o,i ng`i bn canh.
Ta xet truoc v`
a sau Gi chuyn qun b`
ai cho mi
.
,
,
,

Nu i = 1 th` S t
ang ln 1 1 + 2 + 20 = 20 (v` G1 ch c
o th chuyn cho
, , ,
,i s da
gi

G2 v`
a G20 m`
a thi, khi d
a tri. qun b`
ai t
ang ln tuong ung vo
o
,

g
an cho c
ac c g
ai v`
a mt di gi
a tri. cua G1 , c g
ai c
o qun b`
ai c
o gi
a tri. l`
a
,

1). Nu 1 < i < 20 th` S khng thay di g`, v` ii+(i1)+(i+1) = 0 (c


,
chuyn qun b`
g
ai Gi d
ai c
o gi
a tri. i th`
anh i 1 v`
a i + 1 cho hai c ng`i
a
,
canh
a Gi+1 ). Nu i = 20 th` S giam d
i 20 v` 2020+1+19 = 20
. Gi1 v`
,
,,
,
,
,
,,
`
(tuong tu. nhu phn trn). Nhu vy
a sau khi chuyn qun b`
ai,
. truoc khi v`
,
, ,,
,

mi c g
S lun lun l`
a bi
o choi c
o kt th
uc. Khi do
ai
. cua 20. Gia su tr`
,
`

cm mt
ai v`
a S = 1 + 2 + + 20 = 210, s n`
ay khng phai l`
a s
. qun b`
,
,
,
,
,
,
bi
o choi khng th kt th
uc duo. c.
. cua 20, v l. Nhu vy
. tr`
1 Johann

,
Peter Gustav Lejeune Dirichlet (1805-1859): Nh`
a to
an hoc
uc.
.

,
,
,c to
5.2. Kin thu
an voi tr`
o choi

213

,
,
,
,
,,
,
`i hoi hai
b) thy duo. c tr`
o choi phai kt th
uc nu n < 20, ta phai do
,
nh du lai
c g
ai da
ai khi l`n d`u tin mt
ai chuyn
. qun b`
. trong hai c g
,
,,
,,
qun b`
ai cho ngu`oi kia. Khi m`
a mt
ai chuyn mt
ai cho ngu`oi
. c g
. qun b`
,
,
,
`i hoi c g
da
nh du giua hai
ng`i canh
m`nh, do
ai phai d`
ung qun b`
ai da
.
,,
c
ngu`oi nu d
o (v` theo di`u kin
ai to
an khi c
o hai qun b`
ai trong tay
a
. b`
,
,
,
,
,
c lai
i d
mo
d
nh du se ta
th` c g
ai do
ai da
uo. c chuyn). Nhu vy
a
. qun b`
.
,

`
. p hai c g
. p canh
d
nh du n
giua ca
ai da
o. Nu n < 20, se tn tai
a
. mt
. ca
.
,
,

nh du, do d
khng bao gi`o
nhau m`
a hai c n`
ay khng c
o qun b`
ai d da
o
,
trao di mt
ai n`
ao cho nhau.
. qun b`
,
, ,,
, ,
c g
Gia su tr`
o choi tin toi v han
ai chuyn
. l`n, ta xet s l`n mi
, ,
,
,
nhung qun b`
ai. V` tr`
o choi khng th d`ung, nn khng c
o mt
ai n`
ao
. c g
,
,
,
,

`
`

chuyn qun b`
ai mt
s
h
u
u
h
an
l
n.
Nhu
ng
ta
x
e
t
b
a
t
d

u
v
o
i
c
a
p

.
.
. c
,
,
,
c g
g
ai m`
a khng c
o su. chuyn di n`
ao (mi
ai n`
ay c
o nhi`u nht mt
.
,
,
,
,
,
,
,
qun b`
ai, v` nu hon th` phai chuyn duo. c sang hai ngu`oi bn canh)
v`
a
di
.
,
,
,
`

chuyn theo chi`u kim d`ng h` mt


c
g
a
i
m
t
l
n
(c
o
th
nh
u
ng c g
ai
.
.
,
,
,
. p Gi v`
n`
ay ch chuyn huu han
ung t`n tai
a Gi+1 sao
. l`n), cui c`
. mt
. ca
,
,
,
cho Gi chuyn qun b`
ai huu han
a Gi+1 chuyn qun b`
ai v han
. l`n v`
. l`n
,
,
,
,
. p n`
(nu khng c
o ca
ao nhu vy
ac c g
ai ch chuyn huu han
a
. th` c
. l`n v`
,
,
,
,
tr`
o choi kt th
uc). i`u n`
ay r
o r`
ang khng d
ung se d`ung
uo. c v` Gi cui c`
,
,
,
,
chuyn qun b`
ai v`
a c
ung d`ung nhn
ai t`u Gi+1 .
. nhung qun b`

` tp
Bai
.
,
,
t d
,i mt
. 5.13. Ba
a B thay nhau thu. c hin
`u vo
. dng n qun c`o. A v`
.
,
,,
,, `
,
,
`
nhung buo
c di. Buo
c du tin, A ly t nht mt
qun
c
o
,
nhu
ng
khng
.
,
,`,
,
, ,
y bt k` s qun c`o,, m`
, mt
ht qun c`o. T`u khi d
ngu
o
i
cho
i
c
o
th
l
a
o
.
,
,
,
,
,
,
,
,
,
,
,
,

c s cua s qun c`o da


c truo
c do
ng
ly o buo
. Ngu`oi tha
s n`
ay l`
a mt
. uo
,
,`,
,,
,,
,
,

ng cuc?
cuc
a nguoi c
o th d
ung. Ai se l`
a ngu`oi tha
i d
uo. c buoc cui c`
. l`
.
(Phu. thuc
v`
a
o
n.)
.

214

,,
,
,ng tr`
Chuong 5. Nhu
o choi to
an hoc
.

,
,
,,
,
. 5.14. A v`
a B thay nhau t m`u t`ung vung cua b`
an c`o 4 4 . Ngu`oi
,
,,
,,
thua cuc
a ngu`oi t m`u ho`
an chnh h`nh vung 2 2 vung. Ngu`oi
. l`
,
,
ng? B c
n`
ao c
o kha n
ang tha
o th c
o mt
oa khng.
. chin thut
. h`
,
. 5.15. A v`
a B thay phin nhau d
a trn b`
an c`o 1994 1994
i qun m
, ,
,,
vung. A ch thu. c hin
cd
i theo chi`u ngang (x, y) (x 2, y 1), B
. buo
,
t d`u chon
ch di theo chi`u doc
. (x, y) (x 1, y 2). A ba
. mt
. vung
,
,
,
,
,
c di. Nhung vung qun m
di qua th`
trn b`
an c`o v`
a thu. c hin
a da
. buo
,
,,
,`,
,`,
,,
,
a nguoi khng th di duo. c nua.
khng duo. c phep di lai.
. Nguoi thua cuc
. l`
`ng A c
ng.
,ng minh ra
Chu
o mt
. chin thut
. tha

,
,
` toan
thi hoc
5.3. Nhung bai
sinh
gi
oi
.
,
,,
`ng rt nhi`u b`
Ta thy ra
ai to
an duo. c ra trong c
ac k` thi hoc
. sinh gioi tai
.
,
,
,, ,,
,
,
,

mt
ai to
an d
ac phuong
in h`nh nht. C
. s nuoc, o dy ch lit
. k nhung b`
,
,
,

ph
ap giai c
ung rt phong ph
u, ban
ay tham khao v`
a t`m ra c
ach giai
. doc
. h
,
cua m`nh.
,,
V du. 5.16 (Lin X 1984). Cho mt
a hai
. khung h`nh khi lp
. phuong v`
,
,`,
,
,
,
loai
au d
a m`
au xanh, hai nguoi quyt d
o choi nhu sau:
o v`
inh
. son m`
. mt
. tr`
,
,
,
,`, ,
,
Nguoi thu nht chon
a t ch
ung ba
`ng m`
au d
o.
. ba canh
. cua khi lp
. phuong v`
,
,,
,
,ng canh
ngu`oi thu hai chon
Sau d
on lai
o
. ba canh
. trong s nhu
. c`
. cua khi lp
.
,,
,,
,`, ,

o
t
ngu
o
i
th
u
nh
t
ch
on
ba
c
anh
phuong v`
a t ch
ung m`
au xanh. Lai
d

n
lu

.
.
.
.
,
,
,`, ,

trong s c`
on lai
t
m`
a
u
d
o.
Cu
i
c`
u
ng
ngu
o
i
th
u
hai
t
n
t
m`
a
u
xanh
c
ua
ba

.
,`,
,`, `
, ,
canh
c`
on lai.
ng cuc
a nguoi ba
ng m`
au son cua m`nh t tron
.
. Nguoi tha
. l`
. ven
.
,
,
,
,
,
,
,

`
4 canh
c
ua
m
t
m
a

t
kh
i
l
p
phu
o
ng.
Ngu
o
i
th
u
nh
t
s
e
th
a

ng
v
o
i
m
t
chi
.
.
.
.
.
,
ng d
thut
n trong cuc
u
a
. d
. choi khng?
,
,,
,
,
`ng vo
`,i giai. Ta se chu
,ng minh ra
,i mt
Lo
. chin thut
. thch ho. p ngu`oi choi
,
,
ng du,o.,c. Ta thy ra
`ng d khng bi. thua, ngu,`o,i cho,i
, nht khng th tha
thu

,
,ng b`
5.3. Nhu
ai to
an thi hoc
. sinh gioi

215

,
t cheo nhau (khng giao nhau) l`
`ng c
, hai t ba canh
thu
va
a du, v` ba
ach
.
,`, ,
,,
,
,`, ,

. t. Nhu vy,
n`
ay nguoi thu hai t d
uo. c mi canh
. trn mi ma
. nguoi thu nht
,
,
,
,
,
,
,
t cheo
hai t ba canh
c t d`u tin phai ng
. n ngu`oi thu
ngay t`u buo
an cha
. va
,
, , ,
,
`ng d
,ng minh ra
nhau. Ta se chu
ay ngu`oi thu
nht khng th thu. c hin
i`u n`
.
,
,
,,
,

t cheo c`
duo. c. Tht
va
ung nhau. Tng
. vy,
. ta xet tt ca b. ba nhung canh
.
,
,

tham gia trong hai b. ba nhu


s l`
a 8 b. ba nhu vy,
trong do
. v` mi canh
.
,
,,
, nht c
vy
o). Mt
ngu`oi thu
o th t
oc
. (ban
. d
. ve h`nh se r
. l`n t 3 canh
.
,
,
,
,
`
`

, hai
mt
nhiu nht l`
a trong 6 b.
. phn nhung canh
.
. Suy ra ngu`oi choi thu
,
,
t cheo c`
c
o th t mt
ac canh
va
ung nhau c`
on lai
a
. trong nhung b. ba c
.
. v`
,
,

nhu vy
oa.
. kt qua se h`

,,
,c 1984). Trn mt
V du. 5.17 (u
ang d
uo. c vit lin tip 2n k hiu
. h`
. x canh
.
,`,
,
`
nhau. Hai nguoi choi thay phin nhau thay mt
ng mt
. k hiu
. x ba
. trong
,
,
,
,
,
,
,

ng chu
s 1, 2, 3, 4, 5, 6. Ngu`oi choi thu hai tha
nhu
ng khi v`
a ch khi s nhn
.
,, `
,

s (trong h. s thp
d
ay t`m tt
uo. c s gm 2n-chu
. phn) chia ht cho 9. H
,
,
,
,
,
,
,
ng gi
o chin thut
ng.
ca nhu
a tri. cua n sao cho ngu`oi choi thu hai c
. tha
,
`ng ngu,`o,i cho,i thu
`,i giai. Ta se chu
,ng minh ra
, nht (ngu,`o,i cho,i d
Lo
`u tin)
,
,
ng vo
,
i n 6 0 (mod 9), c`
i n 0 (mod 9) ngu,`o,i thu
c
o chin thut
on vo
. tha

ng.
hai c
o chin thut
. tha
,,
,,
,
, ,
nht chon
Cho n i (mod 9), o dy 0 < i 8 ngu`oi choi thu
. chu s
,
,
d`u tin x0 cua m`nh theo bang sau:
i
x0

1
1

2
2

3
4

4
6

5
1

6
1

7
3

8
5

,,
,,
,
,
, , ,
ngu`oi n`
c`
on sau d
ay lun chon
a chu s ngu`oi thu
o
. chu s 7 y, o dy y l`
,
,,
,,
,,
,
, , ,
,
. Trong tru`ong ho. p nhu vy,
hai v`ua chon
c truo
c do
. buo
. sau khi ngu`oi thu
,
,
,,
,
,
tng S
hai di buo
c cui c`
ung cua m`nh v du. nhu chon
. chu s x1 , khi do
,
,
,
,
cua nhung chu s trong s v`ua tao
a
. ra l`
S = x0 + 7(n 1) + x1 x0 + 7(i 1) + x1

(mod 9).

216

,,
,
,ng tr`
Chuong 5. Nhu
o choi to
an hoc
.

,
,
,
`ng c
Ba
ach kim tra tru. c tip d
o gi
a tri. cho phep
ua dn kt lun
. khng c
,
n`
ao cua x1 sao cho x0 + 7(i 1) + x1 chia ht cho 9.
,,
ng, nu ngu,`o,i n`
, hai se tha
. t
Nu n 0 (mod 9), ngu`oi thu
ay lun d
a
,,
,
,
,`, ,
,,
,

`
chon
chu s 7 y, o d
a chu s nguoi thu nht d
y y l`
a
i vua
. trong buoc d
,
,
,
,,
, ,
dn ht cuc
xong. Khi d
a
o
. choi tng nhung chu s cua s nhn
. duo. c l`
S = 7n 0 (mod 9).

,
, nht
,ng vung
V du. 5.18 (Ai-len 1996). Trn mt
an c`o h`nh chu
o nhu
. b`
. c
,
,,
,,
,,
,
,
, `
. t
kch thuoc 5 9, tr`
o choi sau d
a duo. c d
a
y d
uo. c choi: Khoi d
u, mt
. s d
,
,

`
ng vung, khng c
ngu nhin trn mt
o vung n`
ao chua nhiu
. s nhu
,
,
,
,
`
`

,ng lut
hon mt
ac d
a. Mt
a theo nhu
. d
. ln choi bao gm ly di tt ca c
. choi
sau d
y:
,
, ,
,,
d
i) Mi
o th chuyn t
oi mt
a c
. h`nh vung pha trn, pha duoi, pha
,
tr
ai hoa
. c pha phai;
,
,,
,
chuyn ln hoa
o
ii) Nu mt
. c xung duoi mt
a d
a
. l`n choi r`i, th` n
. d
,
,
,
,
,
phai chuyn v` bn tr
ai hoa
. c bn phai, v`
a nguo. c lai;
.
,
,
,
,
iii) Tai
uc l`n choi, khng c
o vung n`
ao chua hai d
a
. th`oi dim kt th
,
hoa
. c nhi`u hon th.
,
,
,
,
Tr`
o choi kt th
uc nu n
o khng c
o kha n
ang thu. c hin
`y d
u mt
. d
. l`n
,,
,
,
,
,
. t trn b`
an, tr`
o
choi tip. Chung minh ra
`ng nu khoi d
`ng 33 d
`u ba
a duo. c da
,
,,
,
,
`

`
choi cui c`
ung phai kt th
uc. Chung minh ra
ng khoi d
. t 32 d
u ta d
a
a trn
,
b`
an th` tr`
o choi se keo d`
ai v tn.
.
,
,
,,
,,
,
`,i giai. Nu 32 d
. t trn b`
Lo
an g
oc duo
i pha phai chim h`nh chu
a duo. c d
a
,
,
,
,,
nht
c 4 8; ch
ung c
o th tt ca di chuyn ln trn, sang bn tr
ai,
. kch thuo
,
, ,
,,
`

. p di la
. p lai.
ng tr`
xung duoi v`
a sang bn phai mt
ach la
o
. c
. ch ra ra
,
,
, ,

nh nh
choi voi 33 d
ung phai d`ung, ta da
an cho c
ac vung trn
a cui c`
,
b`
an nhu sau:

,
,ng b`
5.3. Nhu
ai to
an thi hoc
. sinh gioi
1
2
1
2
1

2
3
2
3
2

1
2
1
2
1

2
3
2
3
2

1
2
1
2
1

217
2
3
2
3
2

1
2
1
2
1

2
3
2
3
2

1
2
1
2
1

,
`ng c
chuyn di, mt
Ch
u y ra
o t
am s 3. Mt
a
. da tai
. 1 di dn 3 sau khi 2 d
.
,
,
,

. c 3 tru. c tip v`
da tai
a mt
i dn 1 hoa
a tai
n 2 tru. c tip. Nhu
. 2d
. d
. 3 di d
,
,,
, ,
vy
a k > 8, th` tr`
o choi d`ung lai
a khoi d`u trn 1 v`
u
. nu k d
. v` khng d
, ,
,
3 cho nhu,ng d
`ng s k 8,
i. Nhu, vy
ch
ay chuyn to
a n`
. ta gia thit ra
,,
,`,
,
. c 3 khi khoi d`u (do
trong truong ho. p n`
ay c
o nhi`u nht 16 d
a trn 1 hoa
,
,
c
o 8 s 3 v`
a k 8 l`
a s da nhi`u nht c
o th c
o trn 1) v`
a nhu vy
o t
. c
,
,,
,

`
nht 17 da trn 2. Tu 17 da n`
ay, nhiu nht 8 da duo. c chuyn dn 3 sau
,
,
,
`
mt
o t nht 9 da cui c`
ung dn 1. Nhung da n`
ay
. ln chuyn, nhu vy
. c
,
,
,
,
,

. Nhu vy
khng th tt ca chuyn d
o choi se kt th
uc.
n 3 sau do
. tr`

,,
,
,
an c`o rng
V du. 5.19 (Israel 1995). Hai ngu`oi choi trn mt
. b`
. v han
. bao
,`,
,
,,
, ,
`

gm nhung vung kch thuoc 1 1. Nguoi choi thu nht chon


. mt
. vung
,`,
, ,
`

nh du n
nguoi choi thu hai chon
v`
ad
o ba
ng O. Khi d
ac v`
a
a
o
. mt
. vung kh
,
,`,
,
,
,
`

nh du ba
nh
da
ng X. Ho. choi nhu vy
n khi mt
a
. d
. trong nhung nguoi choi d
,`,
,,
,
,,
`
o
c
m
t
h`
a
ng
ho
a

c
m
t
c
t
5

vung
li
n
nhau
v`
a
ngu
o
i
l`
a
m
d
u
o
c
nhu
du d
u
.
.
.
.
. .
,
,,
,,
,,
,
vy
ng. Nu khng ngu`oi choi n`
ao c
od
uo. c kt qua mong mun
. truoc se tha
,
,
,,
,
, ,,
,
a h`
oa. Chung minh ra
`ng ngu`oi choi thu hai c
o th ng
an
th` tr`
o choi d
uo. c coi l`
,,
,`,
, ,
cha
. n d
ng cuc.
uo. c nguoi choi thu nht tha
.
,
,
`,i giai. Ta g
Lo
an nh
an cho c
ac vung nhu h`nh 5.1.
s d`u c
. p. S 1 v`
. p theo chi`u doc,
Ch
u y mi
o mt
a s 2 c
o ca
on
. ca
. c`
,`,
,
,

nht d
. p theo chiu ngang. Mi khi nguoi choi thu
nh
s 3 v`
a s 4 c
o ca
a
,
,`,
, ,

. p. V` bt k`
nh du vung kia cua ca
du vung, nguoi choi thu hai se da
,
,
,
,

a mt
. c theo h`
. p cua c`
5 lin tip theo ct
ang phai chu
ung nhung
. hoa
. ca
,
,,
, ,
,
ng.
s n`
ay, nhu vy
nht khng bao gi`o tha
. ngu`oi choi thu

,,
,
,ng tr`
Chuong 5. Nhu
o choi to
an hoc
.

218

..
.
1
1
3
4
1
1
3
4
..
.

..
.
2
2
3
4
2
2
3
4
..
.

..
.
3
4
1
1
3
4
1
1
..
.

..
.
3
4
2
2
3
4
2
2
..
.

..
.
1
1
3
4
1
1
3
4
..
.

..
.
2
2
3
4
2
2
3
4
..
.

..
.
3
4
1
1
3
4
1
1
..
.

..
.
3
4
2
2
3
4
2
2
..
.

H`nh 5.1

,
,ng vung kch thu,o,c 11.
V du. 5.20 (IMO 1993). Trn b`
an c`o v han
. nhu
,
,
,,
,
o choi nhu sau: `u tin xp n2 qun v`
ao mt
Ngu`oi ta thu. c hin
. mt
. tr`
. h`nh
,
vung cua h`nh
vung g`m n n vung canh
. li`n canh
. sao cho trong mi
,
,
,
, , ,,
vung chua mt
ach d
o choi l`
a qun c`o ch d
i trong trong tr`
uo. c
. qun c`o. C
,
,,
,
,
nhay theo h`
ang hoa
. c ct
o chua qun c`o o ngay s
at bn canh,
. qua mt
. c
.
,
,,
,

sang mt

tr
ng
ti
p
ngay
sau
d
o
.
Khi
d
o
qun
c`
o
o

b
i
nh
ay
qua
s
e
b
i

.
.
. loai
.
,
bo.
,
,
,
,
, ,
T`m c
ac gi
a tri. cua n d
o th kt th
uc tr`
o choi sao cho trn b`
an c`o ch
c
,
ng mt
c`
on lai
. du
. qun c`o.
,
,
,
,
,,
`,i giai. Ta xet nhu,ng qun c`o, d
. t tai
Lo
o toa. d.
im luoi (dim c
a
. nhung d
,
2
i k = 0, 1, 2, d
. t Ck = {(x, y) Z :
nguyn): Z = {(x, y) : x, y Z}. Vo
a
,
,,
,
,i
. t ak l`
x + y k (mod 3)}. a
a s qun c`o trn dim luo
i thuc
ao Ck vo
. v`

k = 0, 1, 2.
,
,
,,
,
,
Buo
cd
ang l`
a ly mt
i theo h`
. qun c`o tai
. dim (x, y) nhay qua qun c`o
, ,
,
,
ak
i d
tai
im trng (x 2, y). Sau khi di chuyn, mi
. (x 1, y) chuyn to
,
,
,
,
n le.
. c giam d
t
ang ln hoa
i 1. Nhu vy
i tnh cha
. ak thay d

,
,ng b`
5.3. Nhu
ai to
an thi hoc
. sinh gioi

219

n2
t d
a0 = a1 = a2 =
ngay khi ba
Nu n chia ht cho 3, khi do
`u
3
,
,
,
,
,
n le n`
tai
choi. Do do
o c`
ung tnh cha
ay. Nhu vy
im, ak d`u c
. moi
. th`oi d
.
,
,
,

tr`
o choi khng th kt th
uc voi mt
on lai
o hai ak trn toa.
. qun c`
. v` se c
,
,,

n le).
d. luoi 0 v`
a c
ai kia l`
a 1 (khng c`
ung tnh cha
,
,
,i mt
Nu n khng chia ht cho 3, th` tr`
o choi c
o th kt th
uc vo
. qun
,
,
`ng quy nap.
ng minh ba
i n = 1 hoa
. c 2, d
c`
on lai.
ay d
i`u n`
. Ta se chu
. Vo
, ,
,
,
,
,
,
i n 4 ta xy du. ng phuong ph
thy. Vo
ap d
giam n n qun c`o xung
,
(n 3) (n 3) qun c`o.
,
,
,,
,
Ta xet nhung qun c`o tai
cd
i
. vi. tr (0, 0), (0, 1), (0, 2), (1, 0). Nhung buo
,
,,
,
(1, 0) (1, 0), (0, 2) (0, 0), (1, 0) (1, 0), bo di d
uo. c ba qun c`o
,
,
, , , `
lin tip trong mt
a tra lai
tu v vi. tr ban d
o th
`u. Ta c
. ct
. v`
. qun c`o thu
,
,
, ,
. p lai
p dung
y bn tr
a
ay la
ai 3 (n 3) qun c`o cua n n
. thu thut
. n`
. t`u da
,
,
,
,
,,
,
t`u da
y duo
vung t`u tr
ai qua phai, sau d
i bn phai (n 3) 3 qun c`o
o
,
,
,
,
,
,,
,
, ,
y duo
t`u d
i ln dnh v`
a cui c`
ung t`u dnh phai 3 3 qun c`o t`u phai sang
a
,
,
,ng minh.
tr
ai. Nhu vy
on lai
. se c`
. (n 3) (n 3) qun c`o ta c`n chu

,
,
,
V du. 5.21 (Anh 2000). Alice choi mt
o choi mt
an c`o 2020.
. tr`
. m`nh trn b`
,
,,
,
mt
Khoi d
an c`o trong mi
ac dang
`u Alice trai trn b`
`ng xu g`m c
. d
. sau
d
a 100 quarter. Alice chon
y: 100 penny, 100 nickel, 100 dime, v`
. 59 d`ng xu
,
,

mi l`n Alice ly 1 d`ng xu theo nguyn


bt k` v`
a ly ra khoi b`
an c`o. Sau d
o
ta
c sau d
y:
,
,,
Mt
o th ly d
o bn h`nh vung bn canh
`ng peny c
i d
uo. c nu c
. d
.
,
,
,,
,,
,
,,
,
ng o ngo`
(trn, duoi, phai, tr
ai) bo trng. Nhu
ai b`
an c`o khng d
uo. c
,
,
,ng o, g
tnh l`
a bn trng theo quy ta
c n`
ay. V du,
oc b`
an c`o hoa
. c
. nhu
,,
,
,

ng n`
o bn canh
b`
an c`o, nhu
ay thm
o ba bn canh
trng th`
.
. tr c
.
,,

`
dng xu tai
ay c
ung khng d
c n`
ay.
uo. c tnh theo quy ta
. n`
,
`

Mt
o th ly d
i nu tn tai
. dng Nikel c
. t nht ba trng bn canh
.
,
,

ng ngo`
(nhu
ai b`
an c`o khng tnh l`
a trng).

,,
,
,ng tr`
Chuong 5. Nhu
o choi to
an hoc
.

220

,
,
o t nht hai bn canh
trng
Mt
o th ly d
i ch khi c
`ng Dime c
.
. d
,,
,
,
ng o ngo`
(nhu
ai b`
an c`o c
ung khng tnh l`
a trng).
,
,
Mt
o th ly d
o t nht mt
`ng quarter c
i ch khi c
. d
. trng bn canh
.
,,
,
,

ng o ngo`
(nhu
ai b`
an c`o c
ung khng tnh l`
a trng).
,
,
,
,
,
,
ng d
Alice tha
ng nu Alice ly d
an c`o. Chung minh
uo. c tt ca nhu
`ng xu trn b`
,
,
ra
`ng khng c
o kha n
ang tha
ng cua Alice.
,
, ,,
,
`ng nhau. Ta xet tai
`,i giai. Phu,o,ng ph
Lo
ap 1: Gia su nhung vung ba
. mt
.
,
,
,
,
,
,

th`oi dim cua cuc


ung nhung vung trng (v`
ung n`
ay
. choi, chu vi cua v`
,
, ,,
,
,
`
. c khng lin thng). V du,
ng tai
c
o th lin thng hoa
im
. gia su ra
. th`oi d
,
n vo
,i nhau tao
64 vung l`
n`
ao do
a trng. Nu nhung vung trng ga
. ra
`

ng 4.8 = 32. Ma
. t kh
8 8 vung, chu vi se ba
ac, nu khng c
o mt
ao
. n`

`
ng 4.64 = 256 (mi
trong 64 vung l`
a bn canh
nhau, th` chu vi se ba
.
vung c
o chu vi 4).
X

X
X

H`nh 5.2
,
,
,,
,
,
,
Tai
o choi c
o k = 59 vung trng. Tai
im khoi d`u tr`
im
. th`oi d
. th`oi d
,,
,
n`
ay chu vi ti da l`
a 4k. By gi`o ta xet c
ac d
ac nhau d
`ng xu kh
uo. c ly d
i.
,
,
,i c
mt
o phai c
o xung quanh 4 canh
vo
ac
i, n
. d`ng penny ly d
.
,
,
, `

vung trng. So d
n`
ay: Nhung x
am l`
a c
ac
sau d
y din ta t`nh trang
.
,,
,
,

trng, nhung vung c


o du X bi. chim boi nhung d
on
ng xu bt k`, c`

,
,ng b`
5.3. Nhu
ai to
an thi hoc
. sinh gioi

221

,a d`ng penny, du,`o,ng ve dm


c
o du P l`
a chu
a chu vi.
. l`
,
,a d`ng penny
Sau khi ly d
vung chu
`ng penny di r`i, nhung canh
.
,,
,
,
l`n ly du,o.,c d
khng tnh nua. Nhu vy
`ng penny di th` chu vi d
uo. c
. mi
,
giam di 4.
,
,,
,,
Nu mt
o kha n
ang n
o duo. c bao quanh boi 4
i, c
. d`ng nickel ly d
,
,
,,
,
,
vung bn canh
. trng nhu trn, trong tru`ong ho. p nhu vy
. chu vi se giam
, ,
,
di 4. Tuy nhin d`ng nickel c
o th ch c
o ba trng bn canh
nhu h`nh
.
sau:
X

H`nh 5.3
,
,,
,
,
, ho`
Trong tru`ong ho. p n`
ay, chu vi giam d
an
i 2. Nhu vy,
. trong bt cu
,
,
canh n`
ao, sau khi ly d
i d
`ng nickel chu vi d
`u giam d
i t nht 2.
,,
,
`ng l lun
Ba
ay ve h`nh) ta c
o kt lun
a nu mt
. tuong tu. (h
. l`
. d`ng
,
,,
. c 4), v`
dime duo. c ly di, chu vi khng t
ang ln (n
o c
ung giam di 2 hoa
a khi
,
,
,
,
mt
o th t
ang ln, nhung nhi`u nht l`
a
i, chu vi c
. d`ng quarter duo. c ly d
,
,
,
. c giam d
. c 4).
2 (n
o c
o th khng thay di hoa
i 2 hoa
,
,
,
,
`ng trn b`
By gi`o ta se ch ra ra
an c`o khng th khng c`
on mt
. d`ng
, ,,
,,
,
xu n`
ao. Gia su ly d
an c`o th` chu vi se l`
a 4.20 = 80.
uo. c ht d
`ng xu trn b`
, ,,
,
,

`
`
Gia su tai
at c
o p dng penny, n dng nickel, d d`ng
. th`oi dim xut ph
,
. t cho
ly di (nhu vy
dime, v`
a q d`ng quarter da
. p + n + d + q = k). a

222

,,
,
,ng tr`
Chuong 5. Nhu
o choi to
an hoc
.

,
,,
,
duo. c ly di, th` t2 p
gon
on lai
a
. t = 10. Nu tt ca nhung d`ng xu c`
. d
,
,,
,
ng 4(t2 p). C
d`ng penny se duo. c ly di, chu vi giam d
ung nhu vy,
i d
u
.
,
,
,
,
,
,
2

o se giam di t
t nd
o th duo. c ly d
i, nhung chu vi cua n
`ng nickel c
,
,,
2
2

`
nht 2(t n). (t d) dng dime duo. c ly di v`
a chu vi giam di t nht
,,
`ng 0. Cui c`
ba
ung, (t2 q) d`ng quarter duo. c ly di v`
a chu vi t
ang nhi`u
nht l`
a 2(t2 q).
,
,
c lai
Ta nha
a gi
a tri. ban d`u cua chu vi nhi`u nht l`
a 4k. Nhu vy,
. l`
. nu
,
,
,
,
tt ca c
ac d`ng xu d
a tri. cui c`
ung cua chu vi nhi`u nht l`
a
uo. c ly d
i, gi
4k 4(t2 p) 2(t2 n) + 2(t2 q) = 4k 4t2 + 4p + 2n 2q.
, ,
,
`ng 4(p + n + d + q) = 4k, nhu, vy
. c ba
Nhung 4p + 2n 2q l`
a nho hon hoa
.
2
2
2

chu vi cui c`
ung nhiu nht l`
a 4k4t +4k = 8k4t = 8.594.10 = 72,
,
,
,
`

i thu. c t l`
ng 80.
tr
ai vo
a chu phai ba
,,
,
,
,,
Phuong ph
ap 2: inh
a chu vi nhung v`
ung trng (nhu dinh
. ngha P l`
. ngha o
,
,
, , ,
c
ach giai trn) v`
a cho q, d, n l`
a s nhung d
ng quarter, dime, v`
a
`ng tuong u
,
,
,
,
,
d
nickel trn b`
an c`o tai
at. Khi do
im xut ph
ai
. th`oi d
. luo. ng P4q2d+2n
, ,
,
,
l`
a khng d
a n
o khng t
ang (khng thay d
i don diu:
i khi nhung
. Ngha l`
,
,,
,,
,,
d`ng xu duo. c phep ly di). Gi
a tri. khoi d`u cua dai
. luo. ng trn nhi`u nht
,
,
gi
l`
a 72, khi d
a tri. m`
a tt ca c
ac trng trn b`
an c`o l`
a 80. V l, d
ay
o
i`u n`
,
,
,
d
dn
an c`o khng bao gi`o tt ca c
ac d`u trng.
n trn b`

,, `
,
,
V du. 5.22 (Anh 2002). Mt
o choi hai ngu`oi ba
t d
oi mt
u v
. tr`
. ct
. ti`n n
,
,
,
,
c
d
o th`
anh hai ct
`ng xu (n 3). Ngu`oi choi thu nht chia ct
a
. ti`n d
. ti`n
,`,
, ,
c
tu`y y. Nguoi choi thu hai chon
ac ct
o trn b`
an)
a
. mt
. ct
. ti`n (trong c
. ti`n d
,
,`,
`

v`
a lai
o ra th`
anh hai ct
uy y. Tip tuc
ng cuc
. chia n
. tin t`
. nhu vy,
. nguoi tha
.
,
,
,`,
,,
,

`
`
l`
a nguoi choi dn buoc d
am cho tt ca ct
o mt
. c hai d
i l`
ng
. tin ch c
. hoa
,
,
,, `
,`,
,

`
`
tin. V
oi gi
a tri. ban d
ao cua n th` nguoi di buoc d
ng, gia
u n`
u tin se tha
,
,,
,,
,
,
su hai ngu`oi choi d`u t`
ai gioi nhu nhau?
,
,
ng khi v`
n;
`,i giai. Ngu,`o,i cho,i thu
, nht tha
. c n cha
Lo
a ch khi n = 3 hoa

,
,ng b`
5.3. Nhu
ai to
an thi hoc
. sinh gioi

223

,
, ,
,,
, ,
ng vo
,i moi
ngu`oi choi thu
hai tha
a n > 3. Ta c
o th kim tra kt lun
. n le v`
.
,
`
ng minh ba
ng quy nap.
ta chu
trn dn n = 6 v`
a sau do
.
,
n, ngu,`o,i thu
, nht tao
1) Nu n > 6 l`
a cha
o 1 d`ng xu) v`
a
. ra ct
. co 1 (c
,
,
,
,
,

`
ng
ct
on1d
a le, nguoi thu nht se tha
ng xu). V` n 1 l`
. co n 1 (c
,
,,
,`,
,`, ,
, ,

theo gia thit quy nap


anh nguoi thu hai v`
a
. (v` nguoi choi thu nht tro th`
,
,,

`
o vi. tr xut ph
at c
o s le dng xu).
,
,,
, ,
n
2) Nu n 7 l`
a s le, ngu`oi choi thu
nht tao
o s cha
. ra mt
. ct
. c
,
,
,
,
,
hai lai
d`ng xu v`
a mt
o s le d`ng xu. Ngu`oi choi thu
. ct
. c
. chia ct
. s
,
,
,,
n d
cha
o s le d
ach nhu vy,
`ng xu ra hai ct
`ng xu. Tip tuc
. d`u c
. c
. ngu`oi
,
,,
,,
,
, ,
, ,
p lai
choi thu
hai lun lun da
ac buo
cd
nht v`
ad
i cua ngu`oi choi thu
ua
. c
,
,
,,
, ,
,i nhu,ng s le d`ng xu. Khi
ngu`oi choi thu
nht v`
ao t`nh th ch c
o s ct
. vo
,
,i co, 1 (ch c`
c
ac ct
on 1 d`ng xu), ch
ung khng c`
on lin quan dn
. tin to
,
,,
,,
,
,
,
,
,
buoc choi. Vn d` nguy kich
a c
o co 3 o mt
. cho ngu`oi choi l`
. ct
. ti`n: Ch
,
,,
,,
, ,
,
c
o mt
ach duy nht m`
a ngu`oi choi thu
hai c
o th thua l`
a nu ngu`oi choi
. c
,,
, ,
,
, hai tao
thu
nht mt
a rt nhi`u
. cho ngu`oi choi thu
. ct
. duy nht co 3 (v`
,
,
,
,
,
,
,
,
,
, nht phai
ct
`ng xu). Nhung trong tru`ong ho. p nhu vy
. 1d
. ngu`oi choi thu
. c l`
. c l`
tao
a mt
a mt
a
`ng xu, hoa
. ra hoa
. ct
. duy nht hai d`ng xu v`
. ct
. 3d
,
,
,
,
,
,
,
hai
mt
u trong tru`ong ho. p n`
ao, ngu`oi choi thu
. ct
. duy nht 4 d`ng xu. D`
,
,
,
,
,
,
,
,
`ng c
ng o ngay buo
c di tip theo ba
ach l`
am giam d
tha
i: Trong tru`ong ho. p
,,
, nht, ct
thu
a ct
a
. ba d`ng xu duo. c chia ra ct
. 1 d`ng xu v`
. hai d`ng xu, v`
,`,
,
,
,
,
,
hai, ngu`oi thu
hai chia ct
trong truong ho. p thu
anh hai ct
`ng xu th`
. 4d
.
,
,
,
,
,

mi ct
hai
d

ng
xu.
T
o
m
l
ai,
chi
n
thu
t
th
a
ng
c
ua
ngu
o
i
cho
i
th
u
hai
l`
a

.
.
.
, ,
,
,

lun lun tao


an l`
a nhung ct
n t`nh th tt ca
. ra to`
. ti`n le tr`u khi dn d
,
,,
c
ac ct
a mt
o3d
`ng xu v`
`ng xu; trong tru`ong
. ch mt
. d
. ct
. duy nht c
,
,
,,
,
,
,
ng d
ho. p nhu vy
uo. c thu. c hin
. th` chin thut
. chin tha
. nhu m ta trn.

,,
,
, `
,ng d`ng xu tr`
V du. 5.23. Hai ngu`oi choi mt
o choi ba
ng nhu
on b
an knh
. tr`
,
,
,
,
,
,

nht
1, trn ma
. t b`
an h`nh chu
o kch thuoc m n. Mi ngu`oi choi c
o quy`n
. c

224

,,
,
,ng tr`
Chuong 5. Nhu
o choi to
an hoc
.

d
. t mt
an sao cho khng d
ac. Khi ba
t d
a
`ng xu trn b`
`e ln d
`ng xu kh
`u
. d
, ,,
,
. Gia su moi
`i hoi v han
ma
. t b`
an khng c
o mt
ao trn d
`ng xu n`
o
o
. d
. d
. v`
,
,
,
,
,
,
,
,
,
,
ng gi
p ung, v
d
oi nhu
a tri. n`
ao cua m v`
a n ngu`oi choi thu
`ng xu d
`u d
uo. c d
a
nht c
o chin thut
ng?
. tha
,
,
`,i giai. Nu m < 2 hoa
. c n < 2, th` khng th da
. t mt
Lo
ao trn
. d`ng xu n`
,
,`, ,
. t kh
b`
an nhu vy,
ac, nu n 2 v`
a m 2,
. vy
. nguoi thu nht thua ngay. Ma
,
,
,
,`,
, ,
`

ng nguoi choi thu nht c


ng. Goi
ta se ch ra ra
o th tha
a tm cua b`
an,
. O l`
,
,`, ,
,
nht cua n
. t d`ng xu thu
v`
a cho nguoi thu nht d
o sao cho tm d`ng xu
a
,
ng qua tm (ngha l`
chnh l`
a O. Cu h`nh c`
on lai
a di xu
a ph`n c`
on lai
a
. l`
. l`
,
,,
,
,
,
,
,
,

hai d
i O). By gi`o ngu`oi choi thu
. t
nhu nhau duoi phep quay 180 d
i vo
a
,
,
,
,
,
,
,
`
nht da
i tm A n`
c d
ng buo
. Ngu`oi choi thu
p lai
d`ng xu vo
ao d
a
o
i l`
. ba
,
,
,
,
,

i O. Ngu`oi choi
. t dng xu c
da
o tm tai
a phep quay 180 cua A d
i vo
. B, l`
,
,
,
,
,
,
,
`ng c

nht se lun lun thu. c hin


c d
. t d
thu
ach da
uo. c buo
i cua m`nh ba
i
. d
,
,
,
`ng bu,o
,ng qua tm. Ta phai ch ra ra
,c di nhu, vy
xu
a lun lun thu. c hin
. l`
.
,,
ng lut
duo. c (d
u
. cho phep).
,ng nn bu,o
,c
. t d
ng lut,
V` vic
a du
a
`ng xu tai
i xu
. d
. tm A l`
. do tnh d
,
,,
, hai
. t d`ng xu tai
ng lut.
di da
ung d
u
. tm B c
. Nhu vy
. sau khi ngu`oi thu
,,
,,
,,
,
, ,
. t d`ng xu tai
d
nht d
c ho. p l. tai
a d`ng
a
i duo. c buo
. B m`
. A, ngu`oi choi thu
,
,
i d`ng xu c
xu c
o tm tai
o tm tai
. B khng th giao vo
. A. Tht
. vy,
. nu P
,
,
l`
a mt
a BP 1, vy
a trung
. dim giao, th` AP 1 v`
. AB 2. Nhung O l`
,
,
AB
`
m trn ma
. t d`ng
1. i`u n`
ay ngha l`
a A na
dim cua AB, nn AO =
2 ,,
,
c d
. t d
buo
xu c
o tm O, v l. Do do
o tm tai
a thu. c hin
i d
a
`ng xu c
. B l`
.
,,
duo. c.

` tp
Bai
.
,
,,
,
,
,c 1974) Peter v`
. 5.24. (u
a Paul choi mt
o choi nhu sau: Hai ngu`oi l`n
. tr`
,
,
,
,,
,,
s tu., nhin cua : 1, 2, 3, ...
,n nht cua mi
luo. t x
ac d
c s chung le lo
inh
. uo
,
,,
,
,,
Nu uo
c s n`
ay chia cho 4 c`
on du 1, th` Peter mt mt
a Paul duo. c
. dim v`

,
5.4. Chuyn d
o choi Nim
` chin thut
. tr`

225

,
,
,,
,
mt
c s n`
ay chia cho 4 du 3, th` Paul mt mt
a Peter
im. Nu uo
. d
. dim v`
,
,
,
,,
,
,
`ng
ng minh ra
duo. c mt
im. Sau mt
. d
. th`oi gian ho. d`ung cuc
. choi. Chu
ng.
chin tha
Paul da

,
` choi Nim
5.4. Chuyn d
` chin thut
. tro
` v du.
5.4.1. .inh ngha va
, ,,
,
,,
,
,
Tr`
o choi Nim l`
a tr`
o choi giua hai ngu`oi trn mt
. s dng soi (o dy ta
,
,
,

c: L`n lu,o.,t mi
xet cuc
o bn d
ng soi nhu h`nh ve 5.4) theo quy ta
. choi c
,
,
,, ,,
hoa
ngu`oi duo. c phep ly d
ao d
. c
i mt
o
. s bt k` vin soi trong mt
. dng soi n`
,
,`,
,`, , ,

. Nguoi thua cuc


ly d
ao d
a nguoi dn luo. t m`
a khng
i tt ca mt
ng n`
o
. d
. l`
,
,

c`
on vin soi n`
ao d ly.

10

H`nh 5.4
,
,,
,
,
,
Tr`
o choi Nim l`
a tr`
o choi dn gian cua ngu`oi Trung Quc, trong nhung
, ,,
,
,,
,
,
c d
duo. c truy`n b
th k truo
a sang chu u v`
a mt
a tr`
o choi ph
a
. th`oi l`
,
,,
,
,,
,
, ,,
c
bin o c
ac nuo
c n`
ay. T`u tr`
o choi n`
ay d
o h`
ang loat
o choi tuong
a
. kiu tr`
,
,
,
,
,
,
,
n nhung l thuyt d
a t`m ra
tu. ra d
o ha
giai thch v`
`oi, khng nhung vy,
. c
,
,
,
chin luo. c, chin thut
o choi Nim. Trong ph`n n`
ay ch
ung ti
. cho loai
. tr`
,
,
,
,
,

c u
ng dung
ch bin soan
a c
ach thu
biu din d
ay s
. mt
. ph`n d hiu v`
.
,
,
ng trong tr`
nhi. phn d
o choi n`
ay.
tao
. ra chin thut
. tha
,
,
,
,
,,
,
,
,i tr`
cu. th ta ch d` cp
o choi c
o 4 dng soi v`
a s luo. ng nhu trong
. to
,,
,,
,,
,
,,
l`
h`nh ve v`
a ngu`oi di d
a ngu`oi thu nht v`
a ngu`oi d
a ngu`oi
`u tin l`
i tip d
o
,
,
,,
,
,
t d
. t b`
. c
thu hai. S luo. ng soi v`
a c
ac dng soi trn ma
an l
uc ba
`u choi hoa

,,
,
,ng tr`
Chuong 5. Nhu
o choi to
an hoc
.

226
L`n di
1
2
3
2
2
2
2

,`,
, nht di
Nguo
i thu
[8, 10, 3, 1]
[7, 7, 3, 1]
[6, 5, 3, 1]
[5, 4, 3, 1]
[5, 3, 0, 1]
[0, 3, 0, 1]
[0, 0, 0, 1]

,`,
, hai d
Nguo
i thu
i
[8, 7, 3, 1]
[7, 5, 3, 1]
[6, 4, 3, 1]
[5, 4, 0, 1]
[2, 3, 0, 1]
[0, 1, 0, 1]
[0, 0, 0, 0]

,
,,
,
Bang 5.1. C
ac buo
c di trong tr`
o choi Nim
,
,
,,
,,
,
l`
a sau mt
c di cua ngu`oi choi ta goi
a mt
th
ai. C
o th mt
. trang
.
. buo
. l`
.
,,
, ,
, ,,
,

n tr`
phuong a
o choi duo. c ghi lai
ac trang
th
ai nhu bang 5.1. Sau mi
. voi c
.
,
,
,
,,
,
,`,
, , ,
,
`
buoc di cua tung nguoi choi s vin soi v`
a c
ac dng soi c`
on lai
ng
. tuong u
,
, ,
,
,,
,,
,

nhu trong bang trang


ai. Tr`
o choi khng th choi v han
. th
. buoc v` s luo. ng
,
,
,
,,

trong d
a huu han,
uc khng c`
on vin soi n`
ao, ngu`oi n`
ao
ng soi l`
. dn mt
. l
,,
dn luo. t l`
a thua.
,
,,
,
,,
ng d
c
o nhung buo
c di th`
anh chin thut
o
i phuong trong tr`
. tha
,
,
,
,
,
ng di thu
u l thuyt sau dy d c
choi n`
ay ta nghin cu
o th chin tha
,
bu,o
`ng c
,c di. Ta di dinh
ach theo d
oi thay d
ac trang
th
ai sau mi
ba
i c
.
.
,
,
ngha tng c
ac s trong tr`
o choi Nim:
,
,
,,
,
inh
ngha 5.2. Tng-Nim cua hai s trong tr`
o choi Nim duo. c k hiu
a
. l`
.

c sau:
] theo hai nguyn ta
,
,
,
, ,
(A) Tng-Nim hai s c
o l
uy th`ua kh
ac nhau cua 2 ging nhu tng b`nh
,,
thu`ong trong s hoc.
. V du. 4 ] 8 = 12.
,
, ,
`ng 0. V du. 4 ] 4 = 0.
(B) Tng-Nim hai s c
o c`
ung l
uy th`ua cua 2 ba
,
,,
`ng moi
Ch
u y: Ta bit ra
`u biu din theo h. nhi.
. s nguyn duong d
,,
phn (c
o dang
a = a0 2n + a1 2n1 + + an , o dy n 0, a0 6= 0,
.
,
,
,
,i i = 1, 2, ..., n). Do d
Tng-Nim cua hai s khng phai l
0 a i < 2 vo
uy
o

,
5.4. Chuyn d
o choi Nim
` chin thut
. tr`

227

,
,
, ,
,
,, `
,
ng c
th`ua cua 2 c
ach chuyn qua biu din co s 2
ung thu. c hin
uo. c ba
. d
,
c trn. D thy t`u, dinh
r`i cng
o nhung tnh
. ngha trn ta c
. theo nguyn ta
cht sau:
,,
(C) 0 ] a = a ] 0 = a (ph`n tu 0).

(D) a ] b = b ] a (tnh giao ho


an).

,
(E) (a ] b) ] c = a ] (b ] c) (tnh kt ho. p).
,
,
,
,,
`ng quy ta
c trn ta c
, Tng-Nim cua hai s bt
Ba
o th tnh d
uo. c bt cu
,
`ng c
c trn nhu, sau:
p dung
k`. V du. tnh Tng-Nim 5 ] 3 ba
ach a
c
ac quy ta
.
5 ] 3 = (4 + 1) ] (2 + 1)
= (4 ] 1) ] (2 ] 1)
=4]61]2]61
=4]2
=6

c (A)
theo quy ta
c (B)
theo quy ta

c (B)
theo quy ta

c (A).
theo quy ta

,
,
,
,
Ta lit
`u tin (bang 5.2, trang sau).
. k Tng-Nim cua 11 s tu. nhin d
]
0
1
2
3
4
5
6
7
8
9
10

0
0
1
2
3
4
5
6
7
8
9
10

1
1
0
3
2
5
4
7
6
9
8
11

2
2
3
0
1
6
7
4
5
10
11
8

3
3
2
1
0
7
6
5
4
11
10
9

4
4
5
6
7
0
1
2
3
12
13
14

5
5
4
7
6
1
0
3
2
13
12
15

6
6
7
4
5
2
3
0
1
14
15
12

7
6
6
5
4
3
2
1
0
15
14
13

8
8
9
10
11
12
13
14
15
0
1
2

9
9
8
11
10
13
12
15
14
1
0
3

,
,
,
,
Bang 5.2. Tng-Nim cua 11 s tu. nhin d`u tin
,
,
,
Khng c`n bang 5.2 ta c
o th tnh Tng-Nim c
ac s nguyn:

10
10
11
8
9
14
15
12
13
2
3
0

,,
,
,ng tr`
Chuong 5. Nhu
o choi to
an hoc
.

228
,
V du. 5.25. Tnh Tng-Nim:
a) 8 ] 10 ] 7 ] 1;
b) 5 ] 1 ] 3 ] 7.

,
`,i giai. a) 8 ] 10 ] 7 ] 1 =6 8 ] (6 8 ] 6 2) ] (4 ] 6 2 ] 6 1) ] 6 1 = 4.
Lo

b) 5 ] 1 ] 3 ] 7 = (6 4 ] 6 1) ] 6 1 ] (6 2 ] 6 1) ] (6 4 ] 6 2 ] 6 1) = 0.
,
,,
,
,
`ng n
,u k dinh
Ta nghin cu
o tuong tu. nhu
. ngha Tng-Nim th` thy ra
,
,
,,
c
vic
ac s biu din duo
i dang
ung ti khng nha
. tnh tng c
. nhi. phn. Ch
,
,,
,
lai
ach biu din mt
a c
ach cng
y c
. od
. s co s 10 sang dang
. nhi. phn v`
.
,,
,

c
ac s nhi. phn. Ta d`
ung c
ach cng
c
a
c
s
o
d
ang
nh
i
phn
nhu
ng
khng
.
.
.
,
c
o nh
o. V du.
]
8
3
6
4
9

=
=
=
=
=

23
1

22
0

1
1
0

21
0
1
1
0
0

20
0
1
0
0
1

,
,
l
uy th`ua co s 2

,
Tng-Nim.

, ,
,
`ng c
ach tnh
Ban
o th kim tra lai
oc
. d
. c
. Tng-Nim 8 ] 3 ] 6 ] 4 = 9 ba
,
,,
,
,,
cua v du. trn. Trong khi thu. c hin
. phep tnh duoi dang
. nhi. phn o trn ta
,
,
,
, sang h`
khng d`
ung phep nho
ang kh
ac, ch don gian l`
a dm s 1 trn mt
.
,
,
,
,

ct
n
u
l`
a
s
ch
a
n
th`

cho
k
t
qu
a
0
v`
a
n
u
l`
a
s
l
e
cho
k
t
qu
a
1
(th
u
.
.c
,
,,
cht dy l`
a phep to
an XOR trong tin hoc
ung). Ta kim tra
. ngu`oi ta hay d`
, ,,
ng khng:
lai
on du
. hai kt qua o v du. trn xem c`
3
2
1
0
]
2
2
2
2
]
22 21 20
8
= 1
0
0
0
5 = 1
0
1
10 = 1
0
1
0
1 = 0
0
1
7
= 0
1
1
1
3 = 0
1
1
1
= 0
0
0
1
7 = 1
1
1
4
= 0
1
0
0
0 = 0
0
0

,
5.4. Chuyn d
o choi Nim
` chin thut
. tr`

229

,
,
,,
`ng c
ch ra ra
Ngu`oi ta d
ach tnh tng c
ac s theo dang
nhi. phn cho
a
.
,
,
,
i c
kt qua tr`
ung vo
ach tnh Tng-Nim.
,
` choi Nim
5.4.2. Chin thut
. trong tro
,
, ,
,
,,
Ta se d`
ung Tng-Nim d du. do
an buo
c tip theo cua ta. Ph`n c`
on lai
.
,
,
,
,
,,
,
`
Tng-Nim cua c
cua chuyn d ta di xet nhung trang
th
ai o do
ac hat
.
. soi
,
,
`
`

ng minh nu Tng-Nim
ng 0 hoa
. c kh
c`
on lai
a ba
ac 0. Ta se chu
. sau ln di l`
,,
,,
,,

, nht;
ng cho ngu`oi thu
tai
ac 0 th` c
o chin thut
i tha
. buoc di kh
. buoc d
,
,
,
,
,
,
`ng 0 th` ngu`oi thu
ng. Tai
hai c
nhung nu Tng-Nim ba
o th tha
ai
. trang
. th
,
,
,
,
,
i Tng-Nim kh
,i
vo
ac 0 ngu`oi ta goi
a trang
th
ai m`o v`
a tai
th
ai vo
.
. l`
. trang
.
,
,
,
`ng 0 ngu,`o,i ta goi
Tng-Nim ba
a trang
ai r
o. Nhu vy
o bang chin
. th
. l`
. ta c
,,
,
,
thut
. cho hai ngu`oi choi nhu sau:
,
Tng-Nim
6= 0
0

Tn goi
ai
. trang
. th
,
M`o
R
o

,,
ng
Ngu`oi tha
,`, ,
Nguoi thu nht
, , ,
Ngu`oi thu
hai

,
,
,
Bang 5.3. Bang chin thut
o choi Nim
. trong tr`
,
,
,
,,
,
Nh`n v`
ao bang chin thut
cd
ua ra nhung buo
i
. trn trong khi choi se d
,
,
,
,
,
,
`ng 0 hoa
c khi d
. c kh
ac 0. Truo
ao giai thch
thch ho. p d cho Tng-Nim ba
i v`
,
,
. c bit
tai
o bang 5.3, ta h
ay xet mt
ai da
. sao lai
. c
. s trang
. th
. xay ra khi hai
,,
,
ngu`oi c`
ung choi.
,

[0, 0, 0, ..., 0] Trang


th
ai khng c
o mt
ao trn b`
an, khi d
o
.
. vin soi n`
,
,`, ,
,,
, , ,

a thua ngay.
nguoi thu nht dn luo. t v`
a khng th d
i d
uo. c nua, vy
. l`
,
,`,
,`,
,
,
,
`

hai tha
ng 0, nguoi thu
ng, du
ng nhu
Truong ho. p n`
ay Tng-Nim ba
,
,
,,
bang 5.3. Trang
th
ai nhu th n`
ay ngu`oi ta goi
a trang
ai kt th
uc
. th
.
. l`
,
tr`
o choi.
,
,
, , ,
[1, 0, 0, ..., 0] Trn b`
an ch c`
on mt
nht se ly vin
. vin soi, ngu`oi thu
,
,
,
,,
ng. Tng-Nim cua trang
soi duy nht n`
ay v`
a l`
a ngu`oi chin tha
th
ai
.

230

,,
,
,ng tr`
Chuong 5. Nhu
o choi to
an hoc
.

, ,
, , ,
,
ng.
ng nhu bang 5.3, ngu`oi thu
n`
ay l`
a 1, trang
ai m`o v`
a du
nht tha
. th
,
,
, , ,
,
[n, 0, 0, ..., 0] Ch c
o n vin soi tai
. mt
. dng. Theo lut
. choi ngu`oi thu
,
,
,,
,,
,
ng. Tng-Nim trong tru`ong ho. p
nht ly ht vin soi v`
a l`
a ngu`oi tha
,
,
, , ,
,
ng l`
n`
ay l`
a n, trang
ai n`
ay c
ung m`o v`
a nhu bang 5.3 d
a ngu`oi thu
u
. th
ng.
nht tha
,
,
`ng nhau, Tng-Nim ba
`ng
[n, n, 0, 0, ..., 0] Trn b`
an c
o hai d
ng soi ba
,
,
,,
0 (cng
theo biu din nhi. phn thy ngay) v`
a theo bang 5.3 ngu`oi
.
ng. Ngu,`o,i thu
ng th` cu
, hai se tha
, hai mun chin tha
, d
ng s
thu
i du
,
,
,,
,
,`,
,
,
,
nht d
vin soi nhu nguoi thu
ac v`
a nhu vy
i, nhung o dng soi kh
.
,
,`, ,
`

ng 0, nguoi thu hai se tha


ng.
tng Nim lun lun ba
,
,
,

[1, 1, 1, ..., 1] Trn b`


an c
o n vin soi v`
a mi d
o mt
ng soi ch c
. vin.
,`,
,`,
,
,
Trong truong ho. p n`
ay hai nguoi choi thay phin nhau di d
n khi
,
,
,
,
,
,
ng
nht tha
khng c`
on vin soi n`
ao nua, vy
. th` nu n le, ngu`oi thu
,
,
,
,
,
`ng 1; nu n cha
n ngu`oi thu
ng v` Tng-Nim
hai se tha
v` Tng-Nim ba
,
,
`ng 0. i`u n`
i bang 5.3.
ng vo
ba
ay c
ung du
,
,
,
,ng minh bang 5.3 l`
ng, ph`n sau
Nhung v du. trn khng phai chu
ad
u
,
,
,
,
,
`ng
ng minh d
i nhung mnh
chi tit. Ta d y ra
vo
d` cu. th ta se chu
i`u d
o
.
, ,,
,
, ,
,
,
,
ng, ta phai lu`ong truo
c nhung kha n
d choi d`
anh chin tha
ang trang
th
ai
.
,
,
,
,
,
,
i Tng-Nim cua n
xay ra c`
ung vo
o. Ngha l`
a ta phai d
a
i tnh Tng-Nim v`
,
,
,
i Tng-Nim nhu v du. sau:
c
ac trang
ai vo
. th
,
,
,ng trang
V du. 5.26. V
oi nhu
th
ai sau d
ay tnh c
ac Tng-Nim cho c
ac
y h
.
,,
,
buoc d
i ho. p l:
. a) [2, 2, 3] b) [5, 1].
,
,
,
,
`,i giai. a) Tng-Nim 2 ] 2 ] 3 = 3. Nhu,ng bu,o
,c d
Lo
i ho. p l. nhu sau: [1, 2, 3],
,
, , ,
,i Tng-Nim
[0, 2, 3], [2, 1, 3], [2, 0, 3], [2, 2, 2], [2, 2, 1], [2, 2, 0] v`
a tuong u
ng vo
0, 1, 0, 1, 2, 1 v`
a 0.
,
,,
,
b) Tng-Nim 5 ] 1 = 4. C
ac buo
c di ho. p l. [4, 1], [3, 1], [2, 1], [1, 1], [0, 1],
,
, , ,
ng l`
a 5, 2, 3, 0, 1 v`
a 5.
[5, 0] v`
a c
o Tng-Nim tuong u

,
5.4. Chuyn d
o choi Nim
` chin thut
. tr`

231

,
,
,
inh
a Tng-Nim cua trang
ai ban d
`u
. th
. ngha 5.3. Tng-Nim nguyn gc l`
,
,
,
,,
,,
,
,
truoc khi choi. Tng-Nim m
oi l`
a Tng-Nim cua trang
ai sau mt
. th
. buoc di
.
n`
ao d
o
,
T`u v du. trn ta c
o mnh
d` rt hay sau d
y:
.
,
,
,a nhu
,ng
Mnh
d
` 5.1. Tng-Nim nguyn gc khng bao gi`o xut hin
. giu
.
,
,
Tng-Nim m
oi.

H`nh 5.5
,
,
,
,ng minh. Vo
,i vic
Chu
ac s nhi. phn ta
. biu din Tng-Nim theo tng c
,
,
,
,
`ng mt
c di ho. p l. bt k` l`
ng mt
thy ra
am thay di du
. buo
. s hang
. trong
,
,
,
,
,
. c tt ca s cua n
tng nhu h`nh 5.5. S hang
n`
ay se thay d
o
i mt
.
. s hoa
,
,
,
,
,
ba cua biu
trong c
ach biu din nhi. phn (trong h`nh 5.5 k tu. s thu
,
,
,
,
din c
u l`
a 1 chuyn th`
anh 0). Moi
o su. thay di cua s hang
se xut
. ct
. c
.
,
, ,
,
,,
n le, ngha l`
hin
a s 1 trong ct
ay thay di t`u s luo. ng
i tnh cha
. thay d
. n`
,
,
,
,
,,
n sang le hoa
n, nhu, vy
,i kh
. c t`u s luo. ng le sang cha
ac
cha
. Tng-Nim mo
,
,
,
i Tng-Nim nguyn gc t nht l`
vo
a tai
ay (trong h`nh 5.5 chu
. vi. tr ct
. n`
,
,
, ba trong Tng-Nim nguyn gc se thay d
s trong ct
i).
. thu
,
,
,ng minh trn ta ch d`
Ch
u y trong l lun
ung mt
. chu
. s hang
. thay di
,
,
, khng quan tm dn t
. c giam Tng-Nim.
chu
ang hoa

,,
,
,ng tr`
Chuong 5. Nhu
o choi to
an hoc
.

232

,
, ,
,
,ng gi
a tri. nho hon Tng-Nim nguyn gc d
`m
Mnh
d` 5.2. Tt ca nhu
`u na
.
,
,
trong s c
ac Tng-Nim m
oi.
,
,
,
,ng minh. Ta kim tra kha
,ng
ng dinh
ng. Chu
Chu
ad
u
. qua v du. 5.26 qua l`
,
,
,,
,
minh mnh
d` n`
ay hoi kh
o mt
ut. d hiu ta di qua ba buo
c sau:
.
. ch
,
,
,
cho sang mt
1. V du. v` giam Tng-Nim cua mt
ai d
ai
a
. trang
. th
. trang
. th
,
, ,
,
`

c
o Tng-Nim nho hon ba
ng c
ach thay d
V du. trang
th
ai
i mt
. s hang:
.
.
,
,
`
`

ng 6, mun t`m trang


ng 3
[8, 7, 5, 12] c
o Tng-Nim ba
ai c
o Tng-Nim ba
. th
,
,
`

ng c
ba
ach thay d
am th n`
ao? H`nh 5.6 ch cho ta thy
i mt
. s hang
. th` l`
,
,
,
,
biu din cua c
ac s 8, 7, 5, 12, Tng-Nim 6 v`
a Tng-Nim mong mun 3
,
,
,,
,
,i Tng-Nim hin
duo
i dang
nhi. phn. Tng-Nim mong mun kh
ac vo
.
. th`oi
]
8
7
5
12
6
3

=
=
=
=
=
=

23
1
0
0
1
0
0

22 21 20
0
0
0
1
1
1
1
0
1
1
0
0
1
1
0
0
1
1

|
{z
}
vi. tr thay

,
,
Tng-Nim hin
. th`oi
,
Tng-Nim mun c
o

,
,
,
, ,,
,
2
`i hoi thay d
H`nh 5.6. Giam Tng-Nim t`u 6 xung 3 do
a 20
i o ct
. 2 v`
,
,
,
,,
2
a 20 . d
tai
at
. duo. c Tng-Nim mong mun th` ta phai
. hai vi. tr ct
. 2 v`
,
,
,
chon
a di gi
a tri. tai
ay. Nhung ta chon
. mt
. s hang
. v`
. nhung ct
. n`
. s hang
.
, , ,
,

c
n`
ao trong s c
ac s hang
da
o? tra l`oi cu hoi n`
ay ta xet h`nh 5.7
.
,
,
,
2
0

i su. bin di c
vo
ac s 8, 7, 5, 12 tai
ac ct
a 2 . Ta ch
u y l`
a s hang
. c
. 2 v`
.
,
,
,,
,

`
8 t
ang, c`
on lai
ac s hang
kh
ac giam. Nhu vy
i ho. p l.
. c
.
. tn tai
. ba buoc d
,
,
`

ng 3 l`
t`u trang
th
ai [8, 7, 5, 12] dn trang
th
ai c
o Tng-Nim ba
a [8, 2, 5, 12]
.
.
,,
,
. c [8, 7, 0, 12] hoa
. c [8, 7, 5, 9], (thm v`
mt
hoa
ao do
i khng ho. p l.
. buoc d
[13, 5, 7, 12]).

,
5.4. Chuyn d
o choi Nim
` chin thut
. tr`
8
7
5
12

=
=
=
=

1
0
0
1

0 0 0
1 1 1
1 0 1
1 0 0

| {z }
vi. tr thay

1
0
0
1

1
0
0
0

233
0
1
0
0

1
0
0
1

=
=
=
=

13
2
0
9

(t
ang)
,
(giam)
,
(giam)
,
(giam)

,
,
2
H`nh 5.7. Kt qua thay di tai
a 20
. ct
. 2 v`

,
,
,,
,
,
nhi. phn: nhn
2. Quy ta
c nhn
on hon duoi dang
. biu din
. bit s l
. bit
,
,
c sau: T`m v` bn tr
n hon trong dang
s n`
ao lo
nhi. phn theo nguyn ta
ai
.
,
,,
nhi. phn; s c
hai s kh
nht vi. tr o d
ac nhau trong dang
o s 1
o
. biu din
,
,
,
,
,
n hon 0111(7), 11011(27) nho ho,n
tai
ay l`
a l
on hon. V du. 1000(8) lo
. vi. tr n`
s 11101(29).
, , ,
,
,
,
3. Chung minh mnh
ac
`: di t`u mt
. d
. Tng-Nim sang mt
. Tng-Nim kh
, ,
,
,
,
,,

nho hon, truoc tin ta so s


anh biu din nhi. phn cua hai Tng-Nim v`
a x
ac
,
,
l`
dinh
a ch
ung kh
ac nhau, ta goi
a D-vi. tr.
. nhung vi. tr m`
. nhung vi. tr do
,
,
,
,

Nu bt k` mt
a tri. tai
i gi
. s hang
. duo. c thay d
. mi D-vi. tr, th` Tng-Nim
,
mong mun se cho kt qua. Nu s hang
c
o 0 tai
ai nht, th` se
.
. D-vi. tr tr
,
,
,
i nhung s hang
s hang
thay th`
anh 1 v`
a khi do
ang; nhung vo
o 1 tai
. se t
. c
.
,
,
`
ng 0 v`

D-vi. tr tr
ai nht, n
o se thay ba
a nhu vy
se giam. Do d
o
. s hang
.
,
,
,
,,
,`,
,
d t`m buoc di ho. p l. cho phep thay di trong Tng-Nim, nguoi ta chon
.
,
,

i s 1 tai
mt
vo
ai nht, v`
a thay d
o tai
i n
. s hang
.
. D-vi. tr tr
. mi D-vi. tr.
2
0
`
(Trong phn 1 v du. h`nh 5.6 th` D-vi. tr l`
a ct
a 2 . S hang
12 c
o
. 2 v`
.
,
,
,
2

1 tai
a thay d
o tai
i n
. D-vi. tr (ct
. 2 ) v`
. nhung D-vi. tr se sinh ra s nho
,
,
`ng lun lun t`n tai
,
hon). thy ra
c
o 1 tai
. mt
. s hang
.
. D-vi. tr, ta nho
,
, ,
,
,,
,,
,
lai
a Tng-Nim mong mun nho hon Tng-Nim hin
c2o
. l`
. th`oi, theo buo
,
,
,
,
,
trn, Tng-Nim hin
o 1 v`
a Tng-Nim mong mun phai c
o 0 tai
. th`oi phai c
.
,
,
2

D-vi. tr tr
ai nht (trong v du. trn tai
o 1 v`
a
. 2 vi. tr, Tng-Nim hin
. th`oi c

,,
,
,ng tr`
Chuong 5. Nhu
o choi to
an hoc
.

234

,
,
,
Tng-Nim mong mun c
o 1 tai
ai
o 0). V` Tng-Nim hin
. D-vi. tr tr
. th`oi c
,
,
,
,
i 1 tai
nht, nn phai t`n tai
vo
ai nht
. mt
. s le nhung s hang
.
. D-vi. tr tr
,
,
,
,i 1 tai
n`
ay. V` 0 khng phai l`
a s le, nn phai t`n tai
. t nht mt
. s hang
. vo
.
,
,
,,

D-vi. tr tr
ai nht n`
ay, v`
a nhu vy
o tai
ay l`
a mt
i n
. thay d
. D-vi. tr n`
. buoc
,
,

di ho. p l. cho ra Tng-Nim mong mun.


,
, ,
,,
V du. sau ch ra k n
ang su dung
mnh
d` trn d thu. c hin
an
.
.
. tnh to
,
,

su. bin di mt
. s hang.
.
,
,
`, trang
V du. 5.27. Ba
t d
ai tr`
o choi Nim [11, 22, 33] c
o Tng-Nim l`
a
`u tu
. th
,
,
, ,
,
,
,
,
,
60. H
ay ch ra l`
am th n`
ao d
o buoc d
i ho. p l. d
chuyn d
i Tng-Nim d
n
c

a) 55;

b) 0.

,
,
,
`,i giai. Ta kim tra lai
Lo
. Tng-Nim:
]
11
22
33
60

=
=
=
=

25
0
0
1
1

24
0
1
0
1

23
1
0
0
1

22
0
1
0
1

21
1
1
0
0

20
1
0
1
0

,
,
,
,
a) Ta so s
anh hai biu din nhi. phn cua Tng-Nim hin
o v`
a Tng-Nim
. c
mun c
o:
60
55

=
=

25
1
1

24
1
1

23 22 21
20
1
1
0
0
0
1
1
1

|
{z
}

ct
ac nhau
. s kh

,
Vi. tr ct
ac nhau bn tr
ai nht l`
a 23 . Trong c
ac s hang
o
. kh
. 11, 22, 33 ch c
,
,
,,
,
,,
,
`

ng
s 11 c
o 1 o vi. tr n`
ay v`
a nhu vy
o buoc d
i ho. p l. cho Tng-Nim ba
. d c
,
, ,
3
1
0
55 l`
a thay d
a 2 , ngha l`
a di t`u 11(001011)2 th`
anh
i 11 tai
. vi. tr 2 , 2 v`
0(000000)2 .

,
5.4. Chuyn d
o choi Nim
` chin thut
. tr`

235

,
,
,,
b) Ta so s
anh hai s biu din duo
i dang
a 0:
. nhi. phn cua 60 v`

60
0

=
=

25 24 23 22
1
1
1
1
0
0
0
0

|
{z
}

ct
s
kh
a
c
nhau
.

21
0
0

20
0
0

,
,,
Ct
ac tr
ai nht o vi. tr 25 . Trong c
ac s hang
11, 22, 33 ch c
o 33 c
o
. sai kh
.
,
,
5
4
3
2

1 tai
ay v`
a nu ta thay di tai
a 2 cua 33, ta nhn
. vi. tr n`
. vi. tr 2 , 2 , 2 v`
.
,
,
,
,,
,,
,
`
buoc d
duo. c (011101)2 = 29. Do d
a giam
o
i ho. p l. chuyn Tng-Nim v 0 l`
,

t`u 33 xung c`
on 29.
,
,
,
ng minh bang chin thut
ng thng qua mnh
Chu
o choi Nim du
d`:
. tr`
.

,
,ng mnh
ng:
Mnh
d
o choi Nim, nhu
` sau d
u
` 5.3. Trong tr`
. d
.
,
,
,,
`,.
` mt
th
ai r
o d`u d
th
ai mo
(N1 ) Buoc d
i bt k` tu
u a d
n mt
. trang
. trang
.
.
,
,, ,,
`, bt k` d
`, mt
(N2 ) Tu
th
ai mo
o th chon
i dn trang
`u c
uo. c buoc d
. trang
. d
.
.
th
ai r
o.

,
,,
, , ,
`, d
khng th c
a) l`
a mt
(N3 ) Trang
th
ai kt th
uc (tu
o buoc di d
o
uo. c nu
.
.
trang
th
a
i
r
o
.
.
,,
,
,ng minh. Mnh
duo. c n
Chu
oi dn trong ph`n nhung trang
ai
. d` (N3 ) da
. th
,,
,
,
. c bit
da
o choi Nim o trn. C`
on (N1 ) v`
a (N2 ) d d`
ang suy ra t`u
. trong tr`
,
mnh
d
a mnh
d` 5.2 v` Tng-Nim.
` 5.1 v`
.
.
,
,
`ng 0 v`
a nhu vy
Nu ta c
o mt
ai r
o th` Tng-Nim ba
. theo mnh
.
. trang
. th
,
,
,
,
,
c di ho. p l,
d` 5.1 sau buo
T
ng-Nim
khng
th
l`
a
0,
ngh
a
l`
a
tr
ang
th
ai
.
.
,
,
i se l`
mo
a m`o.
,
,
,n ho,n 0 v`
Nu mt
ai l`
a m`o th` Tng-Nim se l`
a mt
a theo
. trang
. th
. s lo
,
,,
,,
,
,
`
i trang
mnh
d 5.2, n
o c
o kha n
ang t`m d
uo. c mt
i ho. p l. to
.
. mt
. buoc d
.
th
ai r
o.

236

,,
,
,ng tr`
Chuong 5. Nhu
o choi to
an hoc
.

,
,
,
ng dung

`u
no
5.4.3. Ch
ung minh bang chin thut
. va
.
,
,
,ng minh bang chin thut
ng:
By gi`o ta di chu
. 5.3 du
, ,,
,
,,
,
theo (N2 ) ngu`oi thu nht
a) Gia su trang
ai nguyn gc l`
a m`o: Khi do
. th
,
,,
,,
,
,,
,
c
o th c
o buo
cd
ai r
o; ngu`oi n`
ay goi
c di nhu vy
i dn trang
. th
. nhung buo
.
,
,,
,,
,,
,
nu ngu`oi thu hai c
l`
a buoc d
o th c`
on di duo. c (nu trang
th
ai
i tt. Sau do
.
,
,
, ,
,
,
,
khng phai l`
a kt th
uc tr`
o choi), ngu`oi n`
ay theo (N1 ) ch c
o th dua v`
,
,
,
,
,,
,
,
,
,
c d
trang
ai m`o. Tip tuc
i tt, ngu`oi thu
. thu. c hin
. buo
. th
. ngu`oi thu nht lai
,,
,
,,
,
, cho,i theo
,i trang
hai se lai
o buo
cd
ai m`o, ... Nu ngu`oi thu nht cu
i t o
. c
. th
, , ,
,,
,
,,
i d
ay trang
c
ach n`
ay (buo
c di tt), th` ngu`oi thu hai lun lun dn luo. t vo
.
,
,
,,
,
,
,

c. n mt
th
ai r
o, mi buoc d
l
u
c
n`
ao
i keo theo giam s vin soi hon truo
.
,
,`, ,
,
i trang
. t vo
nguoi thu hai se phai di ma
d
ai kt th
uc v`
a bi. thua.
o
. th
,,
,
,,
,
,
,
b) Tuong tu. , nu trang
ai nguyn gc l`
a r
o: ngu`oi thu nht se c
o buo
c
. th
,
,
,
,
,
,
,
,
,

i d
d
th
ai m`o v`
a ngu`oi thu hai c
o th dn ngu`oi thu nht vo
ay
i d
n trang
.
,,
,`, ,

buoc d
th
ai r
o; tt nhin nguoi thu nht d
uc n`
ao do
i cho trang
n mt
.
. l
,
,
,
,
,

i trang
. t vo
ng.
ngu`oi thu hai se tha
phai di ma
ai kt th
uc v`
a khi d
o
. th

,
,
,
H`nh 5.8. So d` b quyt cua tr`
o choi Nim
,
,
,
,
T
om lai,
o choi Nim, th` trang
th
ai se phai thay nhau
. d choi tt tr`
.
,
,
,
,
,i khi d
,i trang
chuyn di giua r
o v`
a m`o cho to
th
ai kt th
uc. Nu
at
. to
.

,
5.4. Chuyn d
o choi Nim
` chin thut
. tr`

237

, , ,
,
khng c
o g` sai l`m th` nu trang
ai nguyn gc l`
a m`o th` ngu`oi thu
nht
. th
,
,
,
,
,

ng
hai phai tha
ng, nu trang
phai tha
th
ai nguyn gc l`
a r
o th` ngu`oi thu
.
,
,
,
, `
nhu bang chin thut
a thu. c hin
5.8.
. 5.3 v`
. theo so d
, ,,
,
`,
`, nhu
,ng trang
V du. 5.28. Tu
ai m`o sau d
ay t`m tt ca buoc d
i tt tu
y h
. th
ch
ung:
a) [3, 4, 8, 9]

b) [8, 10, 7, 1]

c) [9, 25, 49]

d) [6, 5, 3, 1].

,
,
,
`,i giai. a) Tng-Nim 3 ] 4 ] 8 ] 9 = 6, ta kim tra lai:
Lo
.
]
3
4
8
9
6

=
=
=
=
=

23
0
0
1
1
0

22
0
1
0
0
1

21
1
0
0
0
1

20
1
0
0
1
0

,
,
,
,
,
,,
`ng 0, nh`n v`
c
o buo
cd
a ta phai d
ao bang
i tt ngha l`
i Tng-Nim ba
,
,
2
trn ta phai thay d
a 21 mt
ac s hang.
Tai
i tai
. vi. tr ct
. 2 v`
. trong c
.
. vi. tr
,
,
,,
2

. Do do
buoc di tt l`
ct
o s 4 l`
a c
o biu din nhi. phn 1 tai
a
o
. 2 ch c
. d
,
,
,,
,

di 4(= 0100) th`


anh 2(= 0010). Ngha l`
a ly o d
ng thu hai di hai vin soi.
, , ,
,
,
. c
Meo
o choi Nim: Trong khi choi tr`
o n`
ay khng th ke bang hoa
. trong tr`
,
,
,,
bin di nhi. phn c
ac s r`i tnh tng rt phi`n h`
a. C
o mt
ach l`
a su dung
. c
.
,
,,
,
phep Tng-Nim duo. c tin h`
anh nhu sau (h`nh 5.9):
,
, ,
dng soi
1. Chia mi
ly ph`n l
uy th`ua cua 2 g`n nht, v du. [3, 4, 8, 9]
chia th`
anh d
ay 1, 2, 4, 8, 8, 1.
,
,
,,
2. Theo Tng-Nim trong d
ay c
o hai 8, 8 v`
a 1, 1 d
on
`u bi. truo. t tiu, ch c`
,
,
,
,
`
`
ng 0 th` tng hai s n`
ng
lai
am Tng-Nim ba
ay phai ba
l`
. dng 2, 4 d
,
,,
0 ngha l`
a phai ly d
i 2 vin o d
ng 4 vin.
,
`ng 4:
b) Tng-Nim ba

,,
,
,ng tr`
Chuong 5. Nhu
o choi to
an hoc
.

238

?
4

]
8
10
7
1
4

8
H`nh 5.9
23
1
1
0
0
0

=
=
=
=
=

22
0
0
1
0
1

21
0
1
1
0
0

20
0
0
1
1
0

, ,
,
,
,,
2
`ng 0, ta phai thay di mt
Tng-Nim ba
ay
. s hang
. o ct
. 2 . Trong ct
. n`
,
,
,
,
c di tt nht l`
ch c
o 7 c
o s 1, vy
a chuyn 7(= 0111) th`
anh 3(= 0011),
. buo
,
,,
,
ba d
ngha l`
a ly o d
ng thu
i 4 vin soi.
,
`ng 33:
c) Tng-Nim ba
]
9
25
49
33

25
0
0
1
1

=
=
=
=

24
0
1
1
0

23
1
1
0
0

22
0
0
0
0

21
0
0
0
0

20
1
1
1
1

, ,
,
,
,,
5
`ng 0, ta phai thay di mt
Tng-Nim ba
o ct
a mt
. s hang
.
. 2 v`
. s
,
,,
,
,
0
5
c di tt nht l`
hang
o ct
o 49 c
o s 1, vy
a
.
. 2 . Trong ct
. 2 ch c
. buo
,
,,
,

chuyn 49(= 110001) th`


anh 16(= 010000), ngha l`
a ly o d
ng thu ba di
,
33 vin soi.
,
`ng 1:
d) Tng-Nim ba
]
6
5
3
1
1

=
=
=
=
=

22
1
1
0
0
0

21
1
0
1
0
0

20
0
1
1
1
1

,
5.4. Chuyn d
o choi Nim
` chin thut
. tr`

239

, ,
,
,
,,
0
`ng 0, ta phai thay di mt
Tng-Nim ba
ay
. 2 . Trong ct
. n`
. s hang
. o ct
,
,
,
c d
c
o ba s 5, 3, 1 c
o s 1, vy
o ba kha n
ang l`
a ly d
i tt nht c
i 1 vin
. buo
,,
.
o mt
ng do
. trong ba d

` tp
Bai
.
,
,,
,
,
. 5.29. (Tr`
o choi Nim giai ) Mt
ang giy d`
ai d
anh nhung
uo. c chia th`
. b
vung v`
a g
an nh
an 0, 1, 2, 3, ...(h`nh 5.10).

H`nh 5.10
,
,,
,
vung c
. t mt
. c mt
Trn mi
o th da
ai qun c`o. Hai ngu`oi l`n
. hoa
. v`
,
,,
,
,,
,,
,
ng 1 qun v`
luo. t thu. c hin
ac buo
cd
cd
a chuyn d
i: Mt
i ho. p l. l`
u
. c
. buo
,
,
,
,
,
,
ng cuc
c cui c`
pha tr
ai (v` s 0). Ngu`oi tha
a ngu`oi di buo
ung v` chu`ng
. l`
,
,
,
,
i tr`
( nh
an s 0). H
ay t`m mi quan h. giuu tr`
o choi n`
ay vo
o choi Nim v`
a
,,
,
,
,

c.
t`m buoc d
ai cho truo
i tt t`u mt
. trang
. th
,
,
. 5.30. (Tr`
o choi Nim qun c`o)
,
, ,,
,
du,o.,c d
. t mt
. c
Tr`
o choi d
an c`o vua. Trn mi
uo. c thu. c hin
a
. trn b`
. hoa
,`,
,
,,
,,
,
,
v`
ai qun c`o. Hai nguoi choi thay phin nhau thu. c hin
i: Mt
. buoc d
. buoc
,
,
,
,

ng mt
di ho. p l. l`
a chuyn du
ao
. qun c`o t`u mt
. sang bt k` vung n`
,
,
`
`
`
. c v pha tr
v pha trn hoa
ai (nhung khng dng th`oi). Muc
a d`n
. dch l`
,
,
,,
,
,
nh du l`
ht tt ca qun c`o v` vung duo. c da
a chu`ng (h`nh ??) v`
a ngu`oi
,
,
,,
,
ng cuc.
ay giai thch mi
dua qun c`o cui c`
ung v` chu`ng l`
a ngu`oi tha
. H
,,
,
,
,
,i tr`
lin quan tr`
o choi n`
ay vo
o choi Nim v`
a t`m buo
c di tt t`u mt
ai
. trang
. th
,
cho (v du. nhu h`nh ??).
da

,
` LIU
TAI
. THAM KHAO
,
,
,,
[1] Kvant (Tap
an hoc
a tru`ong ph thng cua Lin X c
u v`
a
. ch to
. cho nh`
ng`
ay nay l`
a Lin bang Nga, n
am 1970-2002)
,ng suy lun
[2] G. Polya, To
an hoc
a nhu
o l, NXBGD, 1995.
. v`
. c
[3] R. j. McEliece, R. B. Ash, C. Ash, Introduction to Discrete Mathematics,
McGraw-Hill Book Co., 1989.
,
,
[4] G. H. Hardy, J.E. Littlewood, G. Polya Bt d
ng thuc, NXB HQG H`
a
a
Ni,
2002.
.
,,
[5] Nguyn V
an Mu,
am, NXB GD, 1998.
. Phuong tr`nh h`
,
,
,,
,
[6] Nguyn Huu in, Phuong ph
ap irichle v`
a ung dung,
NXB KHKT,
.
1999.
,
,
,,
[7] Nguyn Huu in, Phuong ph
ap Quy nap
an hoc,
. to
. NXB GD, 2000.
,
,
,
,,
, ,
[8] Nguyn Huu in, Phuong ph
ap S phuc v
oi h`nh hoc
ng, NXB
. pha
HQG, 2000.
,
,
,
,
,
,ng phu,o,ng ph
[9] Nguyn Huu in, Nhu
ap d
an ph
in h`nh trong giai to
thng, NXB GD, 2001.
,
,
,
,
,ng phu,o,ng ph
[10] Nguyn Huu in, Nhu
ap giai b`
ai to
an cu. c tri. trong h`nh
hoc,
. NXB KHKT, 2001.
,
,
,
,
[11] Nguyn Huu in, S
ang tao
an ph thng, NXB GD, 2002.
. trong giai to
,
,
,
,
[12] Nguyn Huu in, a thuc v`
a ung dung,
NXB GD, 2003.
.

240

NHUNG KI HIU
.

,
,i c
Trong cun s
ach n`
ay ta d`
ung nhung ky hiu
ac y ngha x
ac dinh
.
. vo
,
,,
trong bang duoi d
y:

N
Z
Q
R
C

Ckm
..
.
.
6 ..
USCLN
BSCNN
deg

IMO
APMO

,
,
tp
. ho. p s tu. nhin
,
tp
. ho. p s nguyn
, ,
,
tp
. ho. p s huu t
,
,
tp
. ho. p s thu. c
, ,
tp
. ho. p s phuc
,
du d`ng du
, ,
,
,,
t ho. p chp
. k cua m ph`n tu
phep chia ht
khng chia ht
,,
,n nht
uo
c s chung lo
,
bi
. s, chung nho nht
,c
bc
a thu
. cua d
,
ng minh
Kt th
uc chu
International Mathematics Olympiad
Asian Pacific Mathematics Olympiad

241

MUC
. LUC
.
`,i n
Lo
oi d
`u . . . . . . . . . . . . . . . . . . . . . . . . . . . . . . . . . . . . . . . . . . . . . . . . . . . . . . . . .
,,
Chuong 1. Nguyn l bt bin. . . . . . . . . . . . . . . . . . . . . . . . . . . . . . . . . . . . . .
,,
,,
,i thiu
1.1. Gio
ap dai
. phuong ph
. luo. ng bt bin . . . . . . . . . . . . . . . . . . . .

3
7
7

1.2. Ph
at hin
ai to
an . . . . . . . . . . . . . . . . . . . . . . . . . . . . . 9
. bt bin trong b`
,
,
,
`ng dai
1.3. Giai to
an ba
. luo. ng bt bin . . . . . . . . . . . . . . . . . . . . . . . . . . . 24
,
1.4. Bt bin d
on d
iu
. . . . . . . . . . . . . . . . . . . . . . . . . . . . . . . . . . . . . . . . . . . . 27
,
1.5. Nhung b`
ai to
an nng cao . . . . . . . . . . . . . . . . . . . . . . . . . . . . . . . . . . . . 43
1.6. Chuyn d` v` h`
am bt bin. . . . . . . . . . . . . . . . . . . . . . . . . . . . . . . . . .

51

1.6.1. inh
am bt bin trn trang
ai . . . . . . . . . . . . . . . .
. ngha h`
. th
,
1.6.2. H. thng bt bin d
`y d
u . . . . . . . . . . . . . . . . . . . . . . . . . . . . . . .
,,
,
,
c d
ng hai bin . . . . . . . . . . . . . . . . . . . . . . . . . . . .
Chuong 2. a thu
i xu

52
58
64

2.1. inh
a tnh cht . . . . . . . . . . . . . . . . . . . . . . . . . . . . . . . . . . . . . 65
. ngha v`
,
,
,c hai bin . . . . . . . . . . . . . . . . . . . . . . . . . . 67
2.2. inh
. l co ban cho da thu
,
,,
,ng . . . . . . . . . . . . . . . . . . . . . . . . . . . . . . . 71
2.3. Giai h. phuong tr`nh di xu
,
,,
,ng . . . . . . . . . . . . . . . . . . . . . . 77
2.4. ua v` h. phuong tr`nh dang
i xu
. d
,
,c d
,ng . . . . . . . . . . . . . . . . . . . . . . . . . 82
,ng minh bt d
ng thu
2.5. Chu
i xu
a
,c bc
2.6. B`
ai to
an v` tam thu
. hai . . . . . . . . . . . . . . . . . . . . . . . . . . . . . . . . 87
,c d
,ng ra th`u,a s . . . . . . . . . . . . . . . . . . . . . . . 91
2.7. Phn tch da thu
i xu
,
2.8. Nhung b`
ai to
an kh
ac . . . . . . . . . . . . . . . . . . . . . . . . . . . . . . . . . . . . . . . . 95
,,
,ng . . . . . . . . . . . . . . . . . 101
2.9. Chuyn d` v` phuong tr`nh h. s d
i xu
,
, ,
,c h. s di xu
,ng . . . . . . . . . . . . . . 101
2.9.1. inh
. l co ban cua da thu
242

MUC
. LUC
.
,
,
,,
2.9.2. Nhung v du. giai phuong tr`nh bc
. cao. . . . . . . . . . . . . . . . .
, ,
,
,
,
2.10. Go. i y v`
a tra l`oi b`
ai tp
. chuong 2 . . . . . . . . . . . . . . . . . . . . . . . . . .
,
,
,,
,
, tu., . . . . . . . . . . .
c cua c
ng thu
Chuong 3. Bt d
ac d
ay s d`ng thu
a
, tu., hai hoa
. p thu
. c ba s. . . . . . . . . . . . . . . . . . . . . . . . . . . . . . . . . .
3.1. Ca
,
3.2. Tng qu
at h
oa . . . . . . . . . . . . . . . . . . . . . . . . . . . . . . . . . . . . . . . . . . . . .
,,
, tu., . . . . . . . . . . . . . . . . . . . . . . . . . . . .
3.3. Su dung
c
ac d
ay s d
`ng thu
.
,
,c Karamata . . . . . . . . . . . . . . . . . . . . .
ng thu
3.4. Chuyn d` v` bt da
,
,c Karamata . . . . . . . . . . . . . . . . . . .
ng thu
3.4.1. H`
am l`i v`
a bt da
,
,,
,c Karamata . . . . . . . . . . . . . . . . . . . . . .
ng thu
3.4.2. Su dung
bt d
a
.
,
,c kh
ng thu
3.4.3. Mt
ac . . . . . . . . . . . . . . . . . . . . . . . . . . . .
. s bt da
,
,
,c trong tam gi
ng thu
3.4.4. Nhung bt d
ac . . . . . . . . . . . . . . . . . . .
a
,
,,
,
,
3.5. Go. i y v`
a tra l`oi b`
ai tp
. chuong 3 . . . . . . . . . . . . . . . . . . . . . . . . . . .
,,
,,
Chuong 4. Phuong tr`nh h`
am . . . . . . . . . . . . . . . . . . . . . . . . . . . . . . . . . . .
,,
,
4.1. Phuong ph
ap th nhung gi
a tri. d
i s . . . . . . . . . . . . . . . . . . . . . .
,
,,
4.2. Phuong ph
ap d
im bt dng
. ...............................
,c giao ho
4.3. Chuyn d
ac d
an . . . . . . . . . . . . . . . . . . . . . . . .
` v` c
a thu
4.3.1. inh
. ngha . . . . . . . . . . . . . . . . . . . . . . . . . . . . . . . . . . . . . . . . . . . .
,c Chebyshev . . . . . . . . . . . . . . . . . . . . . . . . . . . . . . . . . . . .
4.3.2. a thu
,
,c giao ho
4.3.3. B`
ai to
an tng qu
at v` c
ac d
an . . . . . . . . . . .
a thu
, ,
,
,
,
4.4. Go. i y v`
a tra l`oi b`
ai tp
. chuong 4 . . . . . . . . . . . . . . . . . . . . . . . . . . .
,,
,
,
ng tr`
Chuong 5. Nhu
o choi to
an hoc
. .............................
5.1. Chin thut
. bt bin . . . . . . . . . . . . . . . . . . . . . . . . . . . . . . . . . . . . . . .
,
,c to
,i tr`
5.2. Kin thu
an vo
o choi . . . . . . . . . . . . . . . . . . . . . . . . . . . . . . . . .
,
,
5.3. Nhung b`
ai to
an thi hoc
. sinh gioi . . . . . . . . . . . . . . . . . . . . . . . . . . .

243
105
110
132
134
143
149
156
156
163
166
168
171
176
176
187
194
194
197
200
203
205
206
210
214

244

MUC
. LUC
.

,
5.4. Chuyn d` chin thut
o choi Nim . . . . . . . . . . . . . . . . . . . . . . . .
. tr`

225

5.4.1. inh
a v du. . . . . . . . . . . . . . . . . . . . . . . . . . . . . . . . . . . .
. ngha v`
,
5.4.2. Chin thut
o choi Nim . . . . . . . . . . . . . . . . . . . . . . . .
. trong tr`
,
,ng minh bang chin thut
,ng dung
5.4.3. Chu
au
n
o.........
. v`
.
,
T`
ai liu
. tham khao . . . . . . . . . . . . . . . . . . . . . . . . . . . . . . . . . . . . . . . . . . . . .
,
Nhung k hiu
. ..................................................

225

241

Muc
. luc
. ........................................................

242

229
236
240

,
Chiu
ach nhim
. tr
. xut ban:
,
,
`
Chu tich
am d
c NG TRN AI
. HQT kim Tng Gi
,
,
, ,
DUONG THUY
Ph
o Tng Gi
am dc kim Tng bin tp
. VU
.
Bin tp
. ni
. dung:
,
`

TRN HUU NAM


Tr`nh b`
ay b`a:
`
HANH
TRN THI. THUY
.
,
Ch ban:
,
HU
,U IN
NGUYN
,, ,
Sua ban in:
,
HNG
`
NGUYN
SON

,
, ,
, ,

`
AI
BANG
PHUONG PHAP
GIAI TOAN
. LUO
. NG BT BIN
M
a s: 8I004T5-TTS
,
,
In 3.000 ban (10TK), kh 14, 3 20, 3 cm Tai
a Ty
. Cng ty In H`
,
S in: 29/TK. S xut ban: 21/727-05.
,
,
In xong v`
a np
ang 4 n
am 2005.
. luu chiu th

You might also like